[ { "question": " 1 - Anneau de matrices [Signaler une erreur] [Ajouter à ma feuille d'exos]Enoncé Soit $A$ l'ensemble des matrices s'écrivant $\\begin{pmatrix}a&b\\\\0&a\\end{pmatrix}$ avec $a$ et $b$ des entiers relatifs. \n\n\n Démontrer que $A$ est un anneau pour les lois d'addition et de produits de matrices.\n\n Déterminer les éléments inversibles de $A$. \n\n", "answer": " \n Il suffit de démontrer que $A$ est un sous-anneau de $\\mathcal M_2(\\mathbb R).$ Pour cela, on remarque que\n\n\n $I_2\\in A$ (choisir $a=1$ et $b=0$).\n\n Si $M=\\begin{pmatrix}a&b\\\\0&a\\end{pmatrix}$ et $M'=\\begin{pmatrix}a'&b'\\\\0&a'\\end{pmatrix}$ sont dans $A$, alors \n\n$$M-M'=\\begin{pmatrix}a-a'&b-b'\\\\0&a-a'\\end{pmatrix}$$\n\net \n\n$$MM'=\\begin{pmatrix}aa'&ab'+ba'\\\\0&aa'\\end{pmatrix}$$\n\nsont dans $A.$\n\n\n Soit $M=\\begin{pmatrix}a&b\\\\0&a\\end{pmatrix}$ inversible dans $A$. Alors il existe $M'=\\begin{pmatrix}a'&b'\\\\0&a'\\end{pmatrix}$ tel que $MM'=I_2$. On obtient $aa'=1$, et comme $a$ et $a'$ sont dans $\\mathbb Z$, on doit avoir $a=\\pm 1$. Dans ce cas, on aura $a'=a$. De plus, on doit aussi avoir \n\n$ab'+ba'=0$ ce qui entraîne $b'=-b$.\n\nRéciproquement, soit $M\\in A$ qui s'écrit\n\n$$M=\\begin{pmatrix}a&b\\\\0&a\\end{pmatrix}$$\n\navec $a=\\pm 1$.\n\nPosons \n\n$$M'=\\begin{pmatrix}a&-b\\\\0&a\\end{pmatrix}.$$\n\nAlors \n\n$$MM'=\\begin{pmatrix}a^2&ab-ba\\\\0&a^2\\end{pmatrix}=I_2.$$\n\nAinsi, $M'$ est inversible. On a donc démontré que les éléments inversibles de $A$ sont ceux pour lesquels $a=1$ ou $a=-1.$\n\n\n" }, { "question": " 2 - Anneau des entiers de Gauss [Signaler une erreur] [Ajouter à ma feuille d'exos]Enoncé On appelle ensemble des entiers de Gauss noté $\\mathbb Z[i]$ l'ensemble des nombres complexes qui s'écrivent $a+ib$, avec $a$ et $b\\in\\mathbb Z.$\n\n\n Démontrer que $\\mathbb Z[i]$ est un anneau.\n\n Pour tout nombre complexe $z$, on note $N(z)=z\\bar z.$\n\n\n Démontrer que, pour tous nombres complexes $z$ et $z'$, $N(z)N(z')=N(zz').$\n\n Démontrer que, pour tout entier de Gauss $z$, $N(z)$ est un entier naturel.\n\n Soit $z$ un entier de Gauss inversible. Déduire des questions précédentes que $N(z)=1$.\n\n Quels sont les éléments inversibles de $\\mathbb Z[i]$?\n\n\n", "answer": " \n On va prouver que $\\mathbb Z[i]$ est un sous-anneau de $\\mathbb C.$\n\nEn effet,\n\n\n $1=1+0i\\in\\mathbb Z[i];$\n\n Soit $z=a+ib$ et $z'=a'+ib'\\in\\mathbb Z[i].$ Alors \n\n$$z-z'=(a-a')+i(b-b')\\in\\mathbb Z[i]$$\n\n(puisque $a-a'$ et $b-b'\\in\\mathbb Z$) et \n\n$$zz'=(aa'-bb')+i(ab'+a'b)\\in\\mathbb Z[i]$$\n\n(puisque $aa-bb'$ et $ab'+a'b\\in\\mathbb Z$).\n\n\n \n On remarque que $N(z)=|z|^2.$ Puisque $|zz'|=|z|\\cdot |z'|,$ en mettant au carré cette égalité, on a le résultat demandé.\n\n Si $z=a+ib$, alors $N(z)=a^2+b^2$ et $a^2,b^2$ sont des entiers naturels, donc $N(z)$ aussi.\n\n Soit $z$ un entier de Gauss inversible et soit $z'$ son inverse. Alors on sait que $zz'=1$ et donc $N(z)\\times N(z')=1$. Or le produit de deux entiers naturels est égal à $1$ si et seulement si ces deux entiers sont égaux à $1$. Donc $N(z)=1.$\n\n Soit $z=a+ib\\in\\mathbb Z[i]$ inversible. Alors $N(z)=1$, et donc $a^2+b^2=1.$ Or, puisque $a^2$ et $b^2$ sont des entiers naturels, ceci n'est possible que dans quatre cas : $(a,b)=(1,0),$ $(a,b)=(-1,0),$ $(a,b)=(0,1),$\n\net $(a,b)=(0,-1).$ Réciproquement, il est facile de vérifier que $1$, $-1$, $i$ et $-i$ sont inversibles dans $\\mathbb Z[i],$ d'inverse respectif $1$, $-1$, $-i$ et $i$. Les éléments inversibles de $\\mathbb Z[i]$ sont donc $1$, $-1$, $i$ et $-i.$\n\n\n\n" }, { "question": " 3 - Endomorphisme de groupes [Signaler une erreur] [Ajouter à ma feuille d'exos]Enoncé Soit $(G,+)$ un groupe commutatif. On note $\\textrm{End}(G)$ l'ensemble des endomorphismes de $G$ sur lequel on définit la loi $+$ par $f+g:G\\to G,\\ x\\mapsto f(x)+g(x)$. Démontrer que $(\\textrm{End}(G),+,\\circ)$ est un anneau.", "answer": " On remarque d'abord que $+$ et $\\circ$ sont bien des lois de composition interne sur $\\textrm{End}(G)$. Ensuite, on vérifie tous les points de la définition d'un anneau.\n\n\n $(\\textrm{End}(G),+)$ est un groupe commutatif. En effet, la loi $+$ est associative et commutative, l'application $0_G:G\\to G,\\ g\\mapsto 0$ est un élément neutre pour la loi $+$, et tout élément $f\\in\\textrm{End}(G)$ admet un inverse $-f:G\\to G,\\ x\\mapsto -f(x)$.\n\n La loi $\\circ$ est associative.\n\n La loi $\\circ$ possède un élément neutre, qui est l'application identité.\n\n La loi $\\circ$ est distributive par rapport à la loi $+$ : pour tous $f,g,h\\in\\textrm{End}(G)$ et tout $x\\in G$, \n\n$$((f+g)\\circ h)(x)=(f+g)(h(x))=f(h(x))+g(h(x))=(f\\circ h+g\\circ h)(x)$$\n\net \n\n$$(f\\circ (g+h))(x)=f((g+h)(x))=f(g(x)+h(x))=f(g(x))+f(h(x))=(f\\circ g+f\\circ h)(x).$$\n\n\n\nAinsi, $(\\textrm{End}(G),+,\\circ)$ est un anneau.\n" }, { "question": " 4 - Rationnels à dénominateur impair [Signaler une erreur] [Ajouter à ma feuille d'exos]Enoncé Soit $\\displaystyle A=\\left\\{\\frac mn;\\ m\\in\\mathbb Z,\\ n\\in 2\\mathbb N+1\\right\\}$ (c'est-à-dire que $A$ est l'ensemble des rationnels à dénominateur impair). Démontrer que $(A,+,\\times)$ est un anneau. Quels sont ses éléments inversibles?", "answer": " On va démontrer que $A$ est un sous-anneau de $(\\mathbb Q,+,\\times)$. Pour cela, soient $x=\\frac mn$ et $y=\\frac{m'}{n'}\\in A$. Alors : \n\n$$x-y=\\frac{mn'-m'n}{nn'}\\textrm{ et }xy=\\frac{mm'}{nn'}.$$\n\nComme $nn'$, produit de deux nombres impairs, est impair, et que $A$ est non vide puisqu'il contient $1$, on en déduit que $A$ est bien un sous-anneau de $(\\mathbb Q,+,\\times)$. \n\nDéterminons ensuite les inversibles de $A$. Soit $x=\\frac mn\\in A$ inversible, et soit $y=\\frac{m'}{n'}\\in A$ tel que $xy=1$. On en déduit que $mm'=nn'$. En particulier, $m$ est nécessairement impair. Réciproquement, si $x=\\frac{m}n$ avec $m$ impair, alors $y=\\frac nm$ est dans $A$ (si jamais $m<0$, il suffit d'écrire $y=\\frac{-n}{-m}$ pour vérifier qu'il est bien dans $A$), et $xy=1$. Ainsi, les inversibles de $A$ sont les éléments $\\frac mn$ avec $m\\in\\mathbb Z$, $n\\in\\mathbb N^*$, et $m,n$ impairs.\n" }, { "question": " 5 - Décimaux [Signaler une erreur] [Ajouter à ma feuille d'exos]Enoncé Soit $\\mathbb D$ l'ensemble des nombres décimaux, \n\n$$\\mathbb D=\\left\\{\\frac{n}{10^k};\\ n\\in\\mathbb Z, k\\in\\mathbb N\\right\\}.$$\n\nDémontrer que $(\\mathbb D,+,\\times)$ est un anneau. Quels sont ses éléments inversibles?", "answer": " On va prouver que $(\\mathbb D,+,\\times)$ est un sous-anneau de $(\\mathbb Q,+,\\times)$. On remarque d'abord que $\\mathbb D\\subset\\mathbb Q$, puis que $1\\in\\mathbb D$. De plus, soient $x=\\frac n{10^k}$ et $y=\\frac{m}{10^l}$ deux éléments de $\\mathbb D$. Alors \n\n$$x-y=\\frac{n10^l-m10^k}{10^{k+l}}\\textrm{ et }xy=\\frac{nm}{10^{k+l}}$$\n\nsont clairement des éléments de $\\mathbb D$, et $(\\mathbb D,+,\\times)$ est bien un sous-anneau de $(\\mathbb Q,+,\\times)$.\n\nDéterminons ensuite les inversibles de $(\\mathbb D,+,\\times)$. Soit $x=\\frac{n}{10^k}$ inversible, d'inverse $y=\\frac{m}{10^l}$. Alors \n\n$$xy=1\\iff nm=10^{k+l}.$$\n\nOn en déduit que les seuls diviseurs premiers de $n$ sont $2$ et $5$, autrement dit que $n$ s'écrit $\\pm 2^p5^q$ pour $p,q\\in\\mathbb N$. Réciproquement, soit $x=\\frac{\\pm 2^p5^q}{10^k}$ et montrons que $x$ est inversible dans $\\mathbb D$. Posons $y=\\frac{\\pm 10^k}{2^p5^q}$. Il suffit de vérifier que $y$ est élément de $\\mathbb D$. Mais on peut aussi écrire\n\n$$y=\\frac{\\pm 10^k 2^q 5^p}{2^{p+q}5^{p+q}}=\\frac{\\pm 10^k 2^q5^p}{10^{p+q}}\\in\\mathbb D.$$\n\nAinsi, les inversibles de $(\\mathbb D,+,\\times)$ sont les éléments $\\frac{\\pm 2^p5^q}{10^k}$, avec $p,q,k\\in\\mathbb N$.\n" }, { "question": " 6 - Un anneau d'entiers [Signaler une erreur] [Ajouter à ma feuille d'exos]Enoncé On considère $\\mathbb Z[\\sqrt 2]=\\{a+b\\sqrt 2;\\ a,b\\in\\mathbb Z\\}$.\n\n\n Montrer que $(\\mathbb Z[\\sqrt 2],+,\\times)$ est un anneau.\n\n On note $N(a+b\\sqrt{2})=a^2-2b^2$. Montrer que, pour tous $x,y$ de $\\mathbb Z[\\sqrt 2]$,\n\non a $N(xy)=N(x)N(y)$.\n\n En déduire que les éléments inversibles de $\\mathbb Z[\\sqrt 2]$ sont ceux s'écrivant $a+b\\sqrt 2$ avec\n\n$a^2-2b^2=\\pm 1$.\n\n", "answer": " \n Il suffit de prouver que c'est un sous-anneau de $(\\mathbb R,+,\\times)$.\n\nMais $\\mathbb Z[\\sqrt 2]$ est\n\n\n stable par la loi + : $(a+b\\sqrt 2)+(a'+b'\\sqrt 2)=(a+a')+(b+b')\\sqrt 2$.\n\n stable par la loi $\\times$ :\n\n$$(a+b\\sqrt 2)\\times (a'+b'\\sqrt 2)=(aa'+2bb')+(ab'+a'b)\\sqrt{2}$$.\n\n stable par passage à l'opposé $-(a+b\\sqrt 2)=-a+(-b)\\sqrt 2$.\n\n\n\nDe plus, $1\\in \\mathbb Z[\\sqrt 2]$, ce qui achève la preuve du fait que $\\mathbb Z[\\sqrt 2]$ est un sous-anneau de $\\mathbb R$.\n\n Posons $x=a+b\\sqrt{2}$ et $y=a'+b'\\sqrt{2}$. En tenant compte de la formule pour le produit \n\nobtenue à la question précédente, on a\n\n\\begin{eqnarray*}\n\nN(xy)&=&(aa'+2bb')^2-2(ab'+a'b)^2\\\\\n\n&=&(aa')^2-2(ab')^2-2(a'b)^2+4(bb')^2.\n\n\\end{eqnarray*}\n\nD'autre part,\n\n\\begin{eqnarray*}\n\nN(x)\\times N(y)&=&(a^2-2b^2)(a'^2-2b'^2)\\\\\n\n&=&(aa')^2-2(ab')^2-2(a'b)^2+4(bb')^2.\n\n\\end{eqnarray*}\n\n Soit $x=a+b\\sqrt 2$. Supposons d'abord que $x$ est inversible, d'inverse $y$.\n\nAlors $N(xy)=N(1)=1$, et donc $N(x)N(y)=1$. Puisque $N(x)$ et $N(y)$ sont tous les deux des\n\nentiers, on a nécessairement $N(x)=\\pm 1$. Réciproquement, si $N(x)=\\pm 1$, alors,\n\nen utilisant la quantité conjuguée :\n\n$$\\frac{1}{a+b\\sqrt 2}=\\frac{a-b\\sqrt 2}{a^2-2b^2}=\\pm(a-b\\sqrt{2})$$\n\nce qui montre que $a+b\\sqrt 2$ est inversible, d'inverse $\\pm(a-b\\sqrt{2})$.\n\n\n" }, { "question": " 7 - Centre d'un anneau [Signaler une erreur] [Ajouter à ma feuille d'exos]Enoncé Soit $A$ un anneau. On appelle centre de $A$ et l'on note $C(A)$ l'ensemble des éléments $a\\in A$ tels que, pour tout $b\\in A$, $ab=ba.$ Démontrer que $C(A)$ est un sous-anneau de $A$.", "answer": " Il suffit de vérifier le théorème de caractérisation des sous-anneaux.\n\n\n $1\\in C(A)$, puisque $1a=a1=a$ pour tout $a\\in A.$\n\n Soit $a,a'\\in C(A)$ et soit $b\\in A$. Alors \n\n$$(a-a')b=ab-a'b=ba-ba'=b(a-a')$$ \n\n(on a utilisé deux fois la distributivité) et donc $a-a'\\in C(A).$\n\n Soit $a,a'\\in C(A)$ et soit $b\\in A$. Alors \n\n$$(aa')b=a(a'b)=a(ba')=(ab)a'=(ba)a'=b(aa')$$\n\n(on a utilisé plusieurs fois l'associativité) et donc $aa'\\in C(A).$\n\n\n" }, { "question": " 8 - Sous-anneaux de $\\mathbb Z^2$ [Signaler une erreur] [Ajouter à ma feuille d'exos]Enoncé Pour $d\\in\\mathbb N$, on note $A_d=\\{(x,y)\\in\\mathbb Z^2;\\ y-x\\in d\\mathbb Z\\}$. \n\n\n Démontrer que, pour tout $d\\in\\mathbb N$, $A_d$ est un sous-anneau de $\\mathbb Z^2$.\n\n Réciproquement, soit $A$ un sous-anneau de $\\mathbb Z^2$. Démontrer que $H=\\{x\\in\\mathbb Z;\\ (x,0)\\in A\\}$ est un sous-groupe de $\\mathbb Z$.\n\n En déduire qu'il existe $d\\in\\mathbb N$ tel que $A=A_d$.\n\n", "answer": " \n Il est clair que $0_{\\mathbb Z^2}$ et $1_{\\mathbb Z^2}$ sont éléments de $A_d$. Considérons ensuite $(x,y),(x',y')\\in A_d$. Que $(x+x',y+y')$ reste élément de $A_d$ ne pose pas de problèmes. Pour le produit, on a \n\n$$(x,y)\\times (x',y')=(xx',yy')$$\n\net on a \n\n$yy'-xx'=(y-x)y'+x(y'-x')\\in d\\mathbb Z$.\n\n $0\\in H$ et si $x,x'\\in H$, alors $(x-x',0)=(x,0)-(x',0)\\in A$ et donc $x-x'\\in H$. $H$ est un sous-groupe de $\\mathbb Z$.\n\n Puisque $\\mathbb Z$ est principal, il existe $d\\in\\mathbb N$ tel que $H=d\\mathbb Z$. Démontrons que $A=A_d$. D'une part, si $(x,y)\\in A$, alors \n\n$$(x-y,0)=(x,y)-y(1,1)\\in A$$\n\n et donc $d|x-y$, c'est-à-dire $(x,y)\\in A_d$. Réciproquement, si $(x,y)\\in A_d$, alors $x-y\\in d\\mathbb Z=H$, ce qui signifie que $(x-y,0)\\in A$. On termine presque comme précédemment en écrivant que\n\n$$(x,y)=(x-y,0)+y(1,1)\\in A.$$ \n\nLes sous-anneaux de $\\mathbb Z^2$ sont donc tous de la forme $A_d$.\n\n\n" }, { "question": " 9 - Sur certains sous-anneaux de $\\mathbb Q$ [Signaler une erreur] [Ajouter à ma feuille d'exos]Enoncé Soit $p$ un nombre premier. On note \n\n$$\\mathbb Z_p=\\left\\{\\frac{a}{b}:\\ (a,b)\\in\\mathbb Z\\times\\mathbb Z^*, p\\wedge b=1\\right\\}.$$\n\n\n Démontrer que $\\mathbb Z_p$ est un sous-anneau de $(\\mathbb Q,+,\\times).$\n\n Démontrer que pour tout nombre rationnel non nul $x$, soit $x\\in\\mathbb Z_p,$ soit $x^{-1}\\in \\mathbb Z_p.$\n\n Soit $B$ un sous-anneau de $\\mathbb Q$ contenant $\\mathbb Z_p.$ Démontrer que $B=\\mathbb Q$ ou que $B=\\mathbb Z_p.$\n\n", "answer": " \n On remarque d'abord que $1=\\frac11\\in\\mathbb Z_p$. Soit ensuite $x=\\frac ab$ et $y=\\frac cd$ deux éléments de $\\mathbb Z_p.$ Alors puisque $p\\wedge b=1$ et $p\\wedge d=1,$ on a $p\\wedge (bd)=1.$ Mais puisque \n\n$$x-y=\\frac{ad-bc}{bd}\\textrm{ et }x\\times y=\\frac{ac}{bd},$$\n\non a $x-y\\in\\mathbb Z_p$ et $x\\times y\\in\\mathbb Z_p.$ Ainsi, $\\mathbb Z_p$ est bien un sous-anneau de $\\mathbb Q.$\n\n Soit $x=\\frac{a}{b}\\in\\mathbb Q,$ $x\\neq 0$, écrit sous forme irréductible. Si $x\\in\\mathbb Z_p$, c'est fini. Sinon, c'est que $p|b$ (rappelons que $p$ est premier) et puisque $a\\wedge b=1$, on a $a\\wedge p=1$. Ainsi, $x^{-1}=\\frac{b}{a}\\in\\mathbb Z_p.$\n\n On suppose que $B\\neq \\mathbb Z_p.$ Soit $x\\in B\\backslash \\mathbb Z_p.$ Alors $x$ s'écrit $x=\\frac{a}{p^n b}$ avec $a\\wedge b=1,$ $a\\wedge p=1$ et $b\\wedge p=1.$ En particulier, on sait que $y=\\frac{p^{n-1}b}{a}\\in\\mathbb Z_p\\subset B$, et puisque $B$ est un sous-anneau, $y\\times x=\\frac{1}{p}\\in B.$ \n\nPrenant les puissances de cet élément, on a $\\frac{1}{p^k}\\in B$ pour tout $k\\geq 0.$ Finalement, considérons $r\\in\\mathbb Q,$ $r\\neq 0.$ On peut écrire $r=\\frac{c}{p^m d},$ avec $p\\wedge d=1.$ Alors \n\n$$r=\\frac{c}{d}\\times\\frac{1}{p^m}\\in B$$\n\npuisque $\\frac cd\\in\\mathbb Z_p\\subset\\mathbb Q$ et $\\frac1{p^m}\\in B.$\n\n\n" }, { "question": " 10 - Anneau des fonctions de $\\mathbb R$ dans $\\mathbb R$ [Signaler une erreur] [Ajouter à ma feuille d'exos]Enoncé L'anneau des fonctions de $\\mathbb R$ dans $\\mathbb R$ est-il intègre?", "answer": " Non, cet anneau n'est pas intègre. Il suffit de trouver deux fonctions non identiquement nulles dont le produit est nulle. Par exemple, si $f=1$ sur $]-\\infty,0]$ et $0$ sur $]0,+\\infty[$, et si $g=0$ sur $]-\\infty,0]$ et $1$ sur $]0,+\\infty[$, alors ni $f$ ni $g$ n'est nulle et pourtant $fg=0.$\n" }, { "question": " 11 - Éléments nilpotents [Signaler une erreur] [Ajouter à ma feuille d'exos]Enoncé Un élément $x$ d'un anneau $A$ est dit nilpotent s'il existe un entier $n\\geq 1$ tel que $x^n=0$.\n\nOn suppose que $A$ est commutatif, et on fixe $x,y$ deux éléments nilpotents.\n\n\n Montrer que $xy$ est nilpotent.\n\n Montrer que $x+y$ est nilpotent.\n\n Montrer que $1_A-x$ est inversible.\n\n Dans cette question, on ne suppose plus que $A$ est commutatif. Soit $u,v\\in A$ tels que $uv$ est nilpotent. Montrer que $vu$ est nilpotent.\n\n", "answer": " Soient $n,m$ tels que $x^n=0$ et $y^m=0$.\n\n\n Puisque $x$ et $y$ commutent, on a $(xy)^n=x^ny^n=0\\times y^n=0$.\n\n Remarquons d'abord que pour $p\\geq n$, on a $x^p=x^{p-n}x^n=0$.\n\nD'après la formule du binôme, $(x+y)^{n+m}=\\sum_{k=0}^{n+m}\\binom{n+m}{k}x^ky^{n+m-k}$.\n\nMais, pour $k\\geq n$, $x^k=0\\implies x^ky^{n+m-k}=0$. D'autre part, pour $k2$, on pourrait trouver $b$ tel que la famille $(1,a,b)$ soit libre. Comme à la question précédente, on trouverait $j\\in \\textrm{vect}(1,b)$ tel que $j^2=-1$. Mais alors, \n\n$$(i-j)(i+j)=0$$\n\net par intégrité de $A$, un des deux facteurs doit être nul. Dans un cas comme dans l'autre, cela implique $j\\in \\textrm{vect}(1,a)$ et donc $b\\in\\textrm{vect}(1,a)$, puisque qu'on peut aussi dire que $b\\in\\textrm{vect}(1,j)$. C'est une contradiction, et donc la dimension de $A$ est deux.\n\n L'isomorphisme est donné par $1_A\\mapsto 1_{\\mathbb C}$ et $i_A\\mapsto i_{\\mathbb C}$, dont on vérifie facilement que c'est un morphisme d'algèbre.\n" } , { "question": " 1 - QCM [Signaler une erreur] [Ajouter à ma feuille d'exos]Enoncé Pour chaque question, une seule réponse est juste. Laquelle?\n\n\n La somme $\\sum_{k=0}^n 2$\n\n$$\\mathbf a.\\textrm{ n'a pas de sens}\\ \\ \\mathbf b. \\textrm{ vaut }2(n+1)\\ \\ \\mathbf c.\\ \\textrm{vaut }2n.$$\n\n La somme $\\sum_{p=0}^{2n+1}(-1)^p$ est égale à \n\n$$\\mathbf a.\\ 1\\ \\ \\mathbf b.\\ -1\\ \\ \\mathbf c.\\ 0.$$\n\n Le produit $\\prod_{i=1}^n (5a_i)$ est égal à \n\n$$\\mathbf a.\\ 5\\prod_{i=1}^n a_i\\ \\ \\mathbf b.\\ 5^n\\prod_{i=1}^n a_i\\ \\ \\mathbf c.\\ 5^{n-1}\\prod_{i=1}^n a_i.$$\n\n", "answer": " \n On somme $(n+1)$ fois le nombre 2. La bonne réponse est b.\n\n On somme $(n+1)$ fois le nombre 1 (pour les $p$ correspondant à $0,2,\\dots 2n$), et $(n+1)$ faut le nombre $-1$ (pour les $p$ correspondant à $1,3,\\dots,2p+1$). La bonne réponse est c. (Si vous n'êtes pas convaincu, essayez le calcul avec $n=2,3,...$).\n\n Dans chaque produit, il y a le terme 5 qui ne dépend pas de $i$ et qu'on peut extraire du produit. Comme il y a $n$ termes dans le produit, la bonne réponse est b.\n\n\n" }, { "question": " 2 - Écrire à l'aide du symbole somme [Signaler une erreur] [Ajouter à ma feuille d'exos]Enoncé Écrire à l'aide du symbole somme les sommes suivantes : \n\n\n $2^3+2^4+\\cdots+2^{12}$.\n\n $\\frac 12+\\frac24+\\frac{3}8+\\cdots+\\frac{10}{1024}$.\n\n $2-4+6-8+\\cdots+50$.\n\n $1-\\frac 12+\\frac13-\\frac 14+\\cdots+\\frac1{2n-1}-\\frac{1}{2n}$.\n\n", "answer": " \n $\\sum_{k=3}^{12}2^k$.\n\n $\\sum_{k=1}^{10}\\frac k{2^k}.$\n\n En remarquant que $2=2\\times 1$, $4=2\\times 2$, jusque $50=2\\times 25$, et que devant les termes du type $2k$, où $k$ est impair, on a un signe $+$ et devant les termes du type $2k$, où $k$ est impair, on a un signe $-$, on peut écrire la somme $\\sum_{k=1}^{25}(-1)^{k+1}\\times 2k=2\\sum_{k=1}^{25} (-1)^{k+1}k.$\n\n $\\sum_{k=1}^{2n} \\frac{(-1)^{k+1}}k$.\n\n\n" }, { "question": " 3 - A l'aide du symbole somme [Signaler une erreur] [Ajouter à ma feuille d'exos]Enoncé Écrire à l'aide du symbole $\\sum$ les sommes suivantes :\n\n\n $n+(n+1)+\\dots+2n$;\n\n $\\frac{x_1}{x_n}+\\frac{x_2}{x_{n-1}}+\\cdots+\\frac{x_{n-1}}{x_2}+\\frac{x_n}{x_1}$.\n\n", "answer": " Pour chaque question, plusieurs réponses sont possibles. Parmi ces réponses :\n\n\n $\\sum_{k=n}^{2n}k$.\n\n $\\sum_{k=1}^n \\frac{x_k}{x_{n+1-k}}$.\n\n\n" }, { "question": " 4 - Différence de deux sommes [Signaler une erreur] [Ajouter à ma feuille d'exos]Enoncé Pour $n\\geq 1$, on pose $u_n=\\sum_{k=n}^{2n}\\frac 1k$. Simplifier $u_{n+1}-u_n$ puis étudier la monotonie de $(u_n)$.", "answer": " Lorsqu'on calcule $u_{n+1}-u_n$, \n\n$$u_{n+1}-u_n=\\sum_{k=n+1}^{2n+2}\\frac 1k-\\sum_{k=n}^{2n}\\frac 1k,$$\n\non s'aperçoit que tous les termes se simplifient, hormis les deux derniers de la première somme, et le premier de la deuxième somme. On trouve donc\n\n$$u_{n+1}-u_n=\\frac{1}{2n+1}+\\frac{1}{2n+2}-\\frac{1}n.$$\n\nOn peut ensuite mettre tout au même dénominateur pour étudier la monotonie de la suite $(u_n)$ : \n\n\\begin{align*}\n\nu_{n+1}-u_n&=\\frac{n(2n+2)+n(2n+1)-(2n+1)(2n+2)}{n(2n+1)(2n+2)}\\\\\n\n&=\\frac{2n^2+2n+2n^2+n-4n^2-6n-2}{n(2n+1)(2n+2)}\\\\\n\n&=\\frac{-3n-2}{n(2n+1)(2n+2)}\\\\\n\n&\\leq 0.\n\n\\end{align*}\n\nLa suite $(u_n)$ est décroissante.\n" }, { "question": " 5 - Changement d'indice [Signaler une erreur] [Ajouter à ma feuille d'exos]Enoncé Soit $n\\geq 1$. Démontrer que \n\n$$\\sum_{k=n+1}^{2n-1}\\ln\\left(\\sin\\left(\\frac{k\\pi}{2n}\\right)\\right)=\\sum_{k=1}^{n-1} \\ln\\left(\\sin\\left(\\frac{k\\pi}{2n}\\right)\\right).$$", "answer": " Il y a deux possibilités naturelles pour effectuer le changement d'indice et obtenir les bonnes bornes. On peut poser $j=k-n$, ou $j=2n-k$. Le deuxième choix permet plus facilement d'arriver au résultat. Posons donc $j=2n-k$. Alors :\n\n\\begin{align*}\n\n\\sum_{k=n+1}^{2n-1}\\ln\\left(\\sin\\left(\\frac{k\\pi}{2n}\\right)\\right)&=\n\n\\sum_{j=1}^{n-1}\\ln\\left(\\sin\\left(\\pi-\\frac{j\\pi}{2n}\\right)\\right)\\\\\n\n&=\\sum_{j=1}^{n-1}\\ln\\left(\\sin\\left(\\frac{j\\pi}{2n}\\right)\\right)\n\n\\end{align*}\n\nen utilisant la formule $\\sin(\\pi-x)=\\sin(x)$. C'est exactement le résultat demandé.\n" }, { "question": " 6 - Changement d'indice magique [Signaler une erreur] [Ajouter à ma feuille d'exos]Enoncé Calculer la somme $\\sum_{k=1}^n \\left(\\frac 1k-\\frac1{n+1-k}\\right)$.", "answer": " On commence par séparer la somme en deux :\n\n$$\\sum_{k=1}^n \\left(\\frac 1k-\\frac1{n+1-k}\\right)=\\sum_{k=1}^n \\frac 1k-\\sum_{k=1}^n \\frac1{n+1-k}.$$\n\nOn effectue ensuite le changement d'indices $j=n+1-k$ dans la deuxième somme. \n\nOn a \n\n$$1\\leq k\\leq n\\iff -1\\geq -k\\geq -n\\iff n\\geq n+1-k\\geq 1.$$\n\nLa deuxième somme devient donc \n\n$$\\sum_{k=1}^n \\frac1{n+1-k}=\\sum_{j=1}^n \\frac1j=\\sum_{k=1}^n \\frac 1k.$$\n\nOn trouve finalement\n\n$$\\sum_{k=1}^n \\left(\\frac 1k-\\frac1{n+1-k}\\right)=0.$$\n" }, { "question": " 7 - Simplifier! [Signaler une erreur] [Ajouter à ma feuille d'exos]Enoncé Simplifier les sommes et produits suivants :\n\n$$\\begin{array}{lcl}\n\n\\mathbf 1.\\ \\sum_{k=1}^n \\ln\\left(1+\\frac 1k\\right)&\\quad\\quad&\\mathbf 2.\\ \\prod_{k=2}^n \\left(1-\\frac1{k^2}\\right)\\\\\n\n\\mathbf 3.\\ \\sum_{k=0}^n \\frac{1}{(k+2)(k+3)}.\n\n\\end{array}$$", "answer": " \n On commence par remarquer que\n\n$$\\ln\\left(1+\\frac 1k\\right)=\\ln\\left(\\frac{k+1}k\\right)=\\ln(k+1)-\\ln k.$$\n\nOn en déduit que \n\n\\begin{eqnarray*}\n\n\\sum_{k=1}^n \\ln\\left(1+\\frac 1k\\right)&=&\\sum_{k=1}^n \\ln(k+1)-\\sum_{k=1}^n \\ln(k)\\\\\n\n&=&\\sum_{k=2}^{n+1}\\ln(k)-\\sum_{k=1}^n \\ln (k)\\\\\n\n&=&\\ln(n+1)-\\ln(1)\\\\\n\n&=&\\ln(n+1).\n\n\\end{eqnarray*}\n\n On commence par remarquer que \n\n$$\\left(1-\\frac1{k^2}\\right)=\\frac{k^2-1}{k^2}=\\frac{(k-1)(k+1)}{k^2}.$$\n\nOn en déduit que\n\n\\begin{eqnarray*}\n\n\\prod_{k=2}^n \\left(1-\\frac1{k^2}\\right)&=&\\frac{\\prod_{k=2}^n (k-1)\\prod_{k=2}^n (k+1)}\n\n{\\left(\\prod_{k=2}^n k\\right)^2}\\\\\n\n&=&\\frac{(n-1)!\\times\\frac12\\times (n+1)!}{(n!)^2}\\\\\n\n&=&\\frac{n+1}{2n}.\n\n\\end{eqnarray*}\n\n On commence par remarquer que \n\n$$\\frac 1{(k+2)(k+3)}=\\frac1{k+2}-\\frac 1{k+3}.$$\n\nOn raisonne alors exactement comme pour la première somme :\n\n\\begin{eqnarray*}\n\n\\sum_{k=0}^n \\frac{1}{(k+2)(k+3)}&=&\\sum_{k=0}^n \\frac1{k+2}-\\sum_{k=0}^n \\frac 1{k+3}\\\\\n\n&=&\\sum_{k=0}^n \\frac1{k+2}-\\sum_{k=1}^{n+1}\\frac{1}{k+2}\\\\\n\n&=&\\frac 12-\\frac 1{n+3}.\n\n\\end{eqnarray*}\n\n\n" }, { "question": " 8 - Télescopage [Signaler une erreur] [Ajouter à ma feuille d'exos]Enoncé Déterminer deux réels $a$ et $b$ tels que, pour tout $k\\in\\mathbb N$,\n\n$$\\frac 1{(k+1)(k+3)}=\\frac a{k+1}+\\frac b{k+3}.$$\n\nEn déduire la valeur de la somme\n\n$$S_n=\\sum_{k=0}^n \\frac{1}{(k+1)(k+3)}.$$", "answer": " On réduit au même dénominateur, et on trouve\n\n$$\\frac a{k+1}+\\frac b{k+3}=\\frac{(a+b)k+(3a+b)}{(k+1)(k+3)}.$$\n\nPar identification, on cherche à résoudre le système\n\n$$\\left\\{\\begin{array}{rcl}\n\na+b&=&0\\\\\n\n3a+b&=&1\n\n\\end{array}\\right.$$\n\ndont la seule solution est $a=1/2$, $b=-1/2$. On en déduit\n\n\\begin{eqnarray*}\n\nS_n&=&\\frac12\\sum_{k=0}^n \\frac 1{k+1}-\\frac 12\\sum_{k=0}^n \\frac 1{k+3}\\\\\n\n&=&\\frac 12\\sum_{k=0}^n \\frac1{k+1}-\\frac12\\sum_{k=2}^{n+2} \\frac{1}{k+1}\\\\\n\n&=&\\frac12\\left(1+\\frac 12-\\frac1{n+2}-\\frac1{n+3}\\right)\\\\\n\n&=&\\frac{3n^2+11n+8}{4(n+2)(n+3)}.\n\n\\end{eqnarray*}\n\n\n" }, { "question": " 9 - Somme télescopique et factorielle [Signaler une erreur] [Ajouter à ma feuille d'exos]Enoncé En utilisant une somme télescopique, calculer $\\sum_{k=1}^n k\\cdot k!$.", "answer": " Pour faire apparaître une somme télescopique, on va écrire $k=(k+1)-1$. Il vient\n\n\\begin{align*}\n\n\\sum_{k=1}^n k\\cdot k!&=\\sum_{k=1}^n (k+1-1)\\cdot k!\\\\\n\n&=\\sum_{k=1}^n \\big( (k+1)!-k!\\big)\\\\\n\n&=(n+1)!-1!=(n+1)!-1.\n\n\\end{align*}\n" }, { "question": " 10 - Transformer en somme télescopique [Signaler une erreur] [Ajouter à ma feuille d'exos]Enoncé \n Déterminer une suite $(u_k)$ telle que, pour tout $k\\geq 0$, on ait\n\n$$u_{k+1}-u_k=(k+2) 2^k.$$\n\n En déduire $\\sum_{k=0}^{n}(k+2)2^k.$\n\n", "answer": " \n On va chercher $u_k$ sous la forme $u_k=(ak+b)2^k$. On a alors\n\n$$u_{k+1}-u_k=(a(k+1)+b)2^{k+1}-(ak+b)2^k=(2ak+2a+2b-ak-b)2^k=(ak+2a+b)2^k.$$\n\nOn obtient donc le résultat si on choisit $a=1$ et $b=0$.\n\n Il s'agit maintenant d'une simple somme télescopique : \n\n\\begin{align*}\n\n\\sum_{k=0}^n (k+2)2^k&=\\sum_{k=0}^n \\big(u_{k+1}-u_k\\big)\\\\\n\n&=u_{n+1}-u_0\\\\\n\n&=(n+1)2^{n+1}.\n\n\\end{align*}\n\n\n" }, { "question": " 11 - Somme et factorielles [Signaler une erreur] [Ajouter à ma feuille d'exos]Enoncé Démontrer que, pour tout $n\\in\\mathbb N^*$, on a \n\n$$(n+1)!\\geq\\sum_{k=1}^n k!\\quad.$$", "answer": " Pour chaque entier $k$ dans $\\{1,\\dots,n\\}$, on a $k!\\leq n!$.\n\nOn obtient donc\n\n$$\\sum_{k=1}^n k!\\leq\\sum_{k=1}^n n!=n\\times n!$$\n\npuisque la somme de droite est une somme de termes constants. Puisque $n\\leq n+1$, on obtient bien\n\n$$\\sum_{k=1}^n k!\\leq (n+1)\\times n!=(n+1)!\\quad .$$\n" }, { "question": " 12 - Somme et somme des carrés [Signaler une erreur] [Ajouter à ma feuille d'exos]Enoncé Soit $n\\geq 1$ et $x_1,\\dots,x_n$ des réels vérifiant \n\n$$\\sum_{k=1}^n x_k=n\\textrm{ et }\\sum_{k=1}^n x_k^2=n.$$\n\nDémontrer que, pour tout $k$ dans $\\{1,\\dots,n\\}$, $x_k=1$.", "answer": " Calculons $\\sum_{k=1}^n (1-x_k)^2$ :\n\n$$\\sum_{k=1}^n (1-x_k)^2=\\sum_{k=1}^n 1-2\\sum_{k=1}^n x_k+\\sum_{k=1}^n x_k^2=n-2n+n=0.$$\n\nOr, une somme de réels positifs est nulle si et seulement si chacun des termes de la somme est nul. On en déduit le résultat demandé.\n" }, { "question": " 13 - Somme des entiers, des carrés,... [Signaler une erreur] [Ajouter à ma feuille d'exos]Enoncé Pour $n\\in\\mathbb N^*$, on note\n\n$$a_n=\\sum_{k=1}^n k,\\ b_n=\\sum_{k=1}^n k^2\\textrm{ et }c_n=\\sum_{k=1}^n k^3.$$\n\nDémontrer que $\\displaystyle a_n=\\frac{n(n+1)}2$, que $\\displaystyle b_n=\\frac{n(n+1)(2n+1)}6$ et que $c_n=a_n^2$.", "answer": " On procède simplement par récurrence sur $n$. Voyons d'abord pourquoi la formule est vraie pour $a_n$. Elle\n\nest vraie pour $n=1$ (car $1=\\frac{1\\times 2}{2}$). Soit $n\\in\\mathbb N$ telle qu'elle est vraie au rang $n$ et prouvons-la\n\nau rang $n+1$. Alors, \n\n$$a_{n+1}=a_n+(n+1)=\\frac{n(n+1)}2+(n+1)=(n+1)\\times\\left(\\frac n2+1\\right)=\\frac{(n+1)\\big((n+1)+1\\big)}2.$$\n\nLa formule est donc vraie au rang $n+1$ et par le principe de récurrence, elle est vraie pour tout entier $n\\geq 0$. \n\nLe raisonnement est rigoureusement identique pour $b_n$ et pour $c_n$. Voyons par exemple le cas de $c_n$.\n\nComme précédemment, la formule est vraie au rang $1$, et si elle est vraie au rang $n$, alors\n\n$$c_{n+1}=c_n+(n+1)^3=\\frac{n^2(n+1)^2}4+(n+1)^3=(n+1)^2\\left(\\frac {n^2}4+(n+1)\\right).$$\n\nIl suffit alors pour conclure de remarquer que \n\n$$\\frac {n^2}4+(n+1)=\\frac{(n+2)^2}4.$$\n\nLa propriété est vraie au rang $n+1$ ce qui clôt la démonstration par récurrence.\n" }, { "question": " 14 - Calculs de sommes arithmétiques [Signaler une erreur] [Ajouter à ma feuille d'exos]Enoncé Calculer les somme suivantes :\n\n\n $A_n=\\sum_{k=1}^n 3$.\n\n $B_n=\\sum_{k=1}^n A_k$.\n\n $S_n=\\sum_{k=0}^{n}(2k+1)$.\n\n\n", "answer": " \n On somme des termes constants, et il y a $n$ termes dans la somme, donc \n\n$A_n=3n$.\n\n Par linéarité, \n\n\\[ B_n = 3\\sum_{k=1}^n k=\\frac{3n(n+1)}2. \\]\n\n On raisonne également par linéarité : \n\n\\begin{align*}\n\nS_n&=2\\sum_{k=0}^n k+\\sum_{k=0}^n 1\\\\\n\n&=n(n+1)+(n+1)\\\\\n\n&=(n+1)^2.\n\n\\end{align*}\n\n\n" }, { "question": " 15 - Calculs de sommes géométriques [Signaler une erreur] [Ajouter à ma feuille d'exos]Enoncé Calculer les sommes suivantes : \n\n\n $S=\\frac{1}{2^{10}}+\\frac{1}{2^{20}}+\\frac{1}{2^{30}}+\\cdots+\\frac{1}{2^{1000}}$.\n\n $T_n=\\sum_{k=0}^n \\frac{2^{k-1}}{3^{k+1}}$.\n\n", "answer": " \n On remarque que \n\n$$S=\\sum_{k=1}^{100}\\frac{1}{2^{10k}}=\\sum_{k=1}^{100}\\left(\\frac1{2^{10}}\\right)^k.$$\n\nPar la formule donnant la somme d'une suite géométrique, on trouve\n\n\\begin{align*}\n\nS&=\\frac{ \\frac{1}{2^{10}}-\\frac 1{2^{1010}}}{1-\\frac1{2^{10}}}\\\\\n\n&=\\frac{1-\\frac 1{2^{1000}}}{2^{10}-1}\\\\\n\n&=\\frac{2^{1000}-1}{2^{1000}(2^{10}-1)}.\n\n\\end{align*}\n\n On factorise par $1/6$ pour se ramener à une somme géométrique.\n\n$$\\sum_{k=0}^n\\frac{2^{k-1}}{3^{k+1}}=\\frac 16\\sum_{k=0}^n \\left(\\frac 23\\right)^k=\\frac 16\\times\\frac{1-\\left(\\frac 23\\right)^{n+1}}{1-\\frac 23}=\n\n\\frac{1-\\left(\\frac 23\\right)^{n+1}}2.$$\n\n\n" }, { "question": " 16 - Calcul de sommes par changements d'indice [Signaler une erreur] [Ajouter à ma feuille d'exos]Enoncé Calculer la somme suivante : \n\n$$\\sum_{k=1}^n (n-k+1).$$", "answer": " Effectuons le changement d'indices $\\ell=n-k+1$. Lorsque $k$ varie de $1$ à $n$, alors $\\ell$ varie aussi de $1$ à $n$. On a donc\n\n$$\\sum_{k=1}^n (n+1-k)=\\sum_{\\ell=1}^n \\ell=\\frac{n(n+1)}2.$$\n" }, { "question": " 17 - Calcul de sommes avec indices négatifs [Signaler une erreur] [Ajouter à ma feuille d'exos]Enoncé Calculer la somme suivante : \n\n$$\\sum_{k=-5}^{15} k(10-k).$$", "answer": " On écrit en développant le produit\n\n$$\\sum_{k=-5}^{15}k(10-k)=10\\sum_{k=-5}^{15}k-\\sum_{k=-5}^{15}k^2.$$\n\nÉtudions ensuite la première somme en la découpant en deux :\n\n\\begin{align*}\n\nS_1=\\sum_{k=-5}^{15}k&=\\sum_{k=-5}^0 k +\\sum_{k=1}^{15}k\n\n\\end{align*}\n\nDans la première de ces deux sommes, on fait le changement de variables $\\ell=-k$. Lorsque $k$ va de $-5$ à $0$, $\\ell$ va de $0$ à $5$. On trouve donc que \n\n\\begin{align*}\n\nS_1&=-\\sum_{\\ell=0}^5 \\ell+\\sum_{k=1}^{15}k\\\\\n\n&=-\\frac{5\\times 6}2+\\frac{15\\times 16}2\\\\\n\n&=105.\n\n\\end{align*}\n\nPour la deuxième somme, on fait le même raisonnement :\n\n\\begin{align*}\n\nS_2=\\sum_{k=-5}^{15}k^2&=\\sum_{k=-5}^0 k^2 +\\sum_{k=1}^{15}k^2\\\\\n\n&=\\sum_{\\ell=0}^5(-\\ell)^2+\\sum_{k=1}^{15}k^2\\\\\n\n&=\\sum_{\\ell=0}^5 \\ell^2+\\sum_{k=1}^{15}k^2\\\\\n\n&=\\frac{5\\times 6\\times 11}{6}+\\frac{15\\times 16\\times 31}6=1295.\n\n\\end{align*}\n\nFinalement, on trouve\n\n$$\\sum_{k=-5}^{15} k(10-k)=1050-1295=-245.$$\n\nOn peut vérifier cette somme par un petit programme Python :\n\n     S=0\n    for k in range(-5,16):\n         S+=k*(10-k)\n    print(S)\n\n" }, { "question": " 18 - Calcul de sommes par découpage [Signaler une erreur] [Ajouter à ma feuille d'exos]Enoncé Soit $n\\in\\mathbb N$.\n\n\n Calculer $A_n=\\sum_{k=2n+1}^{3n}(2n)$.\n\n Calculer $B_n=\\sum_{k=n}^{2n}k$.\n\n En déduire la valeur de $S_n=\\sum_{k=n}^{3n}\\min(k,2n)$.\n\n", "answer": " \n On ne somme que des termes constants, et donc on trouve \n\n$$A_n=n\\times (2n)=2n^2.$$\n\n Notons $T_n=\\sum_{k=0}^n k$. Alors on a \n\n$$B_n=T_{2n}-T_{n-1}=n(2n+1)-\\frac{(n-1)n}2=\\frac{n(4n+2-(n-1))}2=\\frac{3(n+1)n}2.$$\n\n On remarque que si $k\\leq 2n$, alors $\\min(k,2n)=k$ alors que si $k\\geq 2n+1$, on a $\\min(k,2n)=2n$. On en déduit que\n\n$$S_n=A_n+B_n=2n^2+\\frac{3n(n+1)}2=\\frac{n(7n+3)}2.$$\n\n\n" }, { "question": " 19 - Se ramener à une somme classique [Signaler une erreur] [Ajouter à ma feuille d'exos]Enoncé Pour $n\\geq 1$, on pose $u_n=\\frac{1}{n^2}+\\frac{2}{n^2}+\\cdots+\\frac{n}{n^2}$. Calculer explicitement $u_n$, puis en déduire la limite de la suite $(u_n)$.", "answer": " En mettant $1/n^2$ en facteur, on se ramène à une somme classique. En effet, on peut écrire\n\n$$u_n=\\frac1{n^2}\\sum_{k=1}^n k=\\frac{1}{n^2}\\times\\frac{n(n+1)}{2}=\\frac{n+1}{2n}=\\frac{1}2+\\frac{1}{2n}.$$\n\nAinsi, la suite $(u_n)$ tend vers $1/2$.\n" }, { "question": " 20 - Un produit [Signaler une erreur] [Ajouter à ma feuille d'exos]Enoncé Pour $n\\in\\mathbb N^*$ et $x\\in\\mathbb R$, on note\n\n$$P_n(x)=\\prod_{k=1}^n \\left(1+\\frac xk\\right).$$\n\n\n Que valent $P_n(0)$, $P_n(1)$, $P_n(-n)$?\n\n Démontrer que pour tout réel non-nul $x$, on a \n\n$$P_n(x)=\\frac {x+n}xP_n(x-1).$$\n\n Pour $p\\in\\mathbb N^*$, écrire $P_n(p)$ comme coefficient du binôme.\n\n", "answer": " \n On a $P_n(0)=1$ (on ne fait que des produits de 1), $P_n(-n)=0$, car alors\n\n$$1+\\frac{-n}n=0$$\n\net donc on a un terme nul dans le produit. Enfin,\n\n$$P_n(1)=\\prod_{k=1}^n \\frac{k+1}k=\\frac{2\\times 3\\times\\dots\\times (n+1)}{1\\times 2\\times\\dots\\times n}={n+1}.$$\n\n On a \n\n\\begin{eqnarray*}\n\nP_n(x)&=&\\prod_{k=1}^n \\frac{x+k}k\\\\\n\n&=&\\frac{(x+1)(x+2)\\dots (x+n)}{1\\times 2\\times\\dots\\times n}\\\\\n\n&=&\\frac{x+n}{x}\\times\\frac {(x-1+1)(x-1+2)\\dots (x-1+n)}{1\\times 2\\times\\dots\\times n}\\\\\n\n&=&\\frac{x+n}xP_n(x-1).\n\n\\end{eqnarray*}\n\n On a \n\n$$P_n(p)=\\prod_{k=1}^n \\frac{k+p}{k}=\\frac{(p+1)\\dots (p+n)}{n!}=\\frac{(n+p)!}{n!p!}=\\binom{n+p}{p}.$$\n\n\n" }, { "question": " 21 - Somme géométrique dans tous ses états [Signaler une erreur] [Ajouter à ma feuille d'exos]Enoncé Soit pour $n\\in\\mathbb N$, $u_n=(-2)^n$. Calculer les sommes suivantes : \n\n$$\\sum_{k=0}^{2n} u_{k};\\quad \\sum_{k=0}^{2n+1} u_{k};\\quad \\sum_{k=0}^{n} u_{2k};\\quad \\sum_{k=0}^{2n} (u_{k}+n);\\quad \\left(\\sum_{k=0}^{2n} u_{k}\\right)+n;\\quad \\sum_{k=0}^{n} u_{k+n};\\quad \\sum_{k=0}^{n} u_{kn}.$$", "answer": " Dans cet exercice, il faut faire très attention aux notations, puis appliquer la formule de la somme d'une série géométrique. Il vient alors\n\n$$\\sum_{k=0}^{2n} u_{k}=\\sum_{k=0}^{2n}(-2)^k=\\frac{1-(-2)^{2n+1}}{1-(-2)}=\\frac{1+2^{2n+1}}3.$$\n\n$$\\sum_{k=0}^{2n+1} u_{k}=\\frac{1-2^{2n+2}}3.$$\n\n$$\\sum_{k=0}^{n} u_{2k}=\\sum_{k=0}^n 4^k=\\frac{1-4^{n+1}}{1-4}=\\frac{4^{n+1}-1}3.$$\n\n$$\\sum_{k=0}^{2n} (u_{k}+n)=\\sum_{k=0}^{2n} u_k+\\sum_{k=0}^{2n} n=\\frac{1+2^{2n+1}}3+n(2n+1).$$\n\n$$\\left(\\sum_{k=0}^{2n} u_{k}\\right)+n=\\frac{1+2^{2n+1}}3+n.$$\n\n$$\\sum_{k=0}^{n} u_{k+n}=\\sum_{k=0}^n (-2)^{k+n}=(-2)^n\\sum_{k=0}^n (-2)^k=\\frac{(-2)^n(1-(-1)^{n+1} 2^{n+1})}{3}.$$\n\n$$\\sum_{k=0}^{n} u_{kn}=\\sum_{k=0}^n ((-2)^n)^k=\\frac{1-(-2)^{n(n+1)}}{1-(-2)^n},$$\n\nsauf si $n=0$ auquel cas la somme vaut $u_0=1$.\n" }, { "question": " 22 - Sommes alternées [Signaler une erreur] [Ajouter à ma feuille d'exos]Enoncé \n Simplifier la somme $\\sum_{k=1}^{2n}(-1)^k k$ en faisant des sommations par paquets. \n\n Montrer par récurrence que pour tout $n\\in\\mtn^*$, on a \n\n$$S_n=\\sum_{k=1}^n (-1)^k k=\\frac{(-1)^n (2n+1)-1}{4}.$$\n\nRetrouver le résultat précédent.\n\n", "answer": " \n L’ensemble $\\{1, . . . , 2n\\}$ est la réunion des parties deux à deux disjointes $\\{2p − 1, 2p\\}$ pour $p$ variant de $1$ à $n$. Or $(−1)^{2p−1}(2p − 1) + (−1)^{2p}2p = 2p − (2p − 1) = 1$ pour tout $1\\leq p\\leq n$, donc la somme est égale à $n$.\n\n Initialisation : On commence par vérifier la propriété pour $n=1$. On a \n\n$$\\sum_{k=1}^1 (-1)^k k=-1\\textrm{ et }\\frac{(-1)^1 (2\\times 1+1)-1}{4}=-1$$\n\nce qui prouve bien l'égalité voulue.\n\nHérédité : Supposons la propriété vraie au rang $n$ et prouvons-là au rang $n+1$.\n\nOn a \n\n\\begin{eqnarray*}\n\n\\sum_{k=1}^{n+1}(-1)^k k&=&\\sum_{k=1}^n (-1)^k k+(-1)^{n+1}(n+1)\\\\\n\n&=&\\frac{(-1)^n (2n+1)-1}{4}+(-1)^{n+1}(n+1)\\\\\n\n&=&\\frac{(-1)^{n+1}(-2n-1+4n+4)-1}{4}\\\\\n\n&=&\\frac{(-1)^{n+1}(2(n+1)+1)-1}{4}\n\n\\end{eqnarray*}\n\nce qui démontre bien la propriété au rang $n+1$. Remarquons que le passage\n\nde la première à la deuxième ligne utilise l'hypothèse de récurrence.\n\nSi on somme jusque $2n$ au lieu de $n$, on trouve que \n\n$$S_{2n}=\\frac{4n+1-1}4=n.$$\n\n\n" }, { "question": " 23 - Somme de puissances [Signaler une erreur] [Ajouter à ma feuille d'exos]Enoncé Soit $x\\in\\mathbb R$ et $n\\in\\mathbb N^*$.\n\n\n Calculer $S_n(x)=\\sum_{k=0}^n x^k.$\n\n En déduire la valeur de $T_n(x)=\\sum_{k=0}^n k x^k.$\n\n\n", "answer": " C'est un exercice extrêmement classique qu'il faut savoir faire. \n\n\n On reconnait une somme géométrique de raison $x$. Pour $x\\neq 1$, on a \n\n$$S_n(x)=\\frac{1-x^{n+1}}{1-x}.$$\n\nPour $x=1$, $S_n(1)=n+1$.\n\n On distingue là encore le cas $x=1$. Il donne\n\n$$T_n(1)=\\sum_{k=0}^n k=\\frac{n(n+1)}2.$$\n\nSinon, on dérive $S_n$ : pour tout $x\\neq 1$, \n\n$$S_n'(x)=\\sum_{k=1}^n kx^{k-1}\\implies T_n(x)=xS_n'(x).$$\n\nOn calcule alors $S_n'(x)$ avec la formule obtenue à la question précédente et on trouve\n\n$$T_n(x)=x\\frac{nx^{n+1} −(n+1)x^n +1}{ (x−1)^2}.$$\n\n\n\n" }, { "question": " 24 - Sommation d'Abel [Signaler une erreur] [Ajouter à ma feuille d'exos]Enoncé Soient $(a_n)_{n\\in\\mathbb N}$ et $(B_n)_{n\\in\\mathbb N}$ deux suites de nombres complexes. On définit deux suites $(A_n)_{n\\in\\mathbb N}$ et $(b_n)_{n\\in\\mathbb N}$ en posant :\n\n$$A_n=\\sum_{k=0}^n a_k,\\quad\\quad b_n=B_{n+1}-B_n.$$\n\n\n Démontrer que $\\sum_{k=0}^n a_kB_k=A_n B_n-\\sum_{k=0}^{n-1}A_kb_k.$\n\n En déduire la valeur de $\\sum_{k=0}^n 2^kk$.\n\n", "answer": " \n On convient que $A_{-1}=0$. On a alors\n\n\\begin{eqnarray*}\n\n\\sum_{k=0}^n a_k B_k&=&\\sum_{k=0}^n (A_k-A_{k-1})B_k\\\\\n\n&=&\\sum_{k=0}^n A_k B_k-\\sum_{k=0}^n A_{k-1}B_k\\\\\n\n&=&\\sum_{k=0}^n A_k B_k-\\sum_{k=0}^{n-1} A_k B_{k+1}\\\\\n\n&=&A_n B_n+\\sum_{k=0}^{n-1}A_k (B_k-B_{k+1})\\\\\n\n&=&A_nB_n-\\sum_{k=0}^{n-1} A_k b_k.\n\n\\end{eqnarray*}\n\n On pose $a_k=2^k$ et $B_k=k$. Alors $A_n=2^{n+1}-1$ et $b_k=1$. On en déduit que\n\n\\begin{eqnarray*}\n\n\\sum_{k=0}^n k2^k&=&(2^{n+1}-1)n-\\sum_{k=0}^{n-1}(2^{k+1}-1)\\\\\n\n&=&(2^{n+1}-1)n-2(2^n-1)+n\\\\\n\n&=&2^{n+1}(n-1)+2.\n\n\\end{eqnarray*}\n\n\n\nCe procédé de transformation de somme s'appelle une sommation d'Abel.\n" }, { "question": " 25 - Comment permuter une somme double? [Signaler une erreur] [Ajouter à ma feuille d'exos]Enoncé Soit $(a_{i,j})_{(i,j)\\in\\mathbb N^2}$ une suite double de nombres réels. Soit $n$ et $m$ deux entiers naturels. Intervertir les sommes doubles suivantes :\n\n\n $S_1=\\sum_{i=0}^n \\sum_{j=i}^n a_{i,j}$;\n\n $S_2=\\sum_{i=0}^n \\sum_{j=0}^{n-i}a_{i,j}$;\n\n $S_3=\\sum_{i=0}^n \\sum_{j=i}^m a_{i,j}$ où on a supposé $n\\leq m$.\n\n", "answer": " \n On fait la somme sur tous les indices $i,j$ tels que $0\\leq i\\leq j\\leq n$. On a donc\n\n$$S_1=\\sum_{j=0}^n \\sum_{i=0}^j a_{i,j}.$$\n\nUne autre façon de le voir est de poser $b_{i,j}=a_{i,j}$ si $j\\geq i$ et $b_{i,j}=0$ si $j0$ et $y_{n+1}-y_n\\geq 2y_n>0$, donc les deux suites sont strictement croissantes.\n\n Puisque les suites sont strictement croissantes, les couples $(x_n,y_n)$ sont tous différents. Il suffit donc de démontrer que, pour tout entier $n$, on a $x_n^2-2y_n^2=1$. Pour cela, il suffit de remarquer que\n\n$$(3-2\\sqrt 2)^n=x_n-\\sqrt 2 y_n.$$\n\nEn effet, si on développe $(3-2\\sqrt 2)^n$ par la formule du binôme, on trouve\n\n$$(3-2\\sqrt 2)^n =\\sum_{k=0}^n \\binom{n}k 3^k (-1)^{n-k}2^{n-k}(\\sqrt 2)^{n-k}.$$\n\nOn regroupe les termes comme précédemment, sachant que pour les termes entiers, on a $n-k$ pair et donc $(-1)^{n-k}=1$ et pour les termes de la forme $m\\sqrt 2$, on a $n-k$ impair\n\net $(-1)^{n-k}=-1$. Ainsi, on a \n\n$$x_n^2-2y_n^2=(x_n-\\sqrt 2 y_n)(x_n+\\sqrt 2 y_n)=(3-2\\sqrt 2)^n(3+2\\sqrt 2)^n =1^n=1.$$\n" } , { "question": " 1 - QCM [Signaler une erreur] [Ajouter à ma feuille d'exos]Enoncé Pour chaque question, une seule réponse est juste. Laquelle?\n\n\n La somme $\\sum_{k=0}^n 2$\n\n$$\\mathbf a.\\textrm{ n'a pas de sens}\\ \\ \\mathbf b. \\textrm{ vaut }2(n+1)\\ \\ \\mathbf c.\\ \\textrm{vaut }2n.$$\n\n La somme $\\sum_{p=0}^{2n+1}(-1)^p$ est égale à \n\n$$\\mathbf a.\\ 1\\ \\ \\mathbf b.\\ -1\\ \\ \\mathbf c.\\ 0.$$\n\n Le produit $\\prod_{i=1}^n (5a_i)$ est égal à \n\n$$\\mathbf a.\\ 5\\prod_{i=1}^n a_i\\ \\ \\mathbf b.\\ 5^n\\prod_{i=1}^n a_i\\ \\ \\mathbf c.\\ 5^{n-1}\\prod_{i=1}^n a_i.$$\n\n", "answer": " \n On somme $(n+1)$ fois le nombre 2. La bonne réponse est b.\n\n On somme $(n+1)$ fois le nombre 1 (pour les $p$ correspondant à $0,2,\\dots 2n$), et $(n+1)$ faut le nombre $-1$ (pour les $p$ correspondant à $1,3,\\dots,2p+1$). La bonne réponse est c. (Si vous n'êtes pas convaincu, essayez le calcul avec $n=2,3,...$).\n\n Dans chaque produit, il y a le terme 5 qui ne dépend pas de $i$ et qu'on peut extraire du produit. Comme il y a $n$ termes dans le produit, la bonne réponse est b.\n\n\n" }, { "question": " 2 - Écrire à l'aide du symbole somme [Signaler une erreur] [Ajouter à ma feuille d'exos]Enoncé Écrire à l'aide du symbole somme les sommes suivantes : \n\n\n $2^3+2^4+\\cdots+2^{12}$.\n\n $\\frac 12+\\frac24+\\frac{3}8+\\cdots+\\frac{10}{1024}$.\n\n $2-4+6-8+\\cdots+50$.\n\n $1-\\frac 12+\\frac13-\\frac 14+\\cdots+\\frac1{2n-1}-\\frac{1}{2n}$.\n\n", "answer": " \n $\\sum_{k=3}^{12}2^k$.\n\n $\\sum_{k=1}^{10}\\frac k{2^k}.$\n\n En remarquant que $2=2\\times 1$, $4=2\\times 2$, jusque $50=2\\times 25$, et que devant les termes du type $2k$, où $k$ est impair, on a un signe $+$ et devant les termes du type $2k$, où $k$ est impair, on a un signe $-$, on peut écrire la somme $\\sum_{k=1}^{25}(-1)^{k+1}\\times 2k=2\\sum_{k=1}^{25} (-1)^{k+1}k.$\n\n $\\sum_{k=1}^{2n} \\frac{(-1)^{k+1}}k$.\n\n\n" }, { "question": " 3 - A l'aide du symbole somme [Signaler une erreur] [Ajouter à ma feuille d'exos]Enoncé Écrire à l'aide du symbole $\\sum$ les sommes suivantes :\n\n\n $n+(n+1)+\\dots+2n$;\n\n $\\frac{x_1}{x_n}+\\frac{x_2}{x_{n-1}}+\\cdots+\\frac{x_{n-1}}{x_2}+\\frac{x_n}{x_1}$.\n\n", "answer": " Pour chaque question, plusieurs réponses sont possibles. Parmi ces réponses :\n\n\n $\\sum_{k=n}^{2n}k$.\n\n $\\sum_{k=1}^n \\frac{x_k}{x_{n+1-k}}$.\n\n\n" }, { "question": " 4 - Différence de deux sommes [Signaler une erreur] [Ajouter à ma feuille d'exos]Enoncé Pour $n\\geq 1$, on pose $u_n=\\sum_{k=n}^{2n}\\frac 1k$. Simplifier $u_{n+1}-u_n$ puis étudier la monotonie de $(u_n)$.", "answer": " Lorsqu'on calcule $u_{n+1}-u_n$, \n\n$$u_{n+1}-u_n=\\sum_{k=n+1}^{2n+2}\\frac 1k-\\sum_{k=n}^{2n}\\frac 1k,$$\n\non s'aperçoit que tous les termes se simplifient, hormis les deux derniers de la première somme, et le premier de la deuxième somme. On trouve donc\n\n$$u_{n+1}-u_n=\\frac{1}{2n+1}+\\frac{1}{2n+2}-\\frac{1}n.$$\n\nOn peut ensuite mettre tout au même dénominateur pour étudier la monotonie de la suite $(u_n)$ : \n\n\\begin{align*}\n\nu_{n+1}-u_n&=\\frac{n(2n+2)+n(2n+1)-(2n+1)(2n+2)}{n(2n+1)(2n+2)}\\\\\n\n&=\\frac{2n^2+2n+2n^2+n-4n^2-6n-2}{n(2n+1)(2n+2)}\\\\\n\n&=\\frac{-3n-2}{n(2n+1)(2n+2)}\\\\\n\n&\\leq 0.\n\n\\end{align*}\n\nLa suite $(u_n)$ est décroissante.\n" }, { "question": " 5 - Changement d'indice [Signaler une erreur] [Ajouter à ma feuille d'exos]Enoncé Soit $n\\geq 1$. Démontrer que \n\n$$\\sum_{k=n+1}^{2n-1}\\ln\\left(\\sin\\left(\\frac{k\\pi}{2n}\\right)\\right)=\\sum_{k=1}^{n-1} \\ln\\left(\\sin\\left(\\frac{k\\pi}{2n}\\right)\\right).$$", "answer": " Il y a deux possibilités naturelles pour effectuer le changement d'indice et obtenir les bonnes bornes. On peut poser $j=k-n$, ou $j=2n-k$. Le deuxième choix permet plus facilement d'arriver au résultat. Posons donc $j=2n-k$. Alors :\n\n\\begin{align*}\n\n\\sum_{k=n+1}^{2n-1}\\ln\\left(\\sin\\left(\\frac{k\\pi}{2n}\\right)\\right)&=\n\n\\sum_{j=1}^{n-1}\\ln\\left(\\sin\\left(\\pi-\\frac{j\\pi}{2n}\\right)\\right)\\\\\n\n&=\\sum_{j=1}^{n-1}\\ln\\left(\\sin\\left(\\frac{j\\pi}{2n}\\right)\\right)\n\n\\end{align*}\n\nen utilisant la formule $\\sin(\\pi-x)=\\sin(x)$. C'est exactement le résultat demandé.\n" }, { "question": " 6 - Changement d'indice magique [Signaler une erreur] [Ajouter à ma feuille d'exos]Enoncé Calculer la somme $\\sum_{k=1}^n \\left(\\frac 1k-\\frac1{n+1-k}\\right)$.", "answer": " On commence par séparer la somme en deux :\n\n$$\\sum_{k=1}^n \\left(\\frac 1k-\\frac1{n+1-k}\\right)=\\sum_{k=1}^n \\frac 1k-\\sum_{k=1}^n \\frac1{n+1-k}.$$\n\nOn effectue ensuite le changement d'indices $j=n+1-k$ dans la deuxième somme. \n\nOn a \n\n$$1\\leq k\\leq n\\iff -1\\geq -k\\geq -n\\iff n\\geq n+1-k\\geq 1.$$\n\nLa deuxième somme devient donc \n\n$$\\sum_{k=1}^n \\frac1{n+1-k}=\\sum_{j=1}^n \\frac1j=\\sum_{k=1}^n \\frac 1k.$$\n\nOn trouve finalement\n\n$$\\sum_{k=1}^n \\left(\\frac 1k-\\frac1{n+1-k}\\right)=0.$$\n" }, { "question": " 7 - Simplifier! [Signaler une erreur] [Ajouter à ma feuille d'exos]Enoncé Simplifier les sommes et produits suivants :\n\n$$\\begin{array}{lcl}\n\n\\mathbf 1.\\ \\sum_{k=1}^n \\ln\\left(1+\\frac 1k\\right)&\\quad\\quad&\\mathbf 2.\\ \\prod_{k=2}^n \\left(1-\\frac1{k^2}\\right)\\\\\n\n\\mathbf 3.\\ \\sum_{k=0}^n \\frac{1}{(k+2)(k+3)}.\n\n\\end{array}$$", "answer": " \n On commence par remarquer que\n\n$$\\ln\\left(1+\\frac 1k\\right)=\\ln\\left(\\frac{k+1}k\\right)=\\ln(k+1)-\\ln k.$$\n\nOn en déduit que \n\n\\begin{eqnarray*}\n\n\\sum_{k=1}^n \\ln\\left(1+\\frac 1k\\right)&=&\\sum_{k=1}^n \\ln(k+1)-\\sum_{k=1}^n \\ln(k)\\\\\n\n&=&\\sum_{k=2}^{n+1}\\ln(k)-\\sum_{k=1}^n \\ln (k)\\\\\n\n&=&\\ln(n+1)-\\ln(1)\\\\\n\n&=&\\ln(n+1).\n\n\\end{eqnarray*}\n\n On commence par remarquer que \n\n$$\\left(1-\\frac1{k^2}\\right)=\\frac{k^2-1}{k^2}=\\frac{(k-1)(k+1)}{k^2}.$$\n\nOn en déduit que\n\n\\begin{eqnarray*}\n\n\\prod_{k=2}^n \\left(1-\\frac1{k^2}\\right)&=&\\frac{\\prod_{k=2}^n (k-1)\\prod_{k=2}^n (k+1)}\n\n{\\left(\\prod_{k=2}^n k\\right)^2}\\\\\n\n&=&\\frac{(n-1)!\\times\\frac12\\times (n+1)!}{(n!)^2}\\\\\n\n&=&\\frac{n+1}{2n}.\n\n\\end{eqnarray*}\n\n On commence par remarquer que \n\n$$\\frac 1{(k+2)(k+3)}=\\frac1{k+2}-\\frac 1{k+3}.$$\n\nOn raisonne alors exactement comme pour la première somme :\n\n\\begin{eqnarray*}\n\n\\sum_{k=0}^n \\frac{1}{(k+2)(k+3)}&=&\\sum_{k=0}^n \\frac1{k+2}-\\sum_{k=0}^n \\frac 1{k+3}\\\\\n\n&=&\\sum_{k=0}^n \\frac1{k+2}-\\sum_{k=1}^{n+1}\\frac{1}{k+2}\\\\\n\n&=&\\frac 12-\\frac 1{n+3}.\n\n\\end{eqnarray*}\n\n\n" }, { "question": " 8 - Télescopage [Signaler une erreur] [Ajouter à ma feuille d'exos]Enoncé Déterminer deux réels $a$ et $b$ tels que, pour tout $k\\in\\mathbb N$,\n\n$$\\frac 1{(k+1)(k+3)}=\\frac a{k+1}+\\frac b{k+3}.$$\n\nEn déduire la valeur de la somme\n\n$$S_n=\\sum_{k=0}^n \\frac{1}{(k+1)(k+3)}.$$", "answer": " On réduit au même dénominateur, et on trouve\n\n$$\\frac a{k+1}+\\frac b{k+3}=\\frac{(a+b)k+(3a+b)}{(k+1)(k+3)}.$$\n\nPar identification, on cherche à résoudre le système\n\n$$\\left\\{\\begin{array}{rcl}\n\na+b&=&0\\\\\n\n3a+b&=&1\n\n\\end{array}\\right.$$\n\ndont la seule solution est $a=1/2$, $b=-1/2$. On en déduit\n\n\\begin{eqnarray*}\n\nS_n&=&\\frac12\\sum_{k=0}^n \\frac 1{k+1}-\\frac 12\\sum_{k=0}^n \\frac 1{k+3}\\\\\n\n&=&\\frac 12\\sum_{k=0}^n \\frac1{k+1}-\\frac12\\sum_{k=2}^{n+2} \\frac{1}{k+1}\\\\\n\n&=&\\frac12\\left(1+\\frac 12-\\frac1{n+2}-\\frac1{n+3}\\right)\\\\\n\n&=&\\frac{3n^2+11n+8}{4(n+2)(n+3)}.\n\n\\end{eqnarray*}\n\n\n" }, { "question": " 9 - Somme télescopique et factorielle [Signaler une erreur] [Ajouter à ma feuille d'exos]Enoncé En utilisant une somme télescopique, calculer $\\sum_{k=1}^n k\\cdot k!$.", "answer": " Pour faire apparaître une somme télescopique, on va écrire $k=(k+1)-1$. Il vient\n\n\\begin{align*}\n\n\\sum_{k=1}^n k\\cdot k!&=\\sum_{k=1}^n (k+1-1)\\cdot k!\\\\\n\n&=\\sum_{k=1}^n \\big( (k+1)!-k!\\big)\\\\\n\n&=(n+1)!-1!=(n+1)!-1.\n\n\\end{align*}\n" }, { "question": " 10 - Transformer en somme télescopique [Signaler une erreur] [Ajouter à ma feuille d'exos]Enoncé \n Déterminer une suite $(u_k)$ telle que, pour tout $k\\geq 0$, on ait\n\n$$u_{k+1}-u_k=(k+2) 2^k.$$\n\n En déduire $\\sum_{k=0}^{n}(k+2)2^k.$\n\n", "answer": " \n On va chercher $u_k$ sous la forme $u_k=(ak+b)2^k$. On a alors\n\n$$u_{k+1}-u_k=(a(k+1)+b)2^{k+1}-(ak+b)2^k=(2ak+2a+2b-ak-b)2^k=(ak+2a+b)2^k.$$\n\nOn obtient donc le résultat si on choisit $a=1$ et $b=0$.\n\n Il s'agit maintenant d'une simple somme télescopique : \n\n\\begin{align*}\n\n\\sum_{k=0}^n (k+2)2^k&=\\sum_{k=0}^n \\big(u_{k+1}-u_k\\big)\\\\\n\n&=u_{n+1}-u_0\\\\\n\n&=(n+1)2^{n+1}.\n\n\\end{align*}\n\n\n" }, { "question": " 11 - Somme et factorielles [Signaler une erreur] [Ajouter à ma feuille d'exos]Enoncé Démontrer que, pour tout $n\\in\\mathbb N^*$, on a \n\n$$(n+1)!\\geq\\sum_{k=1}^n k!\\quad.$$", "answer": " Pour chaque entier $k$ dans $\\{1,\\dots,n\\}$, on a $k!\\leq n!$.\n\nOn obtient donc\n\n$$\\sum_{k=1}^n k!\\leq\\sum_{k=1}^n n!=n\\times n!$$\n\npuisque la somme de droite est une somme de termes constants. Puisque $n\\leq n+1$, on obtient bien\n\n$$\\sum_{k=1}^n k!\\leq (n+1)\\times n!=(n+1)!\\quad .$$\n" }, { "question": " 12 - Somme et somme des carrés [Signaler une erreur] [Ajouter à ma feuille d'exos]Enoncé Soit $n\\geq 1$ et $x_1,\\dots,x_n$ des réels vérifiant \n\n$$\\sum_{k=1}^n x_k=n\\textrm{ et }\\sum_{k=1}^n x_k^2=n.$$\n\nDémontrer que, pour tout $k$ dans $\\{1,\\dots,n\\}$, $x_k=1$.", "answer": " Calculons $\\sum_{k=1}^n (1-x_k)^2$ :\n\n$$\\sum_{k=1}^n (1-x_k)^2=\\sum_{k=1}^n 1-2\\sum_{k=1}^n x_k+\\sum_{k=1}^n x_k^2=n-2n+n=0.$$\n\nOr, une somme de réels positifs est nulle si et seulement si chacun des termes de la somme est nul. On en déduit le résultat demandé.\n" }, { "question": " 13 - Somme des entiers, des carrés,... [Signaler une erreur] [Ajouter à ma feuille d'exos]Enoncé Pour $n\\in\\mathbb N^*$, on note\n\n$$a_n=\\sum_{k=1}^n k,\\ b_n=\\sum_{k=1}^n k^2\\textrm{ et }c_n=\\sum_{k=1}^n k^3.$$\n\nDémontrer que $\\displaystyle a_n=\\frac{n(n+1)}2$, que $\\displaystyle b_n=\\frac{n(n+1)(2n+1)}6$ et que $c_n=a_n^2$.", "answer": " On procède simplement par récurrence sur $n$. Voyons d'abord pourquoi la formule est vraie pour $a_n$. Elle\n\nest vraie pour $n=1$ (car $1=\\frac{1\\times 2}{2}$). Soit $n\\in\\mathbb N$ telle qu'elle est vraie au rang $n$ et prouvons-la\n\nau rang $n+1$. Alors, \n\n$$a_{n+1}=a_n+(n+1)=\\frac{n(n+1)}2+(n+1)=(n+1)\\times\\left(\\frac n2+1\\right)=\\frac{(n+1)\\big((n+1)+1\\big)}2.$$\n\nLa formule est donc vraie au rang $n+1$ et par le principe de récurrence, elle est vraie pour tout entier $n\\geq 0$. \n\nLe raisonnement est rigoureusement identique pour $b_n$ et pour $c_n$. Voyons par exemple le cas de $c_n$.\n\nComme précédemment, la formule est vraie au rang $1$, et si elle est vraie au rang $n$, alors\n\n$$c_{n+1}=c_n+(n+1)^3=\\frac{n^2(n+1)^2}4+(n+1)^3=(n+1)^2\\left(\\frac {n^2}4+(n+1)\\right).$$\n\nIl suffit alors pour conclure de remarquer que \n\n$$\\frac {n^2}4+(n+1)=\\frac{(n+2)^2}4.$$\n\nLa propriété est vraie au rang $n+1$ ce qui clôt la démonstration par récurrence.\n" }, { "question": " 14 - Calculs de sommes arithmétiques [Signaler une erreur] [Ajouter à ma feuille d'exos]Enoncé Calculer les somme suivantes :\n\n\n $A_n=\\sum_{k=1}^n 3$.\n\n $B_n=\\sum_{k=1}^n A_k$.\n\n $S_n=\\sum_{k=0}^{n}(2k+1)$.\n\n\n", "answer": " \n On somme des termes constants, et il y a $n$ termes dans la somme, donc \n\n$A_n=3n$.\n\n Par linéarité, \n\n\\[ B_n = 3\\sum_{k=1}^n k=\\frac{3n(n+1)}2. \\]\n\n On raisonne également par linéarité : \n\n\\begin{align*}\n\nS_n&=2\\sum_{k=0}^n k+\\sum_{k=0}^n 1\\\\\n\n&=n(n+1)+(n+1)\\\\\n\n&=(n+1)^2.\n\n\\end{align*}\n\n\n" }, { "question": " 15 - Calculs de sommes géométriques [Signaler une erreur] [Ajouter à ma feuille d'exos]Enoncé Calculer les sommes suivantes : \n\n\n $S=\\frac{1}{2^{10}}+\\frac{1}{2^{20}}+\\frac{1}{2^{30}}+\\cdots+\\frac{1}{2^{1000}}$.\n\n $T_n=\\sum_{k=0}^n \\frac{2^{k-1}}{3^{k+1}}$.\n\n", "answer": " \n On remarque que \n\n$$S=\\sum_{k=1}^{100}\\frac{1}{2^{10k}}=\\sum_{k=1}^{100}\\left(\\frac1{2^{10}}\\right)^k.$$\n\nPar la formule donnant la somme d'une suite géométrique, on trouve\n\n\\begin{align*}\n\nS&=\\frac{ \\frac{1}{2^{10}}-\\frac 1{2^{1010}}}{1-\\frac1{2^{10}}}\\\\\n\n&=\\frac{1-\\frac 1{2^{1000}}}{2^{10}-1}\\\\\n\n&=\\frac{2^{1000}-1}{2^{1000}(2^{10}-1)}.\n\n\\end{align*}\n\n On factorise par $1/6$ pour se ramener à une somme géométrique.\n\n$$\\sum_{k=0}^n\\frac{2^{k-1}}{3^{k+1}}=\\frac 16\\sum_{k=0}^n \\left(\\frac 23\\right)^k=\\frac 16\\times\\frac{1-\\left(\\frac 23\\right)^{n+1}}{1-\\frac 23}=\n\n\\frac{1-\\left(\\frac 23\\right)^{n+1}}2.$$\n\n\n" }, { "question": " 16 - Calcul de sommes par changements d'indice [Signaler une erreur] [Ajouter à ma feuille d'exos]Enoncé Calculer la somme suivante : \n\n$$\\sum_{k=1}^n (n-k+1).$$", "answer": " Effectuons le changement d'indices $\\ell=n-k+1$. Lorsque $k$ varie de $1$ à $n$, alors $\\ell$ varie aussi de $1$ à $n$. On a donc\n\n$$\\sum_{k=1}^n (n+1-k)=\\sum_{\\ell=1}^n \\ell=\\frac{n(n+1)}2.$$\n" }, { "question": " 17 - Calcul de sommes avec indices négatifs [Signaler une erreur] [Ajouter à ma feuille d'exos]Enoncé Calculer la somme suivante : \n\n$$\\sum_{k=-5}^{15} k(10-k).$$", "answer": " On écrit en développant le produit\n\n$$\\sum_{k=-5}^{15}k(10-k)=10\\sum_{k=-5}^{15}k-\\sum_{k=-5}^{15}k^2.$$\n\nÉtudions ensuite la première somme en la découpant en deux :\n\n\\begin{align*}\n\nS_1=\\sum_{k=-5}^{15}k&=\\sum_{k=-5}^0 k +\\sum_{k=1}^{15}k\n\n\\end{align*}\n\nDans la première de ces deux sommes, on fait le changement de variables $\\ell=-k$. Lorsque $k$ va de $-5$ à $0$, $\\ell$ va de $0$ à $5$. On trouve donc que \n\n\\begin{align*}\n\nS_1&=-\\sum_{\\ell=0}^5 \\ell+\\sum_{k=1}^{15}k\\\\\n\n&=-\\frac{5\\times 6}2+\\frac{15\\times 16}2\\\\\n\n&=105.\n\n\\end{align*}\n\nPour la deuxième somme, on fait le même raisonnement :\n\n\\begin{align*}\n\nS_2=\\sum_{k=-5}^{15}k^2&=\\sum_{k=-5}^0 k^2 +\\sum_{k=1}^{15}k^2\\\\\n\n&=\\sum_{\\ell=0}^5(-\\ell)^2+\\sum_{k=1}^{15}k^2\\\\\n\n&=\\sum_{\\ell=0}^5 \\ell^2+\\sum_{k=1}^{15}k^2\\\\\n\n&=\\frac{5\\times 6\\times 11}{6}+\\frac{15\\times 16\\times 31}6=1295.\n\n\\end{align*}\n\nFinalement, on trouve\n\n$$\\sum_{k=-5}^{15} k(10-k)=1050-1295=-245.$$\n\nOn peut vérifier cette somme par un petit programme Python :\n\n     S=0\n    for k in range(-5,16):\n         S+=k*(10-k)\n    print(S)\n\n" }, { "question": " 18 - Calcul de sommes par découpage [Signaler une erreur] [Ajouter à ma feuille d'exos]Enoncé Soit $n\\in\\mathbb N$.\n\n\n Calculer $A_n=\\sum_{k=2n+1}^{3n}(2n)$.\n\n Calculer $B_n=\\sum_{k=n}^{2n}k$.\n\n En déduire la valeur de $S_n=\\sum_{k=n}^{3n}\\min(k,2n)$.\n\n", "answer": " \n On ne somme que des termes constants, et donc on trouve \n\n$$A_n=n\\times (2n)=2n^2.$$\n\n Notons $T_n=\\sum_{k=0}^n k$. Alors on a \n\n$$B_n=T_{2n}-T_{n-1}=n(2n+1)-\\frac{(n-1)n}2=\\frac{n(4n+2-(n-1))}2=\\frac{3(n+1)n}2.$$\n\n On remarque que si $k\\leq 2n$, alors $\\min(k,2n)=k$ alors que si $k\\geq 2n+1$, on a $\\min(k,2n)=2n$. On en déduit que\n\n$$S_n=A_n+B_n=2n^2+\\frac{3n(n+1)}2=\\frac{n(7n+3)}2.$$\n\n\n" }, { "question": " 19 - Se ramener à une somme classique [Signaler une erreur] [Ajouter à ma feuille d'exos]Enoncé Pour $n\\geq 1$, on pose $u_n=\\frac{1}{n^2}+\\frac{2}{n^2}+\\cdots+\\frac{n}{n^2}$. Calculer explicitement $u_n$, puis en déduire la limite de la suite $(u_n)$.", "answer": " En mettant $1/n^2$ en facteur, on se ramène à une somme classique. En effet, on peut écrire\n\n$$u_n=\\frac1{n^2}\\sum_{k=1}^n k=\\frac{1}{n^2}\\times\\frac{n(n+1)}{2}=\\frac{n+1}{2n}=\\frac{1}2+\\frac{1}{2n}.$$\n\nAinsi, la suite $(u_n)$ tend vers $1/2$.\n" }, { "question": " 20 - Un produit [Signaler une erreur] [Ajouter à ma feuille d'exos]Enoncé Pour $n\\in\\mathbb N^*$ et $x\\in\\mathbb R$, on note\n\n$$P_n(x)=\\prod_{k=1}^n \\left(1+\\frac xk\\right).$$\n\n\n Que valent $P_n(0)$, $P_n(1)$, $P_n(-n)$?\n\n Démontrer que pour tout réel non-nul $x$, on a \n\n$$P_n(x)=\\frac {x+n}xP_n(x-1).$$\n\n Pour $p\\in\\mathbb N^*$, écrire $P_n(p)$ comme coefficient du binôme.\n\n", "answer": " \n On a $P_n(0)=1$ (on ne fait que des produits de 1), $P_n(-n)=0$, car alors\n\n$$1+\\frac{-n}n=0$$\n\net donc on a un terme nul dans le produit. Enfin,\n\n$$P_n(1)=\\prod_{k=1}^n \\frac{k+1}k=\\frac{2\\times 3\\times\\dots\\times (n+1)}{1\\times 2\\times\\dots\\times n}={n+1}.$$\n\n On a \n\n\\begin{eqnarray*}\n\nP_n(x)&=&\\prod_{k=1}^n \\frac{x+k}k\\\\\n\n&=&\\frac{(x+1)(x+2)\\dots (x+n)}{1\\times 2\\times\\dots\\times n}\\\\\n\n&=&\\frac{x+n}{x}\\times\\frac {(x-1+1)(x-1+2)\\dots (x-1+n)}{1\\times 2\\times\\dots\\times n}\\\\\n\n&=&\\frac{x+n}xP_n(x-1).\n\n\\end{eqnarray*}\n\n On a \n\n$$P_n(p)=\\prod_{k=1}^n \\frac{k+p}{k}=\\frac{(p+1)\\dots (p+n)}{n!}=\\frac{(n+p)!}{n!p!}=\\binom{n+p}{p}.$$\n\n\n" }, { "question": " 21 - Somme géométrique dans tous ses états [Signaler une erreur] [Ajouter à ma feuille d'exos]Enoncé Soit pour $n\\in\\mathbb N$, $u_n=(-2)^n$. Calculer les sommes suivantes : \n\n$$\\sum_{k=0}^{2n} u_{k};\\quad \\sum_{k=0}^{2n+1} u_{k};\\quad \\sum_{k=0}^{n} u_{2k};\\quad \\sum_{k=0}^{2n} (u_{k}+n);\\quad \\left(\\sum_{k=0}^{2n} u_{k}\\right)+n;\\quad \\sum_{k=0}^{n} u_{k+n};\\quad \\sum_{k=0}^{n} u_{kn}.$$", "answer": " Dans cet exercice, il faut faire très attention aux notations, puis appliquer la formule de la somme d'une série géométrique. Il vient alors\n\n$$\\sum_{k=0}^{2n} u_{k}=\\sum_{k=0}^{2n}(-2)^k=\\frac{1-(-2)^{2n+1}}{1-(-2)}=\\frac{1+2^{2n+1}}3.$$\n\n$$\\sum_{k=0}^{2n+1} u_{k}=\\frac{1-2^{2n+2}}3.$$\n\n$$\\sum_{k=0}^{n} u_{2k}=\\sum_{k=0}^n 4^k=\\frac{1-4^{n+1}}{1-4}=\\frac{4^{n+1}-1}3.$$\n\n$$\\sum_{k=0}^{2n} (u_{k}+n)=\\sum_{k=0}^{2n} u_k+\\sum_{k=0}^{2n} n=\\frac{1+2^{2n+1}}3+n(2n+1).$$\n\n$$\\left(\\sum_{k=0}^{2n} u_{k}\\right)+n=\\frac{1+2^{2n+1}}3+n.$$\n\n$$\\sum_{k=0}^{n} u_{k+n}=\\sum_{k=0}^n (-2)^{k+n}=(-2)^n\\sum_{k=0}^n (-2)^k=\\frac{(-2)^n(1-(-1)^{n+1} 2^{n+1})}{3}.$$\n\n$$\\sum_{k=0}^{n} u_{kn}=\\sum_{k=0}^n ((-2)^n)^k=\\frac{1-(-2)^{n(n+1)}}{1-(-2)^n},$$\n\nsauf si $n=0$ auquel cas la somme vaut $u_0=1$.\n" }, { "question": " 22 - Sommes alternées [Signaler une erreur] [Ajouter à ma feuille d'exos]Enoncé \n Simplifier la somme $\\sum_{k=1}^{2n}(-1)^k k$ en faisant des sommations par paquets. \n\n Montrer par récurrence que pour tout $n\\in\\mtn^*$, on a \n\n$$S_n=\\sum_{k=1}^n (-1)^k k=\\frac{(-1)^n (2n+1)-1}{4}.$$\n\nRetrouver le résultat précédent.\n\n", "answer": " \n L’ensemble $\\{1, . . . , 2n\\}$ est la réunion des parties deux à deux disjointes $\\{2p − 1, 2p\\}$ pour $p$ variant de $1$ à $n$. Or $(−1)^{2p−1}(2p − 1) + (−1)^{2p}2p = 2p − (2p − 1) = 1$ pour tout $1\\leq p\\leq n$, donc la somme est égale à $n$.\n\n Initialisation : On commence par vérifier la propriété pour $n=1$. On a \n\n$$\\sum_{k=1}^1 (-1)^k k=-1\\textrm{ et }\\frac{(-1)^1 (2\\times 1+1)-1}{4}=-1$$\n\nce qui prouve bien l'égalité voulue.\n\nHérédité : Supposons la propriété vraie au rang $n$ et prouvons-là au rang $n+1$.\n\nOn a \n\n\\begin{eqnarray*}\n\n\\sum_{k=1}^{n+1}(-1)^k k&=&\\sum_{k=1}^n (-1)^k k+(-1)^{n+1}(n+1)\\\\\n\n&=&\\frac{(-1)^n (2n+1)-1}{4}+(-1)^{n+1}(n+1)\\\\\n\n&=&\\frac{(-1)^{n+1}(-2n-1+4n+4)-1}{4}\\\\\n\n&=&\\frac{(-1)^{n+1}(2(n+1)+1)-1}{4}\n\n\\end{eqnarray*}\n\nce qui démontre bien la propriété au rang $n+1$. Remarquons que le passage\n\nde la première à la deuxième ligne utilise l'hypothèse de récurrence.\n\nSi on somme jusque $2n$ au lieu de $n$, on trouve que \n\n$$S_{2n}=\\frac{4n+1-1}4=n.$$\n\n\n" }, { "question": " 23 - Somme de puissances [Signaler une erreur] [Ajouter à ma feuille d'exos]Enoncé Soit $x\\in\\mathbb R$ et $n\\in\\mathbb N^*$.\n\n\n Calculer $S_n(x)=\\sum_{k=0}^n x^k.$\n\n En déduire la valeur de $T_n(x)=\\sum_{k=0}^n k x^k.$\n\n\n", "answer": " C'est un exercice extrêmement classique qu'il faut savoir faire. \n\n\n On reconnait une somme géométrique de raison $x$. Pour $x\\neq 1$, on a \n\n$$S_n(x)=\\frac{1-x^{n+1}}{1-x}.$$\n\nPour $x=1$, $S_n(1)=n+1$.\n\n On distingue là encore le cas $x=1$. Il donne\n\n$$T_n(1)=\\sum_{k=0}^n k=\\frac{n(n+1)}2.$$\n\nSinon, on dérive $S_n$ : pour tout $x\\neq 1$, \n\n$$S_n'(x)=\\sum_{k=1}^n kx^{k-1}\\implies T_n(x)=xS_n'(x).$$\n\nOn calcule alors $S_n'(x)$ avec la formule obtenue à la question précédente et on trouve\n\n$$T_n(x)=x\\frac{nx^{n+1} −(n+1)x^n +1}{ (x−1)^2}.$$\n\n\n\n" }, { "question": " 24 - Sommation d'Abel [Signaler une erreur] [Ajouter à ma feuille d'exos]Enoncé Soient $(a_n)_{n\\in\\mathbb N}$ et $(B_n)_{n\\in\\mathbb N}$ deux suites de nombres complexes. On définit deux suites $(A_n)_{n\\in\\mathbb N}$ et $(b_n)_{n\\in\\mathbb N}$ en posant :\n\n$$A_n=\\sum_{k=0}^n a_k,\\quad\\quad b_n=B_{n+1}-B_n.$$\n\n\n Démontrer que $\\sum_{k=0}^n a_kB_k=A_n B_n-\\sum_{k=0}^{n-1}A_kb_k.$\n\n En déduire la valeur de $\\sum_{k=0}^n 2^kk$.\n\n", "answer": " \n On convient que $A_{-1}=0$. On a alors\n\n\\begin{eqnarray*}\n\n\\sum_{k=0}^n a_k B_k&=&\\sum_{k=0}^n (A_k-A_{k-1})B_k\\\\\n\n&=&\\sum_{k=0}^n A_k B_k-\\sum_{k=0}^n A_{k-1}B_k\\\\\n\n&=&\\sum_{k=0}^n A_k B_k-\\sum_{k=0}^{n-1} A_k B_{k+1}\\\\\n\n&=&A_n B_n+\\sum_{k=0}^{n-1}A_k (B_k-B_{k+1})\\\\\n\n&=&A_nB_n-\\sum_{k=0}^{n-1} A_k b_k.\n\n\\end{eqnarray*}\n\n On pose $a_k=2^k$ et $B_k=k$. Alors $A_n=2^{n+1}-1$ et $b_k=1$. On en déduit que\n\n\\begin{eqnarray*}\n\n\\sum_{k=0}^n k2^k&=&(2^{n+1}-1)n-\\sum_{k=0}^{n-1}(2^{k+1}-1)\\\\\n\n&=&(2^{n+1}-1)n-2(2^n-1)+n\\\\\n\n&=&2^{n+1}(n-1)+2.\n\n\\end{eqnarray*}\n\n\n\nCe procédé de transformation de somme s'appelle une sommation d'Abel.\n" }, { "question": " 25 - Comment permuter une somme double? [Signaler une erreur] [Ajouter à ma feuille d'exos]Enoncé Soit $(a_{i,j})_{(i,j)\\in\\mathbb N^2}$ une suite double de nombres réels. Soit $n$ et $m$ deux entiers naturels. Intervertir les sommes doubles suivantes :\n\n\n $S_1=\\sum_{i=0}^n \\sum_{j=i}^n a_{i,j}$;\n\n $S_2=\\sum_{i=0}^n \\sum_{j=0}^{n-i}a_{i,j}$;\n\n $S_3=\\sum_{i=0}^n \\sum_{j=i}^m a_{i,j}$ où on a supposé $n\\leq m$.\n\n", "answer": " \n On fait la somme sur tous les indices $i,j$ tels que $0\\leq i\\leq j\\leq n$. On a donc\n\n$$S_1=\\sum_{j=0}^n \\sum_{i=0}^j a_{i,j}.$$\n\nUne autre façon de le voir est de poser $b_{i,j}=a_{i,j}$ si $j\\geq i$ et $b_{i,j}=0$ si $j0$ et $y_{n+1}-y_n\\geq 2y_n>0$, donc les deux suites sont strictement croissantes.\n\n Puisque les suites sont strictement croissantes, les couples $(x_n,y_n)$ sont tous différents. Il suffit donc de démontrer que, pour tout entier $n$, on a $x_n^2-2y_n^2=1$. Pour cela, il suffit de remarquer que\n\n$$(3-2\\sqrt 2)^n=x_n-\\sqrt 2 y_n.$$\n\nEn effet, si on développe $(3-2\\sqrt 2)^n$ par la formule du binôme, on trouve\n\n$$(3-2\\sqrt 2)^n =\\sum_{k=0}^n \\binom{n}k 3^k (-1)^{n-k}2^{n-k}(\\sqrt 2)^{n-k}.$$\n\nOn regroupe les termes comme précédemment, sachant que pour les termes entiers, on a $n-k$ pair et donc $(-1)^{n-k}=1$ et pour les termes de la forme $m\\sqrt 2$, on a $n-k$ impair\n\net $(-1)^{n-k}=-1$. Ainsi, on a \n\n$$x_n^2-2y_n^2=(x_n-\\sqrt 2 y_n)(x_n+\\sqrt 2 y_n)=(3-2\\sqrt 2)^n(3+2\\sqrt 2)^n =1^n=1.$$\n" } , { "question": " 1 - Partie réelle, partie imaginaire, conjugué [Signaler une erreur] [Ajouter à ma feuille d'exos]Enoncé Donner la partie réelle, la partie imaginaire et le conjugué des nombres complexes suivants :\n\n$$\n\n\\begin{array}{lllll}\n\nz_1=-2i+5&\\quad&z_2=15&\\quad&z_3=3i\\\\\n\nz_4=i(2+3i)\n\n\\end{array}$$", "answer": " \n $\\Re e(z_1)=5$, $\\Im m(z_1)=-2$, $\\overline{z_1}=5+2i$.\n\n $\\Re e(z_2)=15$, $\\Im m(z_2)=0$, $\\overline{z_2}=15$.\n\n $\\Re e(z_3)=0$, $\\Im m(z_3)=3$, $\\overline{z_3}=-3i$.\n\n En effectuant le produit, on trouve $z_4=2i-3$, et donc $\\Re e(z_4)=-3$, $\\Im m(z_4)=2$, $\\overline{z_4}=-3-2i$.\n\n\n" }, { "question": " 2 - Forme algébrique - Somme et produits [Signaler une erreur] [Ajouter à ma feuille d'exos]Enoncé Mettre sous forme algébrique les nombres complexes suivants :\n\n$$\\begin{array}{lll}\n\n\\mathbf{1.}\\ z_1=(2+5i)+(i+3)&\\quad \\mathbf{2.}\\ z_2=4(-2+3i)+3(-5-8i)&\\quad\\mathbf{3.}\\ z_3=(2-i)(3+8i)\\\\\n\n\\displaystyle\\mathbf{4.}\\ z_4=(1-i)\\overline{(1+i)}&\\quad\\mathbf{5.}\\ z_5=i(1-3i)^2& \\quad\\mathbf{6.}\\ z_6=(1+i)^3\n\n\\end{array}\n\n$$\n\nAttention! Il y a un symbole de conjugaison dans $z_4$.", "answer": " \n On regroupe simplement les parties réelles et les parties imaginaires. On trouve\n\n$$z_1=5+6i.$$\n\n De la même façon,\n\n$$z_2=(-8+12i)+(-15-24i)=-23-12i.$$\n\n On développe, puis on regroupe pour trouver :\n\n$$z_3=6+16i-3i+8=14+13i.$$\n\n On écrit \n\n$$z_4=(1-i)(1-i)=1-2i-1=-2i.$$\n\n On commence par calculer $(1-3i)^2$ :\n\n\\[ (1-3i)^2= 1-2\\times 1\\times 3i+(3i)^2=1-6i-9=-8-6i. \\]\n\nOn multiplie ensuite par $i$ :\n\n\\[ i(1-3i)^2= -8i-6i^2=6-8i. \\]\n\n Le plus simple est de tout développer, en utilisant la formule du binôme de Newton ou, pour ceux qui ne la connaissent pas (encore), en écrivant $(1+i)^3=(1+i)^2(1+i)$. On trouve\n\n\\begin{align*}\n\nz_6&=(1+i)^3\\\\\n\n&=(1+i)^2(1+i)\\\\\n\n&=(1+2i+i^2)(1+i)\\\\\n\n&=(1+2i-1)(1+i)\\\\\n\n&=2i(1+i)\\\\\n\n&=2i+2i^2\\\\\n\n&=-2+2i.\n\n\\end{align*}\n\nAvec la formule du binôme, on écrit simplement\n\n$$z_6=(1+i)^3=1+3i+3i^2+i^3=1+3i-3-i=-2+2i.$$\n\n\n" }, { "question": " 3 - Forme algébrique - quotients [Signaler une erreur] [Ajouter à ma feuille d'exos]Enoncé Mettre sous forme algébrique les nombres complexes suivants :\n\n$$\\begin{array}{lll}\n\n\\mathbf{1.}\\ z_1=\\frac1{1+i}&\\quad{\\mathbf 2.}\\ z_2=\\frac{-4}{1+i\\sqrt 3}&\n\n\\quad\\mathbf{3.}\\ z_3=\\frac{1-2i}{3+i}\\\\\n\n\\displaystyle{\\mathbf 4.}\\ z_4=\\frac{(3+5i)^2}{1-2i}&\\displaystyle\\quad{\\mathbf 5.}\\ z_5=\\left(\\frac{1+i}{2-i}\\right)^2+\\frac{3+6i}{3-4i}\\\\\n\n\\end{array}\n\n$$", "answer": " Pour mettre un quotient sous forme algébrique, il faut multiplier le numérateur et le dénominateur par la quantité conjuguée du dénominateur.\n\n\n Le conjugué de $1+i$ est $1-i$ et donc on a \n\n\\[ z_1=\\frac{1}{1+i}=\\frac{1-i}{(1+i)(1-i)}=\\frac{1-i}2=\\frac{1}2 -\\frac12 i. \\]\n\n Avec la même méthode, on trouve\n\n$$z_2=\\frac{-4(1-i\\sqrt 3)}{(1+i\\sqrt 3)(1-i\\sqrt 3)}=\\frac{-4(1-i\\sqrt 3)}{1+3}=-1+i\\sqrt 3.$$\n\n On écrit \n\n\\[ \\frac{1-2i}{3+i}=\\frac{(1-2i)(3-i)}{(3-i)(3+i)}=\\frac{1-7i}{9+1}=\\frac1{10}-\\frac{7}{10}i. \\]\n\n On fait la même chose mais on développe d'abord le numérateur. Puisque \n\n$ (3+5i)^2 =3^2+2\\times 3\\times 5i+(5i)^2=9+30i-25=-16+30i$, on trouve :\n\n\\[ z_4=\\frac{-16+30i}{1-2i}\\times\\frac{1+2i}{1+2i}=\\frac{-76-2i}5. \\]\n\n Ce n'est pas plus compliqué que les exemples précédents, mais il faut être méthodique dans les calculs. On écrit d'abord :\n\n$$\\frac{1+i}{2-i}=\\frac{(1+i)(2+i)}{(2-i)(2+i)}=\\frac{1+3i}5.$$\n\nEn élevant au carré,\n\n$$\\left(\\frac{1+i}{2-i}\\right)^2=\\frac{-8+6i}{25}.$$\n\nD'autre part, par la même méthode,\n\n$$\\frac{3+6i}{3-4i}=\\frac{-3+6i}5.$$\n\nEn faisant la somme et en mettant tout au même dénominateur, on trouve finalement :\n\n$$z_5=-\\frac{23}{25}+\\frac{36}{25}i.$$\n\n\n" }, { "question": " 4 - Avec la formule du binôme [Signaler une erreur] [Ajouter à ma feuille d'exos]Enoncé Simplifier les nombres complexes suivants : $(1+i)^5$, $(1-i)^4$.", "answer": " On développe par la formule du binôme de Newton, en utilisant $i^2=-1$ pour simplifier et regrouper partie réelle et partie imaginaire. Il vient\n\n$$(1+i)^5=1+5i-10-10i+5+i=-4-4i.$$\n\nDe même,\n\n$$(1-i)^4=1-4i-6+4i+1=-4.$$\n\nOn aurait aussi pu obtenir ces résultats en mettant les nombres complexes sous forme trigonométrique.\n" }, { "question": " 5 - Calculs avec des conjugués [Signaler une erreur] [Ajouter à ma feuille d'exos]Enoncé Soit $z$ un nombre complexe non nul, de forme algébrique $z=x+iy$. Donner la forme algébrique des nombres complexes suivants :\n\n$$\\begin{array}{lll}\n\n\\mathbf{1.}\\ z_1=\\frac{\\overline z}{z}&\\quad\\mathbf{2.}\\ z_2=\\frac{iz}{\\overline z}.\n\n\\end{array}$$", "answer": " \n On remplace $z$ par $x+iy$ et $\\overline z$ par $x-iy$, puis on mène le calcul comme d'habiture. On a donc\n\n$$z_1=\\frac{x-iy}{x+iy}=\\frac{(x-iy)(x-iy)}{(x+iy)(x-iy)}=\\frac{x^2-y^2-2ixy}{x^2+y^2}=\\frac{x^2-y^2}{x^2+y^2}-2\\frac{xy}{x^2+y^2}i.$$\n\n On a \n\n\\begin{align*}\n\nz_2&=\\frac{i(x+iy)}{x-iy}\\\\\n\n&=\\frac{-y+ix}{x-iy}\\\\\n\n&=\\frac{(-y+ix)(x+iy)}{(x-iy)(x+iy)}\\\\\n\n&=\\frac{-2xy+i(x^2-y^2)}{x^2+y^2}\\\\\n\n&=\\frac{-2xy}{x^2+y^2}+i\\frac{x^2-y^2}{x^2+y^2}.\n\n\\end{align*}\n\n\n" }, { "question": " 6 - Équations du premier degré [Signaler une erreur] [Ajouter à ma feuille d'exos]Enoncé Résoudre les équations suivantes, d'inconnue $z\\in\\mathbb C$ :\n\n$$\n\n\\begin{array}{lll}\n\n{\\mathbf 1.}\\ z+2i=iz-1&\\quad&{\\mathbf 2.}\\ (3+2i)(z-1)=i\\\\\n\n{\\mathbf 3.}\\ (2-i)z+1=(3+2i)z-i&\\quad&{\\mathbf 4.}\\ (4-2i)z^2=(1+5i)z.\n\n\\end{array}$$\n\nOn écrira les solutions sous forme algébrique.", "answer": " \n L'équation est équivalente à \n\n$$z(1-i)=-1-2i\\iff z=\\frac{-1-2i}{1-i}=\\frac{(-1-2i)(1+i)}{(1-i)(1+i)}=\\frac12-\\frac 32i.$$\n\n De la même façon, on a \n\n$$z-1=\\frac{i}{3+2i}=\\frac{i(3-2i)}{(3+2i)(3-2i)}=\\frac 2{13}+\\frac 3{13}i.$$\n\nOn en déduit que \n\n$$z=\\frac{15}{13}+\\frac 3{13}i.$$\n\n L'équation cette fois est équivalente à \n\n\\begin{align*}\n\n(2-i-3-2i)z=-i-1&\\iff (-1-3i)z=-i-1\\\\\n\n&\\iff z=\\frac{-i-1}{-1-3i}=\\frac{(-i-1)(-1+3i)}{(-1-3i)(-1+3i)}=\\frac{2}5-\\frac15 i.\n\n\\end{align*}\n\n On distingue deux cas. D'abord, $z=0$ est solution de l'équation. Si $z\\neq 0$, alors on peut simplifier par $z$ et l'équation est équivalente à \n\n$$(4-2i)z=1+5i\\iff z=\\frac{1+5i}{4-2i}=\\frac{(1+5i)(4+2i)}{(4-2i)(4+2i)}=\\frac{-6+22i}{20}=\\frac{-3}{10}+\\frac{11}{10}i.$$\n\nL'équation admet donc deux solutions, $z=0$ et $z=\\frac{-3}{10}+\\frac{11}{10}i$.\n\n\n" }, { "question": " 7 - Équations avec des conjuguées. [Signaler une erreur] [Ajouter à ma feuille d'exos]Enoncé Résoudre les équations suivantes :\n\n$$\n\n\\begin{array}{lll}\n\n\\displaystyle{\\mathbf 1.}\\ 2z+i=\\overline z+1&\\displaystyle{\\mathbf 2.}\\ 2z+\\overline z=2+3i&\\displaystyle{\\mathbf 3.}\\ 2z+2\\overline z=2+3i.\n\n\\end{array}$$", "answer": " \n On écrit $z=x+iy$ avec $x,y\\in\\mathbb R$ de sorte que $2z+i=2x+i(2y+1)$ et $\\overline z+1=(x-iy)+1=(x+1)-iy$. Puisque deux nombres complexes sont égaux si et seulement s'il ont même partie réelle et même partie imaginaire, l'équation est équivalente à $2x=x+1$ et $2y+1=-y$. On trouve alors $x=1$ et $y=-1/3$. L'équation admet une unique solution, $z=1-\\frac 13 i$.\n\n On reprend la même méthode. Alors $2z+\\bar z=3x+iy=2+3i$ si et seulement si $x=\\frac 23$ et $y=3$. L'équation admet donc une unique solution, $z=\\frac 23+3i$.\n\n On procède de la même façon, mais cette fois on constate que $2z+2\\bar z=4x$ est réel : il ne pourra jamais être égal à $2+3i$. L'équation n'admet pas de solutions.\n\n\n" }, { "question": " 8 - [Signaler une erreur] [Ajouter à ma feuille d'exos]Enoncé Résoudre les systèmes suivants, d'inconnues les nombres complexes $z_1$ et $z_2$ :\n\n\n\n\n $$\\left\\{\n\n \\begin{array}{rcl}\n\n 2z_1-z_2&=&i\\\\\n\n -2z_1+3iz_2&=&-17\n\n \\end{array}\\right.$$\n\n \n\n $$\\left\\{\n\n \\begin{array}{rcl}\n\n 3iz_1+iz_2&=&i+7\\\\\n\n iz_1+2z_2&=&11i\n\n \\end{array}\\right.$$\n\n\n\nOn donnera les résultats sous forme algébrique.", "answer": " Le fait de résoudre des systèmes avec des nombres complexes ne change pas la méthode de résolution.\n\nSeuls les calculs sont plus compliqués!\n\n\n On a \n\n \\begin{eqnarray*}\n\n \\left\\{\n\n \\begin{array}{rcl}\n\n 2z_1-z_2&=&i\\\\\n\n -2z_1+3iz_2&=&-17\n\n \\end{array}\\right.\n\n &\\iff& \n\n \\left\\{\n\n \\begin{array}{rcl}\n\n 2z_1-z_2&=&i\\\\\n\n (3i-1)z_2&=&-17+i\\ \\textrm{(L1)+(L2)}\n\n \\end{array}\\right.\\\\\n\n &\\iff&\n\n \\left\\{\n\n \\begin{array}{rcl}\n\n 2z_1&=&i+z_2\\\\\n\n z_2&=&\\frac{-17+i}{3i-1}.\n\n \\end{array}\\right.\n\n\\end{eqnarray*}\n\nOn met $z_2$ sous forme algébrique en multipliant numérateur et dénominateur par $-3i-1$ et on trouve $z_2=2+5i$. Finalement,\n\nrevenant à $z_1$, on trouve que $z_1=1+3i$. Le système admet une unique solution donnée par $z_1=1+3i$ et $z_2=2+5i$.\n\n On a \n\n \\begin{eqnarray*}\n\n \\left\\{\n\n \\begin{array}{rcl}\n\n 3iz_1+iz_2&=&i+7\\\\\n\n iz_1+2z_2&=&11i\n\n \\end{array}\\right.\n\n &\\iff& \n\n \\left\\{\n\n \\begin{array}{rcl}\n\n 3iz_1+iz_2&=&i+7\\\\\n\n 3iz_1+6z_2&=&33i\\\\\n\n \\end{array}\\right.\\\\\n\n &\\iff&\n\n \\left\\{\n\n \\begin{array}{rcl}\n\n (i-6)z_2&=&7-32i\\\\\n\n iz_1+2z_2&=&11i.\n\n \\end{array}\\right.\n\n\\end{eqnarray*}\n\nOn a donc $z_2=\\frac{7-32i}{i-6}=-2+5i$ après mise sous forme algébrique. On en déduit facilement que $z_1=1-4i$. Ce\n\nsecond système admet donc une unique solution donnée par $z_1=1-4i$ et $z_2=-2+5i$.\n\n\n" }, { "question": " 9 - Entiers de Gauss [Signaler une erreur] [Ajouter à ma feuille d'exos]Enoncé On appelle ensemble des entiers de Gauss noté $\\mathbb Z[i]$ l'ensemble des nombres complexes qui s'écrivent $a+ib$, avec $a$ et $b\\in\\mathbb Z.$\n\n\n Soit $z$ et $z'$ deux entiers de Gauss. Démontrer que $z-z'$ et $zz'$ sont des entiers de Gauss.\n\n Pour tout nombre complexe $z$, on note $N(z)=z\\bar z.$\n\n\n Démontrer que, pour tous nombres complexes $z$ et $z'$, $N(z)N(z')=N(zz').$\n\n Démontrer que, pour tout entier de Gauss $z$, $N(z)$ est un entier naturel.\n\n Soit $z$ un entier de Gauss non nul tel que $1/z$ est un entier de Gauss. Démontrer que $N(z)=1$.\n\n Déterminer l'ensemble des entiers de Gauss tels que $1/z$ est un entier de Gauss.\n\n\n", "answer": " \n Soit $z=a+ib$ et $z'=a'+ib'\\in\\mathbb Z[i].$ Alors \n\n$$z-z'=(a-a')+i(b-b')\\in\\mathbb Z[i]$$\n\n(puisque $a-a'$ et $b-b'\\in\\mathbb Z$) et \n\n$$zz'=(aa'-bb')+i(ab'+a'b)\\in\\mathbb Z[i]$$\n\n(puisque $aa-bb'$ et $ab'+a'b\\in\\mathbb Z$).\n\n \n On remarque que $N(z)=|z|^2.$ Puisque $|zz'|=|z|\\cdot |z'|,$ en mettant au carré cette égalité, on a le résultat demandé.\n\n Si $z=a+ib$, alors $N(z)=a^2+b^2$ et $a^2,b^2$ sont des entiers naturels, donc $N(z)$ aussi.\n\n Soit $z$ un entier de Gauss tel que $1/z$ est un entier de Gauss. Posons $z'=1/z$. Alors on sait que $zz'=1$ et donc $N(z)\\times N(z')=1$. Or le produit de deux entiers naturels est égal à $1$ si et seulement si ces deux entiers sont égaux à $1$. Donc $N(z)=1.$\n\n Soit $z=a+ib\\in\\mathbb Z[i]$ tel que $1/z\\in\\mathbb Z[i].$ Alors $N(z)=1$, et donc $a^2+b^2=1.$ Or, puisque $a^2$ et $b^2$ sont des entiers naturels, ceci n'est possible que dans quatre cas : $(a,b)=(1,0),$ $(a,b)=(-1,0),$ $(a,b)=(0,1),$\n\net $(a,b)=(0,-1).$ Réciproquement, il est facile de vérifier que $1$, $-1$, $i$ et $-i$ sont éléments de $\\mathbb Z[i],$ et sont tels que $1/z\\in\\mathbb Z[i],$ puisque respectivement on a $1/z=1$, $-1$, $-i$ et $i$. Les éléments de $\\mathbb Z[i]$ tels que $1/z\\in\\mathbb Z[i]$ sont donc $1$, $-1$, $i$ et $-i.$\n\n\n\n" }, { "question": " 10 - Déterminer des fonctions complexes [Signaler une erreur] [Ajouter à ma feuille d'exos]Enoncé On se propose dans cet exercice de déterminer toutes les fonctions $f:\\mathbb C\\to\\mathbb C$ vérifiant les trois propriétés suivantes :\n\n\n $\\forall z\\in\\mathbb R$, $f(z)=z$.\n\n $\\forall (z,z')\\in\\mathbb C^2$, $f(z+z')=f(z)+f(z')$.\n\n $\\forall (z,z')\\in\\mathbb C^2$, $f(z\\times z')=f(z)\\times f(z')$.\n\n\n\n Vérifier que les fonctions définies par $f(z)=z$ et $f(z)=\\bar z$ sont solutions du problème.\n\n Réciproquement soit $f$ une fonction du problème. \n\n\n Démontrer que $f(i)=i$ ou $f(i)=-i$.\n\n On suppose que $f(i)=i$. Démontrer que, pour tout $z\\in\\mathbb C$, $f(z)=z$.\n\n On suppose que $f(i)=-i$. Démontrer que, pour tout $z\\in\\mathbb C$, $f(z)=\\bar z$.\n\n\n Qu'a-t-on démontré dans cet exercice?\n\n", "answer": " \n Pour $f(z)=z$, il n'y a rien à faire. Pour $f(z)=\\bar z$, ceci vient des propriétés du cours sur le conjugué d'un nombre complexe.\n\n \n Utilisons la propriété (c) avec $z=z'=i$. On a \n\n$$f(i\\times i)=f(i)\\times f(i)=\\big(f(i)\\big)^2.$$\n\nMais on a $i\\times i=-1$ et d'après la propriété (a), $f(-1)=-1$.\n\nOn a donc \n\n$$\\big(f(i)\\big)^2=-1.$$\n\nOr, les solutions de l'équation $z^2=-1$ sont $z=i$ et $z=-i$. Donc $f(i)=i$ ou $f(i)=-i$.\n\n Soit $z\\in\\mathbb C$. Écrivons-le $z=a+ib$, avec $a,b\\in\\mathbb R$. On a alors\n\n\\begin{align*}\n\nf(z)&=f(a+ib)\\\\\n\n&=f(a)+f(ib)\\\\\n\n&=f(a)+f(i)f(b)\\\\\n\n&=a+ib\\\\\n\n&=z.\n\n\\end{align*}\n\nOn a donc démontré que $f(z)=z$ pour tout nombre complexe $z$.\n\n On procède de la même façon. Soit $z\\in\\mathbb C$ qu'on écrit $z=a+ib$. On a \n\n\\begin{align*}\n\nf(z)&=f(a+ib)\\\\\n\n&=f(a)+f(ib)\\\\\n\n&=f(a)+f(i)f(b)\\\\\n\n&=a-ib\\\\\n\n&=\\bar z.\n\n\\end{align*}\n\nOn a donc démontré que $f(z)=\\bar z$ pour tout nombre complexe $z$.\n\n\n Dans cet exercice, on a démontré que les fonctions solutions du problème sont exactement les fonctions définies par $f(z)=z$ et $f(z)=\\bar z$.\n\n\n" }, { "question": " 11 - Forme exponentielle [Signaler une erreur] [Ajouter à ma feuille d'exos]Enoncé Mettre sous forme exponentielle les nombres complexes suivants :\n\n$$\\begin{array}{lll}\n\n{\\mathbf 1.}\\ z_1=1+i\\sqrt 3&\\quad\\mathbf 2.\\ z_2=9i&\\quad{\\mathbf 3.}\\ z_3=-3\\\\\n\n\\displaystyle{\\mathbf 4.}\\ z_4=\\frac{-i\\sqrt 2}{1+i}&\\displaystyle \\quad\\mathbf{5.}\\ z_5=\\frac{(1+i\\sqrt 3)^3}{(1-i)^5}&\\quad{\\mathbf 6.}\\ z_6=\\sin x+i\\cos x.\n\n\\end{array}\n\n$$", "answer": " \n Le module de $1+i\\sqrt 3$ est\n\n$$|1+i\\sqrt 3|=\\sqrt{1+3}=2.$$\n\nIl vient \n\n$$z_1=2\\left(\\frac 12+i\\frac{\\sqrt 3}2\\right)=2e^{i\\pi/3}.$$\n\n On a $9i=9e^{i\\pi/2}$.\n\n On a $-3=3\\times (-1)=3e^{i\\pi}$.\n\n On écrit sous forme trigonométrique le numérateur et le dénominateur, puis on utilise les propriétés de l'exponentielle complexe. On a donc\n\n$$-i\\sqrt 2=\\sqrt 2e^{i\\frac{3\\pi}2};$$\n\n$$1+i=\\sqrt 2e^{i\\pi/4}.$$\n\nOn conclut que \n\n$$z_4=e^{i\\left(\\frac{3\\pi}2-\\frac\\pi 4\\right)}=e^{i\\frac{5\\pi}4}.$$\n\n On procède de la même façon. On a \n\n$$1+i\\sqrt 3=2e^{i\\pi/3}$$\n\nd'où\n\n$$(1+i\\sqrt 3)^3=8e^{i\\pi}.$$\n\nDe même, \n\n$$1-i=\\sqrt 2\\left(\\frac 1{\\sqrt 2}-\\frac1{\\sqrt 2}i\\right)=\\sqrt 2e^{-i\\pi/4},$$\n\nd'où\n\n$$(1-i)^5=4\\sqrt 2e^{-i\\frac{5\\pi}4}.$$\n\nOn fait le quotient et on trouve :\n\n$$z_5=\\sqrt 2e^{i\\pi\\left(1+\\frac 54\\right)}=\\sqrt 2e^{i\\frac{9\\pi} 4}=\\sqrt 2e^{i\\frac\\pi 4}.$$\n\n Il faut faire attention, il faut avoir quelque chose de la forme $\\cos \\theta+i\\sin \\theta$ pour obtenir une forme trigonométrique. L'idée ici est d'utiliser les formules de trigonométrie, et notamment le fait que \n\n$$\\sin\\left(\\frac\\pi 2-x\\right)=\\cos x\\textrm{ et }\\cos\\left(\\frac \\pi2-x\\right)=\\sin x.$$\n\nIl vient \n\n$$z_6=\\cos\\left(\\frac\\pi 2-x\\right)+i\\sin\\left(\\frac\\pi 2-x\\right)=e^{i\\left(\\frac\\pi 2-x\\right)}.$$\n\n\n" }, { "question": " 12 - Forme exponentielle [Signaler une erreur] [Ajouter à ma feuille d'exos]Enoncé On pose\n\n $z_1=4e^{i\\frac{\\pi}{4}},\\;z_2=3ie^{i\\frac{\\pi}{6}},\\;z_3=-2e^{i\\frac{2\\pi}{3}}$. Écrire sous\n\n forme exponentielle les nombres complexes : $z_1$, $z_2$, $z_3$, $z_1z_2$,\n\n $\\frac{z_1z_2}{z_3}$.", "answer": " L'écriture $z_1=4e^{i\\frac{\\pi}{4}}$ est déjà la forme exponentielle de $z_1$. Pour $z_2$, on écrit\n\n$$z_2=3e^{i\\frac\\pi2}e^{i\\frac\\pi6}=3e^{i\\frac{2\\pi}3}.$$\n\nPour $z_3$, on écrit \n\n$$z_3=2e^{i\\pi}e^{i\\frac{2\\pi}3}=2e^{i\\frac{5\\pi}3}.$$\n\nPour $z_1z_2$, on écrit\n\n$$z_1z_2=12e^{i\\left(\\frac\\pi4+\\frac{2\\pi}3\\right)}=12e^{i\\frac{11\\pi}{12}}.$$\n\nPour $z_1z_2/z_3$, on écrit\n\n$$\\frac{z_1z_2}{z_3}=6 e^{i\\left(\\frac{11\\pi}{12}-\\frac{5\\pi}3\\right)}=6e^{-i\\frac{9\\pi}{12}}=6e^{-i\\frac{3\\pi}4}.$$\n" }, { "question": " 13 - Module, argument et opérations [Signaler une erreur] [Ajouter à ma feuille d'exos]Enoncé Soit $z=re^{i\\theta}$ avec $r>0$ et $\\theta\\in\\mathbb R$. Soit $n$ un entier naturel non nul. Donner le module et un argument des nombres complexes suivants :\n\n$$z^2,\\ \\overline{z},\\ \\frac 1z,\\ -z,\\ z^n.$$", "answer": " \n On a $z^2=r^2e^{2i\\theta}$. Le module de $z^2$ est $r^2$, un argument de $z^2$ est $2\\theta$.\n\n On a $\\overline z=re^{-i\\theta}$. Le module de $\\bar z$ est $r$, un argument de $\\bar z$ est $-\\theta$.\n\n On a $\\frac 1z=\\frac 1r e^{-i\\theta}$. Le module de $\\frac 1z$ est $\\frac 1r$, un argument de $\\frac 1z$ est $-\\theta$.\n\n On a $-z=e^{i\\pi}re^{i\\theta}=re^{i(\\theta+\\pi)}$. Le module de $-z$ est $r$, un argument de $-z$ est $\\theta+\\pi$.\n\n On a $z^n=r^n e^{in\\theta}$. Le module de $z^n$ est $r^n$, un argument de $z^n$ est $n\\theta$.\n\n\n" }, { "question": " 14 - Les deux à la fois - avec application [Signaler une erreur] [Ajouter à ma feuille d'exos]Enoncé On considère les nombres complexes suivants :\n\n$$z_1=1+i\\sqrt 3,\\ z_2=1+i\\textrm{ et }z_3=\\frac{z_1}{z_2}.$$\n\n\n Écrire $z_3$ sous forme algébrique.\n\n Écrire $z_3$ sous forme trigonométrique.\n\n En déduire les valeurs exactes de $\\cos\\frac\\pi{12}$ et $\\sin\\frac\\pi{12}$.\n\n", "answer": " \n On utilise (classiquement!) la quantité conjuguée du dénominateur :\n\n$$z_3=\\frac{1+i\\sqrt 3}{1+i}\\times\\frac{1-i}{1-i}=\\frac{1+\\sqrt 3}2+\\frac{\\sqrt 3-1}2i.$$\n\n On a \n\n$$z_1=2e^{i\\pi/3}\\textrm{ et }z_2=\\sqrt 2 e^{i\\pi/4},$$\n\nd'où \n\n$$z_3=\\sqrt 2e^{i\\left(\\frac \\pi 3-\\frac \\pi 4\\right)}=\\sqrt 2\\left(\\cos(\\pi/12)+i\\sin({\\pi}/{12})\\right).$$\n\n D'après l'écriture trigonométrique, on obtient une autre écriture algébrique de $z_3$ :\n\n$$z_3=\\sqrt 2\\cos\\frac\\pi{12}+i\\sqrt 2\\sin \\frac\\pi{12}.$$\n\nPar unicité de l'écriture sous forme algébrique, on en déduit\n\n$$\\cos\\frac\\pi {12}=\\frac{1+\\sqrt 3}{2\\sqrt 2}\\textrm{ et }\\sin\\frac\\pi{12}=\\frac{\\sqrt 3-1}{2\\sqrt 2}.$$\n\nRemarquons qu'on aurait aussi pu obtenir ce résultat à partir de la formule de $\\cos(a-b)$ avec $a=\\frac{\\pi}{3}$ et $b=\\frac{\\pi}4$.\n\n\n" }, { "question": " 15 - Forme algébrique, le retour [Signaler une erreur] [Ajouter à ma feuille d'exos]Enoncé Déterminer la forme algébrique des nombres complexes suivants :\n\n$$\\mathbf 1. z_1=(2+2i)^6\\quad \\mathbf 2. z_2=\\left(\\frac{1+i\\sqrt 3}{1-i}\\right)^{20}\\quad\\mathbf 3. z_3=\\frac{(1+i)^{2000}}{(i-\\sqrt 3)^{1000}}.$$", "answer": " La méthode la plus facile, ici, consiste à calculer d'abord la forme trigonométrique qui se comporte bien mieux vis à vis des puissances, puis à revenir à la forme algébrique.\n\n\n On écrit \n\n$$2+2i=2\\sqrt 2e^{i\\pi/4}$$\n\nd'où \n\n$$z_1=2^9 e^{3i\\pi/2}=-512i.$$\n\n On commence par passer par la forme trigonométrique :\n\n$$\\frac{1+i\\sqrt 3}{1-i}=\\frac{2\\left(\\frac12+i\\frac{\\sqrt 3}2\\right)}{\\sqrt 2\\left(\\frac{\\sqrt 2}2-i\\frac{\\sqrt 2}{2}\\right)}=\\sqrt 2\\frac{e^{i\\pi/3}}{e^{-i\\pi/4}}=\\sqrt2e^{i7\\pi/12}.$$\n\nOn en déduit que \n\n$$z_2=\\left(\\frac{1+i\\sqrt 3}{1-i}\\right)^{20}=(\\sqrt 2)^{20}e^{i\\frac{140\\pi}{12}}=2^{10}e^{i\\frac{35\\pi}3}.$$\n\nPuisque $35\\pi=3\\times5\\times 2\\pi+5\\pi$, on en déduit que\n\n$$z_2=2^{10}e^{i\\frac{5\\pi}3}=2^9(1-i\\sqrt 3)=512-i512\\sqrt 3.$$\n\n Avec la même méthode, on a \n\n$$(1+i)^{2000}=2^{1000}e^{i500\\pi}=2^{1000}.$$\n\nDe même,\n\n$$(i-\\sqrt 3)^{1000}=2^{1000}e^{i5000\\pi/6}.$$\n\nOr,\n\n$$2500=833\\times 3+1$$\n\nd'où\n\n$$(i-\\sqrt 3)^{1000}=2^{1000}e^{i4\\pi/3}.$$\n\nIl vient finalement :\n\n$$z_3=e^{-i4\\pi/3}=e^{i2\\pi/3}=-\\frac 12+\\frac{\\sqrt 3}2i.$$\n\n\n" }, { "question": " 16 - Identité du parallélogramme [Signaler une erreur] [Ajouter à ma feuille d'exos]Enoncé Soit $z_1$ et $z_2$ deux nombres complexes. Démontrer que \n\n$$|z_1+z_2|^2+|z_1-z_2|^2=2(|z_1|^2+|z_2|^2).$$", "answer": " On va utiliser que pour tout nombre complexe $z$, $|z|^2=z\\cdot \\bar z$ et développer. On a donc \n\n\\begin{align*}\n\n |z_1+z_2|^2+|z_1-z_2|^2&=(z_1+z_2) \\cdot \\overline{(z_1+z_2)}+(z_1-z_2) \\cdot \\overline{(z_1-z_2)}\\\\\n\n &=(z_1+z_2)\\cdot (\\overline{z_1}+\\overline{z_2})+(z_1-z_2)\\cdot (\\overline{z_1}-\\overline{z_2})\\\\\n\n &=z_1\\overline{z_1}+z_2\\overline{z_1}+z_1\\overline{z_2}+z_2\\overline{z_2}+z_1\\overline{z_1}-z_2\\overline{z_1}-z_1\\overline{z_2}+z_2\\overline{z_2}\\\\\n\n &=2z_1\\overline{z_1}+2z_2\\overline{z_2}\\\\\n\n &=2(|z_1|^2+|z_2|^2).\n\n\\end{align*}\n\nCette identité s'appelle l'identité du parallélogramme. Si $A,$ $B$ et $C$ sont les points d'abscisse respective $z_1,z_1+z_2,z_2$ de sorte\n\nque $OABC$ est un parallélogramme, alors l'égalité précédente dit que \n\n$$OB^2+AC^2=2OA^2+2OC^2.$$\n\nLa somme des carrés de la longueur des diagonales vaut la somme des carrés des longueurs des côtés.\n" }, { "question": " 17 - Exponentielle [Signaler une erreur] [Ajouter à ma feuille d'exos]Enoncé Résoudre l'équation $e^z=3\\sqrt 3-3i$.", "answer": " Posons $z=a+ib$, $a,b\\in\\mathbb R$. Alors \n\n$e^z=e^ae^{ib}$. Ceci nous incite à mettre $3\\sqrt 3-3i$ sous forme trigonométrique. On obtient\n\n$$|3\\sqrt 3-3i|=\\sqrt{27+9}=6.$$\n\nIl vient\n\n$$3\\sqrt 3-3i=6\\left(\\frac{\\sqrt 3}2-i\\frac 12\\right)=6e^{-i\\pi/6}.$$\n\nOn obtient alors $\\exp a=6$ et $b=-\\pi/6+2k\\pi,\\ k\\in\\mathbb Z$. Les solutions de l'équation\n\nsont donc les nombres complexes $\\ln(6)+i\\left(-\\frac\\pi6+2k\\pi\\right),\\ k\\in\\mathbb Z$.\n\n\n" }, { "question": " 18 - Réel positif [Signaler une erreur] [Ajouter à ma feuille d'exos]Enoncé Trouver les entiers $n\\in\\mathbb N$ tels que $(1+i\\sqrt 3)^n$ soit un réel positif.", "answer": " On commence par écrire $1+i\\sqrt 3$ sous forme exponentielle :\n\n$$1+i\\sqrt 3=2e^{i\\pi/3}.$$\n\nEn prenant la puissance $n$-ième, on trouve \n\n$$(1+i\\sqrt 3)^n=2^n e^{in\\pi/3}.$$\n\nCeci est un réel positif si et seulement si $\\sin(n\\pi/3)=0$ et $\\cos(n\\pi/3)\\geq 0$. Or, $\\sin(n\\pi/3)=0$ si et seulement si $n=3k$, $k\\in\\mathbb Z$.\n\nMais, pour ces valeurs de $n$, on a $\\cos(n\\pi/3)=\\cos(k\\pi)$, et ceci est positif si et seulement si $k$ est pair. Ainsi, les entiers qui conviennent sont les multiples de 6.\n" }, { "question": " 19 - Formule d'Euler [Signaler une erreur] [Ajouter à ma feuille d'exos]Enoncé Donner l'écriture exponentielle du nombre complexe suivant : \n\n\\begin{equation*}\n\n \\frac{1-e^{i\\frac{\\pi}{3}}}{1+e^{i\\frac{\\pi}{3}}}.\n\n\\end{equation*}", "answer": " On va utiliser les formules d'Euler pour faire les calculs, en utilisant que $1=e^{i0}$, et en factorisant par l'angle moitié. On a donc\n\n\\begin{align*}\n\n \\frac{1-e^{i\\frac{\\pi}{3}}}{1+e^{i\\frac{\\pi}{3}}}&= \\frac{e^{i0}-e^{i\\frac{\\pi}{3}}}{e^{i0}+e^{i\\frac{\\pi}{3}}}\\\\\n\n&=\\frac{e^{i\\frac{\\pi}6}\\left(e^{-i\\frac\\pi6}-e^{i\\frac{\\pi}{6}}\\right)}\n\n{e^{i\\frac{\\pi}6}\\left(e^{-i\\frac\\pi6}+e^{i\\frac{\\pi}{6}}\\right)}\\\\\n\n&=\\frac{-2i\\sin(\\pi/6)}{2\\cos(\\pi/6)}\\\\\n\n&=\\frac{-\\sqrt 3}3i\\\\\n\n&=\\frac{\\sqrt 3}3e^{i\\frac{3\\pi}2}.\n\n\\end{align*}\n" }, { "question": " 20 - Forme exponentielle et formule d'Euler [Signaler une erreur] [Ajouter à ma feuille d'exos]Enoncé Soient $a,b\\in]0,\\pi[$. Écrire sous forme exponentielle les nombres complexes suivants :\n\n$$\\mathbf 1.\\ z_1=1+e^{ia}\\quad \\mathbf 2.\\ z_2=1-e^{ia}\\quad \\mathbf 3.\\ z_3=e^{ia}+e^{ib}\\quad \\mathbf 4. z_4=\\frac{1+e^{ia}}{1+e^{ib}}.$$", "answer": " On utilise la méthode suivante : dans une somme ou une différence de deux complexes de module 1, $e^{ix}\\pm e^{iy}$, on met en facteur $e^{i\\frac{x+y}2}$ puis on utiise les formules d'Euler.\n\n\n $$z_1=e^{ia/2}\\left(e^{-ia/2}+e^{ia/2}\\right)=2\\cos(a/2)e^{ia/2}.$$\n\nDe plus, $\\cos(a/2)>0$ car $a\\in ]0,\\pi[$, et on a bien obtenu l'écriture trigonométrique du complexe.\n\n La même méthode donne\n\n\\begin{align*}\n\nz_2&=e^{ia/2}\\left(e^{-ia/2}-e^{ia/2}\\right)\\\\\n\n&=-2i\\sin(a/2)e^{ia/2}\\\\\n\n&=2\\sin(a/2)e^{i\\left(\\frac a2-\\frac\\pi 2\\right)}.\n\n\\end{align*}\n\n On a cette fois \n\n$$z_3=e^{i\\frac{a+b}2}\\left(e^{i\\frac{a-b}2}+e^{i\\frac{b-a}2}\\right)=2\\cos\\left(\\frac{a-b}2\\right)e^{i\\frac {a+b}2}.$$\n\nC'est bien une forme trigonométrique de $z_3$, car $-\\frac \\pi 2<\\frac{a-b}2<\\frac \\pi 2$ et donc $\\cos\\left(\\frac{a-b}2\\right)>0$.\n\n En utilisant le résultat de la première question, on trouve\n\n$$z_4=\\frac{2\\cos(a/2)e^{ia/2}}{2\\cos(b/2)e^{ib/2}}=\\frac{\\cos(a/2)}{\\cos(b/2)}e^{i\\frac{a-b}2}.$$\n\n\n" }, { "question": " 21 - Transformation de Cayley [Signaler une erreur] [Ajouter à ma feuille d'exos]Enoncé Soient $z$ et $z'$ deux nombres complexes de module 1 tels que $zz'\\neq -1$. Démontrer que \n\n$\\frac{z+z'}{1+zz'}$ est réel, et préciser son module.", "answer": " On écrit $z=e^{i\\theta}$, $z'=e^{i\\theta'}$ et on utilise les formules d'Euler en mettant en facteur\n\n$e^{i\\frac{\\theta+\\theta'}2}$ en facteur au numérateur et au dénominateur. Il vient\n\n\\begin{eqnarray*}\n\n\\frac{z+z'}{1+zz'}&=&\\frac{e^{i\\theta}+e^{i\\theta'}}{1+e^{i(\\theta+\\theta')}}\\\\\n\n&=&\\frac{e^{i\\frac{\\theta-\\theta'}2}+e^{-i\\frac{\\theta-\\theta'}2}}{e^{i\\frac{\\theta+\\theta'}2}+e^{-i\\frac{\\theta+\\theta'}2}}\\\\\n\n&=&\\frac{\\cos\\left(\\frac{\\theta-\\theta'}2\\right)}{\\cos\\left(\\frac{\\theta+\\theta'}2\\right)}.\n\n\\end{eqnarray*}\n\nOn obtient bien un nombre réel, de module $\\left|\\frac{\\cos\\left(\\frac{\\theta-\\theta'}2\\right)}{\\cos\\left(\\frac{\\theta+\\theta'}2\\right)}\\right|$.\n" }, { "question": " 22 - Une inégalité sur les modules [Signaler une erreur] [Ajouter à ma feuille d'exos]Enoncé \n Soit $Z$ un nombre complexe. Démontrer que \n\n$$1+|Z|^2+2\\Re e(Z)\\geq 0.$$\n\n Soit $z$ et $w$ deux nombres complexes. Démontrer que l'on a \n\n$$|z-w|^2\\leq (1+|z|^2)(1+|w|^2).$$\n\n", "answer": " \n Posons $Z=x+iy$, avec $x$ et $y$ des réels. Alors\n\n\\begin{align*}\n\n1+|Z|^2+2\\Re e(Z)&=1+x^2+y^2+2x\\\\\n\n&=(1+x)^2+y^2\\geq 0.\n\n\\end{align*}\n\n Posons $A=(1+|z|^2)(1+|w|^2)-|z-w|^2$. On a \n\n\\begin{align*}\n\nA&=(1+z\\bar z)(1+w\\bar w)-(z-w)\\overline{z-w}\\\\\n\n&=1+z\\bar z+b\\bar w+zw\\overline{zw}-z\\bar z-w\\bar w+z\\bar w+\\bar z w\\\\\n\n&=1+zw\\overline{zw}+z\\bar w+\\bar z w\\\\\n\n&=1+|z\\bar w|^2+2\\Re e(z\\bar w).\n\n\\end{align*}\n\nOn conclut alors en appliquant le résultat de la question précédente à $Z=z\\bar w$.\n\n\n" }, { "question": " 23 - Le même module [Signaler une erreur] [Ajouter à ma feuille d'exos]Enoncé Déterminer les nombres complexes non nuls $z$ tels que $z$, $\\frac 1z$ et $1-z$ aient le même module.", "answer": " De $|z|=\\left|\\frac 1z\\right|$, on tire que $|z|^2=1$, donc que $|z|=1$, c'est-à-dire que $z=e^{i\\theta}$, où $\\theta\\in[0,2\\pi[$.\n\nCalculons maintenant le module de $1-z$. \n\nOn écrit \n\n$$1-z=1-e^{i\\theta}=e^{i\\theta/2}\\left(e^{-i\\theta/2}-e^{i\\theta/2}\\right)=-2i\\sin(\\theta/2)e^{i\\theta/2}.$$\n\nLe module de $1-z$ vaut donc $1$ si et seulement si $|\\sin(\\theta/2)|=1/2$. L'équation $\\sin(\\theta/2)=1/2$, avec \n\n$\\theta\\in[0,2\\pi[$ donne $\\theta=\\pi/3$ ou $\\theta=5\\pi/3$. L'équation $\\sin(\\theta/2)=-1/2$ avec $\\theta\\in[0,2\\pi[$ n'a pas de solutions puisque, si $\\theta\\in[0,2\\pi[$, $\\theta/2\\in [0,\\pi[$ et alors $\\sin(\\theta/2)\\in[0,1].$\n\nL'ensemble des solutions est donc $\\{e^{i\\pi/3},e^{i5\\pi/3}\\}$.\n" }, { "question": " 24 - Homographie [Signaler une erreur] [Ajouter à ma feuille d'exos]Enoncé Soit $z$ un nombre complexe, $z\\neq 1$. Démontrer que :\n\n$$|z|=1\\iff \\frac{1+z}{1-z}\\in i\\mathbb R.$$", "answer": " Supposons d'abord que $|z|=1$. Alors $z$ s'écrit $z=e^{i\\theta}$, avec $\\theta\\in\\mathbb R\\backslash 2\\pi\\mathbb Z$.\n\nOn peut alors écrire :\n\n$$\\frac{1+z}{1-z}=\\frac{1+e^{i\\theta}}{1-e^{i\\theta}}=\\frac{e^{i\\theta/2}(e^{-i\\theta/2}+e^{i\\theta/2})}{e^{i\\theta/2}(e^{-i\\theta/2}-e^{i\\theta/2})}=\\frac{2\\cos(\\theta/2)}{-2i\\sin(\\theta/2)}=i\\frac{\\cos(\\theta/2)}{\\sin(\\theta/2)}$$\n\nqui est bien un élément de $i\\mathbb R$. Remarquons que l'on a le droit d'effectuer ce calcul car $\\sin(\\theta/2)$ ne s'annule pas.\n\nRéciproquement, supposons que $\\frac{1+z}{1-z}=ia$, avec $a$ un réel. On va exprimer $z$ en fonction de $a$, \n\npuis calculer son module. Il vient :\n\n$$\\frac{1+z}{1-z}=ia\\iff 1+z=ia(1-z)\\iff z(1+ia)=-1+ia\\iff z=\\frac{-1+ia}{1+ia}.$$\n\nOn en déduit que\n\n$$|z|=\\left|\\frac{-1+ia}{1+ia}\\right|=\\frac{\\sqrt{1+a^2}}{\\sqrt{1+a^2}}=1,$$\n\nce qui prouve la réciproque.\n" }, { "question": " 25 - Sommes d'arctan [Signaler une erreur] [Ajouter à ma feuille d'exos]Enoncé \n Quelle est la forme algébrique de $(1+i)(1+2i)(1+3i)$?\n\n En déduire la valeur de $\\arctan(1)+\\arctan(2)+\\arctan(3)$.\n\n", "answer": " \n Un calcul rapide montre que $(1+i)(1+2i)(1+3i)=-10$.\n\n Notons $\\theta_1$, $\\theta_2$ et $\\theta_3$ des arguments respectifs de $1+i$, $1+2i$ et $1+3i$. On peut toujours choisir $\\theta_k\\in[0,\\pi/2]$, $k=1,2,3$. Notons aussi $r_k$ le module de $1+ki$, $k=1,2,3$.\n\nOn a $r_k \\sin(\\theta_k)=k$ et $r_k \\cos(\\theta_k)=1$ de sorte que $\\tan\\theta_k=k$. Puisque $\\theta_k\\in [0,\\pi/2]$, on a donc \n\n$$\\theta_k=\\arctan(k).$$\n\nAinsi, on a aussi \n\n$$(1+i)(1+2i)(1+3i)=r_1r_2r_3 e^{i\\big(\\arctan(1)+\\arctan(2)+\\arctan(3)\\big)}.$$\n\nAinsi, $\\arctan(1)+\\arctan(2)+\\arctan(3)$ est un argument de $(1+i)(1+2i)(1+3i)$ et d'après le calcul effectué à la question précédente, on a\n\n$$\\arctan(1)+\\arctan(2)+\\arctan(3)=\\pi\\ [2\\pi].$$\n\nOr, \n\n$$0\\leq \\arctan(1)+\\arctan(2)+\\arctan(3)\\leq \\frac{3\\pi}2.$$\n\nAinsi, on obtient que\n\n$$\\arctan(1)+\\arctan(2)+\\arctan(3)=\\pi.$$\n\n\n" }, { "question": " 26 - Homographie du cercle unité [Signaler une erreur] [Ajouter à ma feuille d'exos]Enoncé Soit $U=\\left\\{z\\in\\mathbb C:\\ |z|=1\\right\\}$ le cercle unité et soit $a\\notin U$. Démontrer que $f_a(z)=\\frac{z+a}{1+\\bar a z}$ définit une bijection de $U$ sur lui-même et donner l'expression de $f_a^{-1}$.", "answer": " On va commencer par vérifier que $f_a$ est bien définie sur $U$. En effet, puisque $a\\notin U$, $|az|=|a|\\neq 1$ pour tout $z\\in U$ et donc $1+\\bar a z\\neq 0$. \n\nEnsuite, on va démontrer que, pour tout $z\\in U$, $f_a(z)\\in U$. Cela revient à démontrer que $|z+a|=|1+\\bar a z|$. Mais, puisque $|z|=1$, $\\frac 1z=\\bar z$ et on a\n\n$$|1+\\bar a z|=\\left|z\\left(\\frac 1z+\\bar a\\right)\\right|=|z|\\times |\\bar z+\\bar a|=|z+a|.$$\n\nAinsi, on a bien $f_a(U)\\subset U$.\n\nDémontrons enfin que $f_a$ est une bijection de $U$ sur lui-même et calculons la bijection réciproque. Soit $w\\in U$ et essayons de résoudre l'équation $f_a(z)=w$, avec $z\\in U$. On a \n\n\\begin{align*}\n\nf_a(z)=w&\\iff \\frac{z+a}{1+\\bar a z}=w\\\\\n\n&\\iff z+a=w+\\bar a w z\\\\\n\n&\\iff z(1-\\bar a w)=w-a.\n\n\\end{align*}\n\nPuisque $1-\\bar aw\\neq 0$ (le raisonnement est le même que ci-dessus), on a donc\n\n$$f_a(z)=w\\iff z=\\frac{w-a}{1-\\bar a w}=f_{-a}(w)$$\n\net ce dernier élément est bien dans $U$. Ainsi, $f_a$ réalise une bijection de $U$ dans lui-même, de bijection réciproque $f_{-a}$.\n" }, { "question": " 27 - Module de la somme et de la différence [Signaler une erreur] [Ajouter à ma feuille d'exos]Enoncé Soient $z=\\rho e^{i\\theta}$ et $z'=\\rho'e^{i\\theta'}$ deux nombres complexes non nuls. Démontrer que \n\n$$|z+z'|=|z-z'|\\Longleftrightarrow{\\theta'=\\theta+\\frac{\\pi}{2}[\\pi]}.$$", "answer": " L'idée est de passer au carré, et de développer.\n\n\\begin{align*}\n\n|z+z'|=|z-z'|&\\iff |z+z'|^2=|z-z'|^2\\\\\n\n&\\iff (z+z')\\overline{(z+z')}=(z-z')\\overline{(z-z')}\\\\\n\n&\\iff |z|^2+|z'|^2+(z\\overline{z'}+\\overline{z}z')=|z|^2+|z'|^2-(z\\overline{z'}+\\overline{z}z')\\\\\n\n&\\iff |z|^2+|z'|^2+2\\Re e(z\\overline{z'})=|z|^2+|z'|^2-2\\Re e(z\\overline{z'}).\n\n\\end{align*}\n\nEn simplifiant par les termes égaux, ceci est donc équivalent à \n\n$$\\Re e(z\\overline{z'})=0.$$\n\nOr, $z\\overline{z'}=\\rho\\rho'e^{i(\\theta-\\theta')}.$ Ceci a une partie réelle nulle si et seulement si $\\cos(\\theta-\\theta')=0$, c'est-à-dire\n\nsi et seulement si $\\theta'=\\theta+\\frac\\pi2\\ [\\pi]$.\n" }, { "question": " 28 - Un ensemble de complexes [Signaler une erreur] [Ajouter à ma feuille d'exos]Enoncé On rappelle que $j=e^{2i\\pi/3}.$ On note $E=\\{z\\in\\mathbb C:\\ \\exists(a,b)\\in\\mathbb Z^2,\\ z=a+jb\\}.$ \n\n\n Soit $z=a+jb\\in E$, avec $(a,b)\\in\\mathbb Z^2.$ Montrer que $|z|=1$ si et seulement si \n\n$$(2a-b)^2+3b^2=4.$$\n\n En déduire explicitement tous les éléments de $U=\\{z\\in E:\\ |z|=1\\}$\n\nen fonction de $1$, de $-1$, de $j$ et $-j.$\n\n", "answer": " \n On écrit $z=a+jb=\\left(a-\\frac b2\\right)+ib\\frac{\\sqrt 3}2.$ On a donc\n\n$$|z|^2=\\left(a-\\frac b2\\right)^2+\\frac{3b^2}4.$$\n\nOn a donc \n\n$$|z|^2=1\\iff 4|z|^2=4\\iff (2a-b)^2+3b^2=4.$$\n\n Puisque $(2a-3b)^2$ est un entier et que $3b^2$ est un multiple de $3$, l'équation précédente ne peut avoir des solutions que si \n\n\n $3b^2=0$ et $(2a-b)^2=4$. Ceci entraîne $b=0$ et $a=\\pm 1$. On obtient donc les deux nombres complexes $1$ et $-1$.\n\n $3b^2=3$ et $(2a-b)^2=1.$ On distingue alors deux sous-cas : \n\n\n $b=1$, dans ce cas, $2a-1=1$ ou $2a-1=-1$ donne les deux solutions $a=1$ et $a=0$ : on obtient les nombres complexes $1+j$ et $j$. \n\n $b=-1$, dans ce cas, $2a+1=1$ ou $2a+1=-1$ donne les deux solutions $a=0$ et $a=-1$ : on obtient les nombres complexes $-j$ et $-1-j$.\n\n\n\n\nFinalement, on a prouvé que $U=\\{1,-1,j,-j,1+j,-1-j\\}.$\n\n\n\n" }, { "question": " 29 - Entiers somme de deux carrés [Signaler une erreur] [Ajouter à ma feuille d'exos]Enoncé On dit qu'un entier naturel $N$ est somme de deux carrés s'il existe deux entiers naturels $a$ et $b$ de sorte que $N=a^2+b^2$.\n\n\n Écrire un algorithme permettant de déterminer si un entier naturel $N$ est somme de deux carrés.\n\n On souhaite prouver que, si $N_1$ et $N_2$ sont sommes de deux carrés, alors leur produit $N_1N_2$ est aussi somme de deux carrés. Pour cela, on écrit $N_1=a^2+b^2$ et $N_2=c^2+d^2$, et on introduit $z_1=a+ib$, $z_2=c+id$. Comment écrire $N_1$ et $N_2$ en fonction de $z_1$ et $z_2$?\n\n En déduire que $N_1N_2$ est somme de deux carrés.\n\n Démontrer que si $N$ est somme de deux carrés, alors pour tout entier $p\\geq 1$, $N^p$ est somme de deux carrés.\n\n", "answer": " \n Voici un algorithme qui convient (mais qui n'est pas du tout optimal!) :\n\n Variables : N,a,b,compteur\nAlgorithme :\n  Lire N\n  compteur=0\n  Pour a allant de 0 à N faire\n     Pour b allant de 0 à N faire\n        Si (N=a*a+b*b) alors compteur=compteur+1; Finsi.\n     Fin pour\n  Fin pour\n  Si (compteur>0) \n      alors Afficher N est somme de deux carrés\n      sinon Afficher N n'est pas somme de deux carrés.\n\n Il est facile de remarquer que $N_1=|z_1|^2$ et que $N_2=|z_2|^2$.\n\n On écrit, en utilisant le résultat de la question précédente, que \n\n$$N_1N_2=|z_1z_2|^2.$$\n\nOr, \n\n$$z_1z_2=(ac-bd)+i(ad+bc)=e+if$$\n\navec $e=ac-bd$ et $f=ad+bc$ qui sont des entiers. Ainsi, \n\n$$N_1N_2=e^2+f^2$$\n\nest somme de deux carrés.\n\n Prouvons par récurrence sur $p$ la propriété $\\mathcal P(p)$ : \"$N^p$ est somme de deux carrés\". Clairement, la propriété $\\mathcal P(1)$ est vraie. Soit $p\\geq 1$ tel que $\\mathcal P(p)$ soit\n\nvérifiée. Alors $N^{p+1}=N^p\\times N$ est le produit de deux nombres qui sont chacun somme de deux carrés. D'après la question précédente, $N^{p+1}$ est lui aussi somme de deux carrés, et\n\n$\\mathcal P(p+1)$ est vraie. Par le principe de récurrence, $\\mathcal P(p)$ est vraie pour tout entier $p\\geq 1$.\n\n\n" }, { "question": " 30 - Automorphisme du disque [Signaler une erreur] [Ajouter à ma feuille d'exos]Enoncé Soit $a$ un complexe de module $|a|<1$.\n\n\n Démontrer que, pour tout nombre complexe $z$ tel que $1-\\bar a z\\neq 0$,\n\n$$1-\\left|\\frac{z-a}{1-\\bar{a}z}\\right|^2 = \\frac{(1-|a|^2)(1-|z|^2)}{|1-\\bar a z|^2}.$$\n\n Déterminer les nombres complexes\n\n$z$ vérifiant $\\displaystyle \\left|\\frac{z-a}{1-\\bar{a}z}\\right|\\leq 1.$\n\n", "answer": " \n Il suffit de développer les modules au carré. Précisément, on a :\n\n\\begin{eqnarray*}\n\n1-\\left|\\frac{z-a}{1-\\bar{a}z}\\right|^2&=&\\frac{|1-\\bar a z|^2-|z-a|^2}{|1-\\bar a z|^2}\\\\\n\n&=&\\frac{1-\\bar a z-a\\bar z+|a|^2|z|^2-|z|^2-|a|^2+\\bar a z+a\\bar z}{|1-\\bar a z|^2}\\\\\n\n&=&\\frac{1+|a|^2|z|^2-|z|^2-|a|^2}{|1-\\bar a z|^2}\\\\\n\n&=& \\frac{(1-|a|^2)(1-|z|^2)}{|1-\\bar a z|^2}.\n\n\\end{eqnarray*}\n\n On commence par remarquer que :\n\n$$\\left|\\frac{z-a}{1-\\bar{a}z}\\right|\\leq 1 \\iff \\left|\\frac{z-a}{1-\\bar{a}z}\\right|^2\\leq 1.$$\n\nEnsuite, on a d'après la question précédente \n\n$$1-\\left|\\frac{z-a}{1-\\bar{a}z}\\right|^2 = \\frac{(1-|a|^2)(1-|z|^2)}{|1-\\bar a z|^2}.$$\n\nAinsi, on a l'équivalence :\n\n$$\\left|\\frac{z-a}{1-\\bar{a}z}\\right|\\leq 1 \\iff\\frac{(1-|a|^2)(1-|z|^2)}{|1-\\bar a z|^2}\\geq 0.$$\n\nOr, $1-|a|^2\\geq 0$ et $|1-\\bar a z|^2\\geq 0$. On a donc la propriété voulue si et seulement si\n\n$$1-|z|^2\\geq 0\\iff |z|\\leq 1.$$\n\n\n" }, { "question": " 31 - Inégalité triangulaire itérée, et cas d'égalité [Signaler une erreur] [Ajouter à ma feuille d'exos]Enoncé \n Justifier que, pour tout nombre complexe $z$, on a $\\Re e(z)\\leq |z|$. Dans quel cas a-t-on égalité?\n\n Démontrer que pour tout couple $(z_1,z_2)$ de nombres complexes, on a $|z_1+z_2|\\leq |z_1|+|z_2|$. \n\n On suppose de plus que $z_1$ et $z_2$ sont des nombres complexes non nuls. Justifier que l'inégalité précédente est une égalité si et seulement s'il existe un réel positif $\\lambda$ tel que $z_2=\\lambda z_1$.\n\n Démontrer que pour tout $n$-uplet $(z_1,\\dots,z_n)$ de nombres complexes, on a \n\n$$|z_1+\\cdots+z_n|\\leq |z_1|+\\cdots+|z_n|.$$\n\n Démontrer que si $z_1,\\dots,z_n$ sont tous non nuls, alors l'inégalité précédente est une égalité si et seulement si il existe des réels positifs $\\lambda_1,\\dots,\\lambda_n$ tels que, pour tout $k=1,\\dots,n$, on a $z_k=\\lambda_k z_1$.\n\n", "answer": " \n Écrivons le nombre complexe $z$ sous la forme $z=x+iy$. Alors on a\n\n$$\\textrm{Re}(z)\\leq |\\textrm{Re}(z)|=|x|= \\sqrt{x^2}\\leq \\sqrt{x^2 +y^2},$$\n\nla dernière inégalité étant une conséquence de la croissance de la fonction racine carrée.\n\nPour déterminer les cas d'égalité, la méthode est simple : on repère toutes les inégalités utilisées, et on aura\n\négalité si et seulement si ces inégalités sont des égalités. Ici, les deux inégalités utilisées sont \n\n$\\textrm{Re}(z)\\leq |\\textrm{Re}(z)|$ et $\\sqrt{x^2}\\leq \\sqrt{x^2+y^2}$. On a donc égalité si et seulement si $|x|=x$ et $\\sqrt{x^2+y^2}=\\sqrt{x^2}$, c'est-à-dire si et seulement si $x\\geq 0$ et $y=0$. On a donc égalité si et seulement si $z$ est un réel positif ou nul.\n\n C'est une question de cours! Quand on travaille avec des modules, on a tout intérêt à se souvenir que ce qui est facile à calculer, c'est le carré du module. Tout étant positif, il suffit ici de démontrer que $|z_1+z_2|^2\\leq (|z_1|+|z_2|)^2$. Mais on a \n\n\\begin{eqnarray*}\n\n|z_1+z_2|^2&=&(z_1+z_2)\\times\\overline{(z_1+z_2)}\\\\\n\n&=&|z_1|^2+|z_2|^2+z_1\\bar{z_2}+z_2\\bar{z_1}\\\\\n\n&=&|z_1|^2+|z_2|^2+2\\textrm{Re}(\\bar{z_1}z_2),\\\\\n\n(|z_1|+|z_2|)^2&=&|z_1|^2+|z_2|^2+2|z_1z_2|.\n\n\\end{eqnarray*}\n\nOn conclut en appliquant le résultat de la première question à $z=\\bar{z_1}z_2$, puisque $|z_1z_2|=|\\bar{z_1}z_2|$.\n\n C'est une question nettement plus difficile. On étudie la preuve de la question précédente pour savoir où on a utilisé des inégalités plutôt que des égalités. La seule inégalité utilisée est $\\textrm{Re}(\\bar{z_1}z_2)\\leq |\\overline{z_1}z_2|$. D'après le résultat de la question A.I.1., on a égalité si et seulement si $\\bar z_1z_2$ est un réel positif ou nul. Puisque $z_1,z_2$ ne sont pas nuls, on a égalité si et seulement s'il existe $\\mu>0$ tel que $\\overline{z_1}z_2=\\mu$. Il faut maintenant passer au résultat demandé. Pour cela, on peut aussi écrire $z_1=re^{i\\theta}$ de sorte que $\\bar{z_1}=re^{-i\\theta}$ ce qui entraine encore que \n\n$$\\frac{1}{\\bar z_1}=\\frac 1r e^{i\\theta}=\\frac1{r^2}z_1.$$\n\nAinsi, on a démontré qu'on avait égalité si et seulement s'il existe $\\mu>0$ tel que \n\n$$z_2=\\frac{\\mu}{r^2}z_1.$$\n\nPosant $\\lambda=\\mu/r^2$, on obtient le résultat. Il s'interprète en disant que $z_1$ et $z_2$ ont le même argument modulo $2\\pi$.\n\n On va procéder par récurrence. Pour $n\\geq 1$, considérons $\\mathcal P(n)$ la propriété suivante :\n\n$$\"\\forall z_1,\\dots,z_n\\in\\mathbb C,\\ |z_1+\\dots+z_n|\\leq |z_1|+\\dots+|z_n|.\"$$\n\nLa propriété $\\mathcal P(1)$ est trivialement vraie et la propriété $\\mathcal P(2)$ a été vérifiée lors d'une question précédente. Soit $n\\geq 2$ tel que $\\mathcal P(n)$ est vraie et prouvons $\\mathcal P(n+1)$. Soit $z_1,\\dots,z_{n+1}$ des nombres complexes et posons $Z_1=z_1+\\dots+z_n$, $Z_2=z_{n+1}$. Puisque $\\mathcal P(2)$ est vraie, on sait que \n\n$$\\left|\\sum_{k=1}^n z_k\\right|=|Z_1+Z_2|\\leq |Z_1|+|Z_2|.$$\n\nOn utilise maintenant $\\mathcal P(n)$ pour trouver que \n\n$$|Z_1|\\leq \\sum_{k=1}^n |z_k|.$$\n\nOn a donc bien prouvé $\\mathcal P(n+1)$ et par le principe de récurrence, le résultat est démontré pour tout entier $n$.\n\n Là encore, une récurrence s'impose et là encore, elle est un peu délicate. L'hypothèse de récurrence $\\mathcal P(n)$ est cette fois :\n\n$$\"\\forall z_1,\\dots,z_n\\in\\mathbb C^*,\\ \\left|\\sum_{k+1}^n z_k\\right|=\\sum_{k=1}^n |z_k|\\iff \\forall k\\in\\{1,\\dots,n\\},\\ \\exists \\lambda_k\\in\\mathbb R_+,\\ z_k=\\lambda_k z_1.\"$$\n\nL'initialisation $\\mathcal P(1)$, $\\mathcal P(2)$ ne pose pas de problèmes. Soit $n\\geq 2$ tel que $\\mathcal P(n)$ est vraie et prouvons $\\mathcal P(n+1)$. Comme précédemment, considérons $z_1,\\dots,z_{n+1}$ des nombres complexes non-nuls et posons $Z_1=z_1+\\dots+z_n$, $Z_2=z_{n+1}$. De la chaine d'inégalités \n\n$$\\left|\\sum_{k=1}^n z_k\\right|=|Z_1+Z_2|\\leq |Z_1|+|Z_2|\\leq \\sum_{k=1}^n |z_k|+|z_{n+1}|=\\sum_{k=1}^{n+1}|z_k|,$$ on déduit que l'on a égalité si et seulement si $|Z_1+Z_2|=|Z_1|+|Z_2|$ et $\\left|\\sum_{k=1}^n z_k\\right|=\\sum_{k=1}^n |z_k|$. Ici, il faut faire attention! On ne peut pas directement appliquer $\\mathcal P(2)$ à $|Z_1+Z_2|$ car pour le moment, rien ne nous dit que $Z_1\\neq 0$. On commence donc par appliquer $\\mathcal P(n)$ à \n\n$\\left|\\sum_{k=1}^n z_k\\right|=\\sum_{k=1}^n |z_k|$ pour en déduire que, pour $k=1,\\dots,n$, il existe $\\lambda_k\\in\\mathbb R_+$ tel que $z_k=\\lambda_k z_1$. On en déduit que $Z_1=(1+\\lambda_2+\\dots+\\lambda_n)z_1$ est non-nul, et on peut donc appliquer $\\mathcal P(2)$ à $|Z_1+Z_2|=|Z_1|+|Z_2|$. Il existe donc $\\mu\\in\\mathbb R_+$ tel que $z_{n+1}=\\mu Z_1$. Posant $\\lambda_{n+1}=\\mu(1+\\lambda_2+\\dots+\\lambda_n)$, on en déduit que $\\mathcal P(n+1)$ est vraie. Il reste à écrire la conclusion du raisonnement par récurrence. \n\n\n" }, { "question": " 32 - Égalité dans l'inégalité triangulaire [Signaler une erreur] [Ajouter à ma feuille d'exos]Enoncé Soient $z_1,\\dots,z_n$ des nombres complexes tous non nuls. Donner une condition nécessaire et suffisante pour que\n\n$$|z_1+\\dots+z_n|=|z_1|+\\dots+|z_n|.$$", "answer": " On va prouver que la propriété est vraie si et seulement s'il existe des réels positifs $\\lambda_i$ tels que $z_i=\\lambda_i z_1$.\n\nUn sens est facile. En effet, si $z_i=\\lambda_i z_1$, alors\n\n$$|z_1+\\dots+z_n|=|z_1|\\times|1+\\lambda_2+\\dots+\\lambda_n|=|z_1|+\\lambda_2|z_1|+\\dots+\\lambda_n |z_1|=|z_1|+\\dots+|z_n|.$$\n\nRéciproquement, on va prouver par récurrence sur $n$ que si \n\n$$|z_1+\\dots+z_n|=|z_1|+\\dots+|z_n|,$$\n\nalors il existe des réels positifs $\\lambda_i$, $1\\leq i\\leq n$ tels que $z_i=\\lambda_i z_1$.\n\nOn commence par traiter le cas $n=2$, et on suppose que $|z_1+z_2|=|z_1|+|z_2|$.\n\nNotons $u=z_1/z_2$. Alors on a $|1+u|=1+|u|$, et en écrivant $u=x+iy$, on obtient\n\n$$|1+u|^2=(1+x)^2+y^2=1+|u|^2+2x\\textrm{ et }(1+|u|)^2=1+2|u|+|u|^2.$$\n\nOn a donc $x=|u|$, ce qui entraine que $y=0$ et que $u$ est un réel positif. Le cas $n=2$ est donc prouvé.\n\nSupposons maintenant la propriété prouvée au rang $n-1$ et prouvons-la au rang $n$. On commence par remarquer que\n\n$$|z_1+\\dots+z_{n-1}|=|z_1|+\\dots+|z_{n-1}|.$$\n\nEn effet, si on avait $|z_1+\\dots+z_{n-1}|<|z_1|+\\dots+|z_{n-1}|$, on aurait aussi \n\n$$|z_1+\\dots+z_n|\\leq |z_1+\\dots+z_{n-1}|+|z_n|<|z_1|+\\dots+|z_{n-1}|+|z_n|,$$\n\nce qui contredit l'hypothèse initiale. \n\nPar hypothèse de récurrence, on sait que pour $i\\in\\{1,\\dots,n-1\\}$, il existe $\\lambda_i>0$ tel que\n\n$z_i=\\lambda_i z_1$. Mais alors il vient\n\n$$|z_1+\\dots+z_n|=|(1+\\dots+\\lambda_{n-1})z_1+z_n|=(1+\\dots+\\lambda_{n-1})|z_1|+|z_n|.$$\n\nOn applique alors le cas $n=2$, et on trouve que $z_n=\\mu_n(1+\\dots+\\lambda_{n-1})z_1$\n\navec $\\mu_n>0$. On a le résultat voulu, quitte à poser $\\lambda_n=\\mu_n(1+\\dots+\\lambda_{n-1})$" } , { "question": " 1 - Équations du premier degré [Signaler une erreur] [Ajouter à ma feuille d'exos]Enoncé Résoudre les équations suivantes, d'inconnue $z\\in\\mathbb C$ :\n\n$$\n\n\\begin{array}{lll}\n\n{\\mathbf 1.}\\ z+2i=iz-1&\\quad&{\\mathbf 2.}\\ (3+2i)(z-1)=i\\\\\n\n{\\mathbf 3.}\\ (2-i)z+1=(3+2i)z-i&\\quad&{\\mathbf 4.}\\ (4-2i)z^2=(1+5i)z.\n\n\\end{array}$$\n\nOn écrira les solutions sous forme algébrique.", "answer": " \n L'équation est équivalente à \n\n$$z(1-i)=-1-2i\\iff z=\\frac{-1-2i}{1-i}=\\frac{(-1-2i)(1+i)}{(1-i)(1+i)}=\\frac12-\\frac 32i.$$\n\n De la même façon, on a \n\n$$z-1=\\frac{i}{3+2i}=\\frac{i(3-2i)}{(3+2i)(3-2i)}=\\frac 2{13}+\\frac 3{13}i.$$\n\nOn en déduit que \n\n$$z=\\frac{15}{13}+\\frac 3{13}i.$$\n\n L'équation cette fois est équivalente à \n\n\\begin{align*}\n\n(2-i-3-2i)z=-i-1&\\iff (-1-3i)z=-i-1\\\\\n\n&\\iff z=\\frac{-i-1}{-1-3i}=\\frac{(-i-1)(-1+3i)}{(-1-3i)(-1+3i)}=\\frac{2}5-\\frac15 i.\n\n\\end{align*}\n\n On distingue deux cas. D'abord, $z=0$ est solution de l'équation. Si $z\\neq 0$, alors on peut simplifier par $z$ et l'équation est équivalente à \n\n$$(4-2i)z=1+5i\\iff z=\\frac{1+5i}{4-2i}=\\frac{(1+5i)(4+2i)}{(4-2i)(4+2i)}=\\frac{-6+22i}{20}=\\frac{-3}{10}+\\frac{11}{10}i.$$\n\nL'équation admet donc deux solutions, $z=0$ et $z=\\frac{-3}{10}+\\frac{11}{10}i$.\n\n\n" }, { "question": " 2 - Équations avec des conjuguées. [Signaler une erreur] [Ajouter à ma feuille d'exos]Enoncé Résoudre les équations suivantes :\n\n$$\n\n\\begin{array}{lll}\n\n\\displaystyle{\\mathbf 1.}\\ 2z+i=\\overline z+1&\\displaystyle{\\mathbf 2.}\\ 2z+\\overline z=2+3i&\\displaystyle{\\mathbf 3.}\\ 2z+2\\overline z=2+3i.\n\n\\end{array}$$", "answer": " \n On écrit $z=x+iy$ avec $x,y\\in\\mathbb R$ de sorte que $2z+i=2x+i(2y+1)$ et $\\overline z+1=(x-iy)+1=(x+1)-iy$. Puisque deux nombres complexes sont égaux si et seulement s'il ont même partie réelle et même partie imaginaire, l'équation est équivalente à $2x=x+1$ et $2y+1=-y$. On trouve alors $x=1$ et $y=-1/3$. L'équation admet une unique solution, $z=1-\\frac 13 i$.\n\n On reprend la même méthode. Alors $2z+\\bar z=3x+iy=2+3i$ si et seulement si $x=\\frac 23$ et $y=3$. L'équation admet donc une unique solution, $z=\\frac 23+3i$.\n\n On procède de la même façon, mais cette fois on constate que $2z+2\\bar z=4x$ est réel : il ne pourra jamais être égal à $2+3i$. L'équation n'admet pas de solutions.\n\n\n" }, { "question": " 3 - [Signaler une erreur] [Ajouter à ma feuille d'exos]Enoncé Résoudre les systèmes suivants, d'inconnues les nombres complexes $z_1$ et $z_2$ :\n\n\n\n\n $$\\left\\{\n\n \\begin{array}{rcl}\n\n 2z_1-z_2&=&i\\\\\n\n -2z_1+3iz_2&=&-17\n\n \\end{array}\\right.$$\n\n \n\n $$\\left\\{\n\n \\begin{array}{rcl}\n\n 3iz_1+iz_2&=&i+7\\\\\n\n iz_1+2z_2&=&11i\n\n \\end{array}\\right.$$\n\n\n\nOn donnera les résultats sous forme algébrique.", "answer": " Le fait de résoudre des systèmes avec des nombres complexes ne change pas la méthode de résolution.\n\nSeuls les calculs sont plus compliqués!\n\n\n On a \n\n \\begin{eqnarray*}\n\n \\left\\{\n\n \\begin{array}{rcl}\n\n 2z_1-z_2&=&i\\\\\n\n -2z_1+3iz_2&=&-17\n\n \\end{array}\\right.\n\n &\\iff& \n\n \\left\\{\n\n \\begin{array}{rcl}\n\n 2z_1-z_2&=&i\\\\\n\n (3i-1)z_2&=&-17+i\\ \\textrm{(L1)+(L2)}\n\n \\end{array}\\right.\\\\\n\n &\\iff&\n\n \\left\\{\n\n \\begin{array}{rcl}\n\n 2z_1&=&i+z_2\\\\\n\n z_2&=&\\frac{-17+i}{3i-1}.\n\n \\end{array}\\right.\n\n\\end{eqnarray*}\n\nOn met $z_2$ sous forme algébrique en multipliant numérateur et dénominateur par $-3i-1$ et on trouve $z_2=2+5i$. Finalement,\n\nrevenant à $z_1$, on trouve que $z_1=1+3i$. Le système admet une unique solution donnée par $z_1=1+3i$ et $z_2=2+5i$.\n\n On a \n\n \\begin{eqnarray*}\n\n \\left\\{\n\n \\begin{array}{rcl}\n\n 3iz_1+iz_2&=&i+7\\\\\n\n iz_1+2z_2&=&11i\n\n \\end{array}\\right.\n\n &\\iff& \n\n \\left\\{\n\n \\begin{array}{rcl}\n\n 3iz_1+iz_2&=&i+7\\\\\n\n 3iz_1+6z_2&=&33i\\\\\n\n \\end{array}\\right.\\\\\n\n &\\iff&\n\n \\left\\{\n\n \\begin{array}{rcl}\n\n (i-6)z_2&=&7-32i\\\\\n\n iz_1+2z_2&=&11i.\n\n \\end{array}\\right.\n\n\\end{eqnarray*}\n\nOn a donc $z_2=\\frac{7-32i}{i-6}=-2+5i$ après mise sous forme algébrique. On en déduit facilement que $z_1=1-4i$. Ce\n\nsecond système admet donc une unique solution donnée par $z_1=1-4i$ et $z_2=-2+5i$.\n\n\n" }, { "question": " 4 - A coefficients réels [Signaler une erreur] [Ajouter à ma feuille d'exos]Enoncé Résoudre dans $\\mathbb C$ les équations suivantes :\n\n$$\\begin{array}{lll}\n\n\\mathbf{1.}\\ 2z^2+6z+5=0&\\quad&\\mathbf{2.}\\ z^2-6z+13=0\\\\\n\n\\mathbf{3.}\\ z^2+z+1=0&\\quad&\\mathbf{4.}\\ \\frac{3z+2}{z+1}=z+3\n\n\\end{array}$$", "answer": " \n On calcule le discriminant : $\\Delta=36-4\\times 5\\times 2=-4=(2i)^2$. L'équation admet donc deux solutions qui sont \n\n$$z_1=\\frac{-6-2i}{4}=\\frac{-3-i}2\\textrm{ et }z_2=\\frac{-6+2i}{4}=\\frac{-3+i}2.$$\n\n On a cette fois $\\Delta=36-4\\times 13=-16=(4i)^2$. L'équation admet donc deux solutions qui sont \n\n$$z_1=\\frac{6+4i}{2}=3+2i\\textrm{ et }z_2=3-2i.$$\n\n On a $\\Delta=1-4=-3=(i\\sqrt 3)^2$. L'équation admet donc deux solutions qui sont \n\n$$z_1=\\frac{-1-i\\sqrt 3}2\\textrm{ et }z_2=\\frac{-1+i\\sqrt 3}2.$$\n\n On doit avoir $z\\neq -1$. Pour $z\\neq -1$, l'équation est équivalente à \n\n$$3z+2=(z+3)(z+1)\\iff z^2+z+1=0.$$\n\nLes solutions sont donc les mêmes qu'à la question précédente (on vérifie bien que $-1$ n'est pas solution).\n\n\n" }, { "question": " 5 - Un système non linéaire [Signaler une erreur] [Ajouter à ma feuille d'exos]Enoncé Résoudre dans $\\mathbb C$ le système suivant :\n\n$$\\left\\{\n\n\\begin{array}{rcl}\n\nz+z'&=&2\\\\\n\nzz'&=&17\n\n\\end{array}\\right.\n\n$$", "answer": " On sait, par le lien entre les coefficients et les racines d'un polynôme du second degré, que $z$ et $z'$ sont racines de l'équation $z^2-2z+17=0$. Son discriminant est $\\Delta=4-4\\times 17=-64=(8i)^2$. Ses solutions sont\n\n$$z_1=\\frac{2-8i}2=1-4i\\textrm{ et }z_2=1+4i.$$\n\nLes solutions du système sont donc les deux couples $(1-4i,1+4i)$ et $(1+4i,1-4i)$.\n" }, { "question": " 6 - Trouver le polynôme [Signaler une erreur] [Ajouter à ma feuille d'exos]Enoncé Soit $P$ un polynôme à coefficients réels de degré $2$ dont l'une des racines est le nombre $-2+i\\sqrt 3.$\n\n\n Indiquer l'autre racine de $P$.\n\n Donner les expressions développées et factorisées de $P$, sachant que $P(0)=35.$\n\n", "answer": " \n Puisque $-2+i\\sqrt 3$ est racine de $P$ polynôme à coefficients réels, son conjugué $-2-i\\sqrt 3$ est aussi racine de $P$.\n\n On sait que $P$ se factorise en \n\n\\begin{align*}\n\nP&=\\lambda \\big(x-(-2+i\\sqrt 3)\\big)\\big(x-(-2-i\\sqrt 3)\\big)\\\\\n\n&=\\lambda (x^2+4x+7).\n\n\\end{align*}\n\nPuisque de plus $P(0)=35,$ on trouve $\\lambda=5.$\n\nFinalement,\n\n\\begin{align*}\n\nP&=5 \\big(x-(-2+i\\sqrt 3)\\big)\\big(x-(-2-i\\sqrt 3)\\big)\\\\\n\n&=5x^2+20x+35.\n\n\\end{align*}\n" }, { "question": " 7 - Au carré avec un conjugué! [Signaler une erreur] [Ajouter à ma feuille d'exos]Enoncé Résoudre dans $\\mathbb C$ l'équation suivante : \n\n$$z^2+2\\bar z+1=0.$$", "answer": " Ce n'est pas une équation du second degré classique. On va la résoudre en posant $z=x+iy$ avec $x$ et $y$ des réels. On a alors \n\n\\begin{align*}\n\nz^2+2\\bar z+1&=(x+iy)^2+2(x-iy)+1\\\\\n\n&=x^2+2ixy-y^2+2x-2iy+1\\\\\n\n&=(x^2-y^2+2x+1)+i(2xy-2y)\n\n\\end{align*}\n\nAinsi, l'équation $z^2+2\\bar z+1=0$ est équivalente au système \n\n$$\\left\\{\n\n\\begin{array}{rcl}\n\nx^2-y^2+2x+1&=&0\\\\\n\n2xy-2y&=&0.\n\n\\end{array}\n\n\\right.$$\n\nLa deuxième équation s'écrit $2y(x-1)=0$ et est donc équivalente à $y=0$ ou $x=1.$ Si $y=0$, la première équation donne \n\n$$x^2+2x+1=0\\iff (x+1)^2=0\\iff x=-1.$$\n\nSi $x=1,$ la première équation donne \n\n$$1-y^2+2+1=0\\iff y^2=4\\iff y=\\pm 2.$$\n\nL'ensemble des solutions est donc \n\n$$\\mathcal S=\\{-1;\\ 1+2i;\\ 1-2i\\}.$$\n" }, { "question": " 8 - Une équation de degré 3 [Signaler une erreur] [Ajouter à ma feuille d'exos]Enoncé On pose $P(z)=z^3+iz^2-iz+1+i$.\n\n\n Calculer $P(-1-i)$.\n\n Déterminer les réels $a$ et $b$ tels que, pour tout nombre complexe $z$, \n\n$P(z)=(z+1+i)(z^2+az+b)$.\n\n Résoudre dans $\\mathbb C$ l'équation $P(z)=0$.\n\n", "answer": " \n On a $(-1-i)^2=1+2i-1=2i$ et $(-1-i)^3=2i(-1-i)=2-2i$. On en déduit que\n\n\\begin{align*}\n\nP(-1-i)&=2-2i+i(2i)-i(-1-i)+1+i\\\\\n\n&=2-2i-2+i-1+1+i\\\\\n\n&=&0.\n\n\\end{align*}\n\n On va procéder par identification. On a en effet \n\n$$(z+1+i)(z^2+az+b)=z^3+(a+1+i)z^2+(a+ia+b)z+(1+i)b$$\n\net on veut que ce soit égal à $P(z)=z^3+iz^2-iz+1$. On doit donc avoir \n\n$a+1+i=i$ soit $a=-1$, puis $a+ia+b=-i$, soit $b=1$. On vérifie alors que $(1+i)b=1+i$.\n\n On a \n\n\\begin{align*}\n\nP(z)=0&\\iff (z+1+i)(z^2-z+1)=0\\\\\n\n&\\iff z+1+i=0\\textrm{ ou }z^2-z+1=0.\n\n\\end{align*}\n\nOn résout $z^2-z+1=0$. Le discriminant $\\Delta$ vaut $-3=(i\\sqrt 3)^2$ et les racines sont donc $(1-i\\sqrt 3)/2$ et $(1+i\\sqrt 3)/2$. Finalement, l'ensemble des solutions de $P(z)=0$ est $\\{-1-i,(1-i\\sqrt 3)/2,(1+i\\sqrt 3)/2\\}$.\n\n\n" }, { "question": " 9 - Degré plus grand [Signaler une erreur] [Ajouter à ma feuille d'exos]Enoncé On cherche à résoudre l'équation\n\n$$z^3+(1+i)z^2+(i-1)z-i=0.$$\n\n\n Rechercher une solution imaginaire pure $ai$ à l'équation.\n\n Déterminer $b,c\\in\\mathbb R$ tels que \n\n$$z^3+(1+i)z^2+(i-1)z-i=(z-ai)(z^2+bz+c).$$\n\n En déduire toutes les solutions de l'équation.\n\n Sur le même modèle, résoudre l'équation $z^3-(2+i)z^2+2(1+i)z-2i=0$.\n\n", "answer": " \n $ai$ est solution de l'équation si et seulement si \n\n$$-i+(i-1)ai-(1+i)a^2-ia^3=(-a-a^2)+i(-1-a-a^2-a^3)=0.$$\n\nLa partie réelle et la partie imaginaire du complexe doivent être nuls, et donc $ai$\n\nest solution de l'équation si et seulement si \n\n$$\\left\\{\n\n\\begin{array}{rcl}\n\na+a^2&=&0\\\\\n\n1+a+a^2+a^3&=&0.\n\n\\end{array}\\right.$$\n\nLa première équation donne $a=0$ ou $a=-1$, et seul $-1$ convient pour la deuxième équation.\n\nDonc $-i$ est solution de l'équation.\n\n On cherche $b,c$ tels que $z^3+(1+i)z^2+(i-1)z-i=(z+i)(z^2+bz+c)$. Pour cela, on développe\n\nle second membre et on trouve\n\n$$(z+i)(z^2+bz+c)=z^3+(b+i)z^2+(ib+c)z+ic.$$\n\nPar identification, on doit avoir\n\n$$\\left\\{\n\n\\begin{array}{rcl}\n\nb+i&=&1+i\\\\\n\nib+c&=&i-1\\\\\n\nic&=&-i\n\n\\end{array}\\right.$$\n\nOn trouve $b=1$ et $c=-1$ et donc la factorisation\n\n$$z^3+(1+i)z^2+(i-1)z-i=(z+i)(z^2+z-1).$$\n\n $z$ est solution de l'équation si et seulement si $z=-i$ ou \n\n$z^2+z-1=0$. Les racines de cette équation sont $\\frac{-1-\\sqrt 5}2$ et\n\n$\\frac{-1+\\sqrt 5}2$. L'équation admet donc trois solutions, qui sont \n\n$-i, \\frac{-1-\\sqrt 5}2$ et $\\frac{-1+\\sqrt 5}2$.\n\n On trouve que les solutions sont $i$, $1+i$ et $1-i$.\n\n\n" }, { "question": " 10 - Racine carrée en détails [Signaler une erreur] [Ajouter à ma feuille d'exos]Enoncé On cherche à déterminer les nombres complexes $z$ tels que $z^2=15-8i$. Pour cela, on pose $z=x+iy$.\n\n\n Montrer que $z^2=15-8i$ si et et seulement si $(x,y)$ est solution du système :\n\n$$\\left\\{\\begin{array}{rcl}\n\nx^2-y^2&=&15\\\\\n\n2xy&=&-8.\n\n\\end{array}\\right.$$\n\n Démontrer que si $z^2=15-8i$, on a aussi $x^2+y^2=17$. \n\n En déduire tous les nombres complexes $z$ tels que $z^2=15-8i$.\n\n", "answer": " \n On a\n\n$$z^2=(x+iy)^2=x^2-y^2+i2xy.$$\n\nPar unicité des parties réelles et imaginaires, $z^2=15-8i$ si et seulement si $(x,y)$ est solution du système proposé.\n\n Si $z^2=15-8i$, on doit avoir $|z^2|=|15-8i|$. Or, \n\n$$|z^2|=x^2+y^2$$\n\net \n\n$$|15-8i|=17.$$\n\n En utilisant le système et la question précédente, on trouve que si $z^2=15-8i$, alors\n\n$2x^2=17+15$. Ceci donne $x^2=16$ et donc $x=\\pm 4$. De plus, si $x=4$, alors l'équation $2xy=-8$ donne $y=-1$ tandis que si $x=-4$, alors $y=1$. On vérifie alors facilement que $4-i$ et $-4+i$ sont bien solution de $z^2=15-8i$. Ce sont les deux seules solutions de cette équation.\n\n\n" }, { "question": " 11 - Racine carrée d'un nombre complexe [Signaler une erreur] [Ajouter à ma feuille d'exos]Enoncé Calculer les racines carrées des nombres complexes suivants :\n\n$z_1=3+4i,\\ z_2=8-6i.$", "answer": " La méthode est toujours la même. On pose $z=a+ib$, de sorte que \n\n$z^2=(a^2-b^2)+2iab$. L'équation $z^2=3+4i$ est donc équivalente à \n\n$$\\left\\{\\begin{array}{rcl}a^2-b^2&=&3\\\\\n\n2ab=4\\end{array}\\right.$$\n\nOn peut ajouter une troisième équation en remarquant que \n\n$$|z|^2=|3+4i|\\iff a^2+b^2=\\sqrt{9+16}=5.$$\n\nOn trouve alors $2a^2=8$, soit $a=\\pm 2$ et $2b^2=2$, soit $b=\\pm 1$.\n\nL'équation $2ab=4$ oblige $a$ et $b$ à avoir même signe, et donc les deux solutions\n\nsont $2+i$ et $-2-i$.\n\nPour l'équation $z^2=8-6i$, on peut suivre une méthode exactement identique, \n\net les solutions sont cette fois $3-i$ et $-3+i$.\n" }, { "question": " 12 - Racine carré de deux façons [Signaler une erreur] [Ajouter à ma feuille d'exos]Enoncé Déterminer les racines carrées de $Z=\\sqrt 3+i$ sous forme algébrique, puis sous forme trigonométrique.\n\nEn déduire la valeur de $\\cos\\left(\\frac\\pi{12}\\right)$.", "answer": " Soit $w=a+ib$ tel que $w^2=Z$. On obtient le système \n\n$$\\left\\{\n\n\\begin{array}{rcl}\n\na^2-b^2&=&\\sqrt 3\\\\\n\n2ab&=&1\\\\\n\na^2+b^2&=&|\\sqrt 3+i|=2.\n\n\\end{array}\\right.\n\n$$\n\nIl vient $a^2=\\frac{\\sqrt{3}+2}2$ et $b^2=\\frac{2-\\sqrt 3}{2}$. Puisque $a$ et $b$ ont le même signe, les solutions sont\n\ndonc\n\n$$w=\\sqrt{\\frac{\\sqrt{3}+2}2}+i\\sqrt{\\frac{2-\\sqrt 3}{2}}\\textrm{ et }w=-\\sqrt{\\frac{\\sqrt{3}+2}2}-i\\sqrt{\\frac{2-\\sqrt\n\n3}{2}}.$$\n\nPour la résolution sous forme trigonométrique, on remarque que \n\n$$Z=2\\left(\\frac{\\sqrt 3}2+i\\frac 12\\right)=2e^{i\\pi/6}.$$\n\nLes racines carrées de $Z$ sont donc\n\n$$w=\\sqrt 2e^{i\\pi/12}\\textrm{ et }w=-\\sqrt 2e^{i\\pi/12}.$$\n\nComme les deux calculs donnent le même résultat, en identifiant les parties réelles, on trouve :\n\n$$\\sqrt 2\\cos\\left(\\frac{\\pi}{12}\\right)=\\sqrt{\\frac{\\sqrt{3}+2}2},$$\n\nd'où on tire :\n\n$$\\cos\\left(\\frac{\\pi}{12}\\right)=\\frac{1}{\\sqrt 2}\\sqrt{\\frac{\\sqrt{3}+2}2}=\\frac{\\sqrt 6+\\sqrt 2}4.$$\n" }, { "question": " 13 - Racine carrée puis équation du second degré [Signaler une erreur] [Ajouter à ma feuille d'exos]Enoncé \n Calculer les racines carrées du nombre complexe $1+2\\sqrt 2 i$ sous forme algébrique.\n\n Résoudre dans $\\mathbb C$ l'équation $z^2+iz-\\frac 12-i\\frac{\\sqrt 2}2=0$.\n\n", "answer": " \n Soit $z=x+iy$ tel que $z^2=1+2\\sqrt 2 i$. Alors en identifiant les parties réelles, les parties imaginaires, et les modules, on trouve que $x$ et $y$ sont solutions du système :\n\n$$\\left\\{\n\n\\begin{array}{rcl}\n\nx^2-y^2&=&1\\\\\n\n2xy&=&2\\sqrt 2\\\\\n\nx^2+y^2&=&3.\n\n\\end{array}\\right .$$\n\nOn trouve facilement que ceci entraîne $2x^2=4$ et donc $x=\\sqrt 2$ ou $x=-\\sqrt 2$. Le premier cas donne $y=1$ et le second $y=-1$. Les racines carrées de $1+2\\sqrt 2 i$ sont donc $\\sqrt 2+i$ et $-\\sqrt 2-i$.\n\n On commence par calculer le discriminant de cette équation du second degré : \n\n$$\\Delta=i^2+4\\left(\\frac 12+i\\frac{\\sqrt 2}2\\right)=1+2i\\sqrt 2=(\\sqrt 2+i)^2.$$\n\nLes solutions de cette équation sont donc\n\n$$z_1=\\frac{-i+(\\sqrt 2+i)}{2}=\\frac{\\sqrt 2}2\\textrm{ et }z_2=\\frac{-i-(\\sqrt 2+i)}2=-\\frac{\\sqrt 2}2-i.$$\n\n\n" }, { "question": " 14 - Équations du second degré [Signaler une erreur] [Ajouter à ma feuille d'exos]Enoncé Résoudre les équations du second degré suivantes :\n\n$$\\begin{array}{lll}\n\n\\mathbf{1.}\\ z^2-2iz-1+2i=0&&\\mathbf{2.}\\ iz^2+(4i-3)z+i-5=0\\\\\n\n\\mathbf{3.}\\ z^2-(7+i)z+12+3i=0.\n\n\\end{array}$$", "answer": " \n Le discriminant de cette équation du second degré vaut :\n\n$$\\Delta=(-2i)^2-4(-1+2i)=-8i.$$\n\nUne racine carré de $\\Delta$ est donnée par\n\n$\\delta=i\\times 2\\sqrt 2\\times e^{i\\pi/4}=-2+2i.$ En appliquant les formules du cours, on trouve que les racines sont :\n\n$$\\frac{2i-2+2i}{2}=-1+2i\\textrm{ et }\\frac{2i+2-2i}{2}=1.$$\n\n Le discriminant de cette équation du second degré est :\n\n$$\\Delta=(4i-3)^2-4i(i-5)=-4i-3.$$\n\nOn en cherche une racine carrée sous la forme $\\delta=a+ib$. Calculant $\\delta^2$, et utilisant\n\naussi la relation $$|\\delta^2|=|-4i-3|=5,$$ on trouve le système :\n\n$$\\left\\{\n\n\\begin{array}{rcl}\n\na^2-b^2&=&-3\\\\\n\n2ab&=&-4\\\\\n\na^2+b^2&=&5\n\n\\end{array}\\right.$$\n\nOn en déduit que $\\delta=1-2i$ est solution de $\\delta^2=\\Delta$ (l'autre solution\n\nest $-1+2i$). Utilisant les formules du cours, les racines de l'équation initiale sont donc :\n\n$$\\frac{-4i+3+1-2i}{2i}=-3-2i\\textrm{ et }\\frac{-4i+3-1+2i}{2i}=-1-i.$$\n\n Le discriminant de cette équation vaut $(7+i)^2-4(12+3i)=2i$.\n\nUne racine carré de ce discriminant est $1+i$. Les racines de l'équation sont donc :\n\n$$\\frac{7+i-1-i}2=3\\textrm{ et }\\frac{7+i+1+i}{2}=4+i.$$\n\n\n" }, { "question": " 15 - Équation un peu abstraite [Signaler une erreur] [Ajouter à ma feuille d'exos]Enoncé Soit $\\alpha$ un nombre complexe non nul. On considère l'équation \n\n$$z^2-\\alpha(\\alpha+i)z+i\\alpha^3=0.$$\n\n\n Déterminer les solutions de cette équation.\n\n Si $\\alpha=\\rho e^{i\\theta}$, donner la forme exponentielle de ces solutions.\n\n", "answer": " \n Calculons le discriminant de cette équation. On a \n\n\\begin{align*}\n\n\\Delta&=\\big(\\alpha(\\alpha+i)\\big)^2-4i\\alpha^3\\\\\n\n&=\\alpha^2 (\\alpha^2+2i\\alpha-1)-4i\\alpha^3\\\\\n\n&=\\alpha^2 (\\alpha^2-2i\\alpha-1)\\\\\n\n&=\\alpha^2 (\\alpha-i)^2\\\\\n\n&=\\big (\\alpha(\\alpha-i)\\big)^2.\n\n\\end{align*}\n\nLes deux solutions de cette équation sont donc \n\n$$z_1=\\frac{\\alpha(\\alpha+i)-\\alpha(\\alpha-i)}{2}=i\\alpha\\textrm{ et }\n\nz_2=\\frac{\\alpha(\\alpha+i)+\\alpha(\\alpha-i)}{2}=\\alpha^2.$$\n\n Très facilement, on a \n\n$$z_1=\\rho e^{i\\left(\\theta+\\frac\\pi 2\\right)}\\textrm{ et }z_2=\\rho^2 e^{2i\\theta}.$$\n\n\n" }, { "question": " 16 - Degré plus grand [Signaler une erreur] [Ajouter à ma feuille d'exos]Enoncé On considère l'équation $z^3-(3+i)z^2-(2+5i)z+8+14i=0.$\n\n\n Vérifier que $2$ est solution de cette équation.\n\n Déterminer les nombres complexes $a,b,c$ tels que, pour tout $z\\in\\mathbb C$, \n\n$$z^3-(3+i)z^2-(2+5i)z+8+14i=(z-2)(az^2+bz+c).$$\n\n Déterminer les racines carrées de $16+30i$.\n\n En déduire toutes les solutions de l'équation $z^3-(3+i)z^2-(2+5i)z+8+14i=0.$\n\n", "answer": " \n Il s'agit d'une simple vérification...\n\n Procédons par identification. On a \n\n$$(z-2)(az^2+bz+c)=az^3+(b-2a)z^2+(c-2b)z-2c.$$\n\nLes nombres complexes $a$, $b$ et $c$ sont donc solutions du système\n\n$$\\left\\{\n\n\\begin{array}{rcl}\n\na&=&1\\\\\n\nb-2a&=&-(3+i)\\\\\n\nc-2b&=&-(2+5i)\\\\\n\n-2c&=&8+14i.\n\n\\end{array}\\right.$$\n\nLa première et la dernière équation donnent $a=1$ et $c=-4-7i$. La deuxième et la troisième donnent ensuite $b=-1-i$.\n\n Soit $z=x+iy$ tel que $z^2=16+30i$. Alors, utilisant la partie réelle, la partie imaginaire et le module de $z^2$, $x$ et $y$ vérifient le système suivant :\n\n$$\\left\\{\n\n\\begin{array}{rcl}\n\nx^2-y^2&=&16\\\\\n\n2xy&=&30\\\\\n\nx^2+y^2&=&\\sqrt{16^2+30^2}=34\n\n\\end{array}\\right.$$\n\nOn trouve alors $x^2=25$ donc $x=\\pm 5$ puis $y=\\pm 3$, $x$ et $y$ devant être de même signe puisque $2xy>0$. Les racines carrées de $16+30i$ sont donc $5+3i$ et $-5-3i$.\n\n On a $z^3-(3+i)z^2-(2+5i)z+8+14i=0$ si et seulement si $z=2$ ou $z^2-(1+i)z-(4+7i)=0$. Résolvons cette dernière équation. Son discriminant est\n\n$$\\Delta=(1+i)^2+4(4+7i)=16+30i.$$\n\nD'après la question précédente, $\\Delta=(5+3i)^2$. Les racines de cette équation sont donc \n\n$$z_1=\\frac{1+i+5+3i}2=3+2i\\textrm{ et }z_2=\\frac{1+i-5-3i}2=-2-i.$$\n\nFinalement, l'ensemble des solutions de l'équation initiale est $\\mathcal S=\\{2;3+2i;-2-i\\}$.\n\n\n" }, { "question": " 17 - Équation bicarrée [Signaler une erreur] [Ajouter à ma feuille d'exos]Enoncé \n Déterminer les racines carrées de $-\\frac 12+i\\frac{\\sqrt 3}2$ et de $-\\frac 12-i\\frac{\\sqrt 3}2$.\n\n En déduire les solutions de l'équation $z^4+z^2+1=0$.\n\n", "answer": " \n Cherchons d'abord les nombres complexes $z=x+iy$ tels que $z^2=-\\frac 12+i\\frac{\\sqrt 3}2$. On peut utiliser la méthode usuelle en posant $z=x+iy$ et en remarquant que $z^2=-\\frac 12+i\\frac{\\sqrt 3}2$ entraîne, en comparant les parties réelles, les parties imaginaires et les modules de chacun des nombres complexes, le système\n\n$$\\left\\{\\begin{array}{rcl}\n\nx^2-y^2&=&-\\frac{1}2\\\\\n\n2xy&=&\\frac{\\sqrt 3}2\\\\\n\nx^2+y^2&=&1\n\n\\end{array}\\right.$$\n\nLa résolution de ce système donne $2x^2=1/2$ et donc $x=1/2$ ou $x=-1/2$. Si $x=1/2$, alors $y=\\sqrt 3/2$ et si $x=-1/2$, alors $y=-\\sqrt 3/2$. Les deux racines carrées de $-\\frac 12+i\\frac{\\sqrt 3}2$ sont donc\n\n$$\\frac 12+i\\frac{\\sqrt 3}2\\textrm{ et }-\\frac 12-i\\frac{\\sqrt 3}2.$$\n\nOn aurait pu aussi passer par la forme exponentielle : en effet, on peut écrire que \n\n$$-\\frac 12+i\\frac{\\sqrt 3}2=e^{2i\\pi/3}.$$\n\nSes racines carrées sont donc $e^{i\\pi/3}$ et $-e^{i\\pi/3}$.\n\nEn effectuant le même calcul, on trouve que les racines carrées de $-\\frac 12-i\\frac{\\sqrt 3}2$ sont \n\n$$\\frac 12-i\\frac{\\sqrt 3}2\\textrm{ et }-\\frac 12+i\\frac{\\sqrt 3}2.$$\n\n Posons $Z=z^2$. L'équation devient $Z^2+Z+1=0$. Son discriminant vaut $\\Delta=-3=(\\sqrt 3 i)^2$ et ses racines sont \n\n$$Z_1=-\\frac 12+i\\frac{\\sqrt 3}2\\textrm{ et }Z_2=-\\frac 12-i\\frac{i\\sqrt 3}2.$$\n\nOn trouve les solutions de l'équation initiale en résolvant les équations $z^2=Z_1$ et $z^2=Z_2$, ce qu'on vient de faire. L'ensemble des solutions est donc \n\n$$\\mathcal S=\\left\\{\\frac 12-i\\frac{\\sqrt 3}2; -\\frac 12+i\\frac{\\sqrt 3}2;\n\n\\frac 12+i\\frac{\\sqrt 3}2;-\\frac 12-i\\frac{\\sqrt 3}2\\right\\}.$$\n\n\n" }, { "question": " 18 - Degré plus grand [Signaler une erreur] [Ajouter à ma feuille d'exos]Enoncé Résoudre l'équation $4iz^3+2(1+3i)z^2-(5+4i)z+3(1-7i)=0$, sachant qu'elle admet une racine réelle.", "answer": " Soit $x$ une racine réelle, ie $4ix^3+2(1+3i)x^2-(5+4i)x+3(1-7i)=0$. Partie\n\nréelle et partie imaginaire du membre de gauche doivent être nulles, on obtient donc\n\naprès identification :\n\n$$\\left\\{\n\n\\begin{array}{rcl}\n\n2x^2-5x+3&=&0\\\\\n\n4x^3+6x^2-4x-21&=&0.\n\n\\end{array}\\right.$$\n\nIl est facile de résoudre la première équation et de vérifier si on obtient une racine de l'autre équation.\n\nOn trouve que $3/2$ est racine. On factorise alors le polynôme par $z-3/2$, et on trouve (par exemple en procédant par identification) :\n\n$$4iz^3+2(1+3i)z^2-(5+4i)z+3(1-7i)=(z-3/2)\\big(4iz^2+2(1+6i)z+2(-1+7i)\\big).$$\n\nReste à résoudre ensuite l'équation :\n\n$$4iz^2+2(1+6i)z+2(-1+7i)=0$$\n\ndont les solutions sont $-2+\\frac32i$ et $-1-i$.\n" }, { "question": " 19 - Racines $n$-ièmes [Signaler une erreur] [Ajouter à ma feuille d'exos]Enoncé Résoudre les équations suivantes : \n\n$$\\begin{array}{lll}\n\n\\mathbf{1.}\\ z^4=-1&&\\mathbf{2.}\\ z^5=-i.\n\n\\end{array}$$", "answer": " \n On écrit $-1=e^{i\\pi}$ puis on utilise le cours :\n\n$$z^4=-1=e^{i\\pi}\\iff z=e^{i\\frac \\pi4+k\\frac{\\pi}2},\\ k\\in\\mathbb Z.$$\n\nOn obtient $4$ racines distinctes correspondant à $k=0,1,2,3$, qui sont $e^{i\\pi/4}$, $e^{i3\\pi/4}$, $e^{5i\\pi/4}$ et $e^{7i\\pi/4}$.\n\n On a $-i=e^{-i\\pi/2}$, et donc\n\n$$z^5=-i=e^{-i\\pi/2}\\iff z= e^{\\frac{2ik\\pi}{5}-\\frac{i\\pi}{10}},\\ k\\in\\mathbb Z;$$\n\non obtient $5$ racines distinctes pour $k=0,\\dots,4$.\n\n\n" }, { "question": " 20 - Racines $n$-ièmes [Signaler une erreur] [Ajouter à ma feuille d'exos]Enoncé Résoudre les équations suivantes :\n\n$$\\begin{array}{lll}\n\n\\mathbf{1.}\\ z^3=1+i\\sqrt 3&\\quad&\\mathbf{2.}\\ z^6=\\frac{-4}{1+i\\sqrt 3}\\\\\n\n\\mathbf{3.}\\ z^5=\\frac{(1+i\\sqrt 3)^4}{(1+i)^2}.&&\n\n\\end{array}$$", "answer": " C'est du cours!\n\n\n On écrit \n\n$$1+i\\sqrt 3=2\\left(\\frac12+i\\frac{\\sqrt 3}2\\right)=2e^{i\\pi/3}.$$\n\nIl vient \n\n$$z^3=1+i\\sqrt 3=2e^{i\\pi/3}\\iff z=2^{1/3} e^{i\\left(\\frac\\pi 9+\\frac{2k\\pi}3\\right)},\\ k\\in\\mathbb Z.$$\n\nOn obtient 3 nombres complexes différentes pour $k=0$, $k=1$ et $k=2$ qui sont $2^{1/3}e^{i\\pi/9}$, $2^{1/3}e^{i7\\pi/9}$ et $2^{1/3}e^{i13\\pi/9}$.\n\n On a \n\n$$1+i\\sqrt 3=2\\left(\\frac12+i\\frac{\\sqrt 3}2\\right)=2e^{i\\pi/3}.$$\n\nOn en déduit que\n\n$$\\frac{-4}{1+i\\sqrt 3}=-2e^{-i\\pi/3}=2e^{2i\\pi/3}.$$\n\nFinalement,\n\n$$z^6=\\frac{-4}{1+i\\sqrt 3}\\iff z=2^{1/6} e^{\\frac{i k\\pi}{3}+\\frac{i\\pi}9},\\ k\\in\\mathbb Z$$\n\n(on peut se restreindre à $k\\in\\{0,\\dots,5\\}$).\n\n On a\n\n$$\\frac{(1+i\\sqrt 3)^4}{(1+i)^2}=\\frac{2^4\\left(\\frac 12+i\\frac{\\sqrt 3}2\\right)^4}{2\\left(\\frac1{\\sqrt 2}+i\\frac{1}{\\sqrt 2}\\right)^2}=2^3\\frac{e^{i\\frac{\\pi}3\\times 4}}{e^{i\\frac{\\pi}4\\times 2}}.$$\n\nL'équation qu'on doit résoudre est donc :\n\n$$z^5=2^3e^{i\\frac{5\\pi}6}\\iff \\left(\\frac{z}{2^{3/5}e^{i\\pi/6}}\\right)^5=1.$$\n\nOn en déduit que les solutions sont les complexes de la forme \n\n$$z=2^{3/5}e^{i\\left(\\frac{\\pi}6+\\frac{2k\\pi}{5}\\right)},\\ k\\in\\mathbb Z$$\n\n(on peut se restreindre à $k\\in\\{0,\\dots,4\\}$).\n\n\n" }, { "question": " 21 - Presque des racines de l'unité [Signaler une erreur] [Ajouter à ma feuille d'exos]Enoncé Résoudre les équations suivantes : \n\n$$\\begin{array}{lll}\n\n\\mathbf{1.}\\ (z-1)^5=(z+1)^5&&\\mathbf{2.}\\ \\left(\\frac{z+1}{z-1}\\right)^3+\\left(\\frac{z-1}{z+1}\\right)^3=0\\\\\n\n\\mathbf{3.}(z+i)^n=(z-i)^n.\n\n\\end{array}$$", "answer": " \n $1$ n'est pas solution, et l'équation est donc équivalente\n\nà $$\\left(\\frac{z+1}{z-1}\\right)^5=1.$$ Posons $w=\\frac{z+1}{z-1}$,\n\nc'est-à-dire $z=\\frac{w+1}{w-1}$. On a $w^5=1$ si et seulement s'il\n\nexiste $k\\in\\{0,\\dots,4\\}$ tel que $w=e^{2ik \\pi/5}$. Si l'on retourne à $z$, on doit exclure \n\n$w=1$ car l'équation $\\frac{z+1}{z-1}=1$ n'admet pas de solutions. \n\nOn a donc 4 solutions qui sont\n\n$$z=\\frac{e^{2ik\\pi/5}+1}{e^{2ik\\pi/5}-1},$$\n\npour $k=1,\\dots,4$.\n\nOn peut encore simplifier en utilisant les formules d'Euler :\n\n$$\\begin{array}{rcl}\\exp \\left(\\frac{2ik\\pi }{5}\\right)+1&=&\\exp \\left(\\frac{2ik\\pi }{5}\\right)+\\exp\n\n\\left(0\\right)\\\\&=&\\exp \\left(\\frac{ik\\pi }{5}\\right)\\left(\\exp \\left(\\frac{ik\\pi }{5}\\right)+\\exp \\left(\\frac{-ik\\pi\n\n}{5}\\right)\\right)\\\\&=&2\\exp \\left(\\frac{ik\\pi}{5}\\right) \\cos(k\\pi/5).\n\n\\end{array}$$\n\nDe même, on trouve\n\n$$\\exp\\left(\\frac{2k\\pi}{5}\\right)-1=2i\\exp \\left(\\frac{ik\\pi}{5}\\right) \\sin(k\\pi/5).$$\n\nL'ensemble des solutions est donc $\\{-i\\textrm{cotan}(k\\pi/5);\\ k=1,\\dots,4\\}$.\n\n Posons $w=\\frac{z+1}{z-1}$. L'équation devient $w^3+\\frac{1}{w^3}=0$,\n\nsoit $w^6=-1=e^{i\\pi}$. Ses\n\nracines sont\n\n$$e^{i\\pi/6+k\\pi/3},\\ k=0,\\dots,5.$$\n\nOn retrouve alors $z$ car $z=\\frac{w+1}{w-1}$. \n\nPour $k=1$ ou $k=4$, on trouve $z=\\pm i$. Pour les autres valeurs de $k$, on trouve\n\n$z=\\pm i(2\\pm \\sqrt 3)$.\n\n Remarquons d'abord que $z=i$ n'est pas solution de l'équation. Ainsi, l'équation est équivalente à \n\n$$\\left(\\frac{z+i}{z-i}\\right)^n=1.$$\n\nCeci est équivalent à dire qu'il existe $k\\in\\{0,\\dots,n-1\\}$ tel que $$\\frac{z+i}{z-i}=\\omega_k,$$\n\nen notant $\\omega_k=e^{2ik\\pi/n}$. Pour $k=0$, $\\omega_k=1$ et l'équation $\\frac{z+i}{z-i}=1$ n'a pas de solutions.\n\nSinon, pour $k=1,\\dots,n-1$, on obtient les solutions\n\n$$z_k=i\\frac{\\omega_k+1}{\\omega_k-1}.$$\n\n\n\n" }, { "question": " 22 - Second degré et racines 3-ièmes [Signaler une erreur] [Ajouter à ma feuille d'exos]Enoncé \n Déterminer, sous forme algébrique, les racines carrées du nombre complexe $3-4i$.\n\n Résoudre dans $\\mathbb C$ l'équation $w^2-iw-1+i=0$.\n\n Rappeler quelles sont les racines cubiques (ou racines 3-ièmes) de $1$. \n\n Écrire $-1+i$ sous forme exponentielle.\n\n Résoudre l'équation $z^3=-1+i$ (donner les solutions sous forme exponentielle).\n\n En déduire les solutions de l'équation $z^6-iz^3-1+i=0$.\n\n", "answer": " \n Soit $z\\in\\mathbb C$ tel que $z^2=3-4i$. Écrivons $z=x+iy$. En développant $(x+iy)^2$ et en écrivant que $|z|^2=|3+4i|=5$, on trouve que $x$ et $y$ sont solutions du système : \n\n$$\\left\\{\n\n\\begin{array}{rcl}\n\nx^2-y^2&=&3\\\\\n\n2xy&=&-4\\\\\n\nx^2+y^2&=&5.\n\n\\end{array}\\right.$$\n\nLa résolution de ce système donne $x=2$ et $y=-1$ ou $x=-2$ et $y=1$. Les deux racines carrées de $3-4i$ sont donc $2-i$ et $-2+i$.\n\n On peut remarquer que $1$ est solution évidente et donc que $-1 + i$ est l'autre solution.\n\nSinon, on calcule le discriminant de l'équation : $\\Delta=(-i)^2-4(-1+i)=3-4i$, puis $\\Delta =(-2+i)^2$ d'après la question précédente.\n\nLes deux solutions de l'équation sont donc \n\n$$w_1=\\frac{i+2-i}{2}=1\\textrm{ et }w_2=\\frac{-2+2i}{2}=-1+i.$$\n\n Les racines cubiques de $1$ sont $1$, $e^{2i\\pi/3}$ et $e^{4i\\pi/3}$.\n\n On a $|-1+i|=\\sqrt 2$. De plus, \n\n$$-1+i=\\sqrt 2\\left(\\frac{-1}{\\sqrt 2}+\\frac{1}{\\sqrt 2}i\\right)=\\sqrt 2e^{3i\\pi/4}$$\n\npuisque $\\cos(3\\pi/4)=-1/\\sqrt 2$ et $\\sin(3\\pi/4)=1/\\sqrt 2$.\n\n On cherche $z$ sous la forme $z=re^{i\\theta}$. On a $z^3=r^3e^{i3\\theta}=\\sqrt 2e^{3i\\pi/4}$. Une solution particulière est donnée par $z_0=2^{1/6}e^{i\\pi/4}$. \n\nOn trouve toutes les racines cubiques en multipliant par les racines cubiques de $1$, et donc les solutions sont \n\n$$2^{1/6}e^{i\\pi/4},\\ 2^{1/6}e^{i\\pi/4}e^{2i\\pi/3},\\ 2^{1/6}e^{i\\pi/4}e^{4i\\pi/3},$$\n\nce qui après remise au même dénominateur donne \n\n$$2^{1/6}e^{i\\pi/4},\\ 2^{1/6}e^{11i\\pi/12},\\ 2^{1/6}e^{19i\\pi/12}.$$\n\n Soit $z\\in\\mathbb{C}$ et $w=z^3$. Alors $z$ est solution de $z^6-iz^3-1+i=0$ si et seulement si $w$ est solution de $w^2-iw-1+i=0$, et donc si et seulement si $w=1$ ou $w=-1+i$. \n\nOn en déduit, puisqu'on a résolu les équations $z^3=1$ et $z^3=-1+i$, que les solutions de l'équation sont \n\n$$1,\\ e^{2i\\pi/3},\\ e^{4i\\pi/3},\\ 2^{1/6}e^{i\\pi/4},\\ 2^{1/6}e^{11i\\pi/12},\\ 2^{1/6}e^{19i\\pi/12}.$$\n\n\n" }, { "question": " 23 - Pas tout à fait une racine $n$-ième [Signaler une erreur] [Ajouter à ma feuille d'exos]Enoncé Déterminer les nombres complexes $z$ solution de l'équation \n\n$$z^6=(1+i)\\overline z^2.$$", "answer": " On remarque d'abord que $z=0$ est solution. Sinon, on pose $z=re^{i\\theta}$, de sorte que $\\bar z=re^{-i\\theta}$. On remarque aussi que $1+i=\\sqrt 2e^{i\\pi/4}$. L'équation est alors équivalente à\n\n$$r^6 e^{i6\\theta}=\\sqrt 2 e^i{\\pi/4} r^2 e^{-2i\\theta}\\iff r^4 e^{i8\\theta}=\\sqrt 2 e^{i\\pi/4}.$$\n\nCeci est équivalent à $r^4=\\sqrt 2$, soit $r=2^{1/8}$, et à \n\n$$8\\theta \\equiv \\frac{\\pi}4\\ [2\\pi].$$\n\nAinsi, il existe $k\\in\\mathbb Z$ tel que \n\n$$8\\theta=\\frac{\\pi}4+2k\\pi\\iff \\theta=\\frac{\\pi}{32}+k\\frac{\\pi}4.$$\n\nAinsi, l'équation admet neuf solutions : \n\n$$\\mathcal S=\\left\\{0,z_0,z_0e^{i\\pi/4},z_0e^{i\\pi/2},z_0e^{3i\\pi/4},z_0e^{i\\pi},z_0e^{i5\\pi/4},z_0e^{i3\\pi/2},z_0e^{i7\\pi/4}\\right\\}$$\n\noù $z_0=2^{1/8}e^{i\\pi/32}$.\n" }, { "question": " 24 - Équation de degré 6 [Signaler une erreur] [Ajouter à ma feuille d'exos]Enoncé \n Donner la forme algébrique des nombres complexes $(-1-i)^3$ et $(1+2i)^3$.\n\n Donner les racines cubiques des nombres complexes $2-2i$ et $-11-2i$.\n\n Résoudre l'équation d'inconnue $z\\in\\mathbb C$ : $z^6+(9+4i)z^3-26+18i=0$.\n\n", "answer": " \n D'après la formule du binôme de Newton,\n\n $$(-1-i)^3=(-1)^3+3(-1)^2(-i)+3(-1)(-i)^2+(-i)^3=-1-3i+3+i=2-2i$$\n\n $$(1+2i)^3=1^3+3(2i)^2+3(2i)^2+(2i)^3=1+6i-12-8i=-11-2i.$$\n\n On cherche à résoudre $z^3=2-2i$. On sait qu'une solution particulière est $-1-i$. On en déduit que \n\n les racines cubiques de $2-2i$ sont $-1-i,(-1-i)e^{i2\\pi/3},(-1-i)e^{i4\\pi/3}$. De même,\n\n puisque $1+2i$ est une solution particulière de $z^3=-11-2i$, les racines cubiques de $-11-2i$ sont\n\n $1+2i,(1+2i)e^{i2\\pi/3},(1+2i)e^{i4\\pi/3}$.\n\n On pose $w=z^3$ et l'équation devient $w^2+(9+4i)z-26+18i=0$. Son discriminant\n\n vaut $$\\Delta=(9+4i)^2-4(-26+18i)=169=13^2$$\n\n et les racines sont donc \n\n $$w_1=\\frac{-9-4i+13}{2}=2-2i\\textrm{ et }w_2=\\frac{-9-4i-13}2=-11-2i.$$\n\n On a donc prouvé que $z$ est solution de l'équation initiale si et seulement $z^3=w_1$\n\n ou $z^3=w_2$. D'après la question précédente, l'ensemble des solutions de l'équation est\n\n $$\\mathcal S=\\{-1-i,(-1-i)e^{i2\\pi/3},(-1-i)e^{i4\\pi/3},1+2i,(1+2i)e^{i2\\pi/3},(1+2i)e^{i4\\pi/3}\\}.$$\n\n\n" }, { "question": " 25 - Variations sur les équations classiques [Signaler une erreur] [Ajouter à ma feuille d'exos]Enoncé Résoudre les équations suivantes :\n\n$$\\begin{array}{lll}\n\n\\mathbf{1.}\\ iz^8+iz^4+1+i=0&&\\mathbf{2.}\\ z^n=\\bar z\\ (n\\geq 2)\\\\\n\n\\mathbf{3.}\\ z^4-z^3+z^2-z+1=0&& \\mathbf{4.}\\ 1+2z+\\dots+2z^{n-1}+z^n=0\n\n\\end{array}$$", "answer": " \n On commence par poser $u=z^4$, et l'équation devient $iu^2+iu+1+i=0$. Son discriminant est\n\n$$\\Delta=-1-4i(1+i)=3-4i.$$\n\nOn cherche une racine carrée $\\delta$ de $\\Delta$ en posant $\\delta=a+ib$, en utilisant $\\delta^2=\\Delta$\n\net $|\\delta|^2=|\\Delta|=5$, et on trouve qu'une des deux racines est $\\delta=2-i$. Les racines de l'équation\n\n$iu^2+iu+1=0$ sont donc les complexes\n\n$$u_1=\\frac{-i-2+i}{2i}=i\\textrm{ et }u_2=\\frac{-i+2-i}{2i}=-1-i.$$\n\nReste à résoudre les équations $z^4=u_1$ et $z^4=u_2$. Pour cela, on pose \n\n$z=re^{i\\theta}$ et on remarque que $u_1=e^{i\\pi/2}$ et que\n\n$u_2=\\sqrt 2 e^{i5\\pi/4}$. On en déduit\n\n$$z^4=e^{i\\pi/2}\\iff z=e^{i\\pi/8+k\\pi/2},\\ k=0,\\dots,3;$$\n\n$$z^4=\\sqrt 2 e^{i5\\pi/4}\\iff z=2^{1/8}e^{i5\\pi/16+k\\pi/2},\\ k=0,\\dots,3.$$\n\n Remarquons d'abord que $z=0$ convient. Supposons désormais $z\\ne 0$. Puisque $|z|=|\\overline z|$, on a $|z|^{n-1}=1$ et donc $|z|=1$. On peut donc poser $z=e^{i\\theta}$\n\net l'équation devient\n\n$$e^{in\\theta}=e^{-i\\theta}\\iff e^{i(n+1)\\theta}=1\\iff \\theta=2k\\pi/(n+1),\\ k\\in\\{0,\\dots,n\\}.$$\n\n Remarquons que $z=-1$ n'est pas racine de l'équation. On reconnait alors le début de la somme géométrique de raison $-z$.\n\nL'équation est donc équivalente à \n\n$$\\frac{1-(-z)^5}{1+z}=0\\iff z^5=-1=e^{i\\pi}.$$\n\nLes solutions de cette dernière équation sont les complexes de la forme $e^{i\\frac{(1+2k)\\pi}{5}}$, $k=0,\\dots,4$ mais il faut faire attention que pour $k=2$, on retrouve $-1$, valeur interdite! Les solutions de l'équation initiale sont donc les quatre nombres complexes $e^{i\\pi/5}$, $e^{3i\\pi/5}$, $e^{7i\\pi/5}$, $e^{9i\\pi/5}$.\n\n On commence par écrire :\n\n$$1+2z+\\dots+2z^{n-1}+z^n=(1+z+\\dots+z^{n-1})+(z+\\dots+z^n).$$\n\nOn reconnait deux sommes géométriques de raison $z$. Comme $z=1$ n'est pas solution de l'équation, celle-ci est équivalente à \n\n$$\\frac{1-z^n}{1-z}+\\frac{z-z^{n+1}}{1-z}=0\\iff 1-z^n+z-z^{n+1}=0\\iff (1+z)(1-z^n)=0.$$\n\nLes solutions sont donc $z=-1$ et les racines $n$-ièmes de l'unité, excepté $1$. Autrement dit, \n\n$-1$ et $e^{2ik\\pi/n}$, $k=1,\\dots,n-1$.\n\n\n" }, { "question": " 26 - Somme et puissances de racines $n$-iemes [Signaler une erreur] [Ajouter à ma feuille d'exos]Enoncé Soit $n\\geq 1$ et $\\omega=e^{2i\\pi/n}$.\n\n\n Calculer le produit des racines $n$-ièmes de l'unité.\n\n Soit $p\\geq 0$. Calculer $\\sum_{k=0}^{n-1}\\omega^{kp}$.\n\n En déduire que $\\sum_{k=0}^{n-1}(1+\\omega^k)^n =2n$.\n\n", "answer": " \n Les racines $n$-ièmes de l'unité sont les complexes $\\omega^k$,\n\navec $k=0,\\dots,n-1$. Leur produit vaut donc :\n\n$$\\prod_{k=0}^{n-1}\\omega^k=\\omega^{\\sum_{k=0}^{n-1}k}=\\omega^{n(n-1)/2}=e^{i\\pi(n-1)}=(-1)^{n-1}$$\n\n(résultat qu'on vérifie facilement pour $n=1,2,3,4$).\n\n On a ici une somme géométrique de raison $\\omega^p$. Si $p$ est un multiple de $n$, \n\nla raison est donc égale à 1, et la somme fait $n$. Sinon, on a\n\n$$\\sum_{k=0}^{n-1}\\omega^{kp}=\\frac{1-\\omega^{np}}{1-\\omega^p}=0$$\n\npuisque $\\omega^n=1$.\n\n On développe la puissance à l'intérieur de la somme en utilisant la formule du binôme de Newton, et on trouve :\n\n\\begin{eqnarray*}\n\n\\sum_{k=0}^{n-1}(1+\\omega^k)^n&=&\\sum_{k=0}^{n-1}\\sum_{p=0}^n \\binom{n}{p}\\omega^{kp}\\\\\n\n&=&\\sum_{p=0}^n \\binom{n}{p} \\sum_{k=0}^{n-1}\\omega^{kp}.\n\n\\end{eqnarray*}\n\nOn utilise le résultat de la question précédente : la somme $\\sum_{k=0}^{n-1}\\omega^{kp}$ est nulle, sauf si $p=0$ ou si $p=n$. Dans ce cas, cette somme fait $n$. On en déduit que \n\n\\begin{eqnarray*}\n\n\\sum_{k=0}^{n-1}(1+\\omega^k)^n&=&\\binom n0 n+\\binom nn n=n+n=2n.\n\n\\end{eqnarray*}\n" }, { "question": " 1 - Placer des points [Signaler une erreur] [Ajouter à ma feuille d'exos]Enoncé \n On considère le nombre complexe $z=3-2i$. Placer dans le plan complexe les points $A,B,C,D$ d'affixes respectives $z$, $\\bar z$, $-z$ et $-\\bar z$.\n\n Placer dans le plan complexe les points $E,F,G,H$ d'affixes respectives\n\n$$z_E=2e^{i\\pi/3},\\ z_F=-e^{i\\pi/6},\\ z_G=-z_E\\times z_F,\\ z_H=\\frac{-z_F}{z_E}.$$\n\n", "answer": " \n On peut placer $B$, $C$ et $D$ en calculant leurs affixes sous forme algébrique ou en utilisant les propriétés de symétrie par rapport à l'axe des abscisses (passage au conjugué) ou par rapport à l'origine (multiplication par $-1$). On trouve\n\n\n Puisque $|z_E|=2$ et qu'un argument de $E$ est $\\pi/3$, on a $OE=2$ et $(\\vec i,\\overrightarrow{OE})=\\pi/3\\ [2\\pi]$. Pour placer $F$, on peut d'abord placer $F'$ d'affixe $e^{i\\pi/6}$ puis procéder par symétrie par rapport à l'origine. Pour placer $G$ et $H$, on calcule\n\n$$z_G=2e^{i\\pi/3}e^{i\\pi/6}=2e^{i\\pi/2}=2i$$\n\n$$z_H=\\frac{1}2 e^{i\\pi/6}e^{-i\\pi/3}=\\frac 12 e^{-i\\pi/6}$$\n\npuis on procède comme précédemment.\n\n\n\n" }, { "question": " 2 - [Signaler une erreur] [Ajouter à ma feuille d'exos]Enoncé Le point $M$ de la figure ci-dessous à pour affixe $z$. Reproduire la figure et tracer :\n\n\n en vert l'ensemble des points dont l'affixe non nulle $z'$ est telle que \n\n$$\\arg(z')=\\arg(z)+\\frac\\pi 2\\ [2\\pi].$$\n\n en bleu l'ensemble des points dont l'affixe non nulle $z'$ est telle que \n\n$$|z'|=2|z|.$$\n\n en noir l'ensemble des points dont l'affixe non nulle $z'$ est telle que\n\n$$\\arg(z')=\\arg(z)\\ [\\pi].$$\n\n en rouge l'ensemble des points dont l'affixe non nulle $z'$ est telle que \n\n$$\\arg(z')=\\arg(z)+\\arg(\\bar z)\\ [2\\pi].$$\n\n\n", "answer": " \n On trace $M'$ l'image de $M$ par la rotation d'angle $\\pi/2$. L'ensemble recherché est la demi-droite $(OM')$.\n\n Il s'agit du cercle de centre $O$ et de rayon $2OM$.\n\n Il s'agit de la droite $(OM)$, excepté le point $O$ (droite et non demi-droite car l'égalité est modulo $\\pi$ et non modulo $2\\pi$).\n\n On a $\\arg(z)+\\arg(\\bar z)=0\\ [2\\pi]$ et donc l'ensemble recherché est la demi-droite $(O,\\vec i)$.\n\n\n\n" }, { "question": " 3 - Configuration simple [Signaler une erreur] [Ajouter à ma feuille d'exos]Enoncé Dans le plan rapporté à un repère orthonormé $(O,\\vec u,\\vec v)$, on considère les points $A$, $B$, $C$ et $D$ d'affixes respectives $a=-1+i$, $b=-1-i$, $c=2i$ et $d=2-2i$.\n\n\n Placer ces points.\n\n Calculer $\\frac{c-a}{d-a}$ et en déduire la nature du triangle $ACD$.\n\n Montrer que les points $A$, $B$, $C$ et $D$ sont sur un même cercle dont on précisera le centre et le rayon.\n\n", "answer": " \n\n\n On a \n\n$$\\frac{c-a}{d-a}=\\frac{1+i}{3-3i}=\\frac{(1+i)(3+3i)}{(3-3i)(3+3i)}=\\frac{6i}{18}=\\frac 13 i.$$\n\nOn en déduit que $|c-a|=\\frac{1}3|d-a|$ et donc que $AD=3AC$, puis que \n\n$$\\arg\\left(\\frac{c-a}{d-a}\\right)=\\frac\\pi2\\ [2\\pi]\\iff (\\overrightarrow{AD},\\overrightarrow{AC})=\\frac{\\pi}2\\ [2\\pi].$$\n\nAinsi, le triangle $ACD$ est un triangle rectangle (non isocèle) en $A$.\n\n Soit $I$ le point d'affixe $1$. Alors on a :\n\n$$IA=|-2+i|=\\sqrt 5,\\ IB=|-2-i|=\\sqrt 5,\\ IC=|2i-1|=\\sqrt 5,\\ ID=|1-2i|=\\sqrt 5.$$\n\nLes points $A$, $B$, $C$ et $D$ sont donc sur le cercle de centre $I$ et de rayon $\\sqrt 5$. Bien entendu, le choix de $I$ était guidé par la figure géométrique.\n\n\n" }, { "question": " 4 - Écriture complexe de transformations [Signaler une erreur] [Ajouter à ma feuille d'exos]Enoncé Déterminer la nature et les éléments caractéristiques des transformations géométriques données par l'écriture complexe suivante :\n\n$$\\begin{array}{ll}\n\n\\mathbf 1.\\ z\\mapsto \\frac 1iz&\\mathbf 2.\\ z\\mapsto z+(2+i)\\\\\n\n\\mathbf 3.\\ z\\mapsto (1+i\\sqrt 3)z+\\sqrt 3(1-i)&\\mathbf 4.\\ z\\mapsto (1+i\\tan\\alpha )z-i\\tan\\alpha,\\ \\alpha\\in [0,\\pi/2[.\n\n\\end{array}$$", "answer": " \n On écrit $\\frac 1iz=e^{-i\\pi/2}z$, et on remarque que l'on a affaire à une rotation d'angle $-\\pi/2$.\n\n On a ici l'écriture d'une translation de vecteur $(2,1)$.\n\n L'application de la forme $z\\mapsto az+b$ est une similitude directe. Cherchons son centre qui est le point invariant,\n\nc'est-à-dire le point vérifiant $z=(1+i\\sqrt 3)z+\\sqrt 3(1-i)$. On trouve $z=1+i$, le centre de la similitude est donc le\n\npoint $A(1,1)$. On a de plus\n\n$$1+i\\sqrt 3=2\\left(\\frac12+i\\frac{\\sqrt 3}2\\right)=2e^{i\\pi/3}.$$\n\nLe rapport de la similitude est donc égal à 2, et l'angle à $\\pi/3$.\n\n Si $\\alpha=0$, la transformation est simplement l'identité. Sinon, on a affaire à une similitude directe. Son point invariant est le nombre complexe $z$ solution de l'équation \n\n$$z=(1+i\\tan \\alpha)z-i\\tan \\alpha\\iff z=1.$$\n\nLe centre de la similitude est donc le point $A(1,0)$. De plus, on a \n\n$$1+i\\tan\\alpha=\\frac 1{\\cos\\alpha}\\times(\\cos \\alpha+i\\sin\\alpha)=\\frac1{\\cos\\alpha}e^{i\\alpha}.$$\n\nAinsi, la similitude est de rapport $\\frac 1{\\cos\\alpha}$ et d'angle $\\alpha$.\n\n\n" }, { "question": " 5 - Maximum du produit des distances [Signaler une erreur] [Ajouter à ma feuille d'exos]Enoncé Dans le plan complexe, on considère les points $A,$ $B,$ $C$ et $D$ d'affixe respective $1,$ $-1$, $i$ et $-i.$ On note $\\mathcal C$ le cercle unité, c'est-à-dire l'ensemble des nombres complexes de module 1. Si $M$ est un point de $\\mathcal C,$ on note $p(M)$ le produit des distances de $M$ aux points $A,B,C,D$ :\n\n$$p(M)=MA\\times MB\\times MC\\times MD.$$\n\nDéterminer le maximum de $p(M)$ lorsque $M$ décrit $\\mathcal C.$", "answer": " On a $AM=|z-1|,$ $BM=|z+1|,$ $CM=|z-i|$ et $DM=|z+i|.$ Alors \n\n\\begin{align*}\n\np(M)&=|z-1|\\times |z+1|\\times |z-i|\\times |z+i|\\\\\n\n&=|z^2-1|\\times |z^2+1|\\\\\n\n&=|z^4-1|.\n\n\\end{align*}\n\nPar l'inégalité triangulaire,\n\n$$p(M)\\leq |z^4|+1=|z|^4+1=2.$$\n\nAinsi, $p(M)\\leq 2.$ De plus, si $M$ est le point d'affixe $e^{i\\pi/4},$\n\nalors \n\n$$p(M)=|e^{i\\pi}-1|=|-2|=2.$$\n\nAinsi, le maximum de $p$ sur $\\mathcal C$ est $2$.\n" }, { "question": " 6 - A partir des racines $n$-ièmes [Signaler une erreur] [Ajouter à ma feuille d'exos]Enoncé Soit $a$ un nombre complexe de module 1, $z_1,\\dots,z_n$ les racines de l'équation $z^n=a$. Montrer que les points du plan complexe dont\n\nles affixes sont $(1+z_1)^n,\\dots,(1+z_n)^n$ sont alignés.", "answer": " Posons $a=e^{i\\theta}$. Alors les racines de $z^n=a$ sont données par $z_k=e^{i(2k\\pi+\\theta)/n}$, $k=0,\\dots,n-1$. \n\nFactorisant par l'angle moitié et utilisant les formules d'Euler, on a \n\n$$1+z_k=2\\cos\\left(\\frac{2k\\pi+\\theta}{2n}\\right)e^{i(2k\\pi+\\theta)/2n}$$\n\nsoit\n\n$$(1+z_k)^n=2^n \\cos^n \\left(\\frac{2k\\pi+\\theta}{2n}\\right)e^{i(2k\\pi+\\theta)/2}=2^n \\cos^n \\left(\\frac{2k\\pi+\\theta}{2n}\\right)e^{i(k\\pi+\\theta/2)}.$$\n\nTous les points d'affixe $(1+z_k)^n$ sont donc situés sur la droite qui fait un angle $\\theta/2$ avec l'axe des abscisses. Ainsi, ils sont alignés.\n" }, { "question": " 7 - Triangle équilatéral [Signaler une erreur] [Ajouter à ma feuille d'exos]Enoncé Montrer que le triangle de sommets $M_1(z_1)$, $M_2(z_2)$ et $M_3(z_3)$ est équilatéral si et seulement si \n\n$$z_1^2+z_2^2+z_3^2=z_1z_2+z_1z_3+z_2z_3.$$", "answer": " Le triangle est équilatéral si et seulement si \n\n$$z_3-z_1=e^{i\\pi/3}(z_2-z_1)\\textrm{ ou }z_3-z_1=e^{-i\\pi/3}(z_2-z_1),$$\n\nc'est-à-dire si et seulement si \n\n$$z_3-(1-e^{i\\pi/3})z_1-e^{i\\pi/3}z_2=0\\textrm{ ou }z_3-(1-e^{-i\\pi/3})z_1-e^{-i\\pi/3}z_2=0.$$\n\nCeci est encore équivalent à dire que le produit de ces deux quantités est nul, c'est-à-dire à \n\n$$(z_3-(1-e^{i\\pi/3})z_1-e^{i\\pi/3}z_2)(z_3-(1-e^{-i\\pi/3})z_1-e^{-i\\pi/3}z_2)=0.$$\n\nEn développant ce produit, on trouve exactement la condition demandée.\n" }, { "question": " 8 - Une condition d'alignement sur les affixes [Signaler une erreur] [Ajouter à ma feuille d'exos]Enoncé Soit $A(a)$, $B(b)$ et $C(c)$ trois points du plan complexe. Démontrer que $A,\\ B$ et $C$ sont alignés si et seulement si \n\n$$a\\bar b+b\\bar c+c\\bar a\\in\\mathbb R.$$", "answer": " On peut supposer que les 3 points sont distincts deux à deux (sinon la relation est trivialement vérifiée). Dire que $A$, $B$ et $C$ sont alignés signifie qu'il existe un réel \n\n$\\lambda$ tel que $\\overrightarrow{AC}=\\lambda\\overrightarrow{AB}.$ Autrement dit, avec les notations précédents, $A,$ $B$ et $C$ sont alignés si et seulement \n\nil existe un réel $\\lambda$ tel que $c-a=\\lambda(b-a)$ c'est-à-dire si et seulement si $(b-a)/(c-a)\\in\\mathbb R.$ On va prouver que la condition de l'énoncé est équivalente à celle-ci. Écrivons $b=a+u$ et $c=a+v$ avec $u,v\\in\\mathbb C.$ On a alors\n\n\\begin{align*}\n\na\\bar b&=|a|^2+a\\bar u\\\\\n\nb\\bar c&=|a|^2+\\bar a u+a\\bar v+u\\bar v\\\\\n\nc\\bar a&=|a|^2+\\bar a v\n\n\\end{align*}\n\nFinalement, utilisant $z+\\bar z=2\\Re e(z),$ on trouve \n\n$$a\\bar b+b\\bar c+c\\bar a=3|a|^2+2\\Re e(a\\bar u+a\\bar v)+u\\bar v.$$\n\nAinsi,\n\n$$a\\bar b+b\\bar c+c\\bar a\\in\\mathbb R\\iff u\\bar v\\in\\mathbb R\\iff (b-a)\\overline{(c-a)}\\in\\mathbb R.$$\n\nMais en utilisant par exemple que \n\n\\begin{align*}\n\n\\textrm{arg}\\left((b-a)\\overline{(c-a)}\\right)&=\\textrm{arg}(b-a)+\\textrm{arg}(\\overline{c-a})\\ [2\\pi]\\\\\n\n&=\\textrm{arg}(b-a)-\\textrm{arg}(c-a)\\ [2\\pi]\\\\\n\n&=\\textrm{arg}\\left(\\frac{b-a}{c-a}\\right)\\ [2\\pi]\n\n\\end{align*}\n\non conclut que c'est équivalent à $(b-a)/(c-a)\\in\\mathbb R$ et donc à l'alignement des 3 points.\n" }, { "question": " 9 - Lieux géométriques et module [Signaler une erreur] [Ajouter à ma feuille d'exos]Enoncé Déterminer le lieu géométrique des points $M$ dont l'affixe $z$ vérifie\n\n$$\n\n\\begin{array}{ll}\n\n\\mathbf{1.}\\ |z-i|=|z+i|&\n\n\\mathbf{2.}\\ \\displaystyle \\frac{|z-3+i|}{|z+5-2i|}=1\\\\\n\n\\mathbf{3.}\\ |(1+i)z-2i|=2&\n\n\\mathbf{4.}\\ \\displaystyle \\ |3+iz|=|3-iz|\n\n\\end{array}$$", "answer": " \n Soit $A$ le point d'affixe $i$, $B$ le point d'affixe $-i$, \n\net $M$ le point d'affixe $z$. Alors $|z-i|$ est la longueur $AM$, $|z+i|$ est la longueur $BM$,\n\net la condition recherchée est $AM=BM$, c'est-à-dire $M$ est sur la médiatrice de $[AB]$,\n\nsoit encore $M$ sur l'axe réel, soit $z$ réel.\n\n $z=-5+2i$ n'est pas solution et l'équation est équivalente à $|z-3+i|=|z+5-2i|$ soit encore $|z-(3-i)|=|z-(-5+2i)|$. Autrement dit, le point $M$ d'affixe $z$ est à égale distance du point $A$ d'affixe $3-i$ et du point $B$ d'affixe $-5+2i$. L'ensemble des points recherché est donc la médiatrice de $[AB]$.\n\n Factorisons par $1+i$ dans le module. On trouve :\n\n$$|1+i|\\left|z-\\frac{2i}{1+i}\\right|=2.$$\n\nPuisque $|1+i|=\\sqrt 2$ et \n\n$\\frac{2i}{1+i}=1+i,$ ceci est équivalent à \n\n$$\\left|z-\\big(1+i\\big)\\right|=\\sqrt 2.$$\n\nAinsi, l'ensemble des points $M$ correspondants est le cercle\n\nde centre le point $A(1,1)$ et de rayon $\\sqrt 2$.\n\n On a $3+iz=i(-3i+z)$ et $3-iz=-i(3i+z)$ et donc l'équation est équivalente à $|z-3i|=|z-(-3i)|$. Autrement dit, le point $M$ d'affixe $z$ est à égale distance du point $A$ d'affixe $3i$ et du point $B$ d'affixe $-3i$. Le point $M$ est donc sur la médiatrice de $[AB]$ c'est-à-dire sur l'axe des abscisses.\n\n\n" }, { "question": " 10 - Lieu géométrique et arguments [Signaler une erreur] [Ajouter à ma feuille d'exos]Enoncé Le plan est rapporté à un repère orthonormé direct $(O,\\vec u,\\vec v)$. Déterminer l'ensemble des points $M$ dont l'affixe $z$ vérifie la relation demandée :\n\n$$\n\n\\begin{array}{lll}\n\n\\mathbf{1.}\\ \\arg(z-2)=\\frac{\\pi}2\\ [2\\pi]&\n\n\\mathbf{2.}\\ \\arg(z-2)=\\frac{\\pi}2\\ [\\pi]&\n\n\\mathbf{3.}\\ \\arg(iz)=\\frac{\\pi}{4}\\ [\\pi]\\\\\n\n\\mathbf{4.}\\ \\arg\\left(\\frac{z}{1+i}\\right)=\\frac{\\pi}2\\ [2\\pi]&\n\n\\mathbf{5.}\\ \\arg\\left(\\frac{z-2i}{z-1+i}\\right)=\\frac{\\pi}2\\ [\\pi]\n\n\\end{array}\n\n$$", "answer": " \n Soit $A$ le point d'affixe $2$. Alors \n\n$$\\arg(z-2)=\\frac{\\pi}2\\ [2\\pi]\\iff (\\vec u,\\overrightarrow{AM})=\\frac\\pi2\\ [2\\pi].$$\n\nL'ensemble recherché est donc la demi-droite passant par $A$ et dirigée par $\\vec v$, sauf le point $A$.\n\n On a\n\n$$\\arg(z-2)=\\frac{\\pi}2\\ [\\pi]\\iff (\\vec u,\\overrightarrow{AM})=\\frac\\pi2\\ [\\pi].$$\n\nL'ensemble recherché est donc la droite passant par $A$ et parallèle à l'axe des ordonnées, sauf le point $A$.\n\n On a $\\arg(iz)=\\arg(i)+\\arg(z)=\\frac{\\pi}2+\\arg(z)\\ [2\\pi]$. On a donc\n\n$$\\arg(iz)=\\frac{\\pi}4\\ [\\pi]\\iff \\arg(z)=-\\frac{\\pi}4\\ [\\pi].$$\n\nL'ensemble recherché est donc la droite $y=-x$, sauf l'origine du repère.\n\n On a \n\n\\begin{align*}\n\n\\arg\\left(\\frac{z}{1+i}\\right)&=\\arg(z)-\\arg(1+i)\\ [2\\pi]\\\\\n\n&=\\arg(z)-\\frac{\\pi}4\\ [2\\pi].\n\n\\end{align*}\n\nOn en déduit que \n\n$$\\arg\\left(\\frac{z}{1+i}\\right)=\\frac{\\pi}2\\ [2\\pi]\\iff \\arg(z)=\\frac{3\\pi}4\\ [2\\pi].$$\n\nSoit $B$ un point tel que $(\\vec u,\\overrightarrow{OB})=\\frac{3\\pi}4,$ par exemple le point d'affixe $-1+i$. Alors l'ensemble recherché est la demi-droite $[OB)$, sauf le point $O$.\n\n Soit $C$ le point d'affixe $2i$ et $D$ le point d'affixe $1-i$. Alors\n\n$$\\arg\\left(\\frac{z-2i}{z-1+i}\\right)=\\arg\\left(\\frac{z-2i}{z-(1-i)}\\right)=(\\overrightarrow{MD},\\overrightarrow{MC})\\ [2\\pi].$$\n\nOn a donc \n\n$$\\arg\\left(\\frac{z-2i}{z-1+i}\\right)=\\frac{\\pi}2\\ [\\pi]\\iff (\\overrightarrow{MD},\\overrightarrow{MC})=\\frac\\pi2\\ [\\pi].$$\n\nL'ensemble recherché est donc le cercle de diamètre $[CD]$, sauf les points $C$ et $D$.\n\n\n" }, { "question": " 11 - Lieu géométrique [Signaler une erreur] [Ajouter à ma feuille d'exos]Enoncé Décrire les points $M$ d'affiche $z\\neq -5+3i$ tels que \n\n\n $\\displaystyle \\frac{z-1+i}{z+5-3i}$ est un réel;\n\n $\\displaystyle \\frac{z-1+i}{z+5-3i}$ est un imaginaire pur.\n\n", "answer": " Notons $M$ le point d'affixe $z$, $A$ le point d'affixe $1-i$ et $B$ le point d'affixe $-5+3i$, de sorte que \n\n$$\\textrm{arg}\\left(\\frac{z-1+i}{z+5-3i}\\right)=(\\overrightarrow{MA},\\overrightarrow{MB})\\ [2\\pi].$$\n\n\n Un nombre complexe (non-nul) est réel si et seulement si son argument est égal à $0$ modulo $2\\pi$. Ainsi, $\\displaystyle \\frac{z-1+i}{z+5-3i}$ est un réel si et seulement si $M=A$ ou bien \n\n$$(\\overrightarrow{MA},\\overrightarrow{MB})=0\\ [\\pi].$$\n\nCeci est équivalent au fait de dire que $M$ est sur la droite $(AB)$, sauf en $B$.\n\n Un nombre complexe (non nul) est imaginaire pur si et seulement si son argument est égal à $\\pi/2$ modulo $2\\pi$. Ainsi, $\\displaystyle \\frac{z-1+i}{z+5-3i}$ est un imaginaire si et seulement si $M=A$ ou bien \n\n$$(\\overrightarrow{MA},\\overrightarrow{MB})=\\frac\\pi 2\\ [\\pi].$$\n\nCeci est équivalent au fait de dire que $M$ est sur le cercle de diamètre $[AB]$, sauf en $B$.\n\n\n" }, { "question": " 12 - Image par une transformation [Signaler une erreur] [Ajouter à ma feuille d'exos]Enoncé Dans le plan complexe muni d'un repère orthonormé $(O,\\vec u,\\vec v)$, on associe à tout point $M$ d'affixe $z$ le point $M'$ d'affixe $z'=\\frac 12\\left(z+\\frac 1z\\right)$. On dit que $M'$ est l'image de $M$ par la transformation $z\\mapsto \\frac 12\\left(z+\\frac 1z\\right)$.\n\n\n Déterminer l'ensemble des points $M$ du plan tels que $M=M'$.\n\n Démontrer que, lorsque $M$ décrit le cercle $\\Gamma$ de centre $O$ et de rayon $1$, alors $M'$ décrit un segment que l'on précisera.\n\n", "answer": " \n Si $M'=M$, on doit avoir \n\n$$\\frac 12\\left(z+\\frac 1z\\right)=z.$$\n\nCeci est équivalent à $\\frac 1z=z$, c'est-à-dire à $z^2=1$, ou alors $z=1$ et $z=-1$. Ceci nous donne deux points invariants, le point $A$ d'affixe $1$ et le point $B$ d'affixe $-1$\n\n Remarquons d'abord que $A$ et $B$ sont dans l'image de $\\Gamma$. Prenons ensuite $z\\in\\Gamma$. Puisque $|z|=1$, on sait que $\\frac 1z=\\bar z$ (on peut redémontrer ceci en utilisant la forme exponentielle de $z$ par exemple). On a alors \n\n$$\\frac 12\\left(z+\\frac 1z\\right)=\\frac 12 (2\\Re e(z))=\\Re e(z).$$\n\nAinsi, $M'$ appartient au segment $[AB]$.\n\nRéciproquement, soit $N$ un point du segment $[AB]$. Alors l'affixe de $N$ est un réel $x\\in [-1,1]$. Soit $M$ un point du cercle unité dont l'affixe a pour partie réelle $x$ (il y a deux points ainsi). Alors l'image $M'$ de $M$ est $N$. Tous les points du segment sont donc décrits.\n\n\n" }, { "question": " 13 - Lieux géométriques [Signaler une erreur] [Ajouter à ma feuille d'exos]Enoncé Pour chacune des conditions suivantes, déterminer le lieu géométrique des points $M$ d'affixe $z$ qui vérifient la condition.\n\n\n $I(i)$ et $M'(iz)$ sont alignés avec $M$; déterminer alors l'ensemble des points $M'$ correspondants;\n\n $\\displaystyle \\Re e\\left(\\frac{z-1}{z-i}\\right)=0$;\n\n $M$, $P$ d'affixe $z^2$ et $Q$ d'affixe $z^3$ sont les sommets d'un triangle rectangle.\n\n", "answer": " \n On sait que les points $I$, $M$ et $M'$ sont alignés si et seulement si\n\n$$(\\vec{IM},\\vec{IM'})=0[\\pi] \\textrm{ ou }M=I\\textrm{ ou }M'=I\\textrm{ ou }M=M'.$$\n\nEn termes de nombres complexes, ceci se traduit par\n\n$$\\arg\\left(\\frac{iz-i}{z-i}\\right)=0\\ [\\pi]\\textrm{ ou }z=i\\textrm{ ou }iz=i\\textrm{ ou }iz=z.$$\n\nIntroduisons le point $A$ d'affixe $1$. Alors, ceci devient \n\n$$\\frac\\pi2+\\arg\\left(\\frac{z-1}{z-i}\\right)=0\\ [\\pi]\\textrm{ ou }M=I\\textrm{ ou }M=A\\textrm{ ou }M=O$$\n\n$$\\iff (\\vec{IM},\\vec{AM})=-\\frac{\\pi}2\\ [\\pi]\\textrm{ ou }M=I\\textrm{ ou }M=A\\textrm{ ou }M=O$$\n\n$$\\iff (IM)\\perp (AM)\\textrm{ ou }M=I\\textrm{ ou }M=A\\textrm{ ou }M=O.$$\n\nLes points $M$ solutions sont donc les points du cercle de diamètre $[AI]$ ($O$ étant également un point de ce cercle). Puisque $M'$ est image de $M$ par rotation de centre\n\n$O$ et d'angle $\\pi/2$, les points $M'$ correspondants sont sur l'image de ce cercle par cette rotation.\n\n Notons $A$ d'affixe $1$ et $I$ d'affixe $i$. La question s'écrit encore \n\n$$\\frac{z-1}{z-i}=ia,\\ \\textrm{avec}\\ a\\in\\mathbb R,$$\n\nc'est-à-dire que les vecteurs $\\overrightarrow{AM}$ et $\\overrightarrow{IM}$ sont orthogonaux.\n\nAutrement dit, la condition est vérifiée si et seulement si $M$ appartient au cercle de diamètre $[AI]$, excepté $I$ (on doit\n\navoir $z\\neq i$ pour définir le quotient).\n\n On va d'abord supposer que $z\\neq 0,1,-1$ pour que les trois points $M,P,Q$ soient distincts et qu'on soit sûr d'avoir affaire\n\nà un vrai triangle. On va utiliser la condition suivante : soit $A(a)$, $B(b)$ et $C(c)$. Les droites $(AB)$ et $(AC)$ sont perpendiculaires si et seulement si\n\n$$\\exists m\\in\\mathbb R,\\ \\frac{c-a}{b-a}=mi\\iff \\Re e\\left(\\frac{c-a}{b-a}\\right)=0.$$\n\nOn distingue alors trois cas :\n\n\n le triangle est rectangle en $M$. Ceci est équivalent à \n\n$$\\Re e\\left(\\frac{z^3-z}{z^2-z}\\right)=0\\iff \\Re e(z+1)=0\\iff \\Re e(z)=-1.$$\n\nLes points $M$ solutions sont alors ceux de la droite d'équation $x=-1$.\n\n le triangle est rectangle en $P$. Ceci est équivalent à \n\n$$\\Re e\\left(\\frac{z^3-z^2}{z-z^2}\\right)=0\\iff \\Re e(z)=0.$$\n\nLes points $M$ solutions sont alors ceux de la droite d'équation $x=0$.\n\n le triangle est rectangle en $Q$. Ceci est équivalent à \n\n$$\\Re e\\left(\\frac{z-z^3}{z^2-z^3}\\right)=0\\iff \\Re e\\left(\\frac{z+1}z\\right)=0.$$\n\nNotons $D$ d'affixe -1 et $O$ d'affixe $0$. On obtient que les droites $(DM)$ et $(OM)$ sont orthogonales,\n\nc'est-à-dire que $M$ décrit le cercle de diamètre $[OD]$.\n\n\n\n" }, { "question": " 14 - Alignement de puissances [Signaler une erreur] [Ajouter à ma feuille d'exos]Enoncé Trouver tous les nombres complexes $z$ tels que les points d'affixe $z$, $z^2$ et $z^4$ soient alignés.", "answer": " On commence par étudier les cas où deux de ces points sont confondus. L'équation $z=z^2$ a pour solution $z=1$ et $z=0$.\n\nL'équation $z=z^4$ a pour solutions $z=0$ et $z=1,j,j^2$. L'équation $z^2=z^4$ a pour solutions $z=0,1,-1$. On suppose désormais que \n\n$z$ est différent des nombres précédemment trouvés, et on remarque que les points sont alignés si et seulement si \n\n$$\\arg\\left(\\frac{z^4-z}{z^2-z}\\right)=0\\ [\\pi]\\iff \\frac{z^4-z}{z^2-z}\\in\\mathbb R.$$\n\nOr, en notant $z=x+iy$, on a \n\n$$\\frac{z^4-z}{z^2-z}=\\frac{z(z-1)(z^2+z+1)}{z(z-1)}=z^2+z+1=(x^2-y^2+x+1)+iy(2x+1).$$\n\nAinsi, on en déduit que, dans ce cas, les points $z$, $z^2$ et $z^4$ sont alignés si et seulement si \n\n$y=0$ ou $x=-1/2$, c'est-à-dire si et seulement si $z$ est réel ou sa partie réelle vaut $-1/2$. Remarquons que l'on retrouve les solutions précédentes, $0$, $1$, $-1$, $j$ et $j^2$.\n" }, { "question": " 15 - Parallélogramme [Signaler une erreur] [Ajouter à ma feuille d'exos]Enoncé Les points $A$, $B$, $C$ et $D$ du plan complexe ont pour affixes respectives $a$, $b$, $c$ et $d$. On note $I$, $J$, $K$ et $L$ les milieux respectifs de $[AB]$, $[BC]$, $[CD]$ et $[DA]$.\n\n\n Calculer les affixes des points $I$, $J$, $K$ et $L$.\n\n En déduire que $IJKL$ est un parallélogramme.\n\n", "answer": " \n C'est du cours. Les affixes respectives de $I$, $J$, $K$ et $L$ sont \n\n$(a+b)/2$, $(b+c)/2$, $(c+d)/2$, $(d+a)/2$.\n\n Le vecteur $\\overrightarrow{IJ}$ a pour affixe \n\n$$\\frac{b+c}2-\\frac{a+b}2=\\frac{c-a}2.$$\n\nLe vecteur $\\overrightarrow{LK}$ a pour affixe\n\n$$\\frac{c+d}2-\\frac{a+d}2=\\frac{c-a}2.$$\n\nAinsi, on a $\\overrightarrow{IJ}=\\overrightarrow{LK}$ et donc $IJKL$ est un parallélogramme.\n\n\n" }, { "question": " 16 - Les huit carrés [Signaler une erreur] [Ajouter à ma feuille d'exos]Enoncé Soit la figure suivante : \n\n\n\nLe but de l'exercice est de démontrer que $\\alpha+\\beta+\\gamma=\\frac{\\pi}{4}\\ [2\\pi]$. On se place dans le repère orthonormé direct $(A,\\vec u,\\vec v)$ où $\\vec u=\\overrightarrow{AB}$ et $\\vec v=\\overrightarrow{AD}$.\n\n\n Reproduire la figure et placer les points $E$ et $F$ sur $[DZ]$ tels que\n\n$\\beta$ et $\\gamma$ soient des mesures respectives de $(\\vec u,\\overrightarrow{AE})$ et $(\\vec u,\\overrightarrow{AF})$. \n\n Quelles sont les affixes $z_Z,$ $z_E$ et $z_F$ des points $Z$, $E$ et $F$?\n\n Démontrer que $z_Z\\times z_E\\times z_F=65(1+i)$.\n\n Conclure.\n\n", "answer": " \n Ces points s'obtiennent par translation (rappelons qu'une translation préserve les angles orientés). On obtient : \n\n\n On a $z_Z=8+i$, $z_E=5+i$ et $z_F=2+i$.\n\n On a $z_Z\\times z_E=39+13i$, puis \n\n$$z_Z\\times z_E\\times z_F=(39+13i)\\times (2+i)=65+65i=65(1+i).$$\n\n On écrit sous forme trigonométrique le nombre complexe précédent. Puisque $1+i=\\sqrt{2}e^{i\\pi/4}$, il vient \n\n$$z_Z\\times z_E\\times z_F=65\\sqrt 2e^{i\\pi/4}.$$\n\nD'où $\\arg(z_Z\\times z_E\\times z_F)=\\pi/4\\ [2\\pi]$. Puisque l'argument d'un produit est la somme des arguments, on a\n\n$$\\arg(z_Z)+\\arg(z_E)+\\arg(z_F)=\\frac\\pi 4\\ [2\\pi].$$\n\nOn conclut car, modulo $2\\pi$, $\\arg(z_Z)=\\alpha$, $\\arg(z_E)=\\beta$ et $\\arg(z_F)=\\gamma$.\n\n\n" }, { "question": " 17 - Une coquille d'escargot [Signaler une erreur] [Ajouter à ma feuille d'exos]Enoncé Dans le plan muni d'un repère orthonormal $(O,\\vec i,\\vec j)$, on note $A_0$ le point d'affixe 6 et $S$ la similitude de centre $O$, de rapport $\\frac{\\sqrt 3}2$ et d'angle $\\frac\\pi 6$. On pose $A_{n+1}=S(A_n)$ pour $n\\geq 1$.\n\n\n Déterminer, en fonction de $n$, l'affixe du point $A_n$. En déduire que $A_{12}$ est sur la demi-droite $(O,\\vec i)$.\n\n Établir que le triangle $OA_nA_{n+1}$ est rectangle en $A_{n+1}$.\n\n Calculer la longueur du segment $[A_0A_1]$. En déduire la longueur $\\ell$ de la ligne polygonale\n\n$A_0A_1A_2\\dots A_{12}.$\n\n", "answer": " \n On va commencer par donner l'écriture complexe de la transformation. On a ainsi\n\n$$S(z)=\\frac{\\sqrt 3}2 e^{i\\pi/6}z.$$\n\nAutrement dit, si on note $z_n$ l'affixe du point $A_n$, on a \n\n$$z_{n+1}=\\frac{\\sqrt 3}2 e^{i\\pi/6}z_n.$$\n\nOn reconnait une suite géométrique (de raison $r=\\frac{\\sqrt 3}2 e^{i\\pi/6}$ qui est un nombre complexe!). Par les théorèmes que l'on connait, on en déduit que\n\n$$z_n=\\left(\\frac{\\sqrt 3}2 e^{i\\pi/6}\\right)^n z_0=\\frac{3^{n/2}}{2^n}e^{in\\pi/6}z_0.$$\n\nEn particulier, $z_{12}=\\frac{3^6}{2^{12}}e^{i2\\pi}6=\\frac{3^7}{2^{11}}.$ Le point $A_{12}$ est bien situé sur\n\nla demi-droite $(O,\\vec i)$.\n\n Le vecteur $\\overrightarrow{OA_n}$ est d'affixe $z_n$, le vecteur $\\overrightarrow{OA_{n+1}}$\n\nest d'affixe $z_{n+1}=\\frac{\\sqrt 3}2 e^{i\\pi/6}z_n$, et le vecteur $\\overrightarrow{A_nA_{n+1}}$\n\nest d'affixe $z_{n+1}-z_n=\\left(\\frac{\\sqrt 3}2 e^{i\\pi/6}-1\\right)z_n$. Pour démontrer que le triangle $OA_nA_{n+1}$ est rectangle en $A_{n+1}$, on peut utiliser la réciproque du théorème de Pythagore.\n\nOr,\n\n$$OA_n^2=|z_n|^2,$$\n\n$$OA_{n+1}^2=\\frac 34 |z_n|^2,$$\n\net\n\n$$\\frac{\\sqrt 3}2 e^{i\\pi/6}-1=-\\frac 14+i\\frac{\\sqrt 3}4,$$\n\nce qui donne\n\n$$A_nA_{n+1}^2=\\frac 14 |z_n|^2.$$\n\nOn a bien \n\n$$OA_n^2=OA_{n+1}^2+A_nA_{n+1}^2.$$\n\nUne autre méthode possible est de vérifier que \n\n$\\frac{z_{n+1}-z_n}{z_{n+1}}$ est un imaginaire pur, ce qui prouve l'orthogonalité des vecteurs $\\overrightarrow{A_nA_{n+1}}$ et $\\overrightarrow{OA_{n+1}}$.\n\n On sait que $OA_0=6$ et que $OA_1=6\\times\\frac{\\sqrt 3}2$. Par le théorème de Pythagore,\n\n$$A_0A_1=6\\sqrt{1^2-\\frac 34}=3.$$\n\nNotons alors $d_n$ la longueur du segment $[A_nA_{n+1}]$, de sorte que $d_0=3$.\n\nPuisqu'une similitude de rapport $r$ multiplie les longueurs par $r$, on a\n\n$$d_{n+1}=\\frac{\\sqrt 3}2d_n.$$\n\nLa quantité recherchée est $d_0+\\dots+d_{11}$. Par la formule de la somme d'une suite géométrique, on trouve\n\n$$d_0+\\dots+d_{11}=3\\frac{1-\\frac{3^6}{2^{12}}}{1-\\frac{\\sqrt 3}2}.$$\n\n\n" }, { "question": " 18 - Points à coordonnées entières [Signaler une erreur] [Ajouter à ma feuille d'exos]Enoncé Soit $ABCD$ un carré dans le plan complexe. Prouver que, si $A$ et $B$ sont à coordonnées entières,\n\nil en est de même de $C$ et $D$. Peut-on trouver un triangle équilatéral dont les trois sommets sont à coordonnées entières?", "answer": " On note $a=x+iy$ et $b=x'+iy'$ les affixes respectives de $A$ et $B$. Par hypothèse, $x$, $x'$, $y$ et $y'$ sont\n\ndes entiers. Puisque $ABCD$ est un carré, $D$ est l'image de $B$ par la rotation de centre $A$ et d'angle $\\pi/2$.\n\nTraduit en termes de nombres complexes, si $d$ est l'affixe de $D$, ceci signifie que \n\n$$d-a=i(b-a)\\implies d=a+i(b-a)=x+iy+i\\big((x'-x)+i(y'-y)\\big)=x+y-y'+i(y+x'-x).$$\n\nAinsi, les coordonnées de $D$ sont bien des entiers. \n\nPour prouver que les coordonnées de $C$ sont des entiers, on procède de la même façon, en utilisant cette fois le fait \n\nque $C$ est l'image de $A$ dans la rotation de centre $D$ et d'angle $\\pi/2$.\n\nImaginons maintenant que $ABC$ soit un triangle équilatéral dont les trois sommets sont à coordonnées entières, et\n\ngardons les notations précédentes. Alors, $C$, d'affixe $c=x''+iy''$ est l'image de $B$ par la rotation de\n\ncentre $A$ et d'angle $\\pi/3$. Autrement dit,\n\n$$c-a=e^{i\\pi/3}(b-a)\\implies c=(x+iy)+\\left(\\frac 12+i\\frac{\\sqrt 3}2\\right)\\big((x'-x)+i(y'-y)\\big).$$\n\nOn développe, et après calcul, on trouve que\n\n$$c=x+\\frac{x'-x}2-\\frac{\\sqrt 3(y'-y)}{2}+i\\left(y+\\frac{y'-y}2+\\frac{\\sqrt 3(x'-x)}{2}\\right).$$\n\nPour que la partie réelle de $c$ soit un entier, il est nécessaire que $y=y'$ et pour que la partie imaginaire de $c$ soit nulle,\n\nil est nécessaire que $x=x'$. Finalement, ceci entraine $A=B$, c'est-à-dire que le triangle est réduit à un point!\n" }, { "question": " 19 - Point de Vecten d'un triangle [Signaler une erreur] [Ajouter à ma feuille d'exos]Enoncé On se place dans le plan rapporté à un repère orthonormé $(O,\\vec i,\\vec j)$.\n\n\n Soit $A$ et $B$ deux points du plan, d'affixes respectives $a$ et $b$. Donner les affixes $p$ et $p'$ des centres $P$ et $P'$ des deux carrés de côté $[AB]$.\n\n Soit $ABC$ un triangle du plan. On considère les trois carrés extérieurs aux côtés du triangle, et on note $P$, $Q$ et $R$ les centres respectifs des carrés de côté $[AB]$, $[BC]$ et $[CA]$.\n\n\n Donner les affixes $p$, $q$ et $r$ des points $P$, $Q$ et $R$ en fonction des affixes $a$, $b$ et $c$ des points $A$, $B$ et $C$.\n\n Montrer que les triangles $ABC$ et $PQR$ ont même centre de gravité.\n\n Démontrer que $PR=AQ$ et que les droites $(AQ)$ et $(PR)$ sont perpendiculaires.\n\n Démontrer que les droites $(AQ)$, $(BR)$ et $(CP)$ sont concourantes.\n\n\n", "answer": " \n Soit $C$ l'image de $B$ par la rotation d'angle $-\\pi/2$ et de centre $A$. Notons $c$ son affixe. Alors $c-a=-i(b-a)$ et donc $c=(1+i)a-ib$. Le point $P$ est le milieu de $[BC]$. Son affixe est donc \n\n$$p=\\frac{(1+i)a+(1-i)b}2.$$\n\nNotons ensuite $D$, d'affixe $d$, l'image de $B$ par la rotation de centre $A$ et d'angle $\\pi/2$, de sorte que $d-a=i(b-a)$. Le point $P'$ est le milieu de $[BD]$ et un calcul similiaire prouve que son affixe $p'$ vérifie \n\n$$p'=\\frac{(1-i)a+(1+i)b}2.$$\n\n\n\n\n Il y a une difficulté ici due à la définition du carré \"extérieur\" au triangle $ABC$. On va supposer, comme dans la figure, que $C$ est dans le même demi-plan défini par la droite $(AB)$ que l'image de $B$ par la rotation de centre $A$ et d'angle $-\\pi/2$. Le centre du carré extérieur à $ABC$ de côté $[AB]$ est alors exactement le point $P$ construit à la question précédente. En particulier, il vérifie \n\n$$p=\\frac{(1+i)a+(1-i)b}2.$$\n\nFaisant tourner le rôle de $A$, $B$ et $C$, on en déduit immédiatement que \n\n$$q=\\frac{(1+i)b+(1-i)c}2$$\n\net \n\n$$r=\\frac{(1+i)c+(1-i)a}2.$$\n\n L'affixe du centre de gravité de $ABC$ est $(a+b+c)/3$. L'affixe du centre de gravité de $PQR$ est $(p+q+r)/3$. C'est un calcul élémentaire de vérifier que \n\n$$\\frac{a+b+c}3=\\frac{p+q+r}3;$$\n\n Soit $z_1$ l'affixe du vecteur $\\overrightarrow{AQ}$ et $z_2$ l'affixe du vecteur $\\overrightarrow{PR}$. Alors on a \n\n\\begin{align*}\n\nz_1&=\\frac{c(1-i)}2+\\frac{b(1+i)}2-a\\\\\n\nz_2&=\\frac{c(1+i)}2+\\frac{b(-1+i)}2-ia.\n\n\\end{align*}\n\nPuisque $i(1-i)=1+i$, on en déduit que $z_2=iz_1$, ce qui démontre à la fois que $AQ=PR$ et que les droites $(AQ)$ et $(PR)$ sont perpendiculaires.\n\n D'après la question précédente, $(AQ)$ est la hauteur du triangle $PQR$ issue de $Q$. De même, $(BR)$ est la hauteur du triangle $PQR$ issue de $R$ et $(CP)$ est la hauteur du triangle $PQR$ issue de $P$. Comme les trois hauteurs d'un triangle sont concourantes, on en déduit que les droites $(AQ)$, $(BR)$ et $(CP)$ sont concourantes.\n\n\n\n" }, { "question": " 20 - Triangle équilatéral [Signaler une erreur] [Ajouter à ma feuille d'exos]Enoncé Soient $A$, $B$ et $C$ trois points non alignés d'affixe $a$, $b$ et $c$. On note $j=e^{2i\\pi/3}$.\n\n\n Montrer que le triangle $ABC$ est équilatéral direct si et seulement si $a+bj+cj^2=0$.\n\n On ne suppose pas nécessairement que $ABC$ est équilatéral. On construit à partir de $ABC$ les trois triangles équilatéraux de base $AB$, $AC$ et $BC$ construits à l'extérieur du premier. Montrer que les centres de gravité de ces trois triangles forme un triangle équilatéral.\n\n", "answer": " \n $ABC$ est équilatéral direct si et seulement si $C$ est l'image de $B$ par la rotation de centre $A$ et d'angle $\\pi/3$. En termes de nombres complexes, ceci se caractérise par\n\n$$\\frac{c-a}{b-a}=e^{i\\pi/3}=-j^2\\iff c-a+j^2b-j^2a=0.$$\n\nOr, $1+j^2=-j$ et en multipliant par $j^2$, on obtient le résultat voulu.\n\n Quitte à échanger les rôles de $B$ et $C$, on peut toujours supposer que le triangle est direct,\n\nc'est-à-dire que l'angle $(\\overrightarrow{AB},\\overrightarrow{AC})$ est dans $]0,\\pi[$.\n\nNotons $AC'B$, $BA'C$ et $CB'A$ les triangles équilatéraux directs obtenus. Soient\n\naussi $a',b',c'$ les affixes respectives de $A'$, $B'$ et $C'$. Alors, par la question précédente, on a les\n\n3 équations :\n\n$$\\left\\{\n\n\\begin{array}{rcl}\n\na+jc'+j^2b&=&0\\\\\n\nb+ja'+j^2c&=&0\\\\\n\nc+jb'+j^2a&=&0.\n\n\\end{array}\\right.$$\n\nSoit $E,F,G$ les centres de gravité respectifs de $AC'B$, $BA'C$ et $CB'A$, d'affixe respectives\n\n$e=\\frac13(a+c'+b)$, $f=\\frac13(b+a'+c)$ et $g=\\frac13(c+b'+a)$. D'après la question précédente,\n\nil suffit de prouver que $e+jf+j^2g=0$. Or, \n\n$$3(e+jf+j^2 g)=a+c'+b+jb+ja'+jc+j^2c+j^2b'+j^2a.$$\n\nOr, $c'=-j^2a-jb$, $ja'=-b-j^2c$ et $j^2b'=-jc-a$, ce qui prouve bien que\n\n$e+jf+j^2g=0$. Le triangle $EFG$ est équilatéral direct.\n" } , { "question": " 1 - Un morphisme de corps est injectif [Signaler une erreur] [Ajouter à ma feuille d'exos]Enoncé Soient $K,L$ deux corps et soit $f:K\\to L$ un morphisme d'anneaux.\n\n\n Démontrer que si $x\\in K\\backslash\\{0_K\\}$, alors $f(x)$ est inversible, et déterminer son inverse.\n\n En déduire qu'un morphisme de corps est injectif.\n\n\n", "answer": " \n Soit $x\\in K\\backslash\\{0_K\\}$. Alors on a $x\\cdot x^{-1}=1_K$. On applique $f$ à cette identité, et en utilisant\n\nque $f$ est un morphisme d'anneaux, on trouve\n\n$$f(x)\\cdot f(x^{-1})=1_L.$$\n\nAinsi, $f(x)$ est inversible, d'inverse $f(x^{-1})$.\n\n Il suffit de démontrer que le noyau de $f$ est réduit à $0_K$. Mais si $f(x)=0$, alors\n\n$x\\notin K\\backslash\\{0_K\\}$ d'après la question précédente, et donc $x=0$.\n\n\n" }, { "question": " 2 - Q n'admet pas de sous-corps strict [Signaler une erreur] [Ajouter à ma feuille d'exos]Enoncé Démontrer que $\\mathbb Q$ n'admet pas d'autre sous-corps que lui-même.", "answer": " Soit $K$ un sous-corps de $\\mathbb Q$. Alors $0,1\\in K$. On a ensuite $2=1+1\\in K$, puis $3=2+1\\in K$. Par récurrence, on prouve facilement que $\\mathbb N\\subset K$. Puis, par stabilité par passage à l'opposé, $\\mathbb Z\\subset K$. Enfin, si $x=p/q$ avec $p\\in\\mathbb Z$ et $q\\in \\mathbb N$, $q\\neq 0$, alors $p\\in K$, $q\\in K$, $q\\neq 0$ et donc $p/q\\in K$. Ainsi $\\mathbb Q\\subset K$.\n" }, { "question": " 3 - Q(i) [Signaler une erreur] [Ajouter à ma feuille d'exos]Enoncé Montrer que $\\mathbb Q(i)=\\{a+ib:\\ a,b\\in\\mathbb Q\\}$ est un corps.", "answer": " On va commencer par démontrer que $\\mathbb Q(i)$ est un sous-anneau de $\\mathbb C.$ Pour cela, on remarque que \n\n\n $1\\in\\mathbb Q(i)$\n\n si $z=a+ib$ et $z'=a+ib'\\in\\mathbb Q(i),$ alors\n\n$$z-z'=(a-a')+i(b-b')\\in\\mathbb Q(i)$$\n\net \n\n$$zz'=(a+ib)(a'+ib')=(aa'-bb')+i(ab'+a'b)\\in\\mathbb Q(i).$$\n\n\n\nEnsuite, soit $z=a+ib\\in\\mathbb Q(i),$ $z\\neq 0.$ Alors \n\n$$\\frac1z=\\frac{a-ib}{a^2+b^2}=\\frac{a}{a^2+b^2}+i\\frac{-b}{a^2+b^2}\\in\\mathbb Q(i).$$\n\nAinsi, $1/z$ est dans $\\mathbb Q(i)$ et donc tout élément non nul de $\\mathbb Q(i)$ admet un inverse dans $\\mathbb Q(i)$. Ceci achève de prouver que $\\mathbb Q(i)$ est un corps.\n" }, { "question": " 4 - $\\mathbb Q[\\sqrt d]$ [Signaler une erreur] [Ajouter à ma feuille d'exos]Enoncé Soit $d\\in\\mathbb N$ tel que $\\sqrt d\\notin \\mathbb Q$. On note \n\n$$\\mathbb Q[\\sqrt d]=\\{a+b\\sqrt d;\\ (a,b)\\in\\mathbb Q ^2\\}.$$\n\nDémontrer que $(\\mathbb Q[\\sqrt d],+,\\times)$ est un corps.", "answer": " On va démontrer qu'il s'agit d'un sous-corps de $(\\mathbb R,+,\\times)$. Remarquons d'abord qu'il est bien contenu dans $\\mathbb R$ et que $0,1\\in \\mathbb Q[\\sqrt d]$. Soient $x,y\\in\\mathbb Q[\\sqrt d]$. \n\nOn les écrit $x=a+b\\sqrt d$ et $y=a'+b'\\sqrt d$. Alors :\n\n\\begin{eqnarray*}\n\nx-y&=&(a-a')+(b-b')\\sqrt d\\\\\n\nxy&=&(aa'+dbb')+(ab'+a'b)\\sqrt d\n\n\\end{eqnarray*}\n\nce qui prouve que $x-y$ et $xy\\in\\mathbb Q[\\sqrt d]$. D'autre part, si $x\\neq 0$, alors \n\n$$\\frac 1x=\\frac{1}{a+b\\sqrt d}=\\frac{a-b\\sqrt d}{a^2-b^2d}=\\frac{a}{a^2-b^2 d}-\\frac{b}{a^2-b^2 d}\\sqrt d\\in\\mathbb Q[\\sqrt d]$$\n\net donc $\\frac 1x\\in \\mathbb Q[\\sqrt d]$. Remarquons qu'il était possible de multiplier par la quantité conjuguée qui est non-nulle car $\\sqrt d\\notin \\mathbb Q$. Finalement, on a bien prouvé que $(\\mathbb Q[\\sqrt d],+,\\times)$ est un sous-corps de $(\\mathbb R,+,\\times)$.\n" }, { "question": " 5 - Un corps de matrices [Signaler une erreur] [Ajouter à ma feuille d'exos]Enoncé Soit $\\mathcal C=\\left\\{\\begin{pmatrix}a&-b\\\\b&a\\end{pmatrix}:\\ (a,b)\\in\\mathbb R^2\\right\\}.$ \n\n\n Démontrer que $(\\mathcal C,+\\times)$ est un corps.\n\n Démontrer que $\\mathcal C$ est isomorphe à $\\mathbb C$.\n\n", "answer": " \n On va commencer par démontrer que $\\mathcal C$ est un sous-anneau de $\\mathcal M_2(\\mathbb R)$. Pour cela, on remarque que \n\n\n la matrice $I_2$ est bien élément de $\\mathcal C$ (prendre $a=1$, $b=0$).\n\n Soit $A,A'$ deux matrices de $\\mathcal C$, écrivons les \n\n$$A=\\begin{pmatrix}a&-b\\\\b&a\\end{pmatrix}, A'=\\begin{pmatrix}a'&-b'\\\\b'&a'\\end{pmatrix}.$$\n\nAlors \n\n$$A+A'=\\begin{pmatrix}(a+a')&-(b+b')\\\\(b+b')&(a+a')\\end{pmatrix}\\in\\mathcal C$$\n\net \n\n$$A\\times A'=\\begin{pmatrix}aa'-bb'&-(ab'+a'b)\\\\ab'+a'b&aa'-bb'\\end{pmatrix}\\in\\mathcal C.$$\n\n \n\nDe plus, $(\\mathcal C,+,\\times)$ est bien un anneau commutatif car le calcul précédent montre facilement que $A\\times A'=A'\\times A.$ Pour conclure, il faut encore démontrer que si on choisit $A\\in\\mathcal C$, $A\\neq 0,$ alors $A$ est inversible et $A^{-1}\\in\\mathcal C$. Notons toujours \n\n$$A=\\begin{pmatrix}a&-b\\\\b&a\\end{pmatrix}.$$\n\nIl est facile de voir que $A$ est inversible (par exemple par ce que son déterminant est $a^2+b^2\\neq 0$). De plus, l'inverse de $A$ est \n\n$$A^{-1}=\\frac{1}{a^2+b^2}\\begin{pmatrix}\n\na&b\\\\\n\n-b&a\n\n\\end{pmatrix}$$\n\net cette matrice est bien dans $\\mathcal C$ (choisir $a'=a/(a^2+b^2)$ et $b'=-b/(a^2+b^2)$). Ainsi, on a bien démontré que $\\mathcal C$ est un corps.\n\n Considérons $\\phi(A)=a+ib$, où on a toujours noté \n\n$$A=\\begin{pmatrix}a&-b\\\\b&a\\end{pmatrix}.$$\n\nOn a bien $\\phi(I_2)=1$. De plus, soit\n\n$$A=\\begin{pmatrix}a&-b\\\\b&a\\end{pmatrix}, A'=\\begin{pmatrix}a'&-b'\\\\b'&a'\\end{pmatrix}.$$\n\nAlors d'après le calcul de $A+A'$ fait ci-dessus,\n\n$$\\phi(A+A')=(a+a')+i(b+b')=\\phi(A)+\\phi(A')$$\n\net de même, d'après le calcul de $A\\times A',$\n\n$$\\phi(AA')=(aa-bb')+i(ab'+a'b)=(a+ib)\\times(a'+ib')=\\phi(A)\\phi(A').$$\n\nAinsi, $\\phi$ est bien un morphisme d'anneaux (ou de corps). C'est même un isomorphisme : si $A\\in \\ker(\\phi)$, $\\phi(A)=0\\iff A=0$ et donc \n\n$\\ker(\\phi)=\\{0\\}$. De plus, $\\textrm{Im}(\\phi)=\\mathbb C^*$, \n\npuisque si $z=a+ib\\in\\mathbb C^*$, $z=\\phi(A)$ où $A=\\begin{pmatrix}a&-b\\\\b&a\\end{pmatrix}.$ Ainsi, les corps $\\mathcal C$ et $\\mathbb C$ sont isomorphes.\n\n\n" }, { "question": " 6 - Anneau intègre fini [Signaler une erreur] [Ajouter à ma feuille d'exos]Enoncé Soit $A$ un anneau intègre commutatif fini. Démontrer que $A$ est un corps.", "answer": " Fixons $a\\in A$ non nul et considérons le morphisme de groupes $(A,+)\\to (A,+),\\ x\\mapsto ax$. Alors ce morphisme de groupes est injectif, car son noyau est réduit à $\\{0_A\\}$ puisque $A$ est intègre. Puisque $A$ est fini, ce morphisme est nécessairement bijectif, et donc il existe $x\\in A$ tel que $ax=1_A$. Par commutativité de $A$, on a aussi $xa=1_A$ et donc $a$ admet un inverse : $A$ est un corps.\n\nRemarquons que l'on peut se passer de l'hypothèse que $A$ est commutatif, par exemple en faisant le même raisonnement avec $x\\mapsto xa$, et en prouvant que l'inverse à droite et l'inverse à gauche coïncident.\n" }, { "question": " 7 - Deux corps non isomorphes [Signaler une erreur] [Ajouter à ma feuille d'exos]Enoncé Démontrer que $-1$ est une somme de deux carrés dans $\\mathbb Q[i\\sqrt 2]$. En déduire que les corps $\\mathbb Q[\\sqrt 2]$ et $\\mathbb Q[i\\sqrt 2]$ ne sont pas isomorphes.", "answer": " Il n'est pas très difficile de trouver que \n\n$$-1=(1)^2+(i\\sqrt 2)^2.$$\n\nImaginons qu'il existe un isomorphisme $\\sigma$ de $\\mathbb Q[i\\sqrt 2]$ sur $\\mathbb Q[\\sqrt 2]$. Alors, on sait que $\\sigma(-1)=-1\\sigma(1)=-1$. \n\nMaintenant, $-1=a^2+b^2$ dans $\\mathbb Q[i\\sqrt 2]$ et donc \n\n$$-1=\\sigma(-1)=c^2+d^2$$\n\noù $c=\\sigma(a)$ et $d=\\sigma(b)$. Ainsi, $-1$ est aussi somme de deux carrés dans $\\mathbb Q[\\sqrt 2]$. C'est impossible, puisque les carrés de $\\mathbb Q[\\sqrt 2]$ sont des réels positifs ou nuls! En réalité, on a même prouvé une propriété plus forte : il n'existe pas de morphisme de corps de $\\mathbb Q[i\\sqrt 2]$ vers $\\mathbb Q[\\sqrt 2]$.\n\n\n" }, { "question": " 8 - Condition d'isomorphisme de deux corps [Signaler une erreur] [Ajouter à ma feuille d'exos]Enoncé On rappelle que si $\\alpha\\in\\mathbb N$ est tel que $\\sqrt \\alpha\\notin \\mathbb Q$, alors $\\mathbb Q(\\sqrt \\alpha)=\\{a+b\\sqrt \\alpha; (a,b)\\in\\mathbb Q^2\\}$ est un corps. Soit $\\alpha,\\beta\\in\\mathbb N$ tels que $\\sqrt \\alpha$ et $\\sqrt\\beta$ sont irrationnels. Donner une condition nécessaire et suffisante pour que $\\mathbb Q(\\sqrt \\alpha)$ et $\\mathbb Q(\\sqrt\\beta)$ soient isomorphes.", "answer": " Procédons par analyse-synthèse en commençant par chercher une condition nécessaire. Supposons donc qu'il existe un isomorphisme $\\varphi$ du corps $\\mathbb Q(\\sqrt\\alpha)$ sur le corps $\\mathbb Q(\\sqrt\\beta)$. Pour tout entier $n$, on a $$\\varphi(n)=\\varphi(1+\\dots+1)=\\varphi(1)+\\dots+\\varphi(1)=n.$$ En particulier, on obtient que \n\n$\\varphi(\\alpha)=\\alpha$. Écrivons maintenant que $\\varphi(\\sqrt \\alpha)=c+d\\sqrt \\beta$, avec $c,d\\in\\mathbb Q$. On a alors\n\n$$\\alpha=\\varphi(\\alpha)=\\big(\\varphi(\\sqrt\\alpha)\\big)^2=(c+d\\sqrt \\beta)^2=c^2+d^2\\beta+2cd\\sqrt \\beta.$$\n\nPuisque $\\sqrt\\beta$ est irrationnel, on a nécessairement $cd=0$. Si $d=0$, on a $\\alpha=c^2$ ce qui contredit que $\\sqrt\\alpha$ est irrationnel. Donc on a $c=0$ et $\\sqrt{\\frac\\alpha\\beta}=d$ est rationnel.\n\nRéciproquement, supposons que $\\sqrt{\\frac\\alpha\\beta}$ est égal à un certain rationnel $r$ et définissons $\\varphi$ sur $\\mathbb Q(\\sqrt\\alpha)$ par \n\n$$\\varphi(a+b\\sqrt\\alpha)=a+br\\sqrt\\beta.$$\n\nOn va démontrer que $\\varphi$ est un isomorphisme de corps. Comme c'est clairement un isomorphisme de groupe additif, il suffit de vérifier la compatibilité avec le produit. Mais, \n\n\\begin{eqnarray*}\n\n\\varphi( (a+b\\sqrt\\alpha)(a'+b'\\sqrt \\alpha))&=&\\varphi(aa'+bb'\\alpha+(ab'+ba')\\sqrt\\alpha)=aa'+bb'\\alpha+(ab'+ba')r\\sqrt\\beta\\\\\n\n\\varphi(a+b\\sqrt\\alpha)\\varphi(a'+b'\\sqrt\\alpha)&=&(a+br\\sqrt\\beta)(a'+b'r\\sqrt\\beta)=aa'+bb'r^2\\beta+(ab'+ba')r\\sqrt\\beta.\n\n\\end{eqnarray*}\n\nComme $r^2\\beta=\\alpha$, ces deux quantités sont égales et $\\varphi$ est bien un (iso)morphisme de corps.\n\n \n" }, { "question": " 9 - Produit de tous les éléments [Signaler une erreur] [Ajouter à ma feuille d'exos]Enoncé Soit $K$ un corps fini. Calculer $\\prod_{x\\in K^*}x$.", "answer": " Dans le produit, on regroupe chaque élément avec son inverse.\n\nCeci est possible s'ils sont distincts, et dans ce cas, on peut simplifier\n\nle produit $x x^{-1}=1$. On en déduit que \n\n$$\\prod_{x\\in K^*}x=\\prod_{x=x^{-1}}x=\\prod_{x^2=1}x.$$\n\nOr, dans un corps $K$, l'équation $x^2=1$ a pour solution $1$ et $-1$. Si ces deux nombres\n\nsont distincts, alors le produit vaut $-1$. Si ces deux nombres sont égaux (cas d'un corps de caractéristique $2$), alors le\n\nproduit de ces deux nombres vaut $1=-1$ également. Finalement, dans tous les cas, on a bien\n\n$$\\prod_{x\\in K^*}x=-1.$$\n" }, { "question": " 10 - Le groupe multiplicatif d'un corps fini est cyclique [Signaler une erreur] [Ajouter à ma feuille d'exos]Enoncé Soit $K$ un corps fini. On souhaite démontrer que le groupe multiplicatif $(K^*,\\times)$ est cyclique. On note $n$ le cardinal de ce groupe.\n\n\n Préliminaire 1 : Soit $G$ un groupe abélien, $x$ et $y$ deux éléments de $G$ d'ordres respectifs $p$ et $q$. On suppose que $p$ et $q$ sont premiers entre eux. Démontrer que $xy$ est d'ordre $pq$.\n\n Préliminaire 2 : Soient $a,b\\in\\mathbb N^*$. Démontrer qu'il existe $a',b'$ tels que $a'|a$, $b'|b$, $a'\\wedge b'=1$ et $a'b'=a\\vee b$.\n\n Démontrer qu'il existe dans $K$ un élément d'ordre égal à $m$, le ppcm des ordres des éléments de $K$.\n\n Démontrer que $m\\geq n$, puis conclure.\n\n", "answer": " \n Notons $d$ l'ordre de $xy$. Remarquons que $(xy)^{pq}=(x^p)^q(y^q)^p=e$, et donc $d|pq$. De plus, puisque $(xy)^d=e$, on en déduit que $x^d=y^{-d}$. Il vient alors\n\n$$x^{dq}=(y^{-d})^q=(y^q)^{-d}=e.$$\n\nAinsi, $p|dq$ et puisque $p$ et $q$ sont premiers entre eux, on en déduit que $p|d$. De la même façon, on a $q|d$ et en utilisant à nouveau que $p$ et $q$ sont premiers entre eux, on conclut que $pq|d$. Ainsi, on a bien que $d=pq$.\n\n Écrivons $a=p_1^{\\alpha_1}\\cdots p_r^{\\alpha_r}$ et $b=p_1^{\\beta_1}\\cdots p_r^{\\beta_r}$. Alors posons $a'=p_1^{\\delta_1}\\cdots p_r^{\\delta_r}$ et $b'=p_1^{\\gamma_1}\\cdots p_r^{\\gamma_r}$, où \n\n$$\\delta_i=\\left\\{\n\n\\begin{array}{ll}\n\n0&\\textrm{si }\\alpha_i\\leq \\beta_i\\\\\n\n\\alpha_i&\\textrm{sinon},\n\n\\end{array}\\right.\n\n\\ \n\n\\gamma_i=\\left\\{\n\n\\begin{array}{ll}\n\n\\beta_i&\\textrm{si }\\alpha_i\\leq \\beta_i\\\\\n\n0&\\textrm{sinon}.\n\n\\end{array}\\right.\n\n$$\n\nAlors $a'$ et $b'$ vérifient les conditions imposées. \n\n Soient $x$ et $y$ deux éléments de $K^*$, d'ordres respectifs $a$ et $b$. Soient $a'$ et $b'$ donnés par la question précédente. Alors $x_1=x^{a/a'}$ et $y_1=y^{b/b'}$ sont d'ordres respectifs $a'$ et $b'$. Par la première question $x_1y_1$ est d'ordre $a\\vee b$. On construit alors facilement par récurrence un élément d'ordre le ppcm des ordres des éléments de $K$.\n\n Considérons le polynôme $P(X)=X^m-1$. Alors tous les éléments de $K^*$ sont racines de ce polynôme. Il a donc $n$ racines, ce qui impose, puisque $K$ est un corps, que $m\\geq n$. Mais d'autre part, l'ordre d'un élément dans un groupe divise l'ordre du groupe, en particulier, il lui est inférieur. On a donc aussi $m\\leq n$ et donc $m=n$. Il existe donc dans $K^*$ un élément d'ordre le cardinal du groupe. Ce groupe est cyclique.\n\n\n" }, { "question": " 11 - Degré d'une extension de corps [Signaler une erreur] [Ajouter à ma feuille d'exos]Enoncé Soit $K,L,M$ trois corps tels que $K$ est un sous-corps de $L$ et que $L$ est un sous-corps de $M$.\n\n\n Démontrer que $L$ peut être muni d'une structure de $K$-espace vectoriel, et que $M$ peut être muni d'une structure de $L$-espace vectoriel et d'une structure de $K$-espace vectoriel.\n\n On suppose que $L$ est un $K$-espace vectoriel de dimension finie $n$ et que $M$ est un $L$-espace vectoriel de dimension finie $p$. Démontrer que $M$ est un $K$-espace vectoriel de dimension finie dont on précisera la dimension.\n\n", "answer": " \n On doit prouver que si $E$ et $k$ sont deux corps tels que $k$ est un sous-corps de $E$, alors $E$ est un $k$-espace vectoriel. Il suffit de vérifier la définition…\n\n\n $(E,+)$ est un groupe commutatif.\n\n Pour tous $(u,v)\\in E$ et tous $(a,b)\\in k$, on a bien :\n\n$$a(bu)=(ab)u$$\n\n$$1u=u$$\n\n$$(a+b)u=au+bu$$\n\n$$a(u+v)=au+bv$$\n\ntous les calculs s'effectuant en réalité dans le corps $E$…\n\n\n Soit $(u_1,\\dots,u_n)$ une base de $L$ comme $K$-espace vectoriel et $(v_1,\\dots,v_p)$ une base de $M$ comme $L$-espace vectoriel. On va prouver que $(u_iv_j)_{i=1,\\dots,n, j=1,\\dots,p}$ est une base de $M$ comme $K$-espace vectoriel. En effet, supposons qu'il existe des scalaires (donc des éléments de $\\mathbb K$) $\\lambda_{i,j}$ de sorte que \n\n$$\\sum_{i,j}\\lambda_{i,j}u_iv_j=0.$$\n\nAlors on regroupe la somme en \n\n$$\\sum_{j=1}^p \\left(\\sum_{i=1}^n \\lambda_{i,j} u_i\\right)v_j=0.$$\n\nAlors, la suite $(v_1,\\dots,v_p)$ est libre comme suite du $L$-espace vectoriel $M$, et donc pour tout $j=1,\\dots,p$, on a \n\n$$\\sum_{i=1}^n \\lambda_{i,j} u_i=0.$$\n\nLa famille $(u_1,\\dots,u_n)$ étant libre comme famille du $K$-espace vectoriel $L$, on en déduit que pour tout $i,j$, on a bien $\\lambda_{i,j}=0$. Démontrons maintenant que la famille est génératrice de $M$. Prenons $x\\in M$. Il existe des éléments $y_1,\\dots,y_p\\in L$ tels que \n\n$$x=\\sum_{j=1}^p y_j v_j.$$\n\nMais pour chaque $j=1,\\dots,p$, il existe des éléments $\\lambda_{1,j},\\dots,\\lambda_{n,j}$ de $K$ tels que \n\n$$y_j=\\sum_{i=1}^n \\lambda_{i,j} u_i.$$\n\nOn en déduit que \n\n$$x=\\sum_{i,j}\\lambda_{i,}u_i v_j$$\n\net donc que la famille est aussi génératrice. En conclusion, on a démontré que $M$ est un $K$-espace vectoriel de dimension finie $np$.\n\n\n" }, { "question": " 12 - Degré d'extension et arithmétique [Signaler une erreur] [Ajouter à ma feuille d'exos]Enoncé Soit $K$ un corps. Soit $H_1$, $H_2$ des corps tels que $K\\subset H_i$ pour $i=1,2$. Montrer que si $[H_1:K]$ et $[H_2:K]$ sont premiers entre eux, alors $H_1\\cap H_2=K$.", "answer": " On pose $M=H_1\\cap H_2$. Par multiplicativité des degrés dans la tour d'extension $K\\subset M\\subset H_1$, on a \n\n$$[H_1:K]=[H_1:M]\\times [M:K].$$\n\nDe même, mais en considérant cette fois la tour $K\\subset M\\subset H_2$, on a \n\n$$[H_2:K]=[H_2:M]\\times [M:K].$$\n\nAinsi, $[M:K]$ est un diviseur commun de $[H_1:K]$ et de $[H_2:K]$. Ces deux nombres étant premiers entre eux, on a $[M:K]=1$, autrement dit, $K=M=H_1\\cap H_2$.\n" }, { "question": " 13 - Une extension de degré premier est monogène [Signaler une erreur] [Ajouter à ma feuille d'exos]Enoncé Soit $K$ un corps et $L$ une extension de $K$ de degré premier. Montrer que, pour tout $\\alpha\\in L\\backslash K$, on a $K(\\alpha)=L$.", "answer": " Appliquons la multiplicativité des degrés à l'extension $K\\subset K(\\alpha)\\subset L$. Alors on a\n\n$$[L:K]=[L:K(\\alpha)]\\times [K(\\alpha):K].$$\n\nComme $[L:K]$ est un nombre premier, on a nécessairement $[K(\\alpha):K]=1$ ou $[K(\\alpha):K]=[L:K]$. Le premier cas est exclu, car on a $K(\\alpha)\\neq K$. C'est donc que $[K(\\alpha):K]=[L:K]$ et $[L:K(\\alpha)]=1$. Autrement dit, $L=K(\\alpha)$.\n" }, { "question": " 14 - Paradoxe de Sierpinski-Mazurkiewicz [Signaler une erreur] [Ajouter à ma feuille d'exos]Enoncé Soit $u$ un nombre complexe transcendant de module 1. Soit\n\n\\begin{eqnarray*}\n\nE&=&\\{P(u);\\ P\\in\\mathbb N[X]\\}\\\\\n\nA&=&\\{(XQ)(u);\\ Q\\in\\mathbb N[X]\\}\\\\\n\nB&=&\\{(R+1)(u);\\ R\\in\\mathbb N[X]\\}.\n\n\\end{eqnarray*}\n\n\n Démontrer que $(A,B)$ forme une partition de $E$.\n\n En déduire qu'il existe un ensemble contenu dans le plan dont il existe une partition en deux parties qui sont isométriques à l'ensemble initial.\n\n", "answer": " \n Il est d'abord clair que $A$ et $B$ sont deux parties de $E$. Prenons $x\\in E$. ll existe $P\\in\\mathbb N[X]$ tel que $x=P(u)$. Si $P(0)=0$, alors $P(X)=XQ(X)$, avec $Q\\in\\mathbb N[X]$ et donc $x=(XQ)(u)\\in A$. Sinon, $P(0)\\in\\mathbb N^*$ et $R(X)=P(X)-1\\in\\mathbb N[X]$. Dans ce cas, $x=(R+1)(u)\\in B$.\n\nProuvons enfin que $A\\cap B=\\varnothing$. Si $x\\in A\\cap B$, $x=XQ(u)=(R+1)(u)$ avec $Q,R\\in\\mathbb N[X]$. Mais alors $S(u)=0$ avec $S=XQ-(R+1)$. Mais $S$ est un polynôme de $\\mathbb Z[X]$ non trivial tel que $S(u)=0$, ce qui contredit que $u$ est transcendant.\n\n On identifie $\\mathbb C$ et le plan euclidien. Alors $A$ est simplement égal à $e^{iu}E$ : c'est donc une rotation de $E$, tandis que $B=E+1$ est un translaté de $B$. $(A,B)$ réalise bien une partition de $E$ en deux parties qui sont toutes les deux isométriques à $E$. Etonnant non? \n\n\n" }, { "question": " 15 - Corps des fractions rationnelles [Signaler une erreur] [Ajouter à ma feuille d'exos]Enoncé Le corps des fractions rationnelles $\\mathbb C(t)$ est-il algébriquement clos?", "answer": " Imaginons que ce corps soit algébriquement clos. Alors le polynôme $X^2-t$ aurait une racine dans\n\n$\\mathbb C(t)$. Autrement dit, on pourrait trouver deux polynômes $P,Q\\in\\mathbb C[t]$ tels que\n\n$$\\left(\\frac{P(t)}{Q(t)}\\right)^2-t=0\\iff P^2(t)=tQ^2(t).$$\n\nMais ceci est impossible, car dans sa décomposition en facteurs irréductibles, le polynôme de gauche $P^2$ n'a\n\nque des facteurs irréductibles à un exposant pair, tandis que celui de droite à un facteur irréductible de degré\n\nimpair, à savoir $t$. Par unicité de la décomposition en facteurs irréductibles dans $\\mathbb C[t]$, on en déduit\n\nle résultat" } , { "question": " 1 - Racine carrée de $X$ [Signaler une erreur] [Ajouter à ma feuille d'exos]Enoncé Démontrer qu'il n'existe pas de fraction rationnelle $F$ tel que $F^2=X$.", "answer": " Si $F$ est une fraction rationnelle, alors $\\deg(F^2)=2\\deg (F)$.\n\nOr, $\\deg(X)=1$, et l'équation $2n=1$ n'a pas de solutions dans $\\mathbb Z$.\n" }, { "question": " 2 - Degré de la dérivée [Signaler une erreur] [Ajouter à ma feuille d'exos]Enoncé Soit $F\\in\\mathbb K(X)$. Montrer que si $\\deg(F')<\\deg(F)-1$, alors $\\deg(F)=0$.", "answer": " On écrit $F=A/B$, avec $\\deg(F)=\\deg(A)-\\deg(B)$. On a $F'=(A'B-AB')/B^2$,\n\net donc $\\deg(F')=\\deg(A'B-AB')-2\\deg(B)$. Or, $\\deg(A'B)=\\deg(AB')=\\deg(A)+\\deg(B)-1$.\n\nSi $\\deg(A'B-AB')=\\deg(A)+\\deg(B)-1$, on aurait $\\deg(F')=\\deg(F)-1$, ce qui n'est pas le cas.\n\nOn a donc $\\deg(A'B-AB')<\\deg(A'B)=\\deg(AB')$. Mais si $A=a_k X^k+\\dots$ et $B=b_nX^n+\\dots,$\n\nalors $A'B=ka_kb_nX^{k+n-1}+\\dots$ et $AB'=na_kb_nX^{k+n-1}+\\dots$. Pour que $\\deg(A'B-AB')$ soit inférieur\n\nstrict à $n+k-1$, il est donc nécessaire que $k=n$, c'est-à-dire que $A$ et $B$ aient le même degré.\n\nCeci signifie exactement $\\deg(F)=0$.\n" }, { "question": " 3 - Pôles et racines [Signaler une erreur] [Ajouter à ma feuille d'exos]Enoncé Soient $p$ et $q$ deux entiers naturels premiers entre eux. Déterminer les racines\n\net les pôles de $(X^p-1)/(X^q-1)$, en précisant leur ordre de multiplicité.", "answer": " Les racines de $X^p-1$ (resp. $X^q-1$) sont les racines $p$-ièmes de l'unité\n\n(resp. $q$-ièmes) et elles sont toutes simples. Mais il peut y avoir des simplifications\n\nentre le numérateur et le dénominateur et on doit déterminer les racines communes.\n\nSoit donc $\\omega$ une racine de $X^p-1$ et de $X^q-1$. Alors $\\omega^p=\\omega^q=1$.\n\nMais, puisque $p$ et $q$ sont premiers entre eux, on peut d'après le théorème de Bézout\n\ntrouver deux entiers $u,v\\in\\mathbb Z$ tels que $pu+qv=1$. Alors\n\n$$\\omega=\\omega^{pu+qv}=(\\omega^p)^u(\\omega^q)^v=1.$$\n\nRéciproquement, $1$ est racine commune des deux polynômes. On a donc prouvé que les racines de la fraction rationnelle\n\nsont les racines $p$-ièmes de l'unité autre que 1. Ses pôles sont les racines $q$-ièmes de l'unité, autre que 1.\n\nLes multiplicités respectives sont à chaque fois égales à 1.\n" }, { "question": " 4 - Primitive de $1/X$ [Signaler une erreur] [Ajouter à ma feuille d'exos]Enoncé Soit $F=P/Q\\in\\mathbb C(X)$ une fraction rationnelle, avec $P\\wedge Q=1$, telle que $F'=1/X$.\n\n\n Démontrer que $X|Q$.\n\n Soit $n\\geq 1$ tel que $X^n|Q$. Démontrer que $X^{n}|Q'$.\n\n Conclure.\n\n", "answer": " \n On a $F'=(P'Q-PQ')/Q^2$, et donc $F'=1/X$ s'écrit $(P'Q-PQ')X=Q^2$. On en déduit que $X|Q^2$,\n\npuis, puisque $X$ est un irréductible de $\\mathbb C[X]$, que $X|Q$. \n\n Si $X^n|Q$, alors on peut écrire $Q^2=X^{2n}R$. L'égalité $(P'Q-PQ')X=Q^2$ devient\n\n$(P'Q-PQ')=X^{2n-1}R$. On en déduit que $X^n|P'Q-PQ'$. Or, $X^n|P'Q$ et donc $X^n|Q'P$.\n\nOr, $P\\wedge Q=1$ et $X^n|Q$, donc $X^n\\wedge P=1$ et donc $X^n|Q'$.\n\n Soit $n$ l'ordre de 0 comme racine de $Q$. La première question nous dit que $n\\geq 1$.\n\nLa deuxième question nous dit que $0$ est aussi racine de $Q'$ de multiplicité $n$. Ainsi,\n\n$0$ doit être racine de $Q$ de multiplicité $n+1$, ce qui est absurde. Il n'existe donc pas\n\nde fraction rationnelle $F$ telle que $F'=1/X$.\n\n\n" }, { "question": " 5 - Fraction rationnelle à valeurs rationnelles sur les entiers [Signaler une erreur] [Ajouter à ma feuille d'exos]Enoncé Soit $R(X)=\\frac{P(X)}{Q(X)}$ une fraction rationnelle de $\\mathbb R[X]$ avec $P\\wedge Q=1$ et telle que $R(n)\\in\\mathbb Q$ pour une infinité d'entiers $n\\in\\mathbb N$. On veut démontrer que $R(x)=\\frac{P_1(X)}{Q_1(X)}$ où $P_1,Q_1\\in\\mathbb Z[X]$. On note $\\omega(R)=\\deg(P)+\\deg(Q)$.\n\n\n Démontrer le résultat si $\\omega(R)=0$.\n\n Soit $d\\geq 0$. On suppose que le résultat est vrai pour toute fraction rationnelle $R$ tel que $\\omega(R)\\leq d$ et on souhaite le prouver pour toute fraction rationnelle telle que $\\omega(R)=d+1$. On fixe donc $R=P/Q$ une fraction rationnelle telle que $\\omega(R)=d+1$ et $R(n)\\in\\mathbb Q$ pour tout\n\n$n\\in\\mathbb N$.\n\n\n Pourquoi peut-on supposer que $\\deg(P)\\geq \\deg(Q)$?\n\n Soit $n\\in\\mathbb N$ tel que $Q(n)\\neq 0$ et $P(n)/Q(n)\\in\\mathbb Q$. On pose \n\n$$P_0(X)=\\frac{P(X)Q(n)-Q(X)P(n)}{Q(n)(X-n)}.$$\n\nJustifier que $P_0\\in\\mathbb R[X]$ avec $\\deg(P_0)<\\deg(P)$.\n\n Conclure.\n\n\n", "answer": " \n Si $\\omega(R)=0$, alors $R$ est le quotient de deux réels $R=p/q$. L'hypothèse entraîne que $R$ est à valeurs dans $\\mathbb Q$, c'est-à-dire que $p/q$ est rationnel. On peut donc écrire $p/q=p_1/q_1$ avec $p_1,q_1\\in\\mathbb Z$.\n\n\n\n Si $\\deg(P)<\\deg(Q)$, il suffira de prouver le résultat à la fraction rationnelle $1/R$ qui vérifie les mêmes hypothèses que $R$, si ce n'est que cette fois le degré de son numérateur sera supérieur à celui de son dénominateur.\n\n Considérons $S$ le polynôme $S(X)=P(X)Q(n)-Q(X)P(n)$. Alors, puisque $\\deg(P)\\geq \\deg(Q)$, $\\deg(S)\\leq \\deg(P)$. De plus, $S(n)=0$. Ainsi, $S$ se factorise en $S(X)=(X-n)S_1(X)$ avec $\\deg(S_1)<\\deg(P)$. Mais alors, $P_0=\\frac{S_1}{Q(n)}$ est bien un élément de $\\mathbb R[X]$ de degré inférieur strict au degré de $P$.\n\n Considérons $T(X)=\\frac{P_0(X)}{Q(X)}$. Alors $\\omega(T)=\\deg(P_0)+\\deg(Q)<\\omega(R)$. De plus, si $m\\neq n$ est un entier tel que $R(m)\\in\\mathbb Q$, on a, \n\n$$T(m)=\\frac{P(m)Q(n)-Q(m)P(n)}{Q(m)Q(n)(m-n)}=\\frac{P(m)}{Q(m)(n-m)}-\\frac{P(n)}{Q(n)(n-m)}\\in\\mathbb Q.$$\n\n Ainsi, $T(m)\\in\\mathbb Q$ pour une infinité d'entiers $m$. D'après l'hypothèse de récurrence, \n\n$T=\\frac{P_2}{Q_2}$ \n\noù $P_2,Q_2\\in \\mathbb Z[X]$. On remonte ensuite à $R$ en remarquant que \n\n$$R(X)=(X-n)T(X)+\\frac{P(n)}{Q(n)}.$$\n\nPuisque $P(n)/Q(n)$ est rationnel, il s'écrit encore $c/d$ avec $c,d\\in\\mathbb Z$.\n\nAinsi, mettant tout au même dénominateur, on a \n\n$$R(X)=\\frac{P_1(X)}{Q_1(X)}$$\n\noù\n\n$$P_1(X)=d(X-n)P_2(X)+c$$\n\net $$Q_1(X)=dQ_2(X)$$\n\nqui sont bien des polynômes à coefficients entiers.\n\n\n\n" }, { "question": " 6 - Tous les cas possibles [Signaler une erreur] [Ajouter à ma feuille d'exos]Enoncé Décomposer sur $\\mathbb R$ les fractions rationnelles suivantes :\n\n$$\\begin{array}{lll}\n\n\\displaystyle\\mathbf{1.}\\quad\\frac{X^2+2X +5}{X^2-3X+2}&\\quad\\quad\\mathbf{2.}\\quad \\displaystyle\\frac{X^2+3X+1}{(X-1)^2(X-2)}\n\n&\\quad\\quad\\mathbf{3.}\\quad \\displaystyle \\frac 1{X^4-1}\n\n\\end{array}$$", "answer": " Méthode pour décomposer une fraction rationnelle en éléments simples et correction de l'exercice\n\n\n\n \n On commence par calculer la partie entière de la fraction rationnelle : \n\n$$X^2+2X+5=1(X^2-3X+2)+5X+3.$$\n\n On factorise le dénominateur $$X^2-3X+2=(X-1)(X-2).$$\n\n On écrit la décomposition en éléments simples a priori : \n\n$$\\frac{X^2+2X+5}{X^2-3X+2}=1+\\frac a{X-1}+\\frac{b}{X-2}.$$\n\n On calcule les coefficients $a$ et $b$ : posons $P(X)=X^2+2X+5$, $Q(X)=X^2-3X+2$, $Q'(X)=2X-3.$ \n\n$$a=\\frac{P(1)}{Q'(1)}=\\frac{8}{-1}=-8$$\n\n$$b=\\frac{P(2)}{Q'(2)}=\\frac{13}{1}=13.$$\n\n Ainsi, \n\n$$ \\frac{X^2+2X+5}{X^2-3X+2}=1-\\frac {8}{X-1}+\\frac{13}{X-2}.$$\n\n\n \n Puisque le degré du dénominateur est strictement supérieur au degré du numérateur, la partie entière est nulle.\n\n On a donc \n\n$$\\frac{X^2+3X+1}{(X-1)^2(X-2)}=\\frac{a}{X-2}+\\frac{b}{X-1}+\\frac{c}{(X-1)^2}.$$\n\n Calculons $a$. On pose $P(X)=X^2+3X+1$, $Q(X)=(X-1)^2(X-2)$, $Q'(X)=2(X-1)(X-2)+(X-1)^2$ et \n\n$$a=\\frac{P(2)}{Q'(2)}=\\frac{11}{1}=11.$$\n\n Calculons $c$. On multiplie par $(X-1)^2$ et on évalue en $X=1$.\n\n\\begin{eqnarray*}\n\n\\frac{X^2+3X+1}{X-2}&=&\\frac{11(X-1)^2}{X-2}+b(X-1)+c\\\\ \n\n\\frac{5}{-1}&=& c\\implies c=-5.\n\n\\end{eqnarray*}\n\nOn a donc \n\n$$\\frac{X^2+3X+1}{(X-1)^2(X-2)}=\\frac{11}{X-2}+\\frac{b}{X-1}+\\frac{-5}{(X-1)^2}.$$\n\n Pour calculer $b$, on multiplie par $X$ et on fait tendre $X$ vers l'infini.\n\n$$\\frac{X(X^2+3X+1)}{(X-1)^2(X-2)}=\\frac{11X}{X-2}+\\frac{bX}{X-1}+\\frac{-5X}{(X-1)^2}.$$\n\nQuand $X\\to+\\infty,$\n\n$$1= 11+ b+ 0 \\implies b=-10.$$\n\nConclusion :\n\n$$\\frac{X^2+3X+1}{(X-1)^2(X-2)}=\\frac{11}{X-2}-\\frac{10}{X-1}-\\frac{5}{(X-1)^2}.$$\n\n\n \n La partie entière est nulle.\n\n On factorise le dénominateur : \n\n$$X^4-1=(X^2-1)(X^2+1)=(X-1)(X+1)(X^2+1).$$\n\n On écrit \n\n$$\\frac1{X^4-1}=\\frac a{X-1}+\\frac{b}{X+1}+\\frac{cX+d}{X^2+1}.$$\n\n On pose $P(X)=1$, $Q(X)=X^4-1$ de sorte que $Q'(X)=4X^3$ et\n\n$$a=\\frac{P(1)}{Q'(1)}=\\frac14$$\n\n$$b=\\frac{P(-1)}{Q'(-1)}=\\frac{-1}{4}.$$\n\n Pour calculer $c$ et $d$, on effectue d'abord la décomposition en éléments neutres sur $\\mathbb C$ et on regroupe les termes conjugués.\n\n Décomposons la fraction rationnelle sur $\\mathbb C.$\n\n$$X^4-1=(X-1)(X+1)(X^2+1)=(X-1)(X+1)(X-i)(X+i).$$\n\nOn écrit \n\n$$\\frac 1{X^4-1}=\\frac{a}{X-1}+\\frac{b}{X+1}+\\frac{e}{X-i}+\\frac{f}{X+i}.$$\n\nOn pose $P(X)=1$, $Q(X)=X^4-1$, $Q'(X)=4X^3.$ On a déjà vu $a=1/4$, $b=-1/4$ et on a \n\n$$e=\\frac{P(i)}{Q'(i)}=\\frac{1}{4i^3}=\\frac{1}{-4i}=\\frac{i}4$$\n\n$$f=\\frac{P(i)}{Q'(-i)}=\\frac{1}{4(-i)^3}=\\frac{1}{4i}=\\frac{-i}4.$$\n\nAinsi, \n\n$$\\frac 1{X^4-1}=\\frac{1/4}{X-1}-\\frac{1/4}{X+1}+\\frac{i/4}{X-i}-\\frac{i/4}{X+i}.$$\n\n On termine en passant de $\\mathbb C$ à $\\mathbb R$.\n\n\\begin{eqnarray*}\n\n\\frac 1{X^4-1}&=&\\frac{1/4}{X-1}-\\frac{1/4}{X+1}+\\frac{i/4}{X-i}-\\frac{i/4}{X+i}\\\\ \n\n&=&\\frac{1/4}{X-1}-\\frac{1/4}{X+1}+\\frac 14\\cdot \\frac{i(X+i)-i(X-i)}{(X-i)(X+i)}\\\\ \n\n&=&\\frac{1/4}{X-1}-\\frac{1/4}{X+1}+\\frac 14\\cdot \\frac{-2}{X^2+1}\\\\ \n\n&=&\\frac{1/4}{X-1}-\\frac{1/4}{X+1}-\\frac{1/2}{X^2+1}.\n\n\\end{eqnarray*}\n\n\n\n" }, { "question": " 7 - Pôles simples [Signaler une erreur] [Ajouter à ma feuille d'exos]Enoncé Décomposer en éléments simples les fractions rationnelles suivantes : \n\n$$\\begin{array}{lll}\n\n\\displaystyle\\mathbf{1.}\\quad\\frac{1}{X^3-X}&\\quad\\quad\\mathbf{2.}\\quad \\displaystyle \\frac{X^3}{(X-1)(X-2)(X-3)}\n\n\\end{array}$$", "answer": " \n La partie entière est nulle, et le dénominateur se factorise en $X(X-1)(X+1)$.\n\nMultipliant par $X$ et faisant $X=0$, on trouve la partie polaire relativement à $X-0$, et ainsi de suite...\n\nOn trouve finalement\n\n$$\\frac{1}{X^3-X}=\\frac{-1}{X}+\\frac{1/2}{X-1}+\\frac{1/2}{X+1}.$$\n\n En appliquant exactement la même méthode, on trouve que la décomposition en éléments simples est\n\n$$1+\\frac{1}{2(X-1)}-\\frac{8}{X-2}+\\frac{27}{2(X-3)}.$$\n\n\n" }, { "question": " 8 - Pôles multiples [Signaler une erreur] [Ajouter à ma feuille d'exos]Enoncé Décomposer en éléments simples les fractions rationnelles suivantes : \n\n$$\\begin{array}{lll}\n\n\\displaystyle\\mathbf{1.}\\quad\\frac{2X^2+1}{(X^2-1)^2}&\n\n\\quad\\quad\\mathbf{2.}\\quad\\displaystyle\\frac{X^3+1}{(X-1)^3}\n\n\\end{array}$$", "answer": " \n Le dénominateur se factorise en $(X^2-1)^2=(X-1)^2(X+1)^2$. Il faut donc étudier la partie polaire relative à +1 et à -1.\n\nCommençons par étudier la partie polaire relative à $-1$. La fraction rationnelle s'écrit sous la forme\n\n$$\\frac{2X^2+1}{(X-1)^2(X+1)^2}=\\frac{\\lambda_1}{X+1}+\\frac{\\lambda_2}{(X+1)^2}+G(X),$$\n\noù $G(X)=\\frac{\\mu_1}{X-1}+\\frac{\\mu_2}{(X-1)^2}$ est une fraction rationnelle dont $-1$ n'est pas un pôle. Multipliant cette égalité\n\npar $(X+1)^2$ et faisant $X=-1$, on trouve $\\lambda_2=3/4$. On calcule ensuite \n\n$$\\frac{2X^2+1}{(X-1)^2(X+1)^2}-\\frac{3/4}{(X+1)^2}=\\frac{(5X+1)/4}{(X+1)(X-1)^2}=\\frac{\\lambda_1}{X+1}+G(X).$$\n\nOn multiplie cette fois par $X+1$, et on fait $X=-1$, et on trouve $\\lambda_1=-1/4$. Pour étudier la partie polaire relative à $1$,\n\non peut procéder de la même façon ou simplement remarquer que la fraction rationnelle est paire. On en déduit que\n\n$$\\frac{2X^2+1}{(X-1)^2(X+1)^2}=\\frac{-1/4}{X+1}+\\frac{3/4}{(X+1)^2}+\\frac{1/4}{X-1}+\\frac{3/4}{(X-1)^2}.$$\n\n La partie entière de cette fraction rationnelle est égale à 1, et on a \n\n$$\\frac{X^3+1}{(X-1)^3}=1+\\frac{3X^2-3X+2}{(X-1)^3}=1+\\frac{a}{(X-1)^3}+\\frac{b}{(X-1)^2}+\\frac{c}{X-1}.$$\n\nPour trouver $a$, on multiplie par $(X-1)^3$ et on fait $X=1$. On trouve\n\n$$a=2.$$\n\nPour trouver $b$, on soustrait $\\frac2{(X-1)^3}$, et on trouve\n\n\\begin{eqnarray*}\n\n\\frac{3X^2-3X+2}{(X-1)^3}-\\frac 2{(X-1)^3}&=&\\frac{3X}{(X-1)^2}\\\\\n\n&=&\\frac{b}{(X-1)^2}+\\frac c{X-1}.\n\n\\end{eqnarray*}\n\nMultipliant par $(X-1)^2$ et faisant $X=1$, on trouve\n\n$$b=3.$$\n\nFinalement, on retranche encore $\\frac{3}{(X-1)^2}$ de sorte que\n\n\\begin{eqnarray*}\n\n\\frac{3X}{(X-1)^2}-\\frac{3}{(X-1)^2}&=&\\frac{3}{X-1}\\\\\n\n&=&\\frac{c}{X-1}.\n\n\\end{eqnarray*}\n\nOn a donc $c=3$. Finalement, la décomposition en éléments simples recherché est\n\n$$1+\\frac{2}{(X-1)^3}+\\frac{3}{(X-1)^2}+\\frac{3}{X-1}.$$\n\n\n" }, { "question": " 9 - Pôle multiple et facteur irréductible de degré $2$ [Signaler une erreur] [Ajouter à ma feuille d'exos]Enoncé Décomposer en éléments simples sur $\\mathbb R$ la fraction rationnelle suivante : \n\n$$\\frac{X^4+1}{(X+1)^2(X^2+1)}$$", "answer": " On commence par réaliser la décomposition en éléments simples sur $\\mathbb C$.\n\nLes pôles sont $-1$ (double), $i$ et $-i$. La fraction rationnelle étant à coefficients réels, \n\nles parties polaires sont conjuguées. On a donc\n\n$$\\frac{X^4+1}{(X+1)^2(X^2+1)}=1+\\frac{a}{(X+1)^2}+\\frac{b}{X+1}+\\frac{c}{X-i}+\\frac{\\bar c}{X+i}.$$\n\nMultipliant par $X-i$ et faisant $X=i$, on trouve\n\n$$c=\\frac{i^4+1}{(i+i)(i+1)^2}=-\\frac12.$$\n\nDe même, on a\n\n$$a=\\frac{1+1}{1+1}=1.$$\n\nEn retranchant $\\frac1{(X+1)^2}$, on trouve \n\n$$\\frac{X^4-X^2}{(X+1)^2(X^2+1)}=\\frac{X^2(X-1)}{(X+1)(X^2+1)}=1+\\frac{b}{X+1}+\\frac c{X-i}+\\frac{\\bar c}{X+i}.$$\n\nMultipliant par $X+1$ et faisant $X=-1$, on trouve\n\n$$b=-1.$$\n\nFinalement,\n\n$$\\frac{X^4+1}{(X+1)^2(X^2+1)}=1+\\frac{1}{(X+1)^2}-\\frac{1}{X+1}-\\frac{1/2}{X-i}-\\frac{1/2}{X+i}.$$\n\nFinalement, en regroupant les deux derniers termes, on trouve la décomposition sur $\\mathbb R$ :\n\n$$\\frac{X^4+1}{(X+1)^2(X^2+1)}=1+\\frac{1}{(X+1)^2}-\\frac{1}{X+1}-\\frac{X}{X^2+1}.$$\n" }, { "question": " 10 - Avec paramètres [Signaler une erreur] [Ajouter à ma feuille d'exos]Enoncé Décomposer en éléments simples les fractions rationnelles suivantes :\n\n$$\\begin{array}{lll}\n\n\\displaystyle\\mathbf{1.}\\quad \\frac{1}{X^n-1}&\n\n\\displaystyle\\quad\\quad\\mathbf{2.}\\quad\\frac{X^{n-1}}{X^n-1}&\n\n\\end{array}$$", "answer": " \n Les pôles de $1/(X^n-1)$ sont les racines $n$-ièmes de l'unité, c'est-à-dire les complexes $x_k=e^{2ik\\pi/n}$, $k=0,\\dots,n-1$.\n\nChaque pôle est simple, la partie polaire correspondante est donc de la forme $\\frac{c_k}{X-x_k}$ avec $c_k=\\frac{1}{P'(x_k)}=\\frac{1}{nx_k^{n-1}}$.\n\nOr, $x_k^{n-1}=\\frac{x_k^n}{x_k}=\\frac1{x_k}=e^{-2ik\\pi/n}$. La décomposition en éléments simples recherché vaut donc\n\n$$\\frac1{X^n-1}=\\frac1n\\sum_{k=0}^{n-1}\\frac{e^{2ik\\pi/n}}{X-e^{2ik\\pi/n}}.$$\n\n Les racines de $X^n-1$ sont les complexes $\\omega_k=e^{2ik\\pi/n}$, $0\\leq k\\leq n-1$. \n\nLa fraction rationnelle admet donc $n$ pôles, qui sont tous simples. Sa décomposition en éléments simples a pour forme\n\n$$\\frac{X^{n-1}}{X^n-1}=\\sum_{k=0}^{n-1}\\frac{\\alpha_k}{X-\\omega_k}$$\n\navec $\\alpha_k=\\frac{\\omega_k^{n-1}}{n\\omega_k^{n-1}}=\\frac{1}{n}$. La décomposition en éléments simples est\n\n$$\\frac{X^{n-1}}{X^n-1}=\\frac1n\\sum_{k=0}^{n-1}\\frac{1}{X-e^{2ik\\pi/n}}.$$\n\n\n\n" }, { "question": " 11 - Avec paramètres [Signaler une erreur] [Ajouter à ma feuille d'exos]Enoncé Décomposer en éléments simples les fractions rationnelles suivantes :\n\n$$\\begin{array}{lll}\n\n\\displaystyle\\mathbf{1.}\\quad \\frac{X^m}{(X-1)^n}&\n\n\\displaystyle\\quad\\quad\\mathbf{2.}\\quad\\frac{1}{X(X+1)\\cdots (X+n)}&\n\n\\end{array}$$", "answer": " \n On écrit $X=(X-1)+1$ et on utilise la formule du binôme. Il vient \n\n$$X^m=\\big((X-1)+1\\big)^m =\\sum_{k=0}^m \\binom mk (X-1)^k.$$\n\nOn distingue alors deux cas pour l'écriture de la décomposition en éléments simples : si $m\\geq n,$ alors \n\n$$\\frac{X^m}{(X-1)^n}=\\sum_{k=n}^m \\binom mk (X-1)^{k-n}+\\sum_{k=0}^{n-1}\\frac{\\binom mk}{(X-1)^{n-k}},$$\n\nle premier terme étant la partie entière et le second la partie polaire.\n\nSi $m1,$ $$f(x)=\\frac{3}{x-1}+\\frac{2}{x+3}-\\frac1{(x+3)^2}.$$\n\nOn intègre chacun des éléments simples de la décomposition précédente, en tenant compte du fait que l'on travaille sur l'intervalle $]1,+\\infty[$. Les primitives de $f$ sur cet intervalle sont donc les fonctions\n\n$$F(x)=3\\ln(x-1)+2\\ln(x+3)+\\frac{1}{x+3}+d.$$\n\nLa primitive qui s'annule en $2$ et celle pour laquelle $d$ vérifie l'équation \n\n$$3\\ln(1)+2\\ln 5+\\frac 15+d=0.$$\n\nLa primitive de $f$ sur l'intervalle $]1,+\\infty[$ qui s'annule en 2 est donc la fonction $F$ définie par\n\n$$F(x)=3\\ln(x-1)+2\\ln(x+3)+\\frac{1}{x+3}-2\\ln 5-\\frac 15.$$\n\n\n" }, { "question": " 15 - Calcul d'intégrale [Signaler une erreur] [Ajouter à ma feuille d'exos]Enoncé Pour $x>0,$ on pose \n\n$$f(x)=\\frac{x^4+x^3+4x^2+2x+2}{x^3+x}.$$\n\n\n Décomposer en éléments simples la fraction rationnelle\n\n$$\\frac{X^4+X^3+4X^2+2X+2}{X^3+X}.$$\n\n En déduire la valeur de $\\int_1^2 f(x)dx.$\n\n", "answer": " \n On commence par effectuer la division euclidienne de $X^4+X^3+4X^2+2X+2$ par $X^3+X$. On trouve que le quotient vaut $X+1$ et le reste vaut $3X2+X+2$. On a donc \n\n\\begin{align*}\n\n\\frac{X^4+X^3+4X^2+2X+2}{X^3+X}&=X+1+\\frac{3X^2+X+2}{X^3+X}\\\\\n\n&=X+1+\\frac{3X^2+X+2}{X(X-i)(X+i)}\\\\\n\n&=X+1+\\frac{a}{X}+\\frac{b}{X-i}+\\frac{\\bar b}{X+i}.\n\n\\end{align*}\n\nPosons $P(X)=3X^2+X+2$ et $Q(X)=X^3+X$. Alors $a=P(0)/Q'(0)=2,$\n\n$b=P(i)/Q'(i)=\\frac12-\\frac i2.$\n\nOn obtient\n\n\\begin{align*}\n\n\\frac{X^4+X^3+4X^2+2X+2}{X^3+X}&=X+1+\\frac{2}{X}+\\frac{1}2\\frac{1-i}{X-i}+\\frac 12\\frac{1+i}{X+i}\\\\\n\n&=X+1+\\frac 2X+\\frac{X+1}{X^2+1}.\n\n\\end{align*}\n\n On calcule l'intégrale en cherchant une primitive de chacun des termes précédents. On écrit encore\n\n$$\\frac{X+1}{X^2+1}=\\frac{X}{X^2+1}+\\frac{1}{X^2+1}.$$\n\nOn obtient donc\n\n\\begin{align*}\n\n\\int_1^2 f(x)dx&=\\left[\\frac{x^2}2+x+2\\ln(x)+\\frac{1}2\\ln(x^2+1)+\\arctan(x)\\right]_1^2\\\\\n\n&=\\frac 52+\\frac 32\\ln(2)+\\frac 12\\ln(5)+\\arctan(2)-\\frac \\pi4.\n\n\\end{align*}\n\n\n" }, { "question": " 16 - Dérivée $n$-ème d'une fraction rationnelle [Signaler une erreur] [Ajouter à ma feuille d'exos]Enoncé Soit $n\\geq 1$. Déterminer la dérivée $n$-ème de la fonction suivante :\n\n$$f(x)=\\frac{1}{x^2-3x+2}.$$ ", "answer": " On va commencer par décomposer en éléments simples la fraction rationnelle\n\n$$\\frac{1}{X^2-3X+2}.$$\n\nOn peut factoriser $X^2-3X+2$ en $(X-1)(X-2).$ On a donc\n\n$$\\frac{1}{X^2-3X+2}=\\frac{a}{X-1}+\\frac b{X-2}.$$\n\nNotons $P(X)=X^2-3X+2$ de sorte que $P'(X)=2X-3$. Alors $a=\\frac1{P'(1)}=-1$ \n\net $b=\\frac 1{P'(2)}=1.$ Ainsi, on a\n\n$$f(x)=\\frac{1}{x-2}-\\frac 1{x-1}.$$\n\nD'autre part, il est facile de voir (par exemple par récurrence) que la dérivée $n$-ème de $x\\mapsto \\frac 1x$ est $x\\mapsto \\frac{(-1)^n n!}{x^{n+1}}.$\n\nOn en déduit finalement que \n\n$$f^{(n)}(x)=\\frac{(-1)^n n!}{(x-2)^{n+1}}-\\frac{(-1)^n n!}{(x-1)^{n+1}}.$$\n" }, { "question": " 17 - Un calcul de somme [Signaler une erreur] [Ajouter à ma feuille d'exos]Enoncé \n Décomposer en éléments simples la fraction rationnelle $\\displaystyle\\frac{1}{X(X+1)(X+2)}$.\n\n En déduire la limite de la suite $(S_n)$ suivante : $\\displaystyle S_n=\\sum_{k=1}^n \\frac{1}{k(k+1)(k+2)}$.\n\n", "answer": " \n On sait que la fraction rationnelle admet une décomposition en éléments simples de la forme\n\n$$\\frac{a}{X}+\\frac{b}{X+1}+\\frac{c}{X+2}.$$\n\nPar les techniques usuelles (identification, multiplication par $X$ et faire $X=0$,...), on trouve\n\n$$\\frac{1}{X(X+1)(X+2)}=\\frac{1/2}X-\\frac{1}{X+1}+\\frac{1/2}{X+2}.$$\n\n Utilisant la décomposition en éléments simples précédente, il vient \n\n\\begin{eqnarray*}\n\nS_n&=&\\sum_{k=1}^n \\frac{1/2}{k}-\\sum_{k=1}^n\\frac{1}{k+1}+\\sum_{k=1}^n \\frac{1/2}{k+2}\\\\\n\n&=&\\sum_{k=1}^n \\frac{1/2}{k}-\\sum_{k=2}^{n+1}\\frac{1}{k}+\\sum_{k=3}^{n+2} \\frac{1/2}{k}\\\\\n\n&=&\\frac12+\\frac14-\\frac{1}2-\\frac{1}{n+1}+\\frac{1/2}{n+1}+\\frac{1/2}{n+2}.\n\n\\end{eqnarray*}\n\nOn en déduit immédiatement que $(S_n)$ converge vers $\\frac14$.\n\n\n" }, { "question": " 18 - Un calcul de somme [Signaler une erreur] [Ajouter à ma feuille d'exos]Enoncé Soit $P\\in\\mathbb R[X]$ un polynôme de degré $n\\geq 1$ possédant $n$ racines distinctes $x_1,\\dots,x_n$ non-nulles.\n\n\n Décomposer en éléments simples la fraction rationnelle $\\displaystyle \\frac1{XP(X)}$.\n\n En déduire que $\\displaystyle\\sum_{k=1}^n \\frac{1}{x_k P'(x_k)}=\\frac{-1}{P(0)}$.\n\n\n", "answer": " \n On écrit la forme à priori de la décomposition en éléments simples qui est ici\n\n$$\\frac{1}{XP(X)}=\\frac{\\alpha_0}{X}+\\sum_{k=1}^n \\frac{\\alpha_k}{X-x_k}.$$\n\nOn calcule $\\alpha_0$ en multipliant tout par $X$ et en faisant tendre $X$ vers 0, et on trouve\n\n$\\alpha_0=\\frac{1}{P(0)}$. Pour $k\\geq 1$, on multiplie tout par $X-x_k$ et on fait tendre $X$ vers $x_k$.\n\nOn trouve cette fois\n\n$$\\alpha_k=\\lim_{x\\to x_k} \\frac{x-x_k}{x_k P(x)}=\\lim_{x\\to x_k} \\frac{x-x_k}{x_k (P(x)-P(x_k))}=\\frac{1}{x_k P'(x_k)}.$$\n\nLa décomposition en éléments simples est donc \n\n$$\\frac{1}{XP(X)}=\\sum_{k=1}^n \\frac{1}{x_k P'(x_k)}\\times\\frac{1}{X-x_k}+\\frac{1}{P(0)}\\times\\frac{1}{X}.$$\n\n On va multiplier par $X$ cette fraction rationnelle, et on va étudier sa limite en $+\\infty$. On trouve d'une part\n\n$$\\lim_{x\\to+\\infty}\\frac x{xP(x)}=0$$\n\net d'autre part\n\n$$\\lim_{x\\to +\\infty}\\sum_{k=1}^n\\frac{1}{x_k P'(x_k)}\\times\\frac{x}{x-x_k}+\\frac{1}{P(0)}\\times\\frac{x}{x}=\\sum_{k=1}^n \\frac{1}{x_k P'(x_k)}+\\frac1{P(0)}.$$\n\nIdentifiant ces deux égalités, on trouve le résultat voulu!\n\n\n" }, { "question": " 19 - Racines des polynômes trigonométriques [Signaler une erreur] [Ajouter à ma feuille d'exos]Enoncé Soit $n\\geq 1$, $a_0,\\dots,a_n,b_0,\\dots,b_n$ des réels et $P$ le polynôme trigonométrique défini par\n\n$$P(x)=\\sum_{k=0}^n\\big(a_k\\cos(kx)+b_k\\sin(kx)\\big).$$\n\nDémontrer que $P$ admet au plus $2n$ racines dans $[0,2\\pi[$.", "answer": " D'après les formules d'Euler, on a\n\n\\begin{align*}\n\nP(x)&=\\sum_{k=0}^n\\left( \\frac{a_k}2(e^{ikx}+e^{-ikx})+\\frac{b_k}{2i}(e^{ikx}-e^{-ikx})\\right)\\\\\n\n&=\\sum_{k=0}^n\\left( \\frac{a_k}2\\left((e^{ix})^k+\\frac 1{(e^{ix})^k}\\right)+\\frac{b_k}{2i}\\left((e^{ix})^k-\\frac 1{(e^{ix})^k}\\right)\\right).\n\n\\end{align*}\n\nCeci incite à considérer la fraction rationnelle\n\n$$R(X)=\\sum_{k=0}^n\\left( \\frac{a_k}2\\left(X^k+\\frac 1{X^k}\\right)+\\frac{b_k}{2i}\\left(X^k-\\frac 1{X^k}\\right)\\right)$$\n\nde sorte que\n\n$$P(x)=R(e^{ix}).$$\n\n$R$ peut s'écrire $R=A/B$ où $\\deg(A)\\leq 2n$. Ainsi, $R$ admet au plus $2n$ racines. L'application $[0;2\\pi[\\to \\mathbb C$, $x\\mapsto e^{ix}$ étant injective,\n\ndes racines distinctes de $P$ dans $[0;2\\pi[$ donnent des racines distinctes de $P$. Donc $P$ admet au plus $2n$ racines dans $[0,2\\pi[$.\n" }, { "question": " 20 - Somme de l'inverse des dérivées aux racines [Signaler une erreur] [Ajouter à ma feuille d'exos]Enoncé Soit $P(X)=\\prod_{k=1}^{n}(X-x_k)\\in\\mathbb R_n[X]$ un polynôme scindé à racines simples de degré $n\\geq 2$.\n\n\n Décomposer en éléments simples $1/P$.\n\n En déduire la valeur de $\\sum_{k=1}^n \\frac1{P'(x_k)}$.\n\n", "answer": " \n Les racines de $P$ étant simples, on a \n\n$$P(X)=\\sum_{k=1}^n \\frac{\\lambda_k}{X-x_k}.$$\n\nDe plus, pour tout $k=1,\\dots,n$,\n\n$$\\lambda_k=\\lim_{x\\to x_k}\\frac{x-x_k}{P(x)}=\\frac 1{P'(x_k)}.$$\n\n Multipliant par $X$, on a pour tout $x\\in\\mathbb R$,\n\n$$\\frac x{P(x)}=\\sum_{k=1}^n \\frac 1{P'(x_k)}\\times \\frac x{x-x_k}.$$\n\nFaisons tendre $x$ vers $+\\infty$. Puisque $\\deg(P)\\geq 2$, on obtient\n\n$$\\sum_{k=1}^n \\frac1{P'(x_k)}=0.$$\n\n\n" }, { "question": " 21 - Polynôme et dérivé [Signaler une erreur] [Ajouter à ma feuille d'exos]Enoncé \n Décomposer en éléments simples la fraction $\\frac{P'}P$, où $P$ est un polynôme de $\\mathbb C[X]$.\n\n En déduire les polynômes $P\\in\\mathbb C[X]$ tels que $P'|P$.\n\n", "answer": " \n On va étudier séparément les parties polaires relatives à chaque racine. Soit donc $a$ une racine de $P$,\n\nde multiplicité $m$. Alors on peut factoriser $P$ en $P(X)=(X-a)^m Q(X)$, soit en dérivant \n\n$P'(X)=m(X-a)^{m-1}Q(X)+(X-a)^m Q'(X)$. On trouve alors\n\n$$\\frac{P'(X)}{P(X)}=\\frac{m}{X-a}+\\frac{Q'(X)}{Q(X)}.$$\n\nOr, $a$ n'est pas racine de $Q$, donc $Q'/Q$ n'admet pas $a$ pour pôle et $\\frac{m}{X-a}$ est la partie\n\npolaire de $P'/P$ relative à $a$. En résumé, si $P(X)=\\lambda(X-a_1)^{m_1}\\dots (X-a_p)^{m_p}$, alors on trouve\n\n$$\\frac{P'}{P}=\\sum_{i=1}^p \\frac{m_i}{X-a_i}.$$\n\n Si $P'|P$, alors $P(X)=\\lambda(X-a)P'$ (car $\\deg P'=\\deg P-1$) et $P'/P=\\frac{1/\\lambda}{X-a}$. \n\nPar unicité de la décomposition en éléments simples et le résultat de la question précédente, \n\nceci entraîne que $a$ est l'unique racine de $P$, et\n\ndonc que $P(X)=\\lambda (X-a)^m$. Réciproquement, les polynômes de cette forme sont tels que $P'|P$.\n\n\n" }, { "question": " 22 - Enveloppe convexe des zéros [Signaler une erreur] [Ajouter à ma feuille d'exos]Enoncé Soit $P\\in\\mathbb C_n[X]$ admettant $n$ racines simples $\\alpha_1,\\dots,\\alpha_n$.\n\nSoient $A_1,\\dots,A_n$ les points du plan complexe d'affixe respectives $\\alpha_1,\\dots,\\alpha_n$.\n\n\n Décomposer la fraction rationnelle $P'/P$ en éléments simples.\n\n Soit $\\beta$ une racine de $P'$, et soit $B$ son image dans le plan complexe. Déduire de la question précédente que \n\n$$\\sum_{j=1}^n \\frac{1}{\\beta-\\alpha_j}=0.$$\n\n En déduire que $B$ est un barycentre de la famille de points $(A_1,\\dots,A_n)$, avec des coefficients positifs.\n\nInterpréter géométriquement cette propriété.\n\n", "answer": " \n On va étudier séparément les parties polaires relatives à chaque racine. \n\nOn peut factoriser $P$ en $P(X)=(X-\\alpha_j) Q(X)$, soit en dérivant \n\n$P'(X)=Q(X)+(X-\\alpha_j) Q'(X)$. On trouve alors\n\n$$\\frac{P'(X)}{P(X)}=\\frac{1}{X-\\alpha_j}+\\frac{Q'(X)}{Q(X)}.$$\n\nOr, $\\alpha_j$ n'est pas racine de $Q$, donc $Q'/Q$ n'admet pas $\\alpha_j$ pour pôle et $\\frac{1}{X-\\alpha_j}$ est la partie\n\npolaire de $P'/P$ relative à $a$. En résumé, la décomposition en éléments simples recherchée est\n\n$$\\frac{P'}{P}=\\sum_{j=1}^p \\frac{1}{X-\\alpha_j}.$$\n\n Il suffit d'évaluer l'équation précédente en $\\beta$.\n\n On multiplie par la quantité conjuguée et on trouve\n\n$$\\sum_{j=1}^n \\frac{\\bar\\beta-\\bar\\alpha_j}{|\\beta-\\alpha_j|^2}=0.$$\n\nPrenant le conjugué de cette expression, on trouve :\n\n$$\\left(\\sum_{j=1}^n \\frac{1}{|\\beta-\\alpha_j|^2}\\right)\\beta=\\sum_{j=1}^n \\frac{1}{|\\beta-\\alpha_j|^2}\\alpha_j,$$\n\nce qui correspond bien au résultat souhaité. On vient donc de prouver que toute racine de $P'$ est dans l'enveloppe convexe des racines de $P$.\n\nCe résultat s'appelle le théorème de Lucas, il est aussi valide si les racines de $P$ ne sont pas simples. La preuve\n\nest similaire, si ce n'est que la décomposition en éléments simples de $P'/P$ est plus difficile à obtenir.\n\nC'est un bon exercice!\n" } , { "question": " 1 - Un exemple de groupes [Signaler une erreur] [Ajouter à ma feuille d'exos]Enoncé On définit, pour $(x,y)$ et $(x',y')$ dans $\\mathbb R^*\\times\\mathbb R$, \n\n$$(x,y)\\star (x',y')=(xx',xy'+y).$$\n\n\n Démontrer que $(\\mathbb R^*\\times \\mathbb R,\\star)$ est un groupe. Est-il commutatif?\n\n Simplifier $(x,y)^n$ pour tout $(x,y)\\in\\mathbb R^*\\times\\mathbb R$ et tout $n\\in\\mathbb N^*$.\n\n", "answer": " \n On n'a pas affaire à une loi \"classique\" et donc on ne peut pas démontrer qu'on a un sous-groupe d'un groupe connu. Il faut donc vérifier à la main les trois propriétés d'un groupe ainsi que le fait qu'il s'agit bien d'une loi interne.\n\n\n Si $(x,y)$ et $(x',y')$ sont dans $\\mathbb R^*\\times \\mathbb R$, alors $xx'\\in\\mathbb R^*$ et $xy'+y\\in\\mathbb R$ et donc il s'agit bien d'une loi interne.\n\n La loi $\\star$ est associative : en effet, si $(x,y)$, $(x',y')$ et \n\n$(x'',y'')$ sont dans $\\mathbb R^*\\times\\mathbb R$, alors d'une part\n\n\\begin{align*}\n\n\\big( (x,y)\\star (x',y')\\big)\\star (x'',y'')&=\n\n(xx',xy'+y)\\star (x'',y'')\\\\\n\n&=(xx'x'',xx'y''+xy'+y)\n\n\\end{align*}\n\net d'autre part\n\n\\begin{align*}\n\n (x,y)\\star \\big((x',y')\\star (x'',y'')\\big)&=\n\n(x,y)\\star (x'x'',x'y''+y')\\\\\n\n&=(xx'x'',x(x'y''+y')+y)\\\\\n\n&=(xx'x'',xx'y''+xy'+y).\n\n\\end{align*}\n\n La loi possède un élément neutre, qui est $(1,0)$. Il est en effet facile de vérifier que \n\n$$(x,y)\\star (1,0)=(1,0)\\star (x,y)=(x,y).$$\n\n Tout élément $(x,y)$ possède un inverse. Expliquons comment le trouver. Il n'est pas très difficile de remarquer qu'il doit s'écrire sous la forme $(1/x,a)$. De\n\n$$(x,y)\\star (1/x,a)=(1,xa+y)$$\n\non voit qu'on doit avoir $xa+y=0$ et donc $a=-y/x$. On vérifie alors que $(1/x,-y/x)$ est un inverse de $(x,y)$ : \n\n$$(x,y)\\star (1/x,-y/x)=(1/x,-y/x)\\star (x,y)=(1,0).$$\n\n\n\nLe groupe n'est pas commutatif. En effet on a \n\n$$(1,1)\\star (2,1)=(2,2)$$ \n\nalors que \n\n$$(2,1)\\star (1,1)=(2,3).$$\n\n On remarque que \n\n$$(x,y)^2=(x^2,xy+y)\\textrm{ et }(x,y)^3=(x^3,x^2y+xy+y).$$\n\nOn prouve alors par récurrence sur $n\\in\\mathbb N^*$ que\n\n$$(x,y)^n=(x^n,x^{n-1}y+x^{n-2}y+\\cdots+xy+y).$$\n\n\n" }, { "question": " 2 - Exemples de groupes - avec des fonctions [Signaler une erreur] [Ajouter à ma feuille d'exos]Enoncé Les ensembles suivants munis des lois considérées sont-ils des groupes?\n\n\n $G$ est l'ensemble des fonctions de $\\mathbb R\\to\\mathbb R$ définies par $x\\mapsto ax+b$, avec $a\\in\\mathbb R^*$ et $b\\in\\mathbb R$, muni de la composition;\n\n $G$ est l'ensemble des fonctions croissantes de $\\mathbb R$ dans $\\mathbb R$, muni de l'addition;\n\n $G=\\{f_1,f_2,f_3,f_4\\}$, où\n\n$$f_1(x)=x,\\ f_2(x)=-x,\\ f_3(x)=\\frac 1x,\\ f_4(x)=-\\frac 1x,$$\n\nmuni de la composition.\n\n", "answer": " \n On remarque d'abord que la composition est une loi de composition interne pour $G$. En effet,\n\n$$(ax+b)\\circ (cx+d)=acx+(ad+b).$$\n\nLa loi $\\circ$ est clairement associative (pour toutes les fonctions $f:\\mathbb R\\to\\mathbb R$, on a effectivement $f\\circ(g\\circ h)=(f\\circ g)\\circ h$. La fonction $x\\mapsto x$ est un élément neutre, et l'inverse de $x\\mapsto ax+b$ est donné par $x\\mapsto \\frac 1a x-\\frac ba$ - on trouve cet élément en résolvant le système (d'inconnues $c$ et $d$) \n\n$$\\left\\{\n\n\\begin{array}{rcl}\n\nac&=&1\\\\\n\nad+b&=&0.\n\n\\end{array}\\right.$$\n\n On aurait aussi pu démontrer que $G$ est un sous-groupe du groupe des permutations de $\\mathbb R$.\n\n Imaginons que $G$ soit un groupe. Son élément neutre est alors forcément la fonction identiquement nulle. Mais prenons la fonction $f(x)=x$ (qui est bien croissante). Son inverse serait la fonction $f(x)=-x$, qui n'est pas croissante! Donc $(G,+)$ n'est pas un groupe.\n\n Calculons d'abord le résultat des différentes compositions :\n\n$$f_1\\circ f_1=f_1,\\ f_1\\circ f_2=f_2\\circ f_1=f_2,\\ f_1\\circ f_3=f_3\\circ f_1=f_3,\\ f_1\\circ f_4=f_4\\circ f_1=f_4$$\n\n$$f_2\\circ f_2=f_1,\\ f_2\\circ f_3=f_3\\circ f_2=f_4,\\ f_2\\circ f_4=f_4\\circ f_2=f_3$$\n\n$$f_3\\circ f_3=f_1,\\ f_3\\circ f_4=f_4\\circ f_3=f_2$$\n\n$$f_4\\circ f_4=f_1.$$\n\nDe ces calculs, on tire que :\n\n\n $\\circ$ est bien une loi de composition interne pour $G$. Elle est de plus clairement associative.\n\n $f_1$ est élément neutre pour cette loi.\n\n Chaque élément admet un inverse : lui-même!\n\n\n\n$(G,\\circ)$ est bien un groupe. On pourrait démontrer, toujours à partir du résultat des différentes compositions, qu'il est isomorphe au groupe classique $\\mathbb Z/2\\mathbb Z\\times \\mathbb Z/2\\mathbb Z$.\n\n\n" }, { "question": " 3 - Exemples de groupes [Signaler une erreur] [Ajouter à ma feuille d'exos]Enoncé Montrer que les lois suivantes munissent l'ensemble $G$ indiqué d'une structure de groupe, et préciser s'il est abélien :\n\n\n $x\\star y=\\frac{x+y}{1+xy}$ sur $G=]-1,1[$;\n\n $(x,y)\\star (x',y')=(x+x',ye^{x'}+y'e^{-x})$ sur $G=\\mathbb R^2$;\n\n", "answer": " \n $(G,\\star)$ est un groupe car \n\n\n $\\star$ est une loi de composition interne sur $G$ : en effet, si $x,y\\in G$, alors \n\n$x\\star y\\in G$. Pour prouver cela, fixons $y\\in[-1,1]$ et étudions la fonction définie sur $[-1,1]$ par \n\n$$f(t)=\\frac{t+y}{1+ty}.$$\n\nElle est dérivable sur $[-1,1]$, et sa dérivée vérifie \n\n$$f'(t)=\\frac{1-y^2}{(1+ty)^2}> 0\\textrm{ sur }]-1,1[.$$\n\n$f$ est donc strictement croissante sur $[-1,1]$ et on a \n\n$$f(-1)< x\\star y=f(x)< f(1).$$\n\nComme $f(-1)=(-1+y)/(1-y)=-1$ et $f(1)=(1+y)/(1+y)=1$, on obtient bien que $x\\star y\\in G$.\n\n la loi est associative : pour tout $(x,y,z)\\in G^3$,\n\n\\begin{eqnarray*}\n\nx\\star (y\\star z)&=&\\frac{x+(y\\star z)}{1+x(y\\star z)}\\\\\n\n&=&\\frac{x+\\frac{y+z}{1+yz}}{1+x\\frac{y+z}{1+yz}}\\\\\n\n&=&\\frac{x+y+z+xyz}{1+xy+xz+yz},\n\n\\end{eqnarray*}\n\net un calcul similaire donne le même résultat pour $(x\\star y)\\star z$.\n\n $0$ est un élément neutre pour la loi $\\star$. En effet, \n\n$$x\\star 0=0\\star x=\\frac{x+0}{1+0}=x.$$\n\n Tout élément $x\\in G$ est inversible, d'inverse $-x$. En effet, on a\n\n$$x\\star (-x)=(-x)\\star x=\\frac{x-x}{1-x^2}=0.$$\n\n\n\nDe plus, le groupe est clairement abélien.\n\n Il est clair que $\\star$ est une loi de composition interne sur $\\mathbb R^2$. De plus,\n\n\n cette loi est associative : \n\n\\begin{eqnarray*}\n\n(x,y)\\star \\big((x',y')\\star (x'',y'')\\big)&=&(x,y)\\star (x'+x'',y'e^{x''}+y''e^{-x'})\\\\\n\n&=&(x+x'+x'',ye^{x'+x''}+y'e^{x''}e^{-x}+y''e^{-x'}e^{-x})\\\\\n\n&=&(x+x'+x'',ye^{x'+x''}+y'e^{-x+x''}+y''e^{-x-x'}).\n\n\\end{eqnarray*}\n\nDe même,\n\n\\begin{eqnarray*}\n\n\\big((x,y)\\star (x',y')\\big)\\star (x'',y'')&=&(x+x',ye^{x'}+y'e^{-x})\\star (x'',y'')\\\\\n\n&=&(x+x'+x'',(ye^{x'}+y'e^{-x})e^{x''}+y''e^{-x-x'})\\\\\n\n&=&(x+x'+x'',ye^{x'+x''}+y'e^{-x+x''}+y''e^{-x-x'})\n\n\\end{eqnarray*}\n\net donc on a bien $(x,y)\\star \\big((x',y')\\star (x'',y'')\\big)=\\big((x,y)\\star (x',y')\\big)\\star (x'',y'')$.\n\n $(0,0)$ est un élément neutre de $G$ :\n\n$$(x,y)\\star (0,0)=(x+0,ye^{0}+0e^{-x})=(x,y)$$\n\n$$(0,0)\\star (x,y)=(0+x,0e^{x}+ye^{-0})=(x,y).$$\n\n Tout élément $(x,y)\\in G$ admet un inverse donnée par $(-x,-y)$. En effet,\n\n$$(x,y)\\star (-x,-y)=(x-x,ye^{-x}-ye^{-x})=(0,0),$$\n\n$$(-x,-y)\\star (x,y)=(-x+x,-ye^{x}+ye^{x})=(0,0).$$\n\n\n\nDe plus, le groupe n'est pas abélien, car \n\n$$(1,0)\\star (0,1)=(1,e^{-1})\\textrm{ tandis que }(0,1)\\star (1,0)=(1,e^{1}).$$\n\n\n" }, { "question": " 4 - Un élément est son propre inverse [Signaler une erreur] [Ajouter à ma feuille d'exos]Enoncé Soit $G$ un groupe fini dont l'élément neutre est noté $e$. On suppose que le cardinal de $G$ est pair. Démontrer qu'il existe $x\\in G$ avec $x\\neq e$ tel que $x=x^{-1}$.", "answer": " Notons, pour $x\\in G$, $F_x=\\{x,x^{-1}\\}$. Alors il est facile de voir que l'on a ou bien $F_x=F_y$, ou bien $F_x\\cap F_y=\\varnothing$ pour tout $x,y\\in G$. En effet, si $x=y^{-1}$, alors $x^{-1}=y$ et on a bien $F_x=F_y$. $G$ s'écrit alors comme la réunion disjointe de tous les $F_x$ différents. Au moins l'un parmi ces $F_x$ est de cardinal 1 : il s'agit de $F_e$. Si tous les autres étaient de cardinal 2, alors le groupe serait de cardinal impair, ce qui n'est pas le cas. Il existe donc $x\\neq e$ tel que le cardinal de $F_x$ soit égal à 1, c'est-à-dire tel que $x=x^{-1}$.\n" }, { "question": " 5 - Tout élément est régulier [Signaler une erreur] [Ajouter à ma feuille d'exos]Enoncé Soit $G$ un ensemble fini muni d'une loi de composition interne $\\star$ associative.\n\nOn dit qu'un élément $a$ de $G$ est régulier si les deux conditions suivantes sont réalisées :\n\n\n l'égalité $a\\star x=a\\star y$ entraine $x=y$;\n\n l'égalité $x\\star a=y\\star a$ entraine $x=y$.\n\n\n\nOn suppose que tous les éléments de $G$ sont réguliers, et on fixe $a\\in G$.\n\n\n Démontrer qu'il existe $e\\in G$ tel que $a\\star e=a$.\n\n Démontrer que, pour tout $x\\in G$, on a $e\\star x=x$.\n\n Démontrer que, pour tout $x\\in G$, on a $x\\star e=x$.\n\n Démontrer que $(G,\\star)$ est un groupe.\n\n Le résultat subsiste-t-il si $G$ n'est pas fini?\n\n", "answer": " \n Considérons l'application $\\phi:G\\to G,\\ x\\mapsto a\\star x$. L'hypothèse nous dit que $\\phi$ est injective. Puisque $G$ est fini, et que $\\phi$ va de $G$ dans lui-même, elle est aussi surjective, et donc il existe $e\\in G$ tel que $a\\star e=\\phi(e)=a$.\n\n On a $a\\star e\\star x=a\\star x$ (car la loi est associative) et donc, puisque $a$ est régulier, on a\n\n$e\\star x=x$.\n\n On a $x\\star e\\star a=x\\star a$ d'après la question précédente, et donc puisque $a$ est régulier, on a $x\\star e=x$.\n\n Il suffit désormais de prouver que tout élément est inversible. Soit $b\\in G$. Puisque $x\\mapsto b\\star x$ est injective, donc\n\nsurjective, il existe $c\\in G$ tel que $b\\star c=e$. De plus, on a $c\\star b\\star c=c\\star e=c$, et donc puisque $c$ est régulier, on en déduit\n\nque $c\\star b=e$. Ainsi, $c$ est un inverse de $b$.\n\n Non, ce n'est pas vrai, car $(\\mathbb N,+)$ vérifie que tout élément est régulier, mais ce n'est pas un groupe.\n\n\n" }, { "question": " 6 - Minimisation des axiomes d'un groupe [Signaler une erreur] [Ajouter à ma feuille d'exos]Enoncé Soit $G$ un ensemble muni d'une loi de composition interne $\\cdot$ associative, qui possède un élément neutre à droite $e$ (ie pour tout $x$ de $G$, $x.e=x$) et tel que tout élément $x$ possède un inverse à droite $x'$ (ie $xx'=e$). Montrer que $G$ est un groupe.", "answer": " Soit $x\\in G$, d'inverse à droite $x'$. On va prouver que $x'$ est aussi un inverse à gauche pour $x$. $x'x$ est un élément de $G$, il possède donc un inverse à droite que l'on note $z$. On a donc :\n\n$$x((x'x).z)=x\\implies (xx')x.z=x\\implies e.x.z=x,$$\n\npar associativité de la loi. On multiplie ensuite à gauche par $x'$, pour trouver :\n\n$$(x'x)z=x'x\\implies e=x'x.$$\n\nOn en déduit ensuite que $e$ est aussi neutre à gauche, car si $x$ est dans $G$,\n\n$$ex=(xx'x)=xe=x,$$\n\noù on a encore utilisé l'associativité de la loi et le fait que $e$ est un inverse à droite. \n" }, { "question": " 7 - Sous-groupes ou non? [Signaler une erreur] [Ajouter à ma feuille d'exos]Enoncé Dans les questions suivantes, déterminer si la partie $H$ est un sous-groupe du groupe $G$.\n\n\n $G=(\\mathbb Z,+)$; $H=\\{\\textrm{nombres pairs}\\}$.\n\n $G=(\\mathbb Z,+)$; $H=\\{\\textrm{nombres impairs}\\}$.\n\n $G=(\\mathbb R,+)$; $H=[-1,+\\infty[$.\n\n $G=(\\mathbb R^*,\\times)$; $H=\\mathbb Q^*$.\n\n $G=(\\{\\textrm{bijections de $E$ dans $E$}\\},\\circ)$; $H=\\{f\\in G;\\ f(x)=x\\}$ où $E$ est un ensemble et $x\\in E$.\n\n $G=(\\{\\textrm{bijections de $E$ dans $E$}\\},\\circ)$; $H=\\{f\\in G;\\ f(x)=y\\}$ où $E$ est un ensemble et $x,y\\in E$ avec $x\\neq y$.\n\n", "answer": " \n $H$ est un sous-groupe de $G$. En effet, $0\\in H$, si $x,y\\in H$, alors $-x$ et $x+y$ sont deux entiers pairs\n\n et donc $-x\\in H$, $x+y\\in H$. Le théorème de caractérisation des sous-groupes nous dit que $H$ est un sous-groupe de $G$.\n\n $0\\notin H$ et donc $H$ n'est pas un sous-groupe de $G$.\n\n $2\\in H$ et $-2\\notin H$ : $H$ n'est pas un sous-groupe de $G$.\n\n $1\\in H$. Si $x=p/q$ et $y=p'/q'$ sont deux rationnels non-nuls, alors $1/x=q/p$ et $x\\times y=\\frac{p\\times p'}{q\\times q'}$\n\n sont deux rationnels non nuls. $H$ est un sous-groupe de $G$.\n\n $Id_E$, l'élément neutre de $G$, est élément de $H$. De plus, si $f,g\\in H$, alors \n\n $$f(x)=x\\implies f^{-1}(f(x))=f^{-1}(x)\\implies f^{-1}(x)=x$$\n\n et\n\n $$f\\circ g(x)=f(g(x))=f(x)=x.$$\n\n Ainsi, $f^{-1}$ et $f\\circ g$ sont éléments de $H$, et $H$ est un sous-groupe de $G$.\n\n L'élément neutre de $G$, $Id_E$, n'est pas élément de $H$ qui n'est donc pas un sous-groupe de $G$.\n\n\n" }, { "question": " 8 - Sous-groupes du groupe des matrices inversibles? [Signaler une erreur] [Ajouter à ma feuille d'exos]Enoncé Dire si les parties suivantes de $GL_n(\\mathbb R)$ sont des sous-groupes de $GL_n(\\mathbb R)$.\n\n\n $H_1=\\{A\\in GL_n(\\mathbb R);\\ A\\textrm{ diagonale avec tous ses coefficients diagonaux non-nuls}\\}.$\n\n $H_2=\\left\\{\\begin{pmatrix}a&b\\\\0&1\\end{pmatrix};\\ a>0,\\ b\\in\\mathbb R\\right\\}$ (ici, $n=2$).\n\n $H_3=\\left\\{\\begin{pmatrix}0&1\\\\a&b\\end{pmatrix};\\ a>0,\\ b\\in\\mathbb R\\right\\}$ (ici, $n=2$).\n\n\n", "answer": " Remarquons pour commencer que nous avons bien affaire à des parties de $GL_n(\\mathbb R)$. \n\nOn va appliquer le théorème de caractérisation des sous-groupes pour vérifier si ce sont, ou non, des sous-groupes.\n\n\n\n\n Notons $D(\\lambda_1,\\dots,\\lambda_n)$ la matrice diagonale dont les coefficients diagonaux sont $\\lambda_1,\\dots,\\lambda_n$. Les éléments de \n\n $H_1$ sont les matrices $D(\\lambda_1,\\dots,\\lambda_n)$ avec tous les $\\lambda_i$ non nuls.\n\nAlors on remarque que $I_n=D(1,\\dots,1)\\in H_1$, que si $D(\\lambda_1,\\dots,\\lambda_n)$, $D(\\mu_1,\\dots,\\mu_n)\\in H_1$, alors\n\n$$D(\\lambda_1,\\dots,\\lambda_n)\\cdot D(\\mu_1,\\dots,\\mu_n)=D(\\lambda_1\\mu_1,\\dots,\\lambda_n\\mu_n)\\in H_1$$\n\n$$D(\\lambda_1,\\dots,\\lambda_n)^{-1}=D(1/\\lambda_1,\\dots,1/\\lambda_n)\\in H_1.$$\n\nAinsi, $H_1$ est bien un sous-groupe de $GL_n(\\mathbb R)$.\n\n Notons $M(a,b)$ la matrice $\\begin{pmatrix}a&b\\\\0&1\\end{pmatrix}$ de sorte que $H_2$ est l'ensemble des matrices $M(a,b)$ avec $a>0$ et $b\\in\\mathbb R$.\n\nAlors on a $M(a,b)M(c,d)=M(ac,ad+b)$. Ainsi, on remarque que $I_2=M(1,0)\\in H_2$, que si $M(a,b)$ et $M(c,d)$ sont dans $H_2$, alors leur produit est dans $H_2$\n\n(car $ac>0$). De plus, calculant l'inverse de la matrice $M(a,b)$, on trouve\n\n$$M(a,b)^{-1}=M\\left(\\frac 1a,\\frac{-b}a\\right)\\in H_2.$$\n\nOn en déduit que $H_2$ est un sous-groupe de $GL_2(\\mathbb R)$.\n\n Remarquons que $I_2\\notin H_3$. Ainsi, $H_3$ n'est pas un sous-groupe de $GL_2(\\mathbb R)$.\n\n\n" }, { "question": " 9 - Quelques exemples de sous-groupes [Signaler une erreur] [Ajouter à ma feuille d'exos]Enoncé Démontrer pour chaque question que $H$ est un sous-groupe de $G$.\n\n\n $G=(\\mathbb C^*,\\times)$ et $H=\\{z\\in \\mathbb C^*:\\ \\exists n\\in\\mathbb N,\\ z^n=1\\}.$\n\n $G=(\\mathbb R^*,\\times)$ et $H=\\{a+b\\sqrt 2:\\ a,b\\in\\mathbb Q,\\ (a,b)\\neq (0,0)\\}$.\n\n", "answer": " \n On commence par vérifier que $H\\subset G$ (c'est-à-dire que $0\\notin H$). On vérifie ensuite que $1\\in H$ puisque $1^1=1$. Soit $z_1,z_2\\in H$. Alors il existe $n_1\\in\\mathbb N$ et $n_2\\in\\mathbb N$ tels que $z_1^{n_1}=1$ et $z_2^{n_2}=1$ (attention! on n'a pas forcément $n_1=n_2$). Mais alors \n\n$$(z_1z_2)^{n_1n_2}=(z_1^{n_1})^{n_2}(z_2^{n_2})^{n_1}=1^{n_2}1^{n_1}=1.$$\n\nAinsi, $z_1z_2\\in H$. De plus,\n\n$$\\left(\\frac 1z_1\\right)^{n_1}=\\frac{1}{z_1^{n_1}}=1$$\n\net donc $1/z_1\\in H$. Ainsi, $H$ est bien un sous-groupe de $G$.\n\n $1=1+0\\sqrt 2\\in H$ (attention! la condition $(a,b)\\neq (0,0)$ signifie qu'on ne peut pas avoir $a=0$ ET $b=0$ en même temps, mais il est tout à fait possible de prendre $a=1$ et $b=0$). Si $x=a+b\\sqrt 2$ et $y=c+d\\sqrt 2$ sont éléments de $H$, alors \n\n $$\\frac 1x=\\frac{1}{a+b\\sqrt 2}=\\frac{a-b\\sqrt 2}{a^2-2b^2}=\\frac{a}{a^2-2b^2}+\\frac{-b}{a^2-2b^2}\\sqrt 2$$\n\n est lui aussi un élément de $H$ (on n'a $a^2-2b^2\\neq 0$ si $(a,b)\\neq (0,0)$ car $\\sqrt 2$ est irrationnel). De même, $xy\\in H$ puisque \n\n $$xy=(ac+2bd)+(ad+bc)\\sqrt 2$$\n\n est un élément de $H$. Remarquons que l'on ne peut pas avoir $ac+2bd=0$ et $ad+bc=0$, sinon on aurait $xy=0$ et donc ou bien $x=0$ ou bien $y=0$\n\n ce qui n'est pas le cas. Ainsi $H$ est un sous-groupe de $G$.\n\n\n" }, { "question": " 10 - Exemple de groupes - centre d'un groupe [Signaler une erreur] [Ajouter à ma feuille d'exos]Enoncé Montrer que l'ensemble $G$ des matrices de la forme $\\begin{pmatrix}1&x&z\\\\0&1&y\\\\ 0&0&1\\end{pmatrix}$ est un groupe pour le produit matriciel. Déterminer son centre, c'est-à-dire les matrices $A$ de $G$ telles que $AB=BA$ pour tout $B\\in G.$", "answer": " On va démontrer que $G$ est un sous-groupe de $GL_3(\\mathbb R)$. En effet, \n\n$I_3\\in G$. De plus, si $A=\\begin{pmatrix}1&x&z\\\\0&1&y\\\\ 0&0&1\\end{pmatrix}$\n\net $B=\\begin{pmatrix}1&x'&z'\\\\0&1&y'\\\\ 0&0&1\\end{pmatrix}$, alors \n\n$$AB=\\begin{pmatrix}1&x+x'&z+z'+xy'\\\\0&1&y+y'\\\\ 0&0&1\\end{pmatrix}$$\n\nest bien un élément de $G.$ De plus, le calcul précédent montre que \n\n$$A^{-1}=\\begin{pmatrix}1&-x&-z+xy\\\\0&1&-y\\\\ 0&0&1\\end{pmatrix}$$\n\net donc que $A^{-1}$ est un élément de $G.$\n\nSoit maintenant $A=\\begin{pmatrix}1&x&z\\\\0&1&y\\\\ 0&0&1\\end{pmatrix}\\in G.$\n\nAlors $A$ est dans le centre de $G$ si et seulement si, pour tout $B=\\begin{pmatrix}1&x'&z'\\\\0&1&y'\\\\ 0&0&1\\end{pmatrix}$, $AB=BA.$\n\nLe seul coefficient qui diffère entre $AB$ et $BA$ est le coefficient à droite de la première ligne, et on trouve \n\n$$AB=BA\\iff xy'=x'y.$$\n\nChoisissant $y'=1$ et $x'=0$, on trouve $x=0$ puis choisissant $x'=1$ et $y'=0$, on trouve $y=0$. Réciproquement, si $x=y=0,$ on a bien $AB=BA.$ Le centre de \n\n$G$ est donc constitué des matrices $\\begin{pmatrix}1&0&z\\\\0&1&0\\\\ 0&0&1\\end{pmatrix}.$\n" }, { "question": " 11 - Quelques sous-groupes usuels [Signaler une erreur] [Ajouter à ma feuille d'exos]Enoncé Soit $(G,\\cdot)$ un groupe. Démontrer que les parties suivantes sont des sous-groupes de $G$ :\n\n\n $C(G)=\\{x\\in G;\\ \\forall y\\in G, xy=yx\\}$ ($C(G)$ s'appelle le centre de $G$);\n\n $aHa^{-1}=\\{aha^{-1};\\ h\\in H\\}$ où $a\\in G$ et $H$ est un sous-groupe de $G$.\n\n On suppose de plus que $G$ est commutatif. On dit que $x$ est un élément de torsion de $G$ s'il existe $n\\in\\mathbb N^*$ tel que $x^n=e$. Démontrer que l'ensemble des éléments de torsion de $G$ est un sous-groupe de $G$.\n\n", "answer": " Il suffit, pour chaque cas, d'appliquer le théorème de caractérisation des sous-groupes.\n\n\n $e$ est élément de $C(G)$ car $ey=ye=y$ pour tout $y\\in G$. Soient $x_1,x_2\\in C(G)$. Alors, \n\npour tout $y\\in G$, on a \n\n$$x_1x_2y=x_1(x_2y)=(x_1y)x_2=yx_1x_2$$\n\net donc $x_1x_2\\in C(G)$. Enfin, si $x\\in C(G)$, alors pour tout $y\\in G$,\n\n$$xy=yx\\implies xyx^{-1}=yxx^{-1}=y\\implies x^{-1}xyx^{-1}=x^{-1}y\\implies yx^{-1}=x^{-1}y$$\n\noù on a multiplié à droite puis à gauche par $x^{-1}$. On en déduit que $x^{-1}\\in C(G)$ qui est donc\n\nun sous-groupe de $G$.\n\n Puisque $H$ est un sous-groupe de $G$, $e\\in H$ et donc $aea^{-1}\\in aHa^{-1}$. Mais $aea^{-1}=e$ et donc $e\\in aHa^{-1}$.\n\nSoient $x=aha^{-1}$ et $y=ah'a^{-1}$ deux éléments de $aHa^{-1}$ avec donc $h,h'\\in H$.\n\nOn a \n\n$$xy=aha^{-1}ah'a^{-1}=ahh'a^{-1}\\in aHa^{-1}$$\n\npuisque $hh'\\in H$ ($H$ est un sous-groupe de $G$). Enfin, si on choisit $h'=h^{-1}$, le calcul précédent montre que \n\n$$xy=yx=e$$\n\net donc $x^{-1}=y\\in aHa^{-1}$\n\npuisque $h^{-1}\\in H$. $aHa^{-1}$ est donc bien un sous-groupe de $G$.\n\n Notons $T$ l'ensemble des éléments de torsion de $G$. On a $e^1=e$, donc $e\\in T$. De plus, si $x,y\\in T$, avec respectivement $x^n=e$ et $y^m=e$, il suffit de remarquer que \n\n$$(y^{-1})^m=(y^m)^{-1}=e^{-1}=e$$\n\npuis d'utiliser le fait que $x$ et $y^{-1}$ commutent pour prouver que\n\n$$(xy^{-1})^{nm}=(x^n)^m((y^{-1})^m)^n=e.$$\n\nAinsi, $xy^{-1}$ est élément de $T$, et $T$ est bien un sous-groupe de $G$.\n\n\n" }, { "question": " 12 - Inversibles à coefficients dans $\\mathbb Z$. [Signaler une erreur] [Ajouter à ma feuille d'exos]Enoncé On note $GL_n(\\mathbb Z)$ l'ensemble des matrices de $\\mathcal M_n(\\mathbb R)$, à coefficients dans $\\mathbb Z$, qui sont inversibles et dont l'inverse est à coefficients dans $\\mathbb Z$.\n\n\n Démontrer que si $M$ est à coefficients dans $\\mathbb Z$, alors $M\\in GL_n(\\mathbb Z)$ si et seulement si $\\det(M)=\\pm 1$.\n\n En déduire que $GL_n(\\mathbb Z)$ est un sous-groupe de $GL_n(\\mathbb R)$.\n\n", "answer": " \n Prenons d'abord $M\\in GL_n(\\mathbb Z)$. Alors on a $$\\det(M)\\times \\det(M^{-1})=\\det (MM^{-1})=\\det(I_n)=1$$\n\net de plus $\\det(M)$ et $\\det(M^{-1})$ sont des éléments de $\\mathbb Z$. Ceci n'est possible que si $\\det(M)$ et $\\det(M^{-1})$ sont égaux à $1$ ou $-1$. Réciproquement, si $\\det(M)=\\pm 1$, alors les formules de Cramer nous disent que \n\n$$M^{-1}=\\frac 1{\\det M} (\\textrm{comat }M)^{T}.$$\n\nLa comatrice d'une matrice à coefficients dans $\\mathbb Z$ étant à coefficients dans $\\mathbb Z$ et $\\det(M)$ valant $\\pm 1$, on a bien que $M^{-1}$ est une matrice à coefficients entiers.\n\n On remarque d'abord que $I_n\\in GL_n(\\mathbb Z)$. Ensuite, si $A,B\\in GL_n(\\mathbb Z)$, des formules\n\n$$\\det(A^{-1})=\\frac1{\\det(A)}$$\n\net \n\n$$\\det(AB)=\\det(A)\\det(B)$$\n\non déduit facilement que $\\det(A^{-1})$ et $\\det(AB)$ sont éléments de $\\{-1,1\\}$ et donc $A^{-1}$, $AB$ sont éléments de $GL_n(\\mathbb Z)$.\n\n\n" }, { "question": " 13 - Sous-groupe d'une courbe [Signaler une erreur] [Ajouter à ma feuille d'exos]Enoncé Montrer que $H=\\{x+y\\sqrt 3;\\ x\\in\\mathbb N,\\ y\\in\\mathbb Z,\\ x^2-3y^2=1\\}$ est un sous-groupe de $(\\mathbb R_+^*,\\times)$.", "answer": " La première chose à remarquer est que $H\\subset \\mathbb R_+^*$. Pour $x+y\\sqrt 3\\in H$, puisque $x^2-3y^2>0$ et $x\\in\\mathbb N$, on a $x>\\sqrt 3|y|$ et donc $x+y\\sqrt 3>0$. On remarque ensuite que $1=1+0\\sqrt 3$ est bien un élément de $H$. Soient $a=x+y\\sqrt 3$ et $b=u+v\\sqrt 3$ deux éléments de $H$. Alors :\n\n$$(x+y\\sqrt 3)(u+v\\sqrt 3)=(xu+3yv)+\\sqrt 3(xv+yu).$$\n\nOn remarque ensuite que \n\n\\begin{eqnarray*}\n\n(xu+3yv)^2-3(xv+yu)^2&=&x^2u^2+9y^2v^2-3x^2v^2-3y^2u^2\\\\\n\n&=&x^2(u^2-3v^2)+3y^2(3v^2-u^2)\\\\\n\n&=&x^2-3y^2\\\\\n\n&=&1.\n\n\\end{eqnarray*}\n\nDe plus, il est clair que $xu+3yv$ et $xv+yu$ sont éléments de $\\mathbb Z$. Il reste à voir que \n\n$xu+3yv$ est élément de $\\mathbb N$. Mais c'est clair car $x\\geq \\sqrt 3 |y|$ et $u\\geq \\sqrt 3 |v|$. Ainsi, $ab\\in H$.\n\nDémontrons finalement que $H$ est bien stable par passage à l'inverse. On a \n\n$$\\frac 1a=\\frac 1{x+y\\sqrt 3}=\\frac {x-y\\sqrt 3}{x^2-3y^2}=x-y\\sqrt 3\\in H$$\n\npuisque $x^2-3y^2=1$. Ainsi, $H$ est bien un sous-groupe de $(\\mathbb R_+^*,\\times)$.\n\n\n" }, { "question": " 14 - Sous-groupe engendré par une partie [Signaler une erreur] [Ajouter à ma feuille d'exos]Enoncé Dans cet exercice, $G$ désigne un groupe.\n\n\n Soit $(H_i)_{i\\in I}$ une famille quelconque de sous-groupes de $G$.\n\nDémontrer que $\\bigcap_{i\\in I}H_i$ est un sous-groupe de $G$.\n\n Soit $X$ une partie de $G$. On note $\\langle X\\rangle$ l'intersection de tous les sous-groupes de $G$ contenant $X$. Démontrer que $\\langle X\\rangle$\n\nest le plus petit sous-groupe de $G$ contenant $X$.\n\n Démontrer que \n\n$$\\langle X\\rangle=\\left\\{x_1^{\\veps_1}\\cdots x_n^{\\veps_n}:\\ n\\in\\mathbb N,\\ x_i\\in X, \\veps_i=\\pm 1\\textrm{ pour }i=1,\\dots,n\\right\\}$$\n\n(avec la convention qu'un produit vide vaut $1_G$).\n\n", "answer": " \n Notons $H=\\bigcap_{i\\in I}H_i$. \n\n\n $1_G\\in H$ puisque $1_G$ appartient à tous les $H_i$. \n\n Soit $x,y\\in H$. Pour tout $i\\in I$, $x\\in H_i$ et $y\\in H_i$. Puisque $H_i$ est un sous-groupe de $G$, on a $xy\\in H_i$ et comme c'est vrai pour tout $i\\in I$, $xy\\in H$. \n\n Soit $x\\in H$. Pour tout $i\\in I$, $x\\in H_i$ et donc $x^{-1}\\in H_i$ puisque $H_i$ est un sous-groupe de $G$. Puisque c'est vrai pour tout $i\\in I$, $x^{-1}\\in H$. \n\n\n\nFinalement, on a prouvé que $H$ est un sous-groupe de $G$. \n\n Notons $(H_i)_{i\\in I}$ la famille des sous-groupes de $G$ qui contiennent $X$, de sorte que $\\langle X\\rangle=\\bigcap_{i\\in I}H_i$. Alors, d'après la question précédente, $\\langle X\\rangle$ est bien un sous-groupe de $G$. Puisque $X\\subset H_i$ pour tout $i\\in I$, $X\\subset \\langle X\\rangle$. Puisque $\\langle X\\rangle$ est contenu dans tout sous-groupe de $G$ contenant $X$, $\\langle X\\rangle$ est bien le plus petit sous-groupe de $G$ contenant $X$.\n\n Posons\n\n$$H=\\left\\{x_1^{\\veps_1}\\cdots x_n^{\\veps_n}:\\ n\\in\\mathbb N,\\ x_i\\in X, \\veps_i=\\pm 1\\textrm{ pour }i=1,\\dots,n\\right\\}.$$\n\nOn commence par remarquer que $H$ est un sous-groupe de $G$. En effet, \n\n\n $1_G\\in H$ (cas du produit vide)\n\n si $x,y\\in H$, on écrit $x=x_1^{\\veps_1}\\cdots x_n^{\\veps_n}$ et $y=y_1^{\\nu_1}\\cdots y_m^{\\nu_m}$. Alors \n\n$$xy=x_1^{\\veps_1}\\cdots x_n^{\\veps_n}y_1^{\\nu_1}\\cdots y_m^{\\nu_m}$$\n\nest bien dans $H$.\n\n si $x\\in H$, on écrit $x=x_1^{\\veps_1}\\cdots x_n^{\\veps_n}$ d'où\n\n$$x^{-1}=x_n^{-\\veps_n}\\cdots x_1^{-\\veps_1}$$\n\nqui est bien élément de $H$ puisque $-\\veps_i\\in\\{-1,1\\}$.\n\n\n\nOn remarque ensuite que $X\\subset H$ (cas du produit avec un seul élément). On obtient donc $\\langle X\\rangle \\subset H$. Reste à démontrer l'inclusion réciproque. Pour cela, on va prouver par récurrence sur $n\\in\\mathbb N$ la propriété suivante :\n\n$$\\mathcal P_n=\"\\textrm{tout }x\\in G\\textrm{ qui s'écrit }x=x_1^{\\veps_1}\\cdots x_n^{\\veps_n}\\textrm{ avec }x_i\\in X,\\ \\veps_i=\\pm 1\\textrm{ appartient à }\\langle X\\rangle\".$$\n\nInitialisation : soit $x\\in G$ tel que $x=x_1$ ou $x=x_1^{-1}$ avec $x_1\\in X$. Alors si $x=x_1$, $x\\in \\langle X\\rangle$. Si $x=x_1^{-1}$, alors, comme $x_1\\in\\langle X\\rangle$ et que $\\langle X\\rangle$ est un sous-groupe de $G$, on a $x=x_1^{-1}\\in \\langle X\\rangle$.\n\nHérédité : soit $n\\in\\mathbb N^*$ tel que $\\mathcal P_n$ est vraie. Soit $x=x_1^{\\veps_1}\\dots x_n^{\\veps_n}x_{n+1}^{\\veps_{n+1}}.$ Alors\n\npar hypothèse de récurrence, $x_1^{\\veps_1}\\dots x_n^{\\veps_n}\\in \\langle X\\rangle$ et comme dans l'initialisation, $x_{n+1}^{\\veps_{n+1}}\\in\\langle X\\rangle$. Puisque $\\langle X\\rangle$ est un groupe, \n\n$$x_1^{\\veps_1}\\dots x_n^{\\veps_n}x_{n+1}^{\\veps_{n+1}}\\in\\langle X\\rangle.$$\n\nOn a donc prouvé par récurrence que pour tout $n\\in\\mathbb N^*$, $\\mathcal P_n$ est vraie. Ceci signifie exactement que $H\\subset\\langle X\\rangle$. On a donc prouvé, par double inclusion, que $H=\\langle X\\rangle$.\n\n\n" }, { "question": " 15 - Sous-groupe engendré par le complémentaire d'un sous-groupe [Signaler une erreur] [Ajouter à ma feuille d'exos]Enoncé Soit $H$ un sous-groupe strict d'un groupe $(G,\\cdot)$. Déterminer le sous-groupe engendré par le complémentaire de $H$.", "answer": " Notons $K$ le complémentaire de $H$ et fixons $a$ un élément de $K$ (rappelons que $H$ est strictement inclus dans $G$). Nous allons prouver que le sous-groupe engendré par $K$, que nous allons noter $L$, est égal à $G$ tout entier. Puisque ce sous-groupe contient déjà $K$, il suffit de prouver qu'il contient également son complémentaire, à savoir $H$. Soit donc $x\\in H$. Alors $ax$ ne peut pas être un élément de $H$, sinon \n\n$a=ax x^{-1}$ serait élément de $H$ lui aussi. Donc $ax$ est élément de $K$. Mais alors, $x=a^{-1}ax$ est un élément de $L$ puisque $a$ et $ax$ sont tous deux éléments de $K$, donc de $L$, et que $L$ est un sous-groupe (ce qui entraîne que $a^{-1}\\in L$ et que le produit $a^{-1}ax$ est aussi dans $L$).\n" }, { "question": " 16 - Intersection de deux sous-groupes [Signaler une erreur] [Ajouter à ma feuille d'exos]Enoncé Soit $G$ un groupe et $H_1,H_2$ deux sous-groupes de $G$. Démontrer que $H_1\\cap H_2$ est un sous-groupe de $G$.", "answer": " On va vérifier les hypothèses du théorème de caractérisation des sous-groupes.\n\n\n On sait que $1_G\\in H_1$ (puisque $H_1$ est un sous-groupe de $G$) et que $1_G\\in H_2$ (puisque $H_2$ est un sous-groupe de $G$). Donc $1_G\\in H_1\\cap H_2$.\n\n Soit $x,y\\in H_1\\cap H_2$. Alors, $x\\in H_1,$ $y\\in H_1$ et donc puisque $H_1$ est un sous-groupe de $G$, $xy\\in H_1$. De la même façon, on prouve que $xy\\in H_2$ et donc que $xy\\in H_1\\cap H_2$.\n\n Soit $x\\in H_1\\cap H_2$. Alors, $x\\in H_1$ et puisque $H_1$ est un sous-groupe de $G$, $x^{-1}\\in H_1$. De même, $x^{-1}\\in H_2$ et finalement $x^{-1}\\in H_1\\cap H_2$.\n\n\n" }, { "question": " 17 - Produit de groupe et sous-groupe du produit [Signaler une erreur] [Ajouter à ma feuille d'exos]Enoncé Un sous-groupe d'un groupe produit est-il nécessairement produit de deux sous-groupes?", "answer": " Non, ce n'est pas le cas. Prenons $\\mathbb Z^2=\\mathbb Z\\times \\mathbb Z$ et $H=\\{(x,x);\\ x\\in\\mathbb Z\\}$. $H$ est clairement un sous-groupe de $\\mathbb Z^2$, et $H$ ne s'écrit pas $H=A\\times B$, sinon on aurait $A=B=\\mathbb Z$ ce qui n'est pas le cas.\n" }, { "question": " 18 - Union de deux sous-groupes [Signaler une erreur] [Ajouter à ma feuille d'exos]Enoncé Soit $G$ un groupe et $H,K$ deux sous-groupes de $G$. Démontrer que $H\\cup K$ est un sous-groupe de $G$ si et seulement si $H\\subset K$ ou $K\\subset H$.", "answer": " Si $H\\subset K$, alors $H\\cup K=K$ qui est un sous-groupe de $G$. De même, si $K\\subset H$, $H\\cup K=H$ qui est un sous-groupe de $G$.\n\nSupposons maintenant que $H\\cup K$ est un sous-groupe de $G$ et que ni $H\\subset K$, ni $K\\subset H$. Alors on peut trouver $x\\in H\\backslash K$ et $y\\in K\\backslash H$. Puisque $H\\cup K$ est un groupe et que $x,y\\in H\\cup K$, on a $xy\\in H\\cup K$. Mais si $xy\\in H$, alors $y=x^{-1}(xy)$\n\nest le produit de deux éléments de $H$, qui est un sous-groupe de $G$, et donc $y\\in H$ ce qui est une contradiction. On obtient de même une contradiction dans l'autre cas possible $xy\\in K$. L'hypothèse de départ est donc fausse, et on a bien $H\\subset K$ ou $K\\subset H$.\n" }, { "question": " 19 - Produit de deux sous-groupes [Signaler une erreur] [Ajouter à ma feuille d'exos]Enoncé Soit $(G,\\cdot)$ un groupe et $A$, $B$ deux sous-groupes de $G$.\n\nOn note $AB=\\{ab;\\ a\\in A,\\ b\\in B\\}$. Montrer que $AB$ est un sous-groupe de $G$ si\n\net seulement si $AB=BA$.", "answer": " Supposons d'abord que $AB=BA$. Alors $AB$ est un sous-groupe de $G$ car :\n\n\n $e\\in AB$, car $e=ee$ avec $e\\in A$ et $e\\in B$ (ce sont des sous-groupes);\n\n $AB$ est stable par passage au produit. En effet, si $x=ab\\in AB$ et $y=a'b'\\in AB$, alors\n\n$xy=aba'b'$. Or, $ba'$ est un élément de $BA$, c'est donc aussi un élément de $AB$ et donc\n\n$ba'=a''b''$ avec $a''\\in A$ et $b''\\in B$. On en déduit que\n\n$$xy=aa''b''b'\\in AB$$\n\npuisque $aa''\\in A$ et $b''b'\\in B$.\n\n $AB$ est stable par passage à l'inverse. En effet, si $x=ab\\in AB$, alors $x^{-1}=b^{-1}a^{-1}$\n\nest élément de $BA$ et $BA=AB$.\n\n\n\nRéciproquement, supposons que $AB$ est un sous-groupe de $G$ et prouvons que $AB=BA$. Soit d'abord $x=ab\\in AB$.\n\nAlors $x^{-1}=b^{-1}a^{-1}\\in AB$ puisque $AB$ est un groupe et donc $b^{-1}a^{-1}=a'b'$ avec $a'\\in A$ et $b'\\in B$. On passe à l'inverse :\n\n$$ab=b'^{-1}a'^{-1}\\in BA.$$\n\nPour l'autre inclusion, considérons $y=ba\\in BA$. Alors $y^{-1}=a^{-1}b^{-1}\\in AB$, et donc $y=(y^{-1})^{-1}\\in AB$ puisque $AB$ est un groupe. .\n" }, { "question": " 20 - Théorème de Lagrange [Signaler une erreur] [Ajouter à ma feuille d'exos]Enoncé Soit $(G,\\cdot)$ un groupe fini et $H$ un sous-groupe de $G$.\n\n\n Montrer que pour tout $a\\in G$, $H$ et $aH=\\{ah;\\ h\\in H\\}$ ont le même nombre d'éléments.\n\n Soient $a,b\\in G$. Démontrer que $aH=bH$ ou $aH\\cap bH=\\varnothing$.\n\n En déduire que le cardinal de $H$ divise le cardinal de $G$.\n\n", "answer": " \n Soit $f:H\\to aH$ définie par $f(h)=ah$. Il s'agit clairement d'une surjection de $H$ sur $aH$. De plus, si $ah_1=ah_2$, alors $h_1=h_2$ car $a$ est inversible, et donc $f$ est aussi injective. $f$ est donc une bijection de $H$ sur $aH$; ces deux ensembles ont le même nombre d'éléments.\n\n Supposons que $aH\\cap bH\\neq \\varnothing$ et prouvons que $aH=bH$. Par symétrie, il suffit de prouver que $aH\\subset bH$. Soit $x\\in aH\\cap bH$, $x=ah_1=bh_2$. Prenons $y=ah\\in aH$. Alors \n\n$a=bh_2h_1^{-1}$ et donc $y=b h_2h_1^{-1}h\\in bH$.\n\n La réunion des ensembles $aH$ est clairement égale à $G$ (si $x\\in G$, il est dans $xH$). On ne garde que les $aH$ deux à deux disjoints et par les deux questions précédentes, on réalise ainsi une partition de $G$ avec des ensembles qui ont tous le même cardinal, à savoir le cardinal de $H$. Si $k$ est le nombre d'ensembles nécessaires pour réaliser cette partition, on a \n\n$$k\\card(H)=\\card(G)$$\n\net donc le cardinal de $H$ divise celui de $G$.\n\n\n" }, { "question": " 21 - Exemples ou contre-exemples de morphismes de groupes [Signaler une erreur] [Ajouter à ma feuille d'exos]Enoncé Les applications $\\phi:G\\to H$ définies ci-dessous sont-elles des morphismes de groupes?\n\n\n $G=(GL_n(\\mathbb R),\\times)$, $H=(\\mathbb R,+)$, $\\phi(A)=\\textrm{tr}(A)$.\n\n $G=(M_n(\\mathbb R),+)$, $H=(\\mathbb R,+)$, $\\phi(A)=\\textrm{tr}(A)$.\n\n $G=(\\mathbb R^*,\\times)$, $H=(\\mathbb R^*,\\times)$, $\\phi(x)=|x|$.\n\n $G=(\\mathbb R^*,\\times)$, $H=(\\mathbb R^*,\\times)$, $\\phi(x)=2x$.\n\n $G=(\\mathbb R,+)$, $H=(GL_2(\\mathbb R),\\times)$, $\\phi(x)=\\begin{pmatrix} 1&x\\\\0&1\\end{pmatrix}$.\n\n", "answer": " \n Si $\\phi$ était un morphisme de groupe, on aurait, puisque $I_n$ est l'élément neutre\n\n du groupe $(GL_n(\\mathbb R),\\times)$ et $0$ celui de $\\mathbb R_+$, $\\phi(I_n)=0$. \n\n Ce n'est pas le cas, car $\\phi(I_n)=n$. On peut aussi trouver deux exemple de matrices\n\n $A$ et $B$ pour lesquelles on n'a pas $\\phi(AB)=\\phi(A)+\\phi(B)$ (ici aussi, $A=B=I_n$ conviennent).\n\n Dans ce cas, $\\phi$ est bien un morphisme de groupe car on a bien $\\textrm{Tr}(A+B)=\\textrm{Tr}(A)+\\textrm{Tr}(B)$.\n\nLa morale des deux premières questions est qu'il faut vraiment faire très attention aux groupes en jeu, pas seulement à l'application.\n\n Ici aussi, $\\phi$ est un morphisme de groupes car pour tous $x,y\\neq 0$, on a\n\n\\[\\phi(xy)=|xy|=|x|\\times |y|=\\phi(x)\\times \\phi(y).\\]\n\n Si $\\phi$ était un morphisme de groupes, on aurait $\\phi(1)=1$ puisque $1$ est l'élément neutre\n\nde $(\\mathbb R^*,\\times)$. Ce n'est pas le cas puisque $\\phi(1)=2$.\n\n On va revenir à la définition. Soit $x,y\\in\\mathbb R$. On a \n\n$$\\phi(x+y)=\\begin{pmatrix} 1&x+y\\\\0&1\\end{pmatrix}$$\n\ntandis que par les règles du produit matriciel\n\n$$\\phi(x)\\times \\phi(y)=\\begin{pmatrix} 1&x+y\\\\0&1\\end{pmatrix}.$$\n\nLes deux résultats coïncident : $\\phi$ est bien un morphisme de groupes.\n\n\n" }, { "question": " 22 - Des propriétés bien connues [Signaler une erreur] [Ajouter à ma feuille d'exos]Enoncé Traduire en termes de morphismes de groupes les propriétés bien connues suivantes (dont le domaine de validité a volontairement été omis) :\n\n\n $\\ln(xy)=\\ln(x)+\\ln(y)$;\n\n $|zz'|=|z||z'|$;\n\n $\\sqrt{xy}=\\sqrt{x}\\sqrt{y}$;\n\n $e^{x+y}=e^xe^y$;\n\n", "answer": " \n La fonction $\\ln$ est un morphisme du groupe $(\\mathbb R_+^*,\\times)$ dans le groupe $(\\mathbb R,+)$.\n\n La fonction $|\\cdot|$ est un morphisme de groupes de $(\\mathbb C^*,\\times)$ dans lui-même. Attention, même si la propriété est vraie pour $z=0$, il faut exclure $0$ du groupe!\n\n La fonction $\\sqrt{}$ est un morphisme du groupe $(\\mathbb R_+^*,\\times)$ dans lui-même.\n\n La fonction $\\exp$ est un morphisme de groupe de $(\\mathbb R,+)$ dans $(\\mathbb R_+^*,\\times)$.\n\n\n" }, { "question": " 23 - Exponentielle complexe [Signaler une erreur] [Ajouter à ma feuille d'exos]Enoncé Justifier que $\\exp$ est un morphisme de $(\\mathbb C,+)$ dans $(\\mathbb C^*,\\cdot)$. Quel est son image? Son noyau?", "answer": " On sait que, pour tous $z,w\\in \\mathbb C$, on a\n\n$$\\exp(z+w)=\\exp(z)\\exp(w).$$\n\nCeci signifie exactement que $\\exp$ est un morphisme de $(\\mathbb C,+)$ dans $(\\mathbb C^*,\\cdot)$ (la fonction exponentielle ne prend jamais la valeur zéro). De plus, soit $w\\in\\mathbb C^*, w=re^{i\\theta}$ avec $r>0$. Soit $a=\\ln r$ et posons $z=a+i\\theta$. Alors\n\n$$\\exp(z)=\\exp(a)\\exp(i\\theta)=re^{i\\theta}=w.$$\n\nL'exponentielle est un morphisme surjectif de $(\\mathbb C,+)$ dans $(\\mathbb C^*,\\cdot)$. Déterminons son noyau. Si $\\exp(z)=1$, posons $z=x+iy$. Alors \n\n$$\\exp(z)=\\exp(x)\\exp(iy)=1=1\\exp(i0).$$\n\nCeci est équivalent à $x=0$ et il existe $k\\in \\mathbb Z$ tel que $y=k2\\pi$. On a donc\n\n$$\\ker(\\exp)=\\{2ik\\pi;\\ k\\in\\mathbb Z\\}.$$\n" }, { "question": " 24 - Exemples de morphismes de groupes - calcul de noyaux et d'images [Signaler une erreur] [Ajouter à ma feuille d'exos]Enoncé Démontrer que les fonctions suivantes sont des morphismes de groupes. Déterminer leur noyau et leur image :\n\n\n $(\\mathbb Z,+)\\to(\\mathbb R^*,\\times),\\ n\\mapsto (-1)^n$;\n\n $(\\mathbb C^*,\\times)\\to(\\mathbb C^*,\\times),\\ z\\mapsto z/|z|$;\n\n $(\\mathbb R_+^*,\\times)\\times (\\mathbb R,+)\\to (\\mathbb C^*,\\times), (r,\\theta)\\mapsto re^{i\\theta}.$\n\n", "answer": " Dans chacun des cas, on notera $\\phi$ la fonction donnée.\n\n\n Soit $n,m\\in\\mathbb Z$. Alors \n\n$$\\phi(n+m)=(-1)^{n+m}=(-1)^n(-1)^m=\\phi(n)\\phi(m).$$\n\nAinsi, $\\phi$ est bien un morphisme de $\\mathbb Z$ dans $\\mathbb R^*$. Soit $n\\in\\mathbb Z$. Alors \n\n$$n\\in\\ker(\\phi)\\iff (-1)^n=1\\iff n\\textrm{ est pair}\\iff \\exists k\\in\\mathbb Z,\\ n=2k\\iff n\\in2\\mathbb Z.$$\n\nAinsi, $\\ker(\\phi)=2\\mathbb Z$. De plus, on a clairement $\\textrm{Im}(\\phi)=\\{-1,1\\}.$\n\n Soit $z_1,z_2\\in\\mathbb C^*$. Alors \n\n$$\\phi(z_1z_2)=\\frac{|z_1z_2|}{z_1z_2}=\\frac{|z_1|}{z_1}\\times \\frac{|z_2|}{z_2}=\\phi(z_1)\\phi(z_2).$$\n\nAinsi $\\phi$ est un morphisme de $\\mathbb C^*$ dans $\\mathbb C^*$. De plus, soit $z\\in\\mathbb C^*$. Alors\n\n$$z\\in\\ker(\\phi)\\iff \\frac{z}{|z|}=1\\iff z=|z|\\iff z\\in\\mathbb R_+^*.$$\n\nAinsi, $\\ker(\\phi)=\\mathbb R_+^*$. Enfin, on a $\\textrm{Im}(\\phi)=\\mathbb U=\\{z\\in\\mathbb C:\\ |z|=1\\}$. Il est d'abord clair que $\\textrm{Im}(\\phi)\\subset\\mathbb U$ puisque \n\n$$\\left|\\frac{z}{|z|}\\right|=\\frac{|z|}{|z|}=1.$$\n\nD'autre part, si $z\\in\\mathbb U$, alors $\\phi(z)=z$ et donc $z\\in\\textrm{Im}(\\phi)$, ce qui démontre l'autre inclusion. \n\n Il faut faire un peu plus attention pour cet exemple, car on part d'un groupe produit et de plus, pour un des éléments du produit, la loi est notée multiplicativement, et pour l'autre, la loi est notée additivement. \n\nSoit $(r_1,\\theta_1)$ et $(r_2,\\theta_2)$ dans $\\mathbb R_+^*\\times\\mathbb R$.\n\nAlors \n\n$$\\phi\\big( (r_1,\\theta_1)\\cdot (r_2,\\theta_2)\\big)=\n\n\\phi(r_1r_2,\\theta_1+\\theta_2)=r_1r_2e^{i(\\theta_1+\\theta_2)}=r_1e^{i\\theta_1}r_2e^{i\\theta_2}=\\phi(r_1,\\theta_1)\\cdot \\phi(r_2,\\theta_2).$$\n\n$\\phi$ est donc bien un morphisme de groupes. De plus, soit $(r,\\theta)\\in\\mathbb R_+^*\\times \\mathbb R$. Alors \n\n$$\\phi(r,\\theta)=1\\iff re^{i\\theta}=1\\iff r=1\\textrm{ et }\\theta=k2\\pi,\\ k\\in\\mathbb Z.$$\n\nOn a donc $\\ker(\\phi)=\\{1\\}\\times (2\\pi\\mathbb Z)$. Enfin, de l'écriture exponentielle d'un nombre complexe, on tire que $\\phi$ est surjective, c'est-à-dire que $\\textrm{Im}(\\phi)=\\mathbb C^*$.\n\n\n" }, { "question": " 25 - Automorphisme intérieur [Signaler une erreur] [Ajouter à ma feuille d'exos]Enoncé Soit $(G,\\cdot)$ un groupe. Pour $a\\in G$, on note $\\tau_a:G\\to G$ défini par $\\tau_a(x)=axa^{-1}$. \n\n\n Démontrer que $\\tau_a$ est un endomorphisme de $G$.\n\n Vérifier que, pour tous $a,b\\in G$, $\\tau_a\\circ \\tau_b=\\tau_{ab}$.\n\n Montrer que $\\tau_a$ est bijective et déterminer son inverse.\n\n En déduire que $\\Theta=\\{\\tau_a;\\ a\\in G\\}$ muni du produit de composition est un groupe.\n\n", "answer": " \n Il suffit d'appliquer la définition : pour tous $x,y\\in G$, on a \n\n$$\\tau_a(xy)=axya^{-1}=axa^{-1}aya^{-1}=\\tau_a(x)\\tau_a(y).$$\n\n Soit $x\\in G$. On a \n\n$$\\tau_a\\circ\\tau_b(x)=\\tau_a(bxb^{-1})=abxb^{-1}a^{-1}$$\n\ntandis que \n\n$$\\tau_{ab}(x)=abx(ab)^{-1}=abxb^{-1}a^{-1}.$$\n\nOn a donc $\\tau_a\\circ\\tau_b=\\tau_{ab}$.\n\n Soit $a\\in G$. On pourrait prouver que $\\tau_a$ est injectif en calculant son noyau, puisqu'il est surjectif, mais c'est plus facile d'appliquer la question précédente. Avec $b=a^{-1}$, elle donne\n\n$$\\tau_a\\circ \\tau_{a^{-1}}=\\tau_{aa^{-1}}=\\tau_e=Id_G$$\n\net en inversant le rôle joué par $a$ et $b$, on a aussi\n\n$$\\tau_{a^{-1}}\\circ\\tau_a=Id_G.$$\n\nAinsi, $\\tau_a$ est inversible d'inverse $\\tau_{a^{-1}}$.\n\n On va prouver que $\\Theta=\\{\\tau_a;\\ a\\in G\\}$ est un sous-groupe de $(S_G,\\circ)$. Il est non-vide parce qu'il contient $\\tau_e$. Si $\\tau_a,\\tau_b\\in\\Theta$, alors \n\n$$(\\tau_a)^{-1}=\\tau_{a^{-1}}\\in\\Theta$$\n\net \n\n$$\\tau_a\\circ\\tau_b=\\tau_{ab}\\in\\Theta.$$\n\nAinsi, $(\\Theta,\\circ)$ est bien un sous-groupe de $(S_G,\\circ)$.\n\n\n" }, { "question": " 26 - Somme des valeurs [Signaler une erreur] [Ajouter à ma feuille d'exos]Enoncé Soit $f$ un morphisme non constant d'un groupe fini $(G,\\cdot)$ dans $(\\mathbb C^*,\\cdot)$. Calculer\n\n$\\sum_{x\\in G}f(x)$.", "answer": " Puisque $f$ n'est pas constante, il existe $a\\in G$ tel que $f(a)\\neq 1$. Maintenant, l'application $x\\mapsto ax$ est une permutation de $G$ : en effet, pour tout $y\\in G$, il existe un unique $x\\in G$ tel que $y=ax$ ($x$ est égal à $a^{-1}y$). On en déduit que\n\n$$\\sum_{x\\in G}f(ax)=\\sum_{x\\in G}f(x).$$\n\nMais d'autre part, puisque $f$ est un morphisme de groupes, on a aussi\n\n$$\\sum_{x\\in G}f(ax)=\\sum_{x\\in G}f(a)f(x)=f(a)\\sum_{x\\in G}f(x).$$\n\nAinsi, il vient\n\n$$\\big(f(a)-1\\big)\\times \\sum_{x\\in G}f(x)=0.$$\n\nPuisque $f(a)\\neq 1$, on en déduit que $\\sum_{x\\in G}f(x)=0.$\n" }, { "question": " 27 - Morphismes de $\\mathbb Z$ dans $\\mathbb Z$ [Signaler une erreur] [Ajouter à ma feuille d'exos]Enoncé Déterminer tous les morphismes de $(\\mathbb Z,+)$ dans lui-même. Lesquels sont injectifs? surjectifs?", "answer": " Soit $f$ un morphisme de $(\\mathbb Z,+)$. Prouvons par récurrence que pour tout $n\\geq 1$, on a\n\n$f(n)=nf(1)$. C'est vrai pour $n=1$, et si c'est vrai pour $n$, alors\n\n$$f(n+1)=f(n)+f(1)=nf(1)+f(1)=(n+1)f(1).$$\n\nDe plus, pour $n\\leq 0$, on a $-n\\geq 0$ et donc $f(-n)=-nf(1)$. On en déduit :\n\n$$0=f(0)=f(n+(-n))=f(n)+f(-n)=f(n)-nf(1).$$\n\nAinsi, on a toujours $f(n)=nf(1)$, quel que soit $n\\in\\mathbb Z$.\n\nCaractérisons maintenant les morphismes surjectifs. Supposons donc que $f$ est surjectif.\n\nTout élément de $\\mathbb Z=f(\\mathbb Z)$ est un multiple de $f(1)$.\n\nOr, les seuls éléments de $\\mathbb Z$ qui divisent tous les autres entiers sont $1$ et $-1$.\n\nOn en déduit que $f(1)=1$ ou $f(1)=-1$, et donc que $f(n)=n$ ou $f(n)=-n$. \n\nRéciproquement, ces deux applications sont clairement des morphismes surjectifs de $(\\mathbb Z,+)$.\n\nDéterminons enfin les morphismes injectifs. Soit $f$ un morphisme et $n\\in \\ker(f)$. Alors $f(n)=nf(1)=0$. Si $f(1)\\neq 0$, alors $f(n)=0\\iff n=0$ et $f$ est injectif, et si $f(1)=0$, alors $f$ n'est pas injectif. Donc tous les morphismes de $(\\mathbb Z,+)$ dans $(\\mathbb Z,+)$ sont injectifs sauf l'application identiquement nulle.\n" }, { "question": " 28 - Morphismes que $(\\mathbb Q,+)$ dans $(\\mathbb Z,+)$ [Signaler une erreur] [Ajouter à ma feuille d'exos]Enoncé Déterminer tous les morphismes de groupes de $(\\mathbb Q,+)$ dans $(\\mathbb Z,+)$.", "answer": " Soit $f$ un tel morphisme de groupe. On va commencer par démontrer que pour $p$ et $q$ des entiers naturels, on a \n\n$f(p)=pf(1)$ et $f\\left(\\frac 1q\\right)=\\frac{1}{q}f(1)$. La première des deux propriétés se démontre aisément par récurrence.\n\nPour la deuxième, on écrit que \n\n$$f(1)=f\\left(\\frac 1q+\\cdots+\\frac 1q\\right)=qf\\left(\\frac 1q\\right).$$\n\nNotons ensuite $a=f(1)$. Alors $a/q$ est un entier pour tout entier $q$, et donc $a=0$. On en déduit que $f(p)=f(1/q)=0$ pour tous les entiers $p$ et $q$,\n\npuis que $f(p/q)=pf(1/q)=0$. Finalement, on trouve que $f$ est le morphisme nul.\n" }, { "question": " 29 - Morphisme entre groupes de torsion et groupes sans torsion [Signaler une erreur] [Ajouter à ma feuille d'exos]Enoncé Dans un groupe $(G,\\cdot)$, un élément $x$ est dit de torsion s'il existe $n\\geq 1$ tel que $x^n=e$. On dit que $G$ est de torsion si tous ses éléments sont de torsion.\n\nOn dit que $G$ est sans torsion si son seul élément de torsion est l'élément neutre.\n\nSoit $G_1$ un groupe de torsion et $G_2$ un groupe sans torsion. Déterminer tous les morphismes de groupe de $G_1$ dans $G_2$.", "answer": " Remarquons d'abord que le morphisme qui à tout $x$ de $G_1$ associe l'élément neutre $e$ de $G_2$ convient.\n\nRéciproquement, soit $x$ un élément de $G_1$ et notons $y=f(x)$. Puisque $x$ est un élément de torsion,\n\nil existe $n\\geq 1$ tel que $x^n=e$. Mais alors, \n\n$$y^n=(f(x))^n=f(x^n)=f(e)=e.$$\n\nComme $G_2$ est sans torsion, on a $y=e$ et donc $f(x)=e$ pour tout $x\\in G_1$.\n" }, { "question": " 30 - Morphismes de $\\mathbb Z/n\\mathbb Z$ dans $\\mathbb Z/m\\mathbb Z$ [Signaler une erreur] [Ajouter à ma feuille d'exos]Enoncé \n Déterminer tous les morphismes de $\\mathbb Z/3\\mathbb Z$ dans $\\mathbb Z/4\\mathbb Z$.\n\n Déterminer tous les morphismes de $\\mathbb Z/6\\mathbb Z$ dans $\\mathbb Z/8\\mathbb Z$.\n\n", "answer": " Nous noterons dans les deux cas $\\bar a$ les éléments de l'ensemble de départ, et $\\tilde a$ les éléments de l'ensemble d'arrivée. Dans les deux cas, on peut remarquer qu'il suffit de déterminer l'image de $\\bar 1$, qui engendre le groupe...\n\n\n Soit $\\phi$ un tel morphisme. Alors $\\phi(\\bar 1)$ a un ordre qui divise 3, puisque $\\phi(\\bar 1)+\\phi(\\bar 1)+\\phi(\\bar 1)=\\phi(\\bar 0)=\\tilde 0$. Mais $\\phi(\\bar 1)$ a aussi un ordre qui divise 4, puisque c'est un élement de $\\mathbb Z/4\\mathbb Z$. Son ordre divise donc le pgcd de 3 et 4, c'est-à-dire que son ordre est 1. Donc $\\phi(\\bar 1)=\\tilde 0$, le seul morphisme est le morphisme trivial.\n\n Reprenons les mêmes notations. Cette fois on a que $\\phi(\\bar 1)$ a pour ordre un diviseur du pgcd de 6 et 8. Son ordre peut donc être égal à 1 ou 2. Si son ordre est égal à 1, il s'agit du morphisme trivial. Si son ordre est égal à 2, on a nécessairement $\\phi(\\bar 1)=\\tilde 4$ qui est le seul élément d'ordre 2 de $\\mathbb Z/8\\mathbb Z$. Maintenant, il s'agit de voir que cette formule définit bien un morphisme de groupes. Par exemple, on peut remarquer qu'elle donne $\\phi(\\bar k)=\\tilde 0$ si $k$ est pair, et $\\phi(\\bar k)=\\tilde 4$ si $k$ est impair. A partir de là, il est facile de voir que $\\phi$ définit bien un morphisme de $\\mathbb Z/6\\mathbb Z$ dans $\\mathbb Z/8\\mathbb Z$.\n\n\n" }, { "question": " 31 - $(\\mathbb Q,+)$ et $(\\mathbb Z,+)$ ne sont pas isomorphes [Signaler une erreur] [Ajouter à ma feuille d'exos]Enoncé Démontrer que $(\\mathbb Q,+)$ et $(\\mathbb Z,+)$ ne sont pas isomorphes", "answer": " On procède par l'absurde et on suppose qu'il existe un isomorphisme de groupes $f:(\\mathbb Q,+)\\to (\\mathbb Z,+)$. Notons $a=f^{-1}(1)$. Alors \n\n$\\frac a2\\in \\mathbb Q$ et \n\n$$\\frac a2+\\frac a2=a.$$\n\nSi on applique $f$ on trouve \n\n$$2f(a/2)=f(a)=1\\implies \\frac 12=f(a/2)\\in \\mathbb Z.$$\n\nC'est bien sûr une contradiction!\n" }, { "question": " 32 - Groupes non isomorphes [Signaler une erreur] [Ajouter à ma feuille d'exos]Enoncé Démontrer que les groupes multiplicatifs $(\\mathbb R^*,\\cdot)$ et $(\\mathbb C^*,\\cdot)$ ne sont pas isomorphes.", "answer": " Supposons que ces deux groupes sont isomorphes et soit $f$ un isomorphisme de $(\\mathbb C^*,\\cdot)$ dans $(\\mathbb R^*,\\cdot)$. Posons $a=f(i)$. Alors\n\n$$f(i^4)=a^4=1$$\n\net donc $a^2=1$ puisque $a^2>0$. D'où $1=a^2=f(i^2)=f(-1)$ et $1=f(1)$. $f$ ne peut pas être injectif, on a obtenu une contradiction.\n" }, { "question": " 33 - Groupes non isomorphes [Signaler une erreur] [Ajouter à ma feuille d'exos]Enoncé Démontrer que les groupes $(\\mathbb R^*,\\times)$ et $(\\mathbb Q^*,\\times)$ ne sont pas isomorphes.", "answer": " Procédons par l'absurde et supposons qu'il existe $f:(\\mathbb R^*,\\times)\\to(\\mathbb Q^*,\\times)$ un isomorphisme. En particulier, il existe $a\\in\\mathbb R^*$ tel que $f(a)=2$. Si $a>0$, alors il existe un réel $b$ tel que $a=b^2$. On a alors $2=f(a)=f(b)\\times f(b)$ et donc $\\sqrt 2=\\pm f(b)$. Mais ceci contredit que $\\sqrt 2$ est irrationnel. \n\nSi $a<0$, alors il existe un réel $b$ tel que $a=-b^2$. On a alors $2=f(a)=f(-1)f(b)\\times f(b)$. Mais $f(-1)^2=f(-1\\times -1)=f(1)=1$ et donc $f(-1)=\\pm 1$. Comme $f$ est un isomorphisme et $f(1)=1$, on a $f(-1)=-1$. On obtient alors $2=-f(b)^2$, ce qui est impossible puisque $2>0$ et $-f(b)^2<0$. \n\nDans les deux cas, on a obtenu une contradiction et l'existence d'un tel isomorphisme est impossible. On aurait aussi pu utiliser des arguments de cardinalité : $\\mathbb Q^*$ est dénombrable, et $\\mathbb R^*$ ne l'est pas. Donc il ne peut pas exister de bijection entre ces deux ensembles. \n" }, { "question": " 34 - Groupes divisibles [Signaler une erreur] [Ajouter à ma feuille d'exos]Enoncé Un groupe $(G,\\cdot)$ est dit divisible si, pour tout $g\\in G$ et tout $n\\in\\mathbb N^*$, il existe $u\\in G$ tel que $u^n=g$.\n\n\n Le groupe $(\\mathbb Q,+)$ est-il divisible?\n\n Montrer que $(\\mathbb Q,+)$ et $(\\mathbb Q_+^*,\\cdot)$ ne sont pas isomorphes.\n\n", "answer": " \n Soit $x\\in\\mathbb Q$, et $n\\in\\mathbb N^*$. Alors, si on pose $y=x/n$, c'est un élément de $\\mathbb Q$ et $ny=x$ : le groupe $(\\mathbb Q,+)$ est divisible.\n\n Procédons en deux temps. On commence par montrer que si $G$ et $H$ sont deux groupes isomorphes et si $G$ est divisible, alors $H$ est divisible. En effet, soit $\\phi:G\\to H$ un isomorphisme. Soit $h\\in H$. Il existe $g\\in G$ tel que $h=\\phi(g)$. Puisque $G$ est divisible, pour tout $n\\geq 1$, il existe $u\\in G$ tel que $u^n=g$. Posons $v=\\phi(u)$. Alors puisque $\\phi$ est un morphisme, on a $v^n=h$ et $h$ est divisible.\n\nPour conclure, il suffit donc de prouver que $(\\mathbb Q_+^*,\\cdot)$ n'est pas divisible. Mais par exemple, pour $g=2$ et $n=2$, il n'existe par de rationnel $u$ tel que $u^2=2$ (car $\\sqrt 2$ est irrationnel). Les deux groupes ne sont donc pas isomorphes.\n\n\n" }, { "question": " 35 - Isométries laissant invariant un triangle équilatéral [Signaler une erreur] [Ajouter à ma feuille d'exos]Enoncé Soit $G$ le groupe des isométries du plan affine euclidien qui laissent invariant un triangle équilatéral $\\Delta$. Démontrer que $G$ est isomorphe à $S_3$.", "answer": " Notons $T=\\{A,B,C\\}$ les trois sommets du triangle. Il suffit de construire un isomorphisme de $G$ sur $S_T$. Considérons $\\phi:G\\to S_T,\\ g\\mapsto g_{|T}$ et prouvons que $\\phi$ est un isomorphisme. Remarquons d'abord que $\\phi$ est bien définie. En effet, les sommets du triangle sont bien envoyés par un élément $g$ de $G$ sur eux-mêmes (pourquoi!!!!), et $g$ étant bijective, sa restriction à l'ensemble fini $T$ ne peut être que bijective. Il est alors clair que $\\phi$ est un morphisme de groupe. Elle est injective. En effet, si $g$ est l'identité sur les $\\{A,B,C\\}$, ceci signifie que $g$ est une isométrie du plan ayant au moins trois points fixes. Ainsi, $g$ ne peut être que l'identité. Prouvons enfin que $\\phi$ est surjective. Les transpositions engendrant $S_3$, il suffit de démontrer que les transpositions sont dans l'image de $\\phi$. Mais la transposition $(A\\ B)$ est dans l'image de $\\phi$. Il suffit en effet de considérer pour $g$ la symétrie orthogonale à la médiatrice de $[AB]$ (cette droite passe donc par $C$), qui échange les points $A$ et $B$ tout en gardant $C$ fixe" } , { "question": " 1 - Pour commencer... [Signaler une erreur] [Ajouter à ma feuille d'exos]Enoncé Quel est l'ordre de $\\bar 9$ dans $(\\mathbb Z/12\\mathbb Z,+)$?", "answer": " On a (tenant compte du fait que la loi est notée additivement) :\n\n$$2\\times \\bar 9=\\bar 6,\\ 3\\times \\bar 9=\\bar 3,\\ 4\\times \\bar 9=\\bar 0.$$\n\n$\\bar 9$ est donc d'ordre 4.\n" }, { "question": " 2 - Ordre du carré [Signaler une erreur] [Ajouter à ma feuille d'exos]Enoncé Soit $G$ un groupe et $x\\in G$ d'ordre $n$. Quel est l'ordre de $x^2$?", "answer": " D'abord, on remarque que $x^2$ est d'ordre fini, car $(x^2)^n=(x^n)^2=e^2=e$. De plus, son ordre que nous allons noter $d$ divise $n$. Distinguons alors deux cas :\n\n\n Si $n$ est pair et s'écrit $2p$, alors $(x^2)^p=x^{n}=e$, et donc l'ordre de $x^2$ divise $p$. De plus, si l'ordre de $x^2$ est inférieur strict à $p$, on a $x^{2d}=e$ avec $1\\leq 2d$ engendré par $a$.\n\n Déterminer un ensemble minimal de générateurs de $(G,\\cdot)$.\n\n $ (G, \\cdot)$ est-il un groupe cyclique ?\n\n Déterminer tous les sous-groupes de $G$ et, pour chaque sous-groupe, préciser un ensemble de générateurs.\n\n Parmi les sous-groupes de $(G,\\cdot)$, lesquels sont isomorphes à un groupe additif $(\\mathbb Z/m\\mathbb Z,+)$?\n\n", "answer": " \n Rappelons que par le théorème de Bézout, $n$ est inversible dans $(\\mathbb Z/20\\mathbb Z,\\cdot)$ si et seulement si $n$ est premier avec 20. On a donc $G = \\{1,3,7,9,11,13,17,19\\}$.\n\n On prend un élément et toutes ses puissances, jusqu'à obtenir l'élément neutre $1$. On obtient \\begin{eqnarray*}\n\n<1> &=& \\{1\\}\\\\\n\n<3> &=& \\{1,3,7,9\\}\\\\\n\n<7> &=& \\{1,3,7,9\\}\\\\\n\n<9> &=& \\{1,9\\}\\\\\n\n<11> &=& \\{1,11\\}\\\\\n\n<13> &=& \\{1,9,13,17\\}\\\\\n\n<17> &=& \\{1,9,13,17\\}\\\\\n\n<19> &=& \\{1,19\\}\\\\ \n\n\\end{eqnarray*}\n\n On vient de voir qu'on ne peut pas engendrer le groupe avec un seul élément. Essayons avec deux éléments. C'est facile à voir. Si on prend par exemple 3 et 11, le groupe engendré comprend au moins $<3>$ et $<11>$, c'est-à-dire au moins 5 éléments. Comme son ordre doit diviser l'ordre du groupe, il contient au moins 8 éléments, c'est-à-dire que c'est $G$ tout entier. Autrement dit, on a prouvé que $<3,11>=G$ et donc $\\{3,11\\}$ est un ensemble minimal de générateurs de $G$.\n\n Aucun élément de $G$ n'engendre seul le groupe. $G$ n'est pas cyclique.\n\n Les sous-groupes de $G$ sont d'ordre 1,2,4 ou 8. Dans $G$, il y a un élément d'ordre 1, 4 éléments d'ordre 4 et 3 éléments d'ordre 2. Si on combine deux éléments d'ordre 4 qui n'engendrent pas le même sous-groupe, ou un élément d'ordre 4 avec un élément d'ordre 2 qui n'est pas dans le sous-groupe engendré (comme à la question 3), on obtiendra $G$ tout entier. Reste à voir les sous-groupes engendrés par les éléments d'ordre 2 : on a \n\n$$<11,19> = \\{1, 11, 19, 9\\}$$\n\n$$<3,11> = <3,13> = <3,19> = <11,13> = <13,19> = G.$$\n\n Parmi les sous-groupes de $G$, ceux de la deuxième question sont cycliques, donc isomorphes à $\\mathbb Z/m\\mathbb Z$ où $m=1,2,4$ suivant le cas. Le sous-groupe $<11,19>$ n'est pas cyclique, car il n'est pas engendré par un seul élément. De même, $G$ n'est pas cyclique.\n\n\n" }, { "question": " 10 - Groupe de cardinal pair [Signaler une erreur] [Ajouter à ma feuille d'exos]Enoncé Soit $G$ un groupe de cardinal $2n$.\n\n\n Démontrer que la relation $\\mathcal R$ définie sur $G$ par\n\n$$x\\mathcal R y\\iff x=y\\textrm{ ou }x=y^{-1}$$\n\nest une relation d'équivalence sur $G$.\n\n En déduire que $G$ admet des éléments d'ordre deux.\n\n", "answer": " \n La relation est clairement réflexive et symétrique. De plus, si $x\\mathcal R y$ et $y\\mathcal R z$, alors \n\n\n si $x=y$ et $y=z$, on a $x=z$;\n\n si $x=y$ et $y=z^{-1}$, on a $x=z^{-1}$;\n\n si $x=y^{-1}$ et $y=z$, on a $x=z^{-1}$;\n\n si $x=y^{-1}$ et $y=z^{-1}$, on a $x=z$.\n\n\n\nDans tous les cas, on a $x\\mathcal R z$ et la relation est transitive.\n\n Puisqu'on n'a pas $x\\mathcal R y$ si $y\\notin \\{x,x^{-1}\\}$, une classe d'équivalence comporte \n\n\n ou bien un seul élément, si $x=x^{-1}$;\n\n ou bien exactement deux élements, si $x\\neq x^{-1}$; ces éléments sont alors $x$ et $x^{-1}$.\n\n\n\nIl y a au moins une classe d'équivalence avec un seul élément : la classe de l'élément neutre. De plus, les classes d'équivalence forment une partition de $G$, et $G$ est de cardinal pair. Il doit donc y avoir au moins une autre classe de cardinal 1 (sinon le cardinal de $G$ serait impair). Cette autre classe de cardinal 1 donne un élément $x$ égal à son inverse.\n\n\n" }, { "question": " 11 - Ordre du produit de deux éléments [Signaler une erreur] [Ajouter à ma feuille d'exos]Enoncé Soit $G$ un groupe abélien, $x$ et $y$ deux éléments de $G$ d'ordres respectifs $p$ et $q$.\n\n\n On suppose que $p$ et $q$ sont premiers entre eux. Démontrer que $xy$ est d'ordre $pq$.\n\n Importance des hypothèses - 1 : Si $H=GL_2(\\mathbb R)$, $A=\\left(\\begin{array}{cc}0&-1\\\\1&0\\end{array}\\right)$ et $B=\\left(\\begin{array}{cc}0&1\\\\-1&-1\\end{array}\\right)$, vérifier que $A$ et $B$ sont d'ordre fini, mais que $AB$ n'est pas d'ordre fini.\n\n Importance des hypothèses - 2 : Si $p$ et $q$ ne sont pas supposés premiers entre eux, démontrer que le produit $xy$ n'est pas nécessairement d'ordre $pq$, ou d'ordre $\\textrm{ppcm}(p,q)$.\n\n Une application :\n\n\n Soit $d$ un diviseur de $p$. Démontrer qu'il existe un élément d'ordre $d$ dans $G$.\n\n En déduire que $G$ admet des éléments d'ordre $\\textrm{ppcm}(p,q)$.\n\n On suppose de plus que $G$ est fini. Démontrer que $G$ admet un élément dont l'ordre est le ppcm de l'ordre des éléments de $G$.\n\n\n", "answer": " \n Notons $d$ l'ordre de $xy$. Remarquons que $(xy)^{pq}=(x^p)^q(y^q)^p=e$, et donc $d|pq$. De plus, puisque $(xy)^d=e$, on en déduit que $x^d=y^{-d}$. Il vient alors\n\n$$x^{dq}=(y^{-d})^q=(y^q)^{-d}=e.$$\n\nAinsi, $p|dq$ et puisque $p$ et $q$ sont premiers entre eux, on en déduit que $p|d$. De la même façon, on a $q|d$ et en utilisant à nouveau que $p$ et $q$ sont premiers entre eux, on conclut que $pq|d$. Ainsi, on a bien que $d=pq$.\n\n On vérifie facilement que $A$ est d'ordre $4$, que $B$ est d'ordre 3 et que \n\n$$AB=\\left(\\begin{array}{cc}\n\n1&1\\\\\n\n0&1\n\n\\end{array}\\right).$$\n\nOn prouve alors par récurrence que, pour tout $n\\geq 1$, \n\n$$(AB)^n=\\left(\\begin{array}{cc}\n\n1&n\\\\\n\n0&1\n\n\\end{array}\\right).$$\n\n$AB$ n'est pas d'ordre fini, et donc l'hypothèse que $G$ est commutatif est importante.\n\n Si $x$ est un élément d'ordre $n\\geq 2$ dans un groupe $G$, son inverse $x^{-1}$ est aussi d'ordre $n$, et pourtant le produit $xx^{-1}$ est d'ordre 1, et non d'ordre $n$ ou $n^2$!\n\n Une application :\n\n\n Considérons $a=x^{p/d}$. Alors on a $a^d=x^p=e$. D'autre part, si $a^r=e$, alors $x^{rp/d}=e$ et donc $rp/d$ est un multiplie de $p$. En particulier $r/d$ est un entier, ce qui signifie que $d|r$. $a$ est donc bien d'ordre $d$.\n\n Décomposons $p$ et $q$ en facteurs premiers (pour avoir les mêmes facteurs, on s'autorise des exposants nuls) : \n\n$$p=p_1^{\\alpha_1}\\cdots p_r^{\\alpha_r}\\ q=p_1^{\\beta_1}\\cdots p_r^{\\beta_r}.$$\n\nOn sait qu'alors\n\n$$\\textrm{ppcm}(p,q)=p_1^{\\max(\\alpha_1,\\beta_1)}\\cdots p_r^{\\max(\\alpha_r,\\beta_r)}.$$\n\nPar la question précédente, il est possible, pour chaque $i=1,\\dots,r$, de fabriquer un élément $a_i$ d'ordre $p_i^{\\max(\\alpha_i,\\beta_i)}$ (on le fabrique à partir de $x$ si $\\alpha_i\\geq \\beta_i$, à partir de $y$ sinon). \n\nEn utilisant le résultat de la première question et une simple récurrence, le produit $a_1\\dots a_r$ est bien d'ordre $\\textrm{ppcm}(p,q)$.\n\n Notons $x_1,\\dots,x_r$ les éléments de $G$, d'ordres respectifs $q_1,\\dots,q_r$. Alors d'après la question précédente, il existe un élément d'ordre $ppcm(q_1,q_2)$. Puis appliquant une nouvelle fois la question précédente, il existe un élément d'ordre $ppcm(ppcm(q_1,q_2),q_3)=ppcm(q_1,q_2,q_3)$. Par une récurrence facile, on construit un élément d'ordre le ppcm que $q_1,\\dots,q_r$.\n\n\n\n" }, { "question": " 12 - Groupe d'ordre 35 [Signaler une erreur] [Ajouter à ma feuille d'exos]Enoncé \n Soit $G$ un groupe et $H,K$ deux sous-groupes de $G$ d'ordre des entiers premiers. Démontrer que $H=K$ ou que $H\\cap K=\\{e\\}$. \n\n Démontrer que dans un groupe d'ordre 35, il existe un élément d'ordre 5 et un élément d'ordre 7.\n\n", "answer": " \n Soit $p$ l'ordre de $H$, qui est premier. Puisque un élément de $H$ a un ordre qui divise $p$, cet ordre ne peut être égal que à 1, si c'est l'élément neutre, ou à $p$. Autrement dit, tout élément de $H$ autre que l'élément neutre génère $H$. Il en est de même pour tout élément de $K$. Ainsi, si $H\\cap K$ contient un élément $x$ différent de $e$, il contient toutes les puissances de $x$, donc $H$ et $K$, et $H=K$.\n\n Soit $G$ un tel groupe. Ses éléments peuvent être d'ordre 1, 5, 7 ou 35. Si $G$ admet un élément d'ordre 35 (ie $G$ est cyclique), que l'on appelle $a$, alors $a^5$ est d'ordre 7 et $a^7$ est d'ordre 5. Supposons donc que $G$ n'est pas cyclique et qu'il n'admet pas d'éléments d'ordre 7. Alors tous ses éléments, sauf l'élément neutre, sont d'ordre $5$, et $G$ est réunion de sous-groupes d'ordre 5.\n\nD'après la première question, l'intersection de deux de sous-groupes, quand ils sont distincts, est restreinte à $\\{e\\}$. Notons $G_1,\\dots,G_n$ ces sous-groupes distincts. Alors chaque $G_i$ s'écrit $G_i=\\{e\\}\\cup H_i$, et les $H_1,\\dots,H_n$ sont deux à deux disjoints. Autrement dit, \n\n$$G=\\{e\\}\\cup H_1\\cup\\dots \\cup H_n$$\n\nest une partition de $G$. Comme chaque $H_i$ est de cardinal 4, ceci implique que $35=4n+1$. Mais alors $34$ serait un multiple de 4, ce qui n'est pas le cas. \n\nLe raisonnement est similaire si on suppose que $G$ n'admet pas d'éléments d'ordre 5. On aurait alors $35=6m+1$ pour un entier $m$, ce qui n'est pas le cas puisque $34$ n'est pas un multiple de 6.\n\n\n" }, { "question": " 13 - Produit de groupes cycliques [Signaler une erreur] [Ajouter à ma feuille d'exos]Enoncé Soient $G$ et $H$ deux groupes.\n\n\n Montrer que si $g$ est un élément d'ordre $p$ de $G$ et $h$ un élément d'ordre $q$ de $H$, alors $(g,h)$ est d'ordre $\\textrm{ppcm}(p,q)$ dans $G\\times H$.\n\n On suppose que $G$ et $H$ sont cycliques. Démontrer que $G\\times H$ est cyclique si et seulement si les ordres de $G$ et $H$ sont premiers entre eux.\n\n", "answer": " \n On a $(g,h)^n=(g^n,h^n)=(e,e)$ si et seulement si on a à la fois $p|n$ et $q|n$, donc si et seulement si $\\textrm{ppcm}(p,q)|n$. Ainsi, l'ordre de $(g,h)$ est bien le ppcm de $p$ et $q$.\n\n Soit $p$ l'ordre de $G$ et $q$ l'ordre de $H$. Si $p\\wedge q=1$, si $x$ est un générateur de $G$ (d'ordre $p$ donc) et si $y$ est un générateur de $H$ (d'ordre $q$ donc), alors $(x,y)$ est d'ordre $\\textrm{ppcm}(p,q)=pq$. Puisque $G\\times H$ est de cardinal $pq$, c'est bien un groupe cyclique.\n\nRéciproquement si $G\\times H$ est cyclique, soit $(g,h)$ un générateur de $G\\times H$. Alors $g$ est un générateur de $G$ et $h$ est un générateur de $H$. Leur ordre respectif est donc $p$ (resp. $q$), et par la première question, $(g,h)$ est d'ordre $\\textrm{ppcm}(p,q)$. Puisqu'on sait qu'il est d'ordre $pq$, on a bien $\\textrm{ppcm}(p,q)=pq$ qui implique que $p$ et $q$ sont premiers entre eux.\n\n\n" }, { "question": " 14 - Sous-groupe d'un groupe cyclique [Signaler une erreur] [Ajouter à ma feuille d'exos]Enoncé Soit $G$ un groupe cyclique et soit $H$ un sous-groupe de $G$. Démontrer que $H$ est cyclique.", "answer": " Soit $a$ un générateur de $G$. L'ensemble des entiers $p\\geq 1$ tels que $a^p\\in H$ est non-vide (puisque $a^{\\textrm{card}(G)}=e\\in H$). Il contient un plus petit élément que nous noterons $n$. On va alors prouver que $H$ est le groupe engendré par $a^n$. Il est d'abord évident que le sous-groupe engendré par $a^n$ est contenu dans $H$.\n\nRéciproquement, soit $x\\in H$. $x$ s'écrit $x=a^p$, et il suffit de prouver que $p=kn$. Effectuons la division euclidienne de $p$ par $n$ : $p=qn+r$ avec $0\\leq rn$. On considère $G$ un groupe d'ordre $pn$, et $H$ un sous-groupe de $G$ d'ordre $p$. Démontrer que $H$ est un sous-groupe normal de $G$.", "answer": " D'après les hypothèses, $p\\wedge n=1$, et donc $H$ est un p-Sylow de $G$. Pour tout $g\\in G$, $gHg^{-1}$ est aussi un $p$-Sylow de $G$. Il suffit donc de démontrer que $G$ admet un seul $p$-Sylow. Notons $N$ le nombre de $p$-Sylow de $G$. Alors par les théorèmes de Sylow, $N$ est congru à $1$ modulo $p$, et $N$ divise $n$. Si $N\\neq 1$, alors $N\\geq p+1$, ce qui contredit que $N$ divise $n$ puisque $n3$, il existe $a_i$ tel que $\\phi(t_i)=(a_1\\ a_i)$.\n\n Montrer que l'application $s$ qui à $i$ associe $a_i$ est bijective.\n\n Montrer que $\\phi$ coïncide avec l'automorphisme intérieur associé à $s$.\n\n", "answer": " \n Il est facile de remarquer que deux transpositions commutent si et seulement si elles ont un support disjoint.\n\nSi $\\phi(t_2)$ et $\\phi(t_3)$ commutaient, alors on aurait $t_2 t_3=\\phi^{-1}(\\phi(t_2)\\phi(t_3))=\\phi^{-1}(\\phi(t_3))\\phi(t_2))=t_3t_2$\n\net donc $t_2$ et $t_3$ commuteraient elles aussi. $\\phi(t_2)$ et $\\phi(t_3)$ n'ont donc pas un support disjoint ce qui donne immédiatement le résultat.\n\n Le support de $\\phi(t_i)$ n'est pas disjoint de $\\phi(t_2)$ pas plus que de celui de $\\phi(t_3)$. Il y a donc deux possibilités.\n\n\n ou bien $\\phi(t_i)=(a_1\\ a_i)$, ce qui est le résultat voulu.\n\n ou bien $\\phi(t_i)=(a_2\\ a_3)$. Mais dans ce cas, \n\n$$\\phi(t_2)\\phi(t_3)\\phi(t_i)=(a_1 \\ a_2)\\circ(a_1\\ a_3)\\circ(a_2\\ a_3)=(a_1\\ a_3)=\\phi(t_3),$$\n\nce qui entraîne $(1\\ 2)(1\\ 3)(1\\ i)=(1\\ 3)$. Cette dernière propriété est fausse et cette alternative\n\nest donc impossible.\n\n\n Si les $a_i$ n'étaient pas tous distincts, alors $\\phi$ ne saurait être bijective.\n\n On a $s(1\\ i)s^{-1}=(s(1)\\ s(i))=(a_1\\ a_i)$. $\\phi$ et l'automorphisme intérieur associé à $s$\n\ncoïncident donc sur les éléments $t_i$. Puisque ces éléments engendrent $S_n$, $\\phi$ est donc\n\nl'automorphisme intérieur associé à $s$.\n" } , { "question": " 1 - Vrai/Faux sur les polynômes [Signaler une erreur] [Ajouter à ma feuille d'exos]Enoncé \n Le polynôme $X^4+X^2+1$ est-il irréductible dans $\\mathbb R[X]$? dans $\\mathbb C[X]$?\n\n La relation $\\mathcal R$ définie sur $\\mathbb R[X]$ par $A\\mathcal R B$ si et seulement si $A$ divise $B$ est-elle une relation d'ordre?\n\n", "answer": " \n Les irréductibles de $\\mathbb R[X]$ sont de degré 1 ou 2, les irréductibles de $\\mathbb C[X]$ sont de degré 1. Le polynôme $X^4+X^2+1$ n'est donc irréductible ni dans $\\mathbb R[X]$, ni dans $\\mathbb C[X]$.\n\n On peut vérifier que la relation $\\mathcal R$ est transitive et réflexive. En revanche, elle n'est pas anti-symétrique. Prenons par exemple $A=X$ et $B=2X$. Alors $A$ divise $B$, $B$ divise $A$, et pourtant $A$ est différent de $B$.\n\n\n" }, { "question": " 2 - Coefficient dominant et degré [Signaler une erreur] [Ajouter à ma feuille d'exos]Enoncé Pour chacun des polynômes suivants, donner son coefficient dominant ainsi que son degré.\n\n\\begin{equation*}\n\n\\begin{aligned}\n\n& P_0 = 1 + \\sqrt 3 X - 2X^2 && \\quad P_1 = X^7 + 4X^8 + (1-\\mathrm i)X^3 \\\\\n\n& P_2 = 3X(1+X^2) + 2X^3 - 1 && \\quad P_3 = (\\mathrm i+X)(1+3X^2-\\mathrm iX) \\\\\n\n& P_4 = \\sum_{k=1}^4 (k!+1) X^k && \\quad P_5 = (3X-7X^2+4)' \\\\\n\n& P_6 = \\big[ X(X+1)(X+2)(X+3)(X+4) \\big]' && \\quad P_7 = (1-X^n)(1+X)^2 + X^{n+2}, \\quad n\\geq 0.\n\n\\end{aligned}\n\n\\end{equation*}", "answer": " \n $P_0$ a pour degré $2$ et coefficient dominant $-2$.\n\n\n\n$P_1 = 4X^8 + X^7 + (1-\\mathrm i)X^3$ et a pour degré $8$ et coefficient dominant $4$.\n\n \n\n$P_2 = 5X^3 + 3X -1$ et a pour degré $3$ et coefficient dominant $5$.\n\n\n\n$P_3$ est le produit d'un polynôme de degré 1 à coefficient dominant 1, avec un polynôme de degré 2 à coefficient dominant 3, $P_3$ est donc de degré 3 et à coefficient dominant $3$.\n\n \n\n$P_4$ est de degré 4 et à coefficient dominant $(4! +1)$, c'est-à-dire $25$.\n\n\n\n$P_5 = -14X+3$ est de degré $1$ et à coefficient dominant $-14$.\n\nC'est en fait le polynôme dérivé d'un polynôme de degré $2$ à coefficient dominant $-7$.\n\n\n\nLe polynôme $X(X+1)(X+2)(X+3)(X+4)$ est de degré $5$ et unitaire (c'est-à-dire de coefficient dominant $1$) ;\n\n$P_6$ est son polynôme dérivé, il est donc de degré $4$, et de coefficient dominant $5$.\n\n\n\n$P_7 = (1-X^n)(1+X)^2 + X^{n+2} = 1 + 2X + X^2 - X^n - 2X^{n+1}$.\n\n\n si $n =0$, $P_7 = X^2$ ; il est de degré $2$, de coefficient dominant $1$ ;\n\n si $n = 1$, $P_7 = 1 + X - X^2$ ; il est de degré $2$, de coefficient dominant $-1$ ;\n\n si $n\\geq 2$, alors $n+1\\geq 3$ donc $P_7$ est de degré $(n+1)$, de coefficient dominant $-2$.\n\n\n\n" }, { "question": " 3 - Carrés [Signaler une erreur] [Ajouter à ma feuille d'exos]Enoncé Soient $a,b$ des réels, et $P(X)=X^4+2aX^3+bX^2+2X+1$. Pour quelles valeurs de $a$ et $b$\n\nle polynôme $P$ est-il le carré d'un polynôme de $\\mathbb R[X]$?", "answer": " Si $P=Q^2$ est le carré d'un polynôme, alors $Q$ est nécessairement de degré 2, et son coefficient\n\ndominant est égal à $1$ ou est égal à $-1$. Dans le premier cas, on peut donc écrire $Q(X)=X^2+cX+d$. On a alors \n\n$$Q^2(X)=X^4+2cX^3+(2d+c^2)X^2+2cdX+d^2.$$\n\nPar identification, on doit avoir $2c=2a$, $2d+c^2=b$, $2cd=2$ et $d^2=1$. On trouve donc $c=a$ et $d=\\pm 1$. \n\nSi $d=1$, alors $c=1$, et donc $a=1$ et $b=3$. Si $d=-1$, alors $c=-1$, $a=-1$ et $b=-1$. Les deux solutions sont donc\n\n\\begin{eqnarray*}\n\nP_1(X)&=&X^4+2X^3+3X^2+2X+1=(X^2+X+1)^2\\\\\n\nP_2(X)&=&X^4-2X^3-X^2+2X+1=(X^2-X-1)^2.\n\n\\end{eqnarray*}\n\nDans le deuxième cas, on écrit $Q(X)=-R(X)$ avec $R(X)=X^2+cX+d$, de sorte que $Q^2(X)=R^2(X)$ et on retrouve en réalité le cas précédent.\n" }, { "question": " 4 - Quelques équations [Signaler une erreur] [Ajouter à ma feuille d'exos]Enoncé Résoudre les équations suivantes, où l'inconnue est un polynôme $P$ de $\\mathbb R[X]$ :\n\n$$\\begin{array}{lll}\n\n\\mathbf{1.}\\ P(X^2 ) = (X^2 + 1)P(X)&\\quad&\\mathbf{2.}\\ P'^2=4P\\\\\n\n\\mathbf{3.}\\ P\\circ P=P.\n\n\\end{array}$$", "answer": " \n Le polynôme nul est évidemment solution. Sinon, si $P$ est solution, alors on a \n\n$$2\\deg(P)=\\deg(P)+2$$\n\nce qui prouve que $\\deg(P)$ doit être égal à 2. Maintenant, si $P(X)=aX^2+bX+c$, alors\n\n\\begin{eqnarray*}\n\nP(X^2)&=&aX^4+bX^2+c\\\\\n\n(X^2+1)P(X)&=&aX^4+bX^3+(a+c)X^2+bX+c.\n\n\\end{eqnarray*}\n\nOn en déduit que $b=0$, puis que $a+c=0$. Les solutions sont donc les polynômes\n\nqui s'écrivent $P(X)=a(X^2-1)$, $a\\in\\mathbb R$.\n\n Là encore, le polynôme nul est solution, et c'est la seule solution constante. Par ailleurs, si $P$ est une solution non constante, alors son degré\n\nvérifie l'équation\n\n$$2(\\deg(P)-1)=\\deg(P)$$\n\nce qui entraîne que $\\deg(P)=2$. Maintenant, si $P(X)=aX^2+bX+c$, alors\n\n\\begin{eqnarray*}\n\nP'^2&=&(2aX+b)^2=4a^2X^2+4abX+b^2\\\\\n\n4P&=&4aX^2+4bX+4c.\n\n\\end{eqnarray*}\n\nCeci entraîne $a^2=a$, donc $a=1$ (le polynôme est de degré $2$, $a\\neq 0$), puis\n\n$c=b^2/4$. Les polynômes solutions sont donc le polynôme nul et les polynômes\n\n$P(X)=X^2+bX+b^2/4$, avec $b\\in\\mathbb R$.\n\n Si $P$ est une solution qui n'est pas le polynôme nul, alors le degré de $P\\circ P$\n\nvaut $\\deg(P)^2$, et donc on a l'équation\n\n$$\\deg(P)^2=\\deg(P).$$\n\net donc $\\deg(P)=1$ ou $\\deg(P)=0$. Maintenant, si $P(X)=aX+b$, alors \n\n\\begin{eqnarray*}\n\nP\\circ P(X)&=&a(aX+b)+b=a^2X+(ab+b)\\\\\n\nP(X)&=&aX+b.\n\n\\end{eqnarray*}\n\nOn doit donc avoir $a^2=a$, soit $a=1$ ou $a=0$, et $ab=0$. Si $a=1$, alors $b=0$ et si $a=0$,\n\nalors $b$ peut être quelconque dans $\\mathbb R$.\n\nFinalement, on trouve que les solutions sont les polynômes constants et le polynôme $P(X)=X$.\n\n\n" }, { "question": " 5 - Polynômes, rationnels et irrationnels [Signaler une erreur] [Ajouter à ma feuille d'exos]Enoncé On souhaite démontrer qu'il n'existe pas de polynôme $P\\in\\mathbb R[X]$ de degré $n\\geq 1$ tel que $P(x)\\in\\mathbb Q$ pour tout $x\\in\\mathbb R\\backslash \\mathbb Q$.\n\n\n Traiter le cas $n=1$. \n\n Traiter le cas général.\n\n", "answer": " \n Supposons qu'il existe $P\\in\\mathbb R[X]$ de degré 1 tel que $P(x)\\in\\mathbb Q$ pour tout $x\\in\\mathbb R\\backslash \\mathbb Q.$ Puisque $\\deg(P)=1$, on peut l'écrire $P(X)=aX+b$ avec $a,b\\in \\mathbb R$ et $a\\neq 0.$ On a $P(\\sqrt 2)\\in\\mathbb Q$, et donc $a\\sqrt 2+b\\in \\mathbb Q$. On a aussi $P(-\\sqrt 2)\\in \\mathbb Q$ et donc $-a\\sqrt 2+b\\in \\mathbb Q$. En effectuant la somme de ces deux quantités, on obtient $2b\\in \\mathbb Q$ et finalement $b\\in \\mathbb Q.$ Ainsi, on obtient $ax\\in\\mathbb Q$ pour tout $x\\in\\mathbb R\\backslash \\mathbb Q.$ En particulier, $a\\sqrt 6\\in\\mathbb Q$ et $a\\sqrt 3\\in\\mathbb Q$ et donc en effectuant le quotient, $\\sqrt 2\\in \\mathbb Q,$ une contradiction.\n\nOn peut donner une démonstration beaucoup plus rapide si on connait la notion d'ensemble dénombrable. Puisque $P$ est un polynôme de degré 1, c'est une fonction injective. Et il ne peut pas exister de fonction injective d'un ensemble qui n'est pas dénombrable, ici $\\mathbb R\\backslash\\mathbb Q,$ dans un ensemble dénombrable (ici $\\mathbb Q$).\n\n Procédons par récurrence sur $n$. Notons, pour $n\\geq 1$, $\\mathcal P(n)$ la propriété suivante :\n\n$$\\mathcal P(n):\"\\textrm{il n'existe pas de polynôme $P\\in\\mathbb R[X]$ de degré $n$ tel que }\\forall x\\in\\mathbb R\\backslash \\mathbb Q,\\ P(x)\\in\\mathbb Q.\"$$\n\nLa question précédente a prouvé que la propriété $\\mathcal P(1)$ est vraie. Soit maintenant $n\\geq 1$ tel que $\\mathcal P(n)$ est vraie. Soit $P\\in\\mathbb R[X]$ de degré $n+1$, et supposons que $P(x)\\in\\mathbb Q$ pour tout $x\\in\\mathbb R\\backslash\\mathbb Q$. Posons $Q(X)=P(X+1)-P(X)$. Alors il est facile de vérifier que $Q$ est de degré $n$ (si $a_{n+1}X^{n+1}$ est le terme de plus haut degré de $P$, alors $(n+1)a_{n+1}X^{n}$ est le terme dominant de $Q$). De plus, pour tout $x\\in\\mathbb R\\backslash \\mathbb Q$, $x+1\\in\\mathbb R\\backslash \\mathbb Q$ et donc $P(x),P(x+1)\\in\\mathbb Q$. On en déduit que $Q(x)\\in\\mathbb Q$, une contradiction avec $\\mathcal P(n)$.\n\nAinsi, par le principe de récurrence, $\\mathcal P(n)$ est vraie pour tout $n\\geq 1$.\n\n\n" }, { "question": " 6 - En pratique! [Signaler une erreur] [Ajouter à ma feuille d'exos]Enoncé Calculer le quotient et le reste de la division euclidienne de \n\n\n $X^4+5X^3+12X^2+19X-7$ par $X^2+3X-1$;\n\n $X^4-4X^3-9X^2+27X+38$ par $X^2-X-7$;\n\n $X^5-X^2+2$ par $X^2+1$.\n\n", "answer": " On trouve les résultats suivants :\n\n\n Le quotient est $X^2+2X+7$, le reste est nul;\n\n Le quotient est $X^2-3X-5$, le reste est $X+3$;\n\n Le quotient est $X^3-X-1$, le reste est $X+3$.\n\n\n" }, { "question": " 7 - Application de la division euclidienne à l'évaluation d'un polynôme [Signaler une erreur] [Ajouter à ma feuille d'exos]Enoncé Soit $A(X)=X^6-X^4+2X^3-X+1$ et $B(X)=X^2-2X+2$.\n\n\n Effectuer la division euclidienne de $A$ par $B$.\n\n En déduire la valeur de $A(1+i)$. \n\n", "answer": " \n On trouve $A(X) = (X^4+2X^3+X^2-2)B(X)-5X+5$.\n\n On vérifie facilement que \n\n$$B(1+i)=(1+i)^2-2(1+i)+2=0.$$\n\nOn en déduit que \n\n$$A(1+i)=-5(1+i)+5=-5i.$$\n\n\n" }, { "question": " 8 - Expression du reste [Signaler une erreur] [Ajouter à ma feuille d'exos]Enoncé Soit $P\\in \\mathbb K[X]$, soit $a,b\\in\\mathbb K$ avec $a\\neq b$.\n\n\n Soit $R$ le reste de la division euclidienne de $P$ par $(X-a)(X-b)$. Exprimer $R$ en fonction de $P(a)$ et de $P(b)$.\n\n Soit $R$ le reste de la division euclidienne de $P$ par $(X-a)^2$. Exprimer $R$ en fonction de $P(a)$ et de $P'(a)$.\n\n", "answer": " Dans les deux cas, on remarquera que $R$ est de degré au plus $1$ et s'écrit donc $R(X)=\\alpha X+\\beta$. \n\n\n Évaluons la relation \n\n$$P(X)=(X-a)(X-b)Q(X)+\\alpha X+\\beta$$\n\nen $a$ et en $b$. On trouve le système \n\n$$\\left\\{\n\n\\begin{array}{rcl}\n\n\\alpha a +\\beta &=&P(a)\\\\\n\n\\alpha b +\\beta &=&P(b).\n\n\\end{array}\\right.$$\n\nOn en déduit alors facilement que \n\n$$\\alpha=\\frac{P(a)-P(b)}{a-b}\\textrm{ et }\\beta=\\frac{aP(b)-bP(a)}{a-b}.$$\n\n Évaluons la relation\n\n$$P(X)=(X-a)^2Q(X)+\\alpha X+\\beta$$\n\nau point $a$. On trouve $P(a)=a\\alpha+\\beta$. Dérivons maintenant la relation précédente :\n\n$$P'(X)=2(X-a)Q(X)+(X-a)^2Q'(X)+\\alpha.$$\n\nOn évalue à nouveau en $a$ et on trouve que \n\n$$\\alpha=P'(a).$$\n\nEn revenant à la première équation, on en déduit que $\\beta=P(a)-aP'(a).$\n\n\n" }, { "question": " 9 - Reste par un produit [Signaler une erreur] [Ajouter à ma feuille d'exos]Enoncé Soit $P\\in\\mathbb R[X]$, $a,b\\in\\mathbb R$, $a\\neq b$. Sachant que le reste de la division euclidienne de $P$ par $(X-a)$ vaut 1 et que le reste de la division euclidienne de $P$ par $X-b$ vaut $-1$, que vaut le reste de la division euclidienne de $P$ par $(X-a)(X-b)$?", "answer": " On sait que $P(X)=(X-a)Q_1(x)+1$, et donc $P(a)=1$. De même, on a $P(b)=-1$. La division euclidienne de $P$ par $(X-a)(X-b)$ s'écrit \n\n$$P(X)=(X-a)(X-b)Q(X)+\\alpha X+\\beta.$$\n\nOn évalue cette relation en $a$ et en $b$, et on trouve le système\n\n$$\\left\\{\n\n\\begin{array}{rcl}\n\n\\alpha a+\\beta&=&1\\\\\n\n\\alpha b+\\beta&=&-1.\n\n\\end{array}\\right.\n\n$$\n\nLa résolution de ce système ne pose pas de difficultés et donne comme unique solution\n\n$$\\alpha=\\frac{2}{a-b}\\textrm{ et }\\beta=\\frac{-a-b}{a-b}.$$\n\nLe reste recherché est donc\n\n$$\\frac{2}{a-b}X+\\frac{-a-b}{a-b}.$$\n" }, { "question": " 10 - Reste et trigonométrie [Signaler une erreur] [Ajouter à ma feuille d'exos]Enoncé Soit $\\alpha$ un nombre réel et $n\\geq 1$ un entier. Quel est le reste de $(X\\sin \\alpha+\\cos\\alpha)^n$ par $X^2+1$.", "answer": " On sait que le reste est de degré au plus $1$, et on peut écrire \n\n$$(X\\sin\\alpha+\\cos\\alpha)^n =(X^2+1)Q(X)+aX+b$$\n\navec $a$ et $b$ des réels. On évalue cette égalité en $i$, racine de $X^2+1$, et on trouve\n\n$$(i\\sin\\alpha+\\cos\\alpha)^n =ai+b.$$\n\nMais \n\n$$(i\\sin\\alpha+\\cos\\alpha)^n=\\cos(n\\alpha)+i\\sin(n\\alpha).$$\n\nAinsi, le reste recherché est $\\sin(n\\alpha)X+\\cos(n\\alpha)$.\n" }, { "question": " 11 - [Signaler une erreur] [Ajouter à ma feuille d'exos]Enoncé Quel est le reste de la division euclidienne de $(X+1)^n-X^n-1$ par\n\n$$\n\n\\mathbf{1.}\\ X^2-3X+2\\quad\\quad\\mathbf{2.}\\ X^2+X+1\\quad\\quad\\mathbf{3.}\\ X^2-2X+1?\n\n$$\n\n", "answer": " On rappelle qu'on note $j$ le nombre complexe $j=-\\frac12+i\\frac{\\sqrt 3}2$. \n\n\n La méthode pour ce type d'exercice est toujours la même. On commence par écrire a priori le résultat de la division euclidienne, par exemple pour le premier polynôme :\n\n$$(X+1)^n-X^n-1=Q(X)(X^2-3X+2)+aX+b,$$\n\noù $a$ et $b$ sont deux réels.\n\nOn évalue ensuite la relation en les racines du diviseur, qui sont ici $1$ et $2$.\n\nOn trouve alors \n\n$$\\left\\{\n\n\\begin{array}{rcl}\n\n2^n-2&=&a+b\\\\\n\n3^{n}-2^n-1&=&2a+b.\n\n\\end{array}\\right.$$\n\nEt finalement on résout le système pour trouver $a$ et $b$, qui sont ici égaux à :\n\n$$\\left\\{\n\n\\begin{array}{rcl}\n\na&=&3^n-2^{n+1}+1\\\\\n\nb&=&-3^n+2^{n+1}+2^n-3.\n\n\\end{array}\\right.$$\n\n On écrit la même chose, \n\n$$(X+1)^n-X^n-1=Q(X)(X^2+X+1)+aX+b,$$\n\net on utilise cette fois que les racines de $X^2+X+1$ sont\n\n$j$ et $j^2$.\n\nIl suffit ici en réalité d'utiliser l'évaluation en $j$, sachant que tout nombre complexe\n\ns'écrit de façon unique sous la forme $x+jy$, avec $x,y\\in\\mathbb R$.\n\nOn trouve :\n\n$$(1+j)^n-j^n-1=Q(j)\\times 0+aj+b.$$\n\nOn distingue ensuite suivant la valeur de $n$ modulo 3, utilisant que\n\n$$(1+j)^n-j^n-1=(-1)^nj^{2n}-j^n-1.$$\n\n\n Si $n\\equiv 0\\ [3]$, alors $j^{2n}=j^n=1$, et donc on a\n\n$$(-1)^n-2=aj+b.$$\n\nOn identifie alors les coefficients et on trouve $a=0$ et $b=(-1)^n-2.$\n\nAinsi le reste est $(-1)^n-2$.\n\n Si $n\\equiv 1\\ [3]$, alors $j^{n}=j$ et donc $j^{2n}=j^2=-1-j$, $j^n=j$, ce qui donne\n\n$$\\big((-1)^{n+1}-1\\big)j+\\big( (-1)^{n+1}-1\\big)=aj+b.$$\n\nOn identifie les coefficients et on trouve\n\n$$a=b=(-1)^{n+1}-1.$$\n\nLe reste est donc $\\big((-1)^{n+1}-1\\big)(X+1).$\n\n Si $n\\equiv 2\\ [3]$, alors $j^{2n}=j$ et $j^{n}=j^2=-1-j$. On trouve\n\n$$\\big( (-1)^n +1\\big)j=aj+b.$$\n\nOn identifie alors les coefficients et on trouve\n\n$a=(-1)^n +1$ et $b=0.$\n\nLe reste est alors $\\big( (-1)^n +1\\big)X.$\n\n\n On recommence en écrivant\n\n$$(X+1)^n-X^n-1=Q(X)(X^2-2X+1)+aX+b,$$\n\net en remarquant que $X^2-2X+1$ a pour racine double 1. Si on évalue en 1, on obtient une seule relation, à savoir\n\n$$2^n-2=a+b.$$\n\nPour obtenir une seconde relation, il faut dériver la relation issue de la division euclidienne et l'évaluer à nouveau en 1 (c'est toujours\n\ncette méthode qui fonctionne pour une racine double). On trouve :\n\n$$n(X+1)^{n-1}-nX^{n-1}=Q'(X)(X^2-2X+1)+2Q(X)(X-1)+a,$$\n\nce qui donne la relation\n\n$$n2^{n-1}-n=a.$$\n\nOn retrouve alors sans problèmes $b$, qui est égal à :\n\n$$b=(2-n)2^{n-1} +n-2.$$\n\n\n" }, { "question": " 12 - Même quotient et même reste en inversant les rôles [Signaler une erreur] [Ajouter à ma feuille d'exos]Enoncé Déterminer les couples $(A,B)$ de polynômes non nuls de $\\mathbb R[X]$ tels que le quotient et le reste dans la division euclidienne de $A$ par $B$ et dans la division euclidienne de $B$ par $A$ soient identiques.", "answer": " Procédons par analyse-synthèse. Supposons donc que la propriété est vraie. Il existe alors un polynôme $Q$ et un polynôme $R$ avec $\\deg(R)<\\min(\\deg(A),\\deg(B))$ tel que $A=BQ+R$ et $B=AQ+R$. Mais alors, on a aussi \n\n\\begin{eqnarray*}\n\nA=(AQ+R)Q+R=AQ^2+RQ+R&\\iff &A(1-Q^2)-R(1+Q)=0\\\\\n\n&\\iff&A(Q+1)(1-Q)-R(1+Q)=0\\\\\n\n&\\iff&(Q+1)\\big( A(1-Q)-R\\big)=0.\n\n\\end{eqnarray*}\n\nSi le produit de deux polynômes est nul, c'est que l'un de ces deux polynômes est nul. Ainsi, on a ou bien $1+Q=0$ ou bien $A(1-Q)-R=0$. On distingue donc deux cas\n\n\n Si $Q=-1$, alors $A=-B+R$ et $B=-A+R$. Autrement dit, il existe deux polynômes $P,R\\in\\mathbb R[X]$ avec $\\deg(R)<\\deg(P)$ tels que $A=P+R$ et $B=-P$.\n\n Si $Q\\neq -1$, alors $A(1-Q)-R=0$. Pour des considérations de degré (rappelons que $\\deg(R)<\\deg(A))$, ceci n'est possible que si $Q=1$ et $R=0$. On obtient alors le cas trivial $A=B$.\n\n\n\nPassons à la synthèse. Supposons que $A=B$ ou qu'il existe un couple de polynômes $(P,R)$ de $\\mathbb R[X]$ avec $\\deg(R)<\\deg(P)$ tels que $A=P+R$ et $B=-P$. Alors les divisions euclidiennes de $A$ par $B$ et de $B$ par $A$ ont bien même quotient et même reste. C'est évident si $A=B$ et dans l'autre cas, l'écriture $A=-B+R$ donne que le quotient et le reste dans la division euclidienne de $A$ par $B$ sont $-1$ et $R$, tandis que l'écriture $B=-P=-A+R$ donne le même reste et le même quotient dans la division euclidienne de $B$ par $A$.\n\nOn a donc démontré que l'ensemble des couples solutions est\n\n$$\\mathcal S=\\{(P,P);\\ P\\in\\mathbb R[X]\\}\\cup \\{(P+R,-P);\\ P,R\\in\\mathbb R[X],\\ \\deg(R)<\\deg(P)\\}.$$\n\n\n" }, { "question": " 13 - Détermination d'un ensemble de polynômes [Signaler une erreur] [Ajouter à ma feuille d'exos]Enoncé Le but de cet exercice est de déterminer\n\n$$E=\\{P\\in \\mathbb R[X];\\ P(X^2)=(X^3+1)P(X)\\}.$$\n\n\n Démontrer que le polynôme nul ainsi que le polynôme $X^3-1$ sont\n\nsolutions du problème.\n\n Analyse du problème. Soit $P\\in E$ non nul.\n\n\n Montrer que $P$ est de degré 3.\n\n Démontrer que $P(1)=0$, puis que $P'(0)=P''(0)=0$ (on pourra penser\n\nà dériver la relation\n\n$P(X^2)=(X^3+1)P(X)$).\n\n En effectuant la division euclidienne de $P$ par $X^3-1$, démontrer\n\nqu'il existe $\\lambda\\in\\mathbb R$ tel que $P(X)=\\lambda (X^3-1)$.\n\n\n Synthèse du problème : en déduire l'ensemble $E$.\n\n", "answer": " \n Il est clair que $0=(X^3+1)0$ et donc le polynôme nul est solution.\n\nPour $P(X)=X^3-1$, on a $P(X^2)=X^6-1$\n\net $(X^3+1)P(X)=(X^3+1)(X^3-1)=X^6-1$. Ce polynôme est aussi solution.\n\n \n Notons $n$ le degré de $P$. Alors $P(X^2)$ est de degré $2n$ et $(X^3+1)P(X)$ est de degré $n+3$. Le degré $n$ vérifie donc l'équation $2n=n+3$, soit $n=3$.\n\n En évaluant la relation en $X=1$, on a $P(1^2)=P(1)=0$. Dérivons maintenant l'équation $P(X^2)=(X^3+1)P(X)$. On trouve\n\n$$2XP'(X^2)=3X^2P(X)+(X^3+1)P'(X).$$\n\nSi on évalue en $X=0$, on trouve $P'(0)=0$. Dérivons une second fois cette équation. On trouve\n\n$$2P'(X^2)+4X^2P''(X^2)=6XP(X)+6X^2P'(X)+(X^3+1)P''(X).$$\n\nOn évalue cette équation en $X=0$ et on trouve, tenant compte du fait que l'on sait déjà que $P'(0)=0$, $P''(0)=0$.\n\n Effectuons la division euclidienne de $P$ par $X^3-1$. On peut écrire \n\n$$P(X)=Q(X)(X^3-1)+R(X)$$\n\noù $\\deg(R)\\leq 2$ et donc $R(X)$ s'écrit $R(X)=aX^2+bX+c$. De plus, en considérant le degré, $Q$ ne peut être qu'un polynôme constant, et donc $Q(X)=\\lambda$ avec $\\lambda\\in\\mathbb R$. Il reste à montrer que $a=b=c=0$. Puisque $P(1)=0$, on a $a+b+c=0$. De plus, dérivons $P(X)=\\lambda(X^3-1)+(aX^2+bX+c)$. On obtient \n\n$$P'(X)=3\\lambda X^2+(2aX+b).$$\n\nPuisque $P'(0)=0$, on a $b=0$. On dérive une seconde fois la relation, on obtient \n\n$$P''(X)=6\\lambda X+2a$$\n\net puisque $P''(0)=0$, on a $a=0$ et finalement également $c=0$.\n\n\n La question précédente nous dit que si $P\\in E$, alors ou bien $P$ est nul ou bien $P(X)=\\lambda (X^3-1)$ pour un certain $\\lambda\\in\\mathbb R^*$. Réciproquement, d'après la première question, le polynôme nul et les polynômes $\\lambda(X^3-1)$, $\\lambda\\in\\mathbb R^*$ sont éléments de $E$. Finalement, on peut conclure que $E=\\{\\lambda (X^3-1);\\ \\lambda\\in\\mathbb R\\}$.\n\n\n" }, { "question": " 14 - Un reste [Signaler une erreur] [Ajouter à ma feuille d'exos]Enoncé Soient $n$, $p$ deux entiers naturels non nuls et soit $P(X)=\\sum_{k=0}^n a_kX^k$\n\nun polynôme de $\\mathbb C[X]$. Pour chaque $k\\in\\{0,\\dots,n\\}$, on note $r_k$ le reste de la division euclidienne\n\nde $k$ par $p$. Démontrer que le reste de la division euclidienne de $P$ par $X^p-1$ est le polynôme\n\n$R(X)=\\sum_{k=0}^n a_kX^{r_k}$.", "answer": " On va démontrer que $X^p-1$ divise $P-R$. En effet, le degré de $R$ est inférieur strict à $p$,\n\net $R$ sera bien le reste dans la division euclidienne de $P$ par $X^p-1$. On écrit alors que\n\n$$P-R=\\sum_{k=0}^n a_k (X^k-X^{r_k}),$$\n\net il suffit de prouver que $X^p-1$ divise chaque $X^k-X^{r_k}$. Si $km$, et on écrit $n=mp+r$, avec $0\\leq rm$, et on écrit $n=mp+r$, avec $0\\leq r0$, on a \n\n$$\\cos\\left(\\frac\\pi{12}\\right)=\\frac{\\sqrt{\\sqrt 3+2}}2.$$\n\nOn en déduit que \n\n$$\\sin^2\\left(\\frac\\pi{12}\\right)=1-\\cos^2\\left(\\frac\\pi{12}\\right)=\\frac{2-\\sqrt 3}{4}.$$\n\nPuisque $\\sin(\\pi/12)>0$, on a donc\n\n$$\\sin\\left(\\frac\\pi{12}\\right)=\\frac{\\sqrt{2-\\sqrt 3}}2.$$\n\nUne autre possibilité pour résoudre cet exercice était de remarquer que \n\n$$\\frac{\\pi}3-\\frac\\pi4=\\frac{\\pi}{12}$$\n\net d'appliquer les formules d'addition $\\sin(a-b)$ et $\\cos(a-b)$.\n" }, { "question": " 7 - Tangente en $\\pi/8$ [Signaler une erreur] [Ajouter à ma feuille d'exos]Enoncé Calculer $\\tan(\\pi/8)$.", "answer": " On remarque que $2\\frac{\\pi}8=\\frac{\\pi}4$. On a donc\n\n\\begin{align*}\n\n\\tan(\\pi/4)&=\\frac{\\tan(\\pi/8)+\\tan(\\pi/8)}{1-\\tan^2(\\pi/8)}\\\\\n\n&=\\frac{2\\tan(\\pi/8)}{1-\\tan^2(\\pi/8)}.\n\n\\end{align*}\n\nPuisque $\\tan(\\pi/4)=1$, $\\tan(\\pi/8)$ est solution de l'équation $x^2+2x-1=0$. Les deux solutions de cette équation sont \n\n$$x_0=\\frac{-2-\\sqrt 8}{2}=-1-\\sqrt{2},\\ x_1=\\frac{-2+\\sqrt 8}2=-1+\\sqrt 2.$$\n\nComme on sait que $\\tan(\\pi/8)>0$ puisque $\\pi/8\\in ]0,\\pi/2[$, on en déduit que \n\n$$\\tan\\left(\\frac\\pi 8\\right)=\\sqrt 2-1.$$\n" }, { "question": " 8 - Cosinus, sinus et tangente en fonction de l'angle moitié [Signaler une erreur] [Ajouter à ma feuille d'exos]Enoncé Soit $x\\in ]-\\pi,\\pi[+2\\pi\\mathbb Z$. On pose $t=\\tan(x/2)$. Démontrer les formules suivantes : \n\n$$\\cos(x)=\\frac{1-t^2}{1+t^2},\\ \\sin(x)=\\frac{2t}{1+t^2},\\ \\tan(x)=\\frac{2t}{1-t^2}.$$", "answer": " Bien sûr, on peut déduire la troisième formule des deux autres. La plus facile est la troisième, car d'après la formule de duplication de $\\tan$ : \n\n$$\\tan(x)=\\tan\\left(2\\frac x2\\right)=\\frac{2t}{1-t^2}.$$\n\nPar ailleurs, on a\n\n\\begin{align*}\n\n\\sin(x)&=\\sin\\left(2\\frac x2\\right)\\\\\n\n&=2\\sin(x/2)\\cos(x/2)\\\\\n\n&=2\\tan(x/2)\\cos^2(x/2).\n\n\\end{align*}\n\nPuisque de plus\n\n$$\\cos^2(x/2)=\\frac{1}{1+\\tan^2(x/2)}$$\n\non obtient le résultat.\n" }, { "question": " 9 - Inégalité sur les sinus [Signaler une erreur] [Ajouter à ma feuille d'exos]Enoncé Démontrer que, pour tout $n\\geq 1$ et tout $x\\in\\mathbb R$, $|\\sin(nx)|\\leq n|\\sin(x)|$.", "answer": " On va démontrer le résultat par récurrence sur $n$. On fixe $x\\in\\mathbb R$ et pour $n\\geq 1$, on note $P_n:\"|\\sin(nx)|\\leq n|\\sin(x)|\"$.\n\nInitialisation : la propriété est clairement vraie pour $n=1$.\n\nHérédité : soit $n\\geq 1$ tel que $P_n$ est vraie et prouvons $P_{n+1}$. Alors on a \n\n$$\\sin((n+1)x)=\\sin(nx)\\cos(x)+\\sin(x)\\cos(nx).$$\n\nEn utilisant l'inégalité triangulaire, puis $|\\cos(x)|\\leq 1$ et $|\\cos(nx)|\\leq 1$, on trouve\n\n\\begin{align*}\n\n|\\sin((n+1)x)|&\\leq |\\sin(nx)|\\cdot |\\cos(x)|+|\\sin(x)|\\cdot |\\cos(nx)|\\\\\n\n&\\leq |\\sin(nx)|\\cdot 1+|\\sin(x)|\\cdot 1\\\\\n\n&\\leq n|\\sin(x)|+|\\sin(x)|\\\\\n\n&\\leq (n+1)|\\sin(x)|\n\n\\end{align*}\n\noù l'avant-dernière ligne vient de l'hypothèse de récurrence. Donc $P_{n+1}$ est vraie.\n\nConclusion : La propriété $P_n$ est vraie pour tout $n\\geq 1$.\n" }, { "question": " 10 - Un produit de cosinus [Signaler une erreur] [Ajouter à ma feuille d'exos]Enoncé Soit $a\\in]0,\\pi[$. Démontrer que pour tout $n\\geq 1$\n\n$$\\prod_{k=1}^n \\cos\\left(\\frac a{2^k}\\right)=\\frac1{2^n}\\cdot \\frac{\\sin(a)}{\\sin\\left(\\frac a{2^n}\\right)}.$$", "answer": " On va démontrer la formule par récurrence sur $n$. Notons $\\mathcal P_n$ la propriété \n\n$$\\mathcal P_n=``\\prod_{k=1}^n \\cos\\left(\\frac a{2^k}\\right)=\\frac1{2^n}\\cdot \\frac{\\sin(a)}{\\sin\\left(\\frac a{2^n}\\right)}''.$$\n\nInitialisation : On a $\\sin(a)=2\\sin(a/2)\\cos(a/2)$ et donc \n\n$$\\frac{\\sin(a)}{2\\sin(a/2)}=\\cos(a/2)$$\n\nce qui prouve que $\\mathcal P_1$ est vraie.\n\nHérédité : Soit $n\\in\\mathbb N$ tel que $\\mathcal P_n$ est vraie, et prouvons $\\mathcal P_{n+1}$. Sachant que\n\n$$\\sin(a/2^n)=2\\sin(a/2^{n+1})\\cos(a/2^{n+1})$$\n\non a \n\n\\begin{align*}\n\n \\frac{\\sin(a)}{2^{n+1}\\sin\\left(\\frac a{2^{n+1}}\\right)}&=\\frac{1}{2^n}\\times\\frac{\\sin(a)}{\\sin\\left(\\frac a{2^n}\\right)}\\times\\cos\\left(\\frac a{2^{n+1}}\\right)\\\\\n\n &=\\prod_{k=1}^n \\cos\\left(\\frac a{2^k}\\right)\\times \\cos\\left(\\frac a{2^{n+1}}\\right)\\\\\n\n &=\\prod_{k=1}^{n+1} \\cos\\left(\\frac a{2^k}\\right).\n\n\\end{align*}\n\nAinsi $\\mathcal P_{n+1}$ est vraie. \n\nConclusion : $\\mathcal P_n$ est vraie pour tout $n\\geq 1$.\n" }, { "question": " 11 - Équations trigonométriques - lecture du cercle trigonométrique [Signaler une erreur] [Ajouter à ma feuille d'exos]Enoncé Résoudre dans $\\mathbb R$ les équations suivantes :\n\n$$\n\n\\begin{array}{lll}\n\n\\displaystyle\\mathbf{1.}\\ \\sin x=\\frac 12&\\displaystyle\\quad\\mathbf{2.}\\ \\tan x=\\sqrt 3&\\displaystyle\\quad\\mathbf{3.}\\ \\cos x=-1\\\\\n\n\\displaystyle\\mathbf{4.}\\ \\sin(3x)=1&\\quad\\displaystyle\\mathbf{5.}\\ \\cos(4x)=-2\n\n\\end{array}$$", "answer": " \n Les seules solutions de l'équation dans $[0,2\\pi[$ sont $x=\\pi/6$ et $x=5\\pi/6$. Par $2\\pi$-périodicité, on obtient que les solutions sont les réels $\\pi/6+2k\\pi$, $k\\in\\mathbb Z$ et les réels $5\\pi/6+2k\\pi$, $k\\in\\mathbb Z$. Une autre façon de rédiger est d'écrire que \n\n$$\\sin(x)=\\sin\\left(\\frac\\pi 6\\right)\\iff \\exists k\\in\\mathbb Z, x=\\frac\\pi 6+2k\\pi\\textrm{ ou }\\exists k\\in\\mathbb Z,\\ x=\\pi-\\frac\\pi6+2k\\pi.$$\n\n On a $\\sqrt 3=\\tan(\\pi/3)$ et donc l'ensemble des solutions est $\\left\\{\\frac\\pi3+k\\pi:\\ k\\in\\mathbb Z\\right\\}$ (attention! ici les solutions sont définies simplement à $\\pi$ près et non à $2\\pi$ près).\n\n Il n'y a qu'une solution à l'équation dans $[0,2\\pi[$, donnée par $x=\n\n\\pi$. Les solutions de l'équation sont donc les réels $\\pi+2k\\pi$, $k\\in\\mathbb Z$.\n\n On écrit \n\n$$\\sin(3x)=1\\iff \\exists k\\in\\mathbb Z,\\ 3x=\\frac\\pi2+2k\\pi\\iff \\exists k\\in\\mathbb Z,\\ x=\\frac\\pi 6+\\frac{2k\\pi}3.$$\n\n L'équation n'admet pas de solutions! En effet, $\\cos$ est à valeurs dans $[-1,1]$.\n\n\n" }, { "question": " 12 - Équations trigonométriques simples [Signaler une erreur] [Ajouter à ma feuille d'exos]Enoncé Résoudre dans $\\mathbb R$ les équations suivantes : \n\n$$\\begin{array}{ll}\n\n\\mathbf{1.}\\ \n\n\\sin(5x)=\\sin\\left(\\frac{2\\pi}3+x\\right)&\n\n\\quad \n\n\\mathbf{2.}\\ \\cos\\left(x+\\frac\\pi4\\right)=\\cos(2x)\\\\\n\n\\mathbf{3.}\\ \\tan\\left(x+\\frac\\pi 4\\right)=\\tan(2x)\n\n\\end{array}$$", "answer": " \n On écrit simplement que \n\n$$\\sin(5x)=\\sin\\left(\\frac{2\\pi}3+x\\right)\\iff 5x=\\frac{2\\pi}3+x+2k\\pi,\\ k\\in\\mathbb Z\\textrm{ ou }5x=\\pi-\\frac{2\\pi}3-x+2k\\pi,\\ k\\in\\mathbb Z.$$\n\nEn résolvant individuellement chaque équation, on trouve que \n\n$$\\sin(5x)=\\sin\\left(\\frac{2\\pi}3+x\\right)\\iff x=\\frac\\pi 6+k\\frac\\pi 2,\\ k\\in\\mathbb Z\\textrm{ ou }x=\\frac\\pi{18}+k\\frac\\pi 3,\\ k\\in\\mathbb Z.$$\n\n On sait que \n\n$$\\cos\\left(x+\\frac\\pi4\\right)=\\cos(2x)\\iff x+\\frac\\pi 4=2x+2k\\pi,\\ k\\in\\mathbb Z \\textrm{ ou }x+\\frac \\pi 4=-2x+2k\\pi,\\ k\\in\\mathbb Z.$$\n\nEn résolvant individuellement chacune de ces deux équations, on trouve que l'ensemble des solutions est \n\n$$\\left\\{\\frac{\\pi}4+2k\\pi:\\ k\\in\\mathbb Z\\right\\}\\cup\\left\\{-\\frac{\\pi}{12}+\\frac{2k\\pi}3:\\ k\\in\\mathbb Z\\right\\}.$$\n\n On remarque d'abord que l'équation a un sens pour $2x\\neq \\frac\\pi2+k\\pi,\\ k\\in\\mathbb Z$ et pour $x+\\frac{\\pi}4\\neq \\frac \\pi2+k\\pi,\\ k\\in\\mathbb Z.$\n\nIl faut donc chercher les solutions dans \n\n$$E=\\mathbb R\\backslash\\left(\\left\\{\\frac \\pi4+\\frac{k\\pi}2:\\ k\\in\\mathbb Z\\right\\}\\cup\\left\\{\\frac{\\pi}4+k\\pi:\\ k\\in\\mathbb Z\\right\\}\\right).$$\n\nPour $x\\in E,$ on écrit alors\n\n\\begin{align*}\n\n\\tan(2x)=\\tan\\left(x+\\frac\\pi 4\\right)&\\iff 2x=x+\\frac\\pi 4+k\\pi,\\ k\\in\\mathbb Z\\\\\n\n&\\iff x=\\frac \\pi 4+k\\pi,\\ k\\in\\mathbb Z.\n\n\\end{align*}\n\nMais $\\frac\\pi4+k\\pi,\\ k\\in\\mathbb Z$ n'est jamais dans $E.$ Ainsi, cette équation n'admet aucune solution.\n\n\n" }, { "question": " 13 - Équations trigonométriques - après formule de trigonométrie [Signaler une erreur] [Ajouter à ma feuille d'exos]Enoncé Résoudre dans $\\mathbb R$ les équations suivantes :\n\n$$\\begin{array}{ll}\n\n\\mathbf 1.\\ \\sin x\\cos x=\\frac 14.\n\n&\\mathbf 2.\\ \\sin\\left(2x-\\frac\\pi3\\right)=\\cos\\left(\\frac x3\\right)\\\\\n\n\\mathbf 3.\\ \\cos(3x)=\\sin(x)&\\mathbf 4. \\tan x=2 \\sin x.\\\\\n\n\\end{array}$$", "answer": " \n On se ramène à une équation simple en remarquant que $\\sin x\\cos x=\\frac 12\\sin(2x)$. L'équation est donc équivalente à $\\sin(2x)=\\frac 12$. Mais, \n\n$$\\begin{array}{rcl}\n\n\\sin(2x)=\\frac12&\\iff&\\exists k\\in\\mathbb Z, 2x=\\frac{\\pi}6+2k\\pi\\textrm{ ou }\\exists k\\in\\mathbb Z,\\ 2x=\\frac{5\\pi}6+2k\\pi\\\\\n\n&\\iff&\\exists k\\in\\mathbb Z, x=\\frac{\\pi}{12}+k\\pi\\textrm{ ou }\\exists k\\in\\mathbb Z,\\ x=\\frac{5\\pi}{12}+k\\pi.\n\n\\end{array}$$\n\n On transforme d'abord l'équation par une formule de trigonométrie :\n\n$$\\sin\\left(2x-\\frac\\pi3\\right)=\\cos\\left(\\frac x3\\right)\\iff \\sin\\left(2x-\\frac\\pi3\\right)=\\sin\\left(\\frac{\\pi}{ 2}-\\frac x3\\right).$$\n\nEn utilisant la même méthode qu'à la question précédente, on trouve :\n\n$$\\sin\\left(2x-\\frac\\pi3\\right)=\\cos\\left(\\frac x3\\right)\\iff x = \\frac{5\\pi}{14}+\\frac{6k\\pi}{7},\\ k\\in\\mathbb Z\\textrm{ ou }x =\\frac{\\pi}2+\\frac{6k\\pi}5,\\ k\\in\\mathbb Z.$$\n\n C'est exactement la même méthode. On trouve que \n\n$$\\cos(3x)=\\sin x\\iff x= \\frac{ \\pi}8+\\frac{k\\pi}2,\\ k\\in\\mathbb Z\\textrm{ ou }x = -\\frac{\\pi}4+k\\pi,\\ k\\in\\mathbb Z.$$\n\n On remarque d'abord que $x\\neq \\frac \\pi 2+k\\pi$, $k\\in\\mathbb Z$. Si tel est le cas, alors\n\n$$\\tan x=2\\sin x\\iff 2\\sin x\\cos x=\\sin x\\iff \\sin(2x)=\\sin x.$$\n\nOr, \n\n$$\\sin(2x)=\\sin x\\iff x=2k\\pi,\\ k\\in\\mathbb Z\\textrm{ ou }x=\\frac\\pi 3+\\frac{2k\\pi}3,\\ k\\in\\mathbb Z.$$\n\nOn vérifie (par exemple, sur le cercle trigonométrique), qu'aucune des solutions ne s'écrit $\\frac{\\pi}2+l\\pi$, $l\\in\\mathbb Z$ et on conclut finalement que :\n\n$$\\tan x=2\\sin x\\iff x=2k\\pi,\\ k\\in\\mathbb Z\\textrm{ ou }x=\\frac\\pi 3+\\frac{2k\\pi}3,\\ k\\in\\mathbb Z.$$\n\n\n" }, { "question": " 14 - Équations trigonométriques plus difficiles [Signaler une erreur] [Ajouter à ma feuille d'exos]Enoncé Résoudre les équations trigonométriques suivantes :\n\n$$\\begin{array}{ll}\n\n\\mathbf{1.}\\ \\cos x=\\sqrt 3\\sin(x)+1&\\quad \\mathbf{2.}\\ \\cos x+\\sin x=1+\\tan x.\n\n\\end{array}\n\n$$", "answer": " \n Pour ce type d'équations, la méthode est toujours la même. On commence par la transformer en \n\n$$\\cos(x)-\\sqrt 3\\sin(x)=1.$$\n\nOn factorise ensuite par $\\sqrt{1+(\\sqrt 3)^2}=2$. L'équation est équivalente à \n\n$$\\frac{1}2\\cos(x)-\\frac{\\sqrt 3}2\\sin(x)=\\frac 12.$$\n\nOn remarque ensuite que $\\cos(\\pi/3)=1/2$ et $\\sin(\\pi/3)=\\sqrt{3}/2.$\n\nL'équation s'écrit donc \n\n$$\\cos\\left(\\frac\\pi 3\\right)\\cos(x)-\\sin\\left(\\frac{\\pi}3\\right)\\sin(x)=\\frac 12.$$\n\nD'après une formule de trigonométrie, elle est encore équivalente à \n\n$$\\cos\\left(\\frac \\pi 3+x\\right)=\\cos\\left(\\frac\\pi 3\\right).$$\n\nLes solutions de cette équation sont les réels $x$ pour lesquels \n\n$$\\frac\\pi 3+x=\\frac\\pi 3+2k\\pi,\\ k\\in\\mathbb Z\\textrm{ ou }\\frac \\pi3+x=-\\frac{\\pi}3+2k\\pi,\\ k\\in\\mathbb Z.$$\n\nL'ensemble des solutions de l'équation est donc \n\n$$\\left\\{2k\\pi:\\ k\\in\\mathbb Z\\right\\}\\cup\\left\\{-\\frac{2\\pi}3+2k\\pi:\\ k\\in\\mathbb Z\\right\\}.$$\n\n Remarquons pour commencer que l'équation a un sens pour les $x$ tels que $x\\neq \\frac\\pi 2+k\\pi$. Pour ces $x$, elle est équivalente à \n\n\\begin{align*}\n\n\\cos x+\\sin x=\\frac{\\cos x+\\sin x}{\\cos x}&\\iff (\\cos x-1)(\\cos x+\\sin x)=0\\\\\n\n&\\iff \\cos x=1\\textrm{ ou }\\cos(x)+\\sin(x)=0\\\\\n\n&\\iff x=2k\\pi,\\ k\\in\\mathbb Z\\textrm{ ou }\\cos(x)=-\\sin(x)=\\cos\\left(\\frac\\pi 2+x\\right)\\\\\n\n&\\iff x=2k\\pi,\\ k\\in\\mathbb Z\\textrm{ ou }x=-\\frac{\\pi}2-x+2k\\pi,\\ k\\in\\mathbb Z\\\\\n\n&\\iff x=2k\\pi,\\ k\\in\\mathbb Z\\textrm{ ou }x=-\\frac{\\pi}4+k\\pi,\\ k\\in\\mathbb Z.\n\n\\end{align*}\n\nOn vérifie qu'aucune de ces solutions ne correspond à une valeur interdite pour laquelle l'équation n'a pas de sens.\n\n\n" }, { "question": " 15 - Equation du second degré [Signaler une erreur] [Ajouter à ma feuille d'exos]Enoncé Déterminer les réels $x$ vérifiant $2\\cos^2(x)+9\\cos(x)+4=0$. ", "answer": " On pose $X=\\cos(x)$, et l'équation devient $2X^2+9X+4=0$. Son discriminant est $\\Delta=49=7^2$, et ses racines sont $X_1=-4$ et $X_2=-1/2$. L'équation $\\cos(x)=-4$ n'a aucune solution. Les solutions de l'équation $\\cos(x)=-1/2$ sont les réels qui s'écrivent $\\frac{4\\pi}3+k2\\pi$, $k\\in\\mathbb Z$, et $\\frac{-4\\pi}3+k2\\pi$, $k\\in\\mathbb Z$. Finalement l'ensemble des solutions\n\nest \n\n$$\\left\\{ \\frac{4\\pi}3+k2\\pi:\\ k\\in\\mathbb Z\\right\\}\\cup \\left\\{\\frac{-4\\pi}3+k2\\pi:k\\in\\mathbb Z\\right\\}.$$\n" }, { "question": " 16 - Inéquations trigonométriques [Signaler une erreur] [Ajouter à ma feuille d'exos]Enoncé Résoudre sur $[0,2\\pi]$, puis sur $[-\\pi,\\pi]$, puis sur $\\mathbb R$ les inéquations suivantes : \n\n$$\\begin{array}{lll}\n\n\\mathbf{1.}\\ \\sin(x)\\geq 1/2&\\quad&\\mathbf{2.}\\cos(x)\\geq 1/2\n\n\\end{array}$$", "answer": " Il faut s'aider du cercle trigonométrique!\n\n\n Pour $x\\in [0,2\\pi]$, on a \n\n$$\\sin(x)\\geq 1/2\\iff x\\in\\left[\\frac{\\pi}6;\\frac{5\\pi}6\\right].$$\n\nPour $x\\in [-\\pi,\\pi]$, on a le même résultat : \n\n$$\\sin(x)\\geq 1/2\\iff x\\in \\left[\\frac{\\pi}6;\\frac{5\\pi}6\\right].$$\n\nFinalement, par $2\\pi$-périodicité, pour $x\\in \\mathbb R$, on a\n\n$$\\sin(x)\\geq 1/2\\iff \\exists k\\in\\mathbb Z,\\ x\\in \\left[\\frac{\\pi}6+2k\\pi;\\frac{5\\pi}6+2k\\pi\\right].$$\n\n Pour $x\\in [0,2\\pi]$, on a\n\n$$\\cos(x)\\geq 1/2\\iff x\\in \\left[0,\\frac\\pi3\\right]\\cup \\left[\\frac{5\\pi}3;2\\pi\\right].$$\n\nPour $x\\in [-\\pi,\\pi]$, on a\n\n$$\\cos(x)\\geq 1/2\\iff x\\in \\left[-\\frac{\\pi}3;\\frac{\\pi}3\\right].$$\n\nOn conclut de la même façon par $2\\pi$-périodicité, mais on utilise plutôt la deuxième expression qui est plus facile. Finalement, pour $x\\in\\mathbb R$, \n\n$$\\cos(x)\\geq 1/2\\iff \\exists k\\in \\mathbb Z,\\ x\\in\\left[-\\frac{\\pi}3+2k\\pi;\\frac{\\pi}3+2k\\pi\\right].$$\n\n\n" }, { "question": " 17 - Un système [Signaler une erreur] [Ajouter à ma feuille d'exos]Enoncé Déterminer l'ensemble des réels $x$ vérifiant :\n\n$$\\left\\{\\begin{array}{rcl}\n\n2\\cos(x)-\\sin(x)&=&\\sqrt 3+\\frac 12\\\\\n\n\\cos(x)+2\\sin(x)&=&\\frac{\\sqrt 3}2-1.\n\n\\end{array}\\right.$$", "answer": " Posons $X=\\cos x$ et $Y=\\sin x$. Alors $X$ et $Y$ sont solutions du système\n\n$$\\left\\{\\begin{array}{rcl}\n\n2X-Y&=&\\sqrt 3+\\frac 12\\\\\n\nX+2Y&=&\\frac{\\sqrt 3}2-1\n\n\\end{array}\\right.$$\n\nOn résout ce système en remarquant par exemple que \n\n$$2\\times 2X+X=2\\times\\left(\\sqrt 3+\\frac 12\\right)+\\frac{\\sqrt 3}2-1=\\frac{5\\sqrt 3}2.$$\n\nOn en déduit que $X=\\sqrt 3/2$ puis que $Y=-1/2$. Ainsi, on cherche $x\\in\\mathbb R$ tel que $\\cos(x)=\\sqrt 3/2$ et $\\sin(x)=-1/2$. La valeur $x=-\\pi/6$ est une solution particulière, et c'est la seule dans $[-\\pi;\\pi]$. On en déduit que l'ensemble des solutions est \n\n$$\\mathcal S=\\left\\{-\\frac\\pi 6+2k\\pi:\\ k\\in\\mathbb Z\\right\\}.$$\n" }, { "question": " 18 - Un système [Signaler une erreur] [Ajouter à ma feuille d'exos]Enoncé Déterminer l'ensemble des couples $(x,y)$ vérifiant les conditions suivantes :\n\n$$\\left\\{\n\n\\begin{array}{rcl}\n\n2\\cos(x)+3\\sin(y)&=&\\sqrt 2-\\frac 32\\\\\n\n4\\cos(x)+\\sin(y)&=&2\\sqrt 2-\\frac 12\\\\\n\nx\\in [-\\pi;\\pi],\\ y\\in [-\\pi;\\pi]\n\n\\end{array}\\right.$$", "answer": " Posons $X=\\cos(x)$ et $Y=\\sin(y)$. Alors le couple $(X,Y)$ est solution du système\n\n$$\\left\\{\n\n\\begin{array}{rcl}\n\n2X+3Y&=&\\sqrt 2-\\frac 32\\\\\n\n4X+Y&=&2\\sqrt 2-\\frac 12.\n\n\\end{array}\\right.$$\n\nOn résout ce système en remarquant que \n\n$$Y-2\\times 3Y=2\\sqrt 2-\\frac 12-2\\times\\left(\\sqrt 2-\\frac 32\\right)$$\n\nce qui donne encore\n\n$$-5Y=\\frac 52\\iff Y=-\\frac 12.$$\n\nOn en déduit ensuite que $X=\\frac{\\sqrt 2}2$. On a donc $\\sin(y)=-\\frac 12$ ce qui entraîne que $y=-\\pi/6$ ou $y=-5\\pi/6$, puis $\\cos x=\\sqrt 2/2$, ce qui entraîne $x=\\pi/4$ ou $x=-\\pi/4$. Finalement, on trouve 4 couples de solutions : \n\n$$\\mathcal S=\\left\\{\\left(\\frac \\pi4,-\\frac \\pi 6\\right);\n\n\\left(\\frac \\pi4,-\\frac{5\\pi} 6\\right);\\left(-\\frac \\pi4,-\\frac \\pi 6\\right);\\left(-\\frac \\pi4,-\\frac{5\\pi} 6\\right)\\right\\}.$$\n" }, { "question": " 19 - Inéquations trigonométriques [Signaler une erreur] [Ajouter à ma feuille d'exos]Enoncé Résoudre sur $\\mathbb R$ les inéquations suivantes :\n\n$$\\begin{array}{ll}\n\n\\mathbf 1.\\ \\tan x\\geq 1& \\mathbf 2.\\ \\cos(x/3)\\leq \\sin(x/3)\\\\\n\n\\mathbf 3.\\ 2\\sin^2 x\\leq 1& \\mathbf 4.\\ \\cos^2x \\geq \\cos2x.\n\n \\end{array}\n\n$$", "answer": " \n On travaille cette fois sur un intervalle de longueur $\\pi$. Puisque $\\tan(\\pi/4)=1$, on en déduit que l'ensemble des solutions est\n\n$$\\mathcal S=\\bigcup_{k\\in\\mathbb Z}\\left[\\frac{\\pi}4+k\\pi,\\frac{\\pi}2+k\\pi\\right[.$$\n\n On va d'abord poser $u=x/3$. Alors, \n\n$$\\cos(x/3)\\leq\\sin(x/3)\\iff \\cos(u)\\leq\\sin(u)\\iff \\exists k\\in\\mathbb Z,\\ u \\in \\left[\\frac\\pi 4+2k\\pi,\\frac{5\\pi}4+2k\\pi\\right].$$\n\nRevenant à $x=3u$, il vient \n\n$$\\cos(x/3)\\leq\\sin(x/3)\\iff \\exists k\\in\\mathbb Z,\\ x \\in \\left[\\frac{3\\pi} 4+6k\\pi,\\frac{15\\pi}4+6k\\pi\\right].$$\n\nOn en déduit que \n\n$$\\mathcal S=\\bigcup_{k\\in\\mathbb Z}\\left[\\frac{3\\pi} {4}+6k\\pi,\\frac{15\\pi}4+6k\\pi\\right].$$\n\n On commence par utiliser la formule de trigonométrie $\\cos(2x)=1-2\\sin^2(x)$. L'équation est donc équivalente à $\\cos(2x)\\geq 0$. Posons $u=2x$. Alors $\\cos(u)\\geq 0$ est équivalent à $u\\in \\bigcup_{k\\in\\mathbb Z}\\left[-\\frac{\\pi}2+2k\\pi;\\frac{\\pi}2+2k\\pi\\right]$. On en déduit que l'ensemble des solutions de l'inéquation initiale est donné par \n\n$$\\bigcup_{k\\in\\mathbb Z}\\left[-\\frac{\\pi}4+k\\pi;\\frac{\\pi}4+k\\pi\\right].$$\n\n On a \n\n$$\\cos^2 x\\geq\\cos 2x\\iff \\cos^2 x\\geq 2\\cos^2 x-1\\iff \\cos^2 x\\leq 1.$$\n\nCette dernière inégalité étant toujours vérifiée, on en déduit que pour tout $x\\in\\mathbb R$, on a\n\n$$\\cos^2 x\\geq\\cos 2x.$$\n\n\n" }, { "question": " 20 - Équation trigonométrique [Signaler une erreur] [Ajouter à ma feuille d'exos]Enoncé Pour quelles valeurs de $m$ l'équation $\\sqrt 3\\cos x-\\sin x=m$ admet-elle des solutions?\n\nLes déterminer lorsque $m=\\sqrt 2$.", "answer": " C'est une méthode classique. On met en facteur $\\sqrt{(\\sqrt 3)^2+1^2}=2$, de sorte que \n\n$$\\sqrt 3\\cos x-\\sin x=2\\left(\\frac{\\sqrt 3}2\\cos x-\\frac 12\\sin x\\right)=2\\big(\\cos(\\pi/6)\\cos x-\\sin(\\pi/6)\\sin x\\big)=2\\cos\\left(x+\\frac\\pi 6\\right).$$\n\nL'équation est donc équivalente à \n\n$$\\cos\\left(x+\\frac\\pi 6\\right)=\\frac{m}2.$$\n\nElle admet donc des solutions si et seulement si $m\\in[-2,2]$. Si $m=\\sqrt 2$, alors on a \n\n$$\\cos\\left(x+\\frac\\pi 6\\right)=\\frac{m}2=\\cos\\left(\\frac{\\pi}4\\right)\\iff x=\\frac{\\pi}{12}+2k\\pi,\\ k\\in\\mathbb Z\\textrm{ ou } x=-\\frac{5\\pi}{12}+2k\\pi,\\ k\\in\\mathbb Z.$$\n" }, { "question": " 21 - Trinôme du second degré? [Signaler une erreur] [Ajouter à ma feuille d'exos]Enoncé Résoudre dans $[0,2\\pi]$ l'équation $\\cos(2x)+\\cos(x)=0$.", "answer": " On sait que $\\cos(2x)=2\\cos^2 (x)-1$ et donc l'équation est équivalente à \n\n$$2\\cos^2(x)+\\cos(x)-1=0.$$\n\nPosons $y=\\cos x$ et déterminons les solutions de l'équation $2y^2+y-1=0$. Calculant le discriminant, on voit que les solutions de cette équation sont $y=-1$ et $y=1/2$. On en déduit donc que \n\n$\\cos(2x)+\\cos(x)=0$ si et seulement si $\\cos(x)=-1$ ou $\\cos(x)=1/2$. Puisque l'on cherche les solutions uniquement dans $[0,2\\pi]$, l'ensemble des solutions est $\\{\\pi,\\pi/3,5\\pi/3\\}$.\n" }, { "question": " 22 - Inéquation plus subtile qu'il n'y parait [Signaler une erreur] [Ajouter à ma feuille d'exos]Enoncé Résoudre dans $]-\\pi;\\pi]$ l'inéquation suivante : $\\tan(x)\\geq 2\\sin(x)$.", "answer": " Pour ne pas se tromper en simplifiant par $\\sin x$, on passe tout à gauche et on factorise : \n\n$$\\tan(x)-2\\sin(x)=\\sin(x)\\left(\\frac{1}{\\cos x}-2\\right)=\\sin(x)\\frac{1-2\\cos x}{\\cos x}.$$\n\nOn conclut par un tableau de signes. \n\n\n\nOn trouve donc que l'ensemble des solutions est \n\n$$\\left]-\\pi,-\\frac\\pi2\\right[\\cup \\left[-\\frac{\\pi}3;0\\right]\\cup\\left[\\frac{\\pi}3,\\frac{\\pi}2\\right[\\cup\\{\\pi\\}.$$\n\nRemarquons qu'on aurait pu simplement étudier le signe de $\\sin(x)\\frac{1-2\\cos x}{\\cos x}$ sur $[0,\\pi]$ puis conclure par imparité de cette fonction.\n" }, { "question": " 23 - Du cosinus à l'angle [Signaler une erreur] [Ajouter à ma feuille d'exos]Enoncé On cherche à déterminer tous les réels $t$ tels que \n\n$$\\cos t=\\frac{1+\\sqrt 5}4.$$\n\n\n Démontrer qu'il existe une unique solution dans l'intervalle $]0,\\pi/4[$. Dans la suite, on notera cette solution $t_0$.\n\n Calculer $\\cos(2t_0)$, puis démontrer que $\\cos(4t_0)=-\\cos(t_0)$.\n\n En déduire $t_0$.\n\n Résoudre l'équation.\n\n\n", "answer": " \n Il est clair que $1=\\cos(0)>\\frac{1+\\sqrt 5}4$ et on vérifie aussi facilement que $\\cos(\\pi/4)=\\frac{\\sqrt 2}2<\\frac{1+\\sqrt 5}4$\n\n(par exemple, à la calculatrice, ou parce que \n\n$$(2\\sqrt 2)^2=8<(1+\\sqrt 5)^2=6+2\\sqrt 5).$$\n\nPuisque la fonction cosinus est continue et strictement décroissante sur l'intervalle $]0,\\pi/4[$, il existe un unique $t_0$ dans $]0,\\pi/4[$ tel que $\\cos(t_0)=\\frac{1+\\sqrt 5}4$.\n\n On a \n\n$$\\cos(2t_0)=2\\cos^2(t_0)-1=\\frac{\\sqrt 5-1}4.$$\n\nDe même, \n\n$$\\cos(4t_0)=2\\cos^2(2t_0)-1=\\frac{-1-\\sqrt 5}4=-\\cos(t_0).$$\n\n L'équation $\\cos(4t_0)=-\\cos(t_0)$ est équivalente à $\\cos(4t_0)=\\cos(\\pi-t_0),$\n\ndont les solutions vérifient\n\n$$4t_0=\\pi-t_0+2k\\pi,\\ k\\in\\mathbb Z\\textrm{ ou }4t_0=t_0-\\pi+2k\\pi,\\ k\\in\\mathbb Z.$$\n\nMais puisque $\\pi-t_0\\in ]3\\pi/4,\\pi[$ et $4t_0\\in ]0,\\pi[$, la première équation n'est possible que \n\npour $k=0$ et on trouve $t_0=\\pi/5$. Et comme $t_0-\\pi\\in]-\\pi,-3\\pi/4[$, on ne peut jamais avoir\n\n$t_0-\\pi+2k\\pi \\in]0,\\pi[$. On en déduit que $t_0=\\frac{\\pi}5.$\n\n Par le résultat du cours, les solutions de $\\cos t=\\frac{1+\\sqrt 5}4$ sont les réels $t=\\frac\\pi 5+2k\\pi$, $k\\in\\mathbb Z$\n\net $t=-\\frac\\pi 5+2k\\pi$, $k\\in\\mathbb Z$.\n\n\n" }, { "question": " 24 - Inéquations trigonométriques [Signaler une erreur] [Ajouter à ma feuille d'exos]Enoncé Résoudre sur $\\mathbb R$ les inéquations suivantes :\n\n\n $2\\cos^2 x-9\\cos x+4\\geq 0$;\n\n $\\cos 5x+\\cos 3x\\geq \\cos x$.\n\n", "answer": " \n Posons $X=\\cos x$. L'inéquation est alors équivalente à \n\n$2X^2-9X+4\\geq 0.$ Le discriminant du polynôme de second degré associé est $49=7^2$. \n\nSes deux racines sont $1/2$ et $4$. Ainsi, $2X^2-9X+4\\geq 0$ si et seulement si \n\n$X\\leq 1/2$ ou $X\\geq 4$. Revenant à $\\cos x=X$, le dernier cas est impossible et on a donc\n\n$$2\\cos^2x-9\\cos x+4\\geq 0\\iff \\cos x\\leq \\frac 12\\iff x\\in\\bigcup_{k\\in\\mathbb Z}\\left[\\frac{\\pi}3+2k\\pi,\\frac{5\\pi}3+2k\\pi\\right].$$\n\n On commence par utiliser une formule de trigonométrie : \n\n$$\\cos 5x+\\cos 3x=2\\cos(4x)\\cos x.$$\n\nAinsi, l'inéquation est équivalente à \n\n$$\\cos x\\left(2\\cos(4x)-1\\right)\\geq 0.$$\n\nOn va résoudre cette inéquation sur un intervalle de longueur $2\\pi$, puis on va utiliser la $2\\pi$-périodicité du cosinus.\n\nRemarquons aussi que l'inégalité est vrai en $\\pm\\frac\\pi 2$, là où le cosinus s'annule. Maintenant, \n\nsi $x\\in\\left]-\\frac{\\pi}2,\\frac{\\pi}2\\right[$, on a $\\cos(x)>0$ et l'inéquation est équivalente à \n\n$$2\\cos(4x)-1\\geq 0\\iff \\cos(4x)\\geq\\frac 12.$$\n\nCeci donne encore :\n\n$$\\exists l\\in\\mathbb Z,\\ x\\in \\left[l\\frac{\\pi}2-\\frac{\\pi}{12},l\\frac{\\pi}2+\\frac{\\pi}{12}\\right].$$\n\nOn cherche les solutions dans l'intervalle $\\left]-\\frac\\pi 2,\\frac\\pi 2\\right[$ et on trouve\n\n$$\\left]-\\frac{\\pi}2,-\\frac{5\\pi}{12}\\right]\\cup\\bigg[-\\frac{\\pi}{12},\\frac{\\pi}{12}\\bigg]\\cup\\left[\\frac{5\\pi}{12},\\frac{\\pi}2\\right[.$$\n\nOn cherche maintenant les solutions dans l'intervalle $\\left]\\frac\\pi 2,\\frac{3\\pi}2\\right[$. L'inéquation y est équivalente à \n\n$$2\\cos(4x)-1\\leq 0$$\n\ndont l'ensemble des solutions sur l'intervalle considéré est\n\n$$\\left[\\frac{7\\pi}{12},\\frac{11\\pi}{12}\\right]\\cup\\left[\\frac{13\\pi}{12},\\frac{17\\pi}{12}\\right].$$\n\nFinalement, l'ensemble des solutions sur $\\mathbb R$ est\n\n\\begin{eqnarray*}\n\n\\bigcup_{k\\in\\mathbb Z}\\bigg(\\left[-\\frac{\\pi}2+2k\\pi,-\\frac{5\\pi}{12}+2k\\pi\\right]\n\n\\cup\\bigg[-\\frac{\\pi}{12}+2k\\pi,\\frac{\\pi}{12}+2k\\pi\\bigg]\\cup\\left[\\frac{5\\pi}{12}+2k\\pi,\\frac{\\pi}2+2k\\pi\\right]\\cup\\\\\n\n\\quad\\quad\n\n\\left[\\frac{7\\pi}{12}+2k\\pi,\\frac{11\\pi}{12}+2k\\pi\\right]\\cup\\left[\\frac{13\\pi}{12}+2k\\pi,\\frac{17\\pi}{12}+2k\\pi\\right]\\Bigg).\n\n\\end{eqnarray*}\n\n\n" }, { "question": " 25 - Avec un déphasage [Signaler une erreur] [Ajouter à ma feuille d'exos]Enoncé On considère la fonction $f$ définie sur $\\mathbb R$ par \n\n$$f(x)=\\cos\\left(\\frac{3x}2-\\frac{\\pi}4\\right).$$\n\n\n Déterminer une période $T$ de $f$.\n\n Déterminer en quels points $f$ atteint son maximum, son minimum, puis résoudre l'équation $f(x)=0$.\n\n Représenter graphiquement la fonction $f$ sur l'intervalle $[-T,T]$.\n\n $f$ est-elle paire?\n\n", "answer": " \n Soit $T$ un nombre réel. On a \n\n$$f(x+T)=\\cos\\left(\\frac{3}2(x+T)-\\frac{\\pi}4\\right)=\\cos\\left(\\frac 32x -\\frac{\\pi}4+\\frac{3T}2\\right).$$\n\nOn a $f(x+T)=f(x)$ par exemple si $3T/2=2\\pi$, donc si $T=4\\pi/3$. La fonction $f$ est donc $4\\pi/3$ périodique.\n\n On a \n\n\\begin{align*}\n\nf(x)=1&\\iff \\exists k\\in\\mathbb Z,\\ \\frac32x-\\frac{\\pi}4=2k\\pi\\\\\n\n&\\iff \\exists k\\in\\mathbb Z,\\ x=\\frac{4k\\pi}3+\\frac{\\pi}6.\n\n\\end{align*}\n\nLa fonction $f$ atteint donc son maximum en les point $\\frac{4k\\pi}3+\\frac{\\pi}6$, $k\\in\\mathbb Z$.\n\nDe même, on a \n\n\\begin{align*}\n\nf(x)=-1&\\iff \\exists k\\in\\mathbb Z,\\ \\frac32x-\\frac{\\pi}4=\\pi+2k\\pi\\\\\n\n&\\iff \\exists k\\in\\mathbb Z,\\ x=\\frac{4k\\pi}3+\\frac{5\\pi}6.\n\n\\end{align*}\n\nLa fonction $f$ atteint donc son maximum en les point $\\frac{4k\\pi}3+\\frac{5\\pi}6$, $k\\in\\mathbb Z$. Finalement, on a aussi\n\n\\begin{align*}\n\nf(x)=0&\\iff \\exists k\\in\\mathbb Z,\\ \\frac32x-\\frac{\\pi}4=\\frac\\pi2+k\\pi\\\\\n\n&\\iff \\exists k\\in\\mathbb Z,\\ x=\\frac{2k\\pi}3+\\frac{\\pi}2.\n\n\\end{align*}\n\nLes solutions de l'équation $f(x)=0$ sont donc les réels $\\frac{2k\\pi}3+\\frac{\\pi}2$, $k\\in\\mathbb Z$.\n\n On peut représenter $f$ en s'inspirant de la courbe représentative de la fonction cosinus, et en utilisant les résultats de la question précédente, qu'on peut compléter par exemple avec $f(0)=\\cos(-\\pi/4)=\\sqrt 2/2$.\n\n On trouve : \n\n\n Puisque $f(0)\\neq 0$, la fonction $f$ n'est pas impaire. De plus, $f(\\pi/2)=0$ alors que $f(-\\pi/2)=\\cos(-\\pi)=-1\\neq -f(\\pi/2)$. La fonction $f$ n'est pas paire!\n\n\n" }, { "question": " 26 - Domaine, parité et périodicité [Signaler une erreur] [Ajouter à ma feuille d'exos]Enoncé Soit $f$ la fonction définie par $f(x)=\\ln\\left(\\left|\\sin\\left(\\frac\\pi2 x\\right)\\right|\\right)$. Quel est le domaine de définition de $f$? La fonction $f$ est-elle paire? impaire? périodique?", "answer": " On sait que $\\ln(u)$ est défini uniquement si $u>0$. Donc $\\ln\\left(\\left|\\sin\\left(\\frac\\pi2 x\\right)\\right|\\right)$ est défini uniquement si $\\left|\\sin\\left(\\frac\\pi2 x\\right)\\right|$ est strictement positif. La valeur absolue d'un réel étant toujours positive ou nulle, la fonction $f$ est bien définie pour les réels $x$ tels que \n\n$$\\sin\\left(\\frac\\pi2 x\\right)\\neq 0.$$\n\nMais on a \n\n\\begin{align*}\n\n\\sin\\left(\\frac\\pi2 x\\right)=0&\\iff \\exists k\\in\\mathbb Z,\\ \\frac\\pi2x=k\\pi\\\\\n\n&\\iff \\exists k\\in\\mathbb Z,\\ x=2k.\n\n\\end{align*}\n\nLa fonction $f$ est donc bien définie pour tous les réels, sauf les entiers pairs : $\\mathcal D_f=\\mathbb R\\backslash 2\\mathbb Z$.\n\nPour déterminer la parité de $f$, remarquons déjà que son domaine de définition est symétrique par rapport à $0$, et donc que si $x\\in\\mathcal D_f$, alors $-x\\in\\mathcal D_f$. Soit donc $x\\in\\mathcal D_f$. Alors\n\n\\begin{align*}\n\nf(-x)&=\\ln\\left(\\left|\\sin\\left(-\\frac\\pi2 x\\right)\\right|\\right) \\\\\n\n&=\\ln\\left(\\left|-\\sin\\left(\\frac\\pi2 x\\right)\\right|\\right)\\textrm{ (car la fonction $\\sin$ est impaire)}\\\\\n\n&=\\ln\\left(\\left|\\sin\\left(\\frac\\pi2 x\\right)\\right|\\right) \\textrm{ (car la fonction $|\\cdot|$ est paire)}\\\\\n\n&=f(x).\n\n\\end{align*}\n\nAinsi, la fonction $f$ est paire. \n\nDe plus, on a\n\n\\begin{align*}\n\nf(x+2)&=\\ln\\left(\\left|\\sin\\left(\\frac\\pi2 (x+2)\\right)\\right|\\right)\\\\\n\n&=\\ln\\left(\\left|\\sin\\left(\\frac\\pi2 x+\\pi\\right)\\right|\\right)\\\\\n\n&=\\ln\\left(\\left|-\\sin\\left(\\frac\\pi2 x\\right)\\right|\\right)\\\\\n\n&=\\ln\\left(\\left|\\sin\\left(\\frac\\pi2 x\\right)\\right|\\right)\\\\\n\n&=f(x)\n\n\\end{align*}\n\noù on a utilisé que $\\sin(t+\\pi)=-\\sin t$ et que $|-a|=|a|$. Ainsi, $f$ est périodique de période $2$. \n" }, { "question": " 27 - Périodique... [Signaler une erreur] [Ajouter à ma feuille d'exos]Enoncé On considère la fonction $f$ définie sur $\\mathbb R$ par\n\n$$f(x)=\\cos(3x)\\cos^3x.$$\n\n\n Pour $x\\in\\mathbb R$, exprimer $f(-x)$ et $f(x+\\pi)$ en fonction de $f(x)$. \n\nSur quel intervalle $I$ peut-on se contenter d'étudier $f$?\n\n Vérifier que $f'(x)$ est du signe de $-\\sin(4x)$, et on déduire le sens de variation de $f$ sur $I$.\n\n Tracer la courbe représentative de $f$.\n\n", "answer": " \n On a \n\n$$f(-x)=\\cos(-3x)(\\cos^3(-x))=\\cos(3x)\\cos^3(x)=f(x).$$\n\nLa fonction $f$ est paire, on peut se contenter de l'étudier sur $[0,+\\infty[$.\n\nDe plus, \n\n$$f(x+\\pi)=\\cos(3x+3\\pi)\\cos^3(x+\\pi)=-\\cos(3x)(-\\cos x)^3=f(x).$$\n\n$f$ est donc $\\pi$-périodique. Finalement, on peut se contenter d'étudier $f$ sur l'intervalle $I=[0,\\pi/2]$. On obtiendra aussi la courbe de $f$ sur $[-\\pi/2,\\pi/2]$ par parité. Cet intervalle est de longueur $\\pi$ et la fonction est $\\pi$-périodique. On va donc déduire le reste de la courbe par des translations de vecteur $k\\pi \\vec i$, $k\\in\\mathbb Z$.\n\n $f$ est dérivable sur $I$ et pour tout $x\\in I$, on a \n\n\\begin{eqnarray*}\n\nf'(x)&=&-3\\sin(3x)\\cos^3(x)-3\\cos(3x)\\sin(x)\\cos^2(x)\\\\\n\n&=&-3\\cos^2(x)\\big(\\sin(3x)\\cos(x)+\\sin(x)\\cos(3x)\\big)\\\\\n\n&=&-3\\cos^2(x)\\sin(4x).\n\n\\end{eqnarray*}\n\nPuisque $\\cos^2(x)\\geq 0$, $f'$ est bien du signe de $-\\sin(4x)$ sur l'intervalle $[0,\\pi/2]$. En particulier, \n\n\n si $x\\in[0,\\pi/4]$, $f'(x)\\leq 0$ et $f$ est décroissante;\n\n si $x\\in[\\pi/4,\\pi/2]$, $f'(x)\\geq 0$ et $f$ est croissante.\n\n\n On obtient le dessin suivant :\n\n\n\n" }, { "question": " 28 - Quotient de sinus [Signaler une erreur] [Ajouter à ma feuille d'exos]Enoncé On considère la fonction $f$ définie par \n\n$$f(x)=\\frac{\\sin x}{1+\\sin x}.$$\n\nOn note $\\Gamma$ sa courbe représentative dans un repère orthonormé.\n\n\n Quel est le domaine de définition de $f$? Vérifier que $f$ est $2\\pi$-périodique.\n\n Comparer $f(\\pi-x)$ et $f(x)$. Que dire sur $\\Gamma$?\n\n Étudier les variations de $f$ sur l'intervalle $\\left]-\\frac\\pi 2,\\frac\\pi 2\\right]$, \n\npuis déterminer la limite de $f$ en $-\\pi/2$. \n\n Construire $\\Gamma$ à l'aide des renseignements précédents.\n\n", "answer": " \n $f(x)$ est défini partout où le dénominateur ne s'annule pas, c'est-à-dire pour tout les $x$ avec $\\sin x\\neq -1$. Le domaine de définition de $f$ est donc \n\n$$\\mathcal D_f=\\mathbb R\\backslash \\left\\{-\\frac\\pi 2+2k\\pi;\\ k\\in\\mathbb Z\\right\\}.$$\n\nDe plus, la $2\\pi$-périodicité de $\\sin$ entraîne facilement la $2\\pi$-périodicité de $f$.\n\n De $\\sin(\\pi-x)=\\sin x$, on déduit que $f(\\pi-x)=f(x)$. Ceci signifie que la droite d'équation $x=\\pi/2$ est un axe de symétrie de $\\Gamma$.\n\n Posons $g(x)=\\frac x{x+1}$ et $h(x)=\\sin x$. On a $f=g\\circ h$. De plus, $h$ est croissante sur l'intervalle $]-\\pi/2,\\pi/2]$ dont l'image est $]-1,1]$. La fonction $g$ est elle croissante sur l'intervalle $]-1,1]$ (par exemple, on peut écrire $g(x)=1-\\frac1{x+1}$. Par composition, $f$ est croissante sur $]-\\pi/2,\\pi/2]$.\n\nOn a $\\sin(x)\\to -1^+$ lorsque $x$ tend vers $-\\pi/2$ et $\\lim_{x\\to -1^+}g(x)=-\\infty$. Ainsi, par composition de limites, $f$ tend vers $-\\infty$ en $-\\pi/2$. \n\n On construit d'abord $\\gamma$ sur $]-\\pi/2,\\pi/2]$. On la déduit sur $]-\\pi/2,3\\pi/2]$ par symétrie d'axe $x=\\pi/2$. Enfin, on l'obtient sur $\\mathbb R$ par périodicité de période $2\\pi$, et donc par des translations de vecteur $k2\\pi\\vec i$, $k\\in\\mathbb Z$. On obtient :\n\n\n" }, { "question": " 29 - Étude d'une fonction trigonométrique [Signaler une erreur] [Ajouter à ma feuille d'exos]Enoncé On considère la fonction $f$ définie par $f(x)=\\frac{\\sin x}{2+\\cos x}$. \n\n\n Déterminer le domaine de définition de $f$. Justifier que $f$ est dérivable sur son domaine de définition.\n\n Pour $x\\in\\mathbb R$, calculer $f(x+2\\pi)$ et $f(-x)$. Que peut-on en déduire sur la courbe représentative de $f$? En déduire qu'il suffit d'étudier $f$ sur $[0,\\pi]$ pour construire toute la courbe représentative de $f$.\n\n Montrer que, pour tout réel $x$, on a\n\n$$f'(x)=\\frac{1+2\\cos x}{(2+\\cos x)^2}.$$\n\n Étudier le signe de $1+2\\cos x$ sur $[0,\\pi]$.\n\n Établir le tableau de variations de $f$ sur $[0,\\pi]$.\n\n Tracer la courbe représentative de $f$.\n\n", "answer": " \n Puisque $\\cos x\\geq -1$ pour tout $x\\in\\mathbb R$, on a $2+\\cos x>0$. Le dénominateur ne s'annule pas, et $f$ est définie sur $\\mathbb R$ tout entier. Comme quotient de deux fonctions dérivables dont le dénominateur ne s'annule pas, $f$ est dérivable sur $\\mathbb R$.\n\n On a \n\n$$f(x+2\\pi)=\\frac{\\sin(x+2\\pi)}{2+\\cos(x+2\\pi)}=\\frac{\\sin x}{2+\\cos x}=f(x)$$\n\npuisque $\\sin$ et $\\cos$ sont $2\\pi$-périodiques. De plus, on a\n\n$$f(-x)=\\frac{\\sin(-x)}{2+\\cos(-x)}=\\frac{-\\sin(x)}{2+\\cos x}=-f(x).$$\n\nLa fonction $f$ est donc impaire. La courbe représentative de $f$ est donc symétrique par rapport à l'origine du repère. De plus, par $2\\pi$-périodicité, on peut limiter l'étude à un intervalle de longueur $2\\pi$ puis déduire la courbe représentative de $f$ par des translations de vecteur $(2\\pi,0)$. Il suffit donc d'étudier la fonction sur $[0,\\pi]$, construire la courbe sur cet intervalle, l'obtenir sur $[-\\pi,\\pi]$ par symétrie par rapport à $O$, puis sur $\\mathbb R$ par périodicité.\n\n En utilisant la formule de dérivabilité d'un quotient, on a \n\n$$f'(x)=\\frac{\\cos x(2+\\cos x)-(-\\sin x)(\\sin x)}{(2+\\cos x)^2}=\\frac{2\\cos x+\\cos^2 x+\\sin^2 x}{(2+\\cos x)^2}=\\frac{1+2\\cos x}{(2+\\cos x)^2}.$$\n\n On a $1+2\\cos x\\geq 0\\iff \\cos x\\geq -1/2$. En s'aidant du cercle trigonométrique, on trouve que $\\cos x\\geq -1/2$ sur $[0,2\\pi/3]$ et $\\cos x\\leq -1/2$ sur $[2\\pi/3,\\pi]$. \n\n On en déduit le tableau de variations suivant :\n\n\n On trouve la courbe suivante :\n\n\n\n" }, { "question": " 30 - Périodicité [Signaler une erreur] [Ajouter à ma feuille d'exos]Enoncé Soit $\\alpha\\in\\mathbb R$ et $f$ la fonction définie sur $\\mathbb R$ par $f(x)=\\cos(x)+\\cos(\\alpha x)$. On veut démontrer que $f$ est périodique si et seulement si $\\alpha\\in\\mathbb Q$.\n\n\n On suppose que $\\alpha=p/q\\in\\mathbb Q$. Démontrer que $f$ est périodique.\n\n On suppose que $\\alpha\\notin\\mathbb Q$. Résoudre l'équation $f(x)=2$. En déduire que $f$ n'est pas périodique.\n\n", "answer": " \n On remarque que \n\n\\begin{align*}\n\nf(x+2\\pi q)&=\\cos(x+2\\pi q)+\\cos\\left(\\frac pqx+2\\pi p\\right)\\\\\n\n&=\\cos(x)+\\cos\\left(\\frac pqx\\right)\\\\\n\n&=f(x).\n\n\\end{align*}\n\nAinsi, $f$ est $2\\pi q$-périodique.\n\n Puisque $\\cos$ est à valeurs dans $[-1,1]$, pour que $f(x)=2$, il est nécessaire et suffisant que $\\cos(x)=1$ et $\\cos(\\alpha x)=1$. \n\nLes solutions de $\\cos(x)=1$ sont les réels de la forme $2k\\pi$, avec $k\\in\\mathbb Z$. De plus,\n\n$$\\cos(\\alpha x)=1\\iff\\exists\\ell\\in\\mathbb Z,\\ \\alpha x=2\\ell\\pi\\iff \\exists \\ell\\in\\mathbb Z,\\ x=2\\ell\\pi/\\alpha.$$\n\nSi $x\\neq 0$ est solution de l'équation $f(x)=2$, il existe donc deux entiers relatifs $k$ et $\\ell$ non-nuls tels que $x=2k\\pi=2\\ell\\pi/\\alpha$. En particulier, $\\alpha=\\ell/k$ est un nombre rationnel, ce qui n'est pas le cas. \n\nDonc la seule solution de $f(x)=2$ est $0$. Ceci empêche $f$ d'être périodique, car si $f$ était périodique de période $T$, on aurait aussi $f(T)=2$.\n\n\n" }, { "question": " 31 - Linéariser! [Signaler une erreur] [Ajouter à ma feuille d'exos]Enoncé \n Établir la formule de trigonométrie \n\n$\\cos^4(\\theta)=\\cos(4\\theta)/8+\\cos(2\\theta)/2+3/8$.\n\n Fournir une relation analogue pour $\\sin^4(\\theta)$. \n\n", "answer": " \n D'après la formule d'Euler, on a \n\n$$\\cos^4(\\theta)=\\left(\\frac{e^{i\\theta}+e^{-i\\theta}}{2}\\right)^4.$$\n\nOn développe ensuite en utilisant la formule du binôme de Newton et on trouve :\n\n\\begin{align*} \n\n \\cos^4(\\theta)&=\\frac1{16}\\left(e^{4i\\theta}+4e^{3i\\theta}e^{-i\\theta}+6e^{2i\\theta}e^{-2i\\theta}+4e^{i\\theta}e^{-3i\\theta}+e^{-4i\\theta}\\right)\\\\\n\n &=\\frac1{16}\\left(e^{4i\\theta}+4e^{2i\\theta}+6+4e^{-2i\\theta}+e^{-4i\\theta}\\right)\\\\\n\n &=\\frac1{16}\\left(\\color{red}{e^{4i\\theta}+e^{-4i\\theta}}+\\color{blue}{4e^{2i\\theta}+4e^{-2i\\theta}}+6\\right)\\\\\n\n &=\\frac1{16}\\left(\\color{red}{2\\cos(4\\theta)}+\\color{blue}{8\\cos(2\\theta)}+6\\right)\\\\\n\n &=\\frac{\\cos(4\\theta)}8+\\frac{\\cos(2\\theta)}2+\\frac 38.\n\n\\end{align*}\n\n On fait la même chose, mais on part de \n\n$$\\sin^4(\\theta)=\\left(\\frac{e^{i\\theta}-e^{-i\\theta}}{2i}\\right)^4.$$\n\nEn utilisant le même argument, ainsi que $i^4=1$, on trouve\n\n\\begin{align*}\n\n \\sin^4(\\theta)&=\\frac1{16}\\left(e^{4i\\theta}-4e^{2i\\theta}+6-4e^{-2i\\theta}+e^{-4i\\theta}\\right)\\\\\n\n &=\\frac{\\cos(4\\theta)}{8}-\\frac{\\cos(2\\theta)}2+\\frac 38.\n\n\\end{align*}\n\n\n" }, { "question": " 32 - Linéariser! [Signaler une erreur] [Ajouter à ma feuille d'exos]Enoncé Linéariser $\\cos^5 x$, $\\sin^5 x$ et $\\cos^2 x\\sin^3 x$.", "answer": " On écrit :\n\n\\begin{eqnarray*}\n\n\\cos^5 x&=&\\left(\\frac{e^{ix}+e^{-ix}}{2}\\right)^5\\\\\n\n&=&\\frac{1}{32}\\left(e^{5ix}+5e^{i3x}+10e^{ix}+10e^{-ix}+5e^{-i3x}+e^{-i5x}\\right)\\\\\n\n&=&\\frac{1}{16}\\big(\\cos(5x)+5\\cos(3x)+10\\cos x\\big).\n\n\\end{eqnarray*}\n\nLe même raisonnement donne\n\n$$\\sin^5 x=\\frac{1}{16}\\big(\\sin(5x)-5\\sin(3x)+10\\sin(x)\\big).$$\n\nPour la dernière expression, on procède ainsi :\n\n\\begin{eqnarray*}\n\n\\cos^2 x\\sin^3 x&=&\\left(\\frac{e^{ix}+e^{-ix}}{2}\\right)^2\\left(\\frac{e^{ix}-e^{-ix}}{2i}\\right)^3\\\\\n\n&=&\\frac{e^{2ix}+2+e^{-2ix}}{4}\\times\\frac{e^{3ix}-3e^{ix}+3e^{-ix}-e^{-3ix}}{-8i}\\\\\n\n&=&\\frac{e^{5ix}-e^{3ix}-2e^{ix}+2e^{-ix}+e^{-3ix}-e^{-5ix}}{-32 i}\\\\\n\n&=&\\frac{2i\\sin(5x)-2i\\sin(3x)-4i\\sin(x)}{-32i}\\\\\n\n&=&\\frac{-1}{16}\\sin(5x)+\\frac1{16}\\sin(3x)+\\frac18\\sin(x).\n\n\\end{eqnarray*}\n" }, { "question": " 33 - Addition [Signaler une erreur] [Ajouter à ma feuille d'exos]Enoncé \n Démontrer la formule de trigonométrie \n\n$\\cos(4\\theta)=\\cos^4(\\theta)-6\\cos^2(\\theta)\\sin^2(\\theta)+\\sin^4(\\theta)$. \n\n Fournir une relation analogue pour $\\sin(4\\theta)$.\n\n", "answer": " \n D'après la formule de de Moivre, on sait que \n\n$$\\cos(4\\theta)+i\\sin(4\\theta)=\\big(\\cos(\\theta)+i\\sin(\\theta)\\big)^4.$$\n\nOn développe le second membre, toujours en utilisant la formule du binôme :\n\n\\begin{align*}\n\n \\big(\\cos(\\theta)+i\\sin(\\theta)\\big)^4&=\\cos^4(\\theta)+4\\cos^3(\\theta)(i\\sin(\\theta))+6\\cos^2(\\theta)(i\\sin(\\theta))^2\\\\\n\n &\\quad\\quad+4\\cos(\\theta)(i\\sin\\theta)^3+(i\\sin(\\theta))^4\\\\\n\n &=\\cos^4(\\theta)+4i\\cos^3(\\theta)\\sin(\\theta)-6\\cos^2(\\theta)\\sin^2(\\theta)\\\\\n\n &\\quad\\quad -4i\\cos(\\theta)\\sin^3(\\theta)+\\sin^4(\\theta).\n\n\\end{align*}\n\nFinalement, en identifiant les parties réelles et imaginaires, on trouve\n\n\\begin{align*}\n\n \\cos(4\\theta)&=\\cos^4(\\theta)-6\\cos^2(\\theta)\\sin^2(\\theta)+\\sin^4(\\theta)\\\\\n\n \\sin(4\\theta)&=4\\cos^3(\\theta)\\sin(\\theta)-4\\cos(\\theta)\\sin^3(\\theta).\n\n\\end{align*}\n\n \n" }, { "question": " 34 - Addition [Signaler une erreur] [Ajouter à ma feuille d'exos]Enoncé Exprimer $\\cos(5x)$ et $\\sin(5x)$ en fonction de $\\cos x$ et $\\sin x$.", "answer": " On va partir, en utilisant la formule de de Moivre, de \n\n$$\\cos(5x)+i\\sin(5x)=e^{i5x}=(e^{ix})^5=(\\cos x+i\\sin x)^5.$$\n\nOn développe ensuite ce produit et on identifie parties réelles et parties imaginaires. On trouve \n\n$$\\cos(5x)=\\cos^5 x −10\\cos^3x \\sin^2 x +5\\cos x \\sin^4 x$$\n\net\n\n$$\\sin(5x)=5\\cos^4x\\sin x −10\\cos^2x \\sin^3x +\\sin^5 x.$$\n" }, { "question": " 35 - Un calcul d'intégrale [Signaler une erreur] [Ajouter à ma feuille d'exos]Enoncé Calculer $\\int_0^{\\pi/2}\\cos^4t\\sin^2tdt$.", "answer": " On linéarise les fonctions trigonométriques à l'aide des nombres complexes :\n\n\\begin{eqnarray*}\n\n\\cos^4t\\sin^2t&=&\\left(\\frac{e^{it}+e^{-it}}{2}\\right)^4\\left(\\frac{e^{it}-e^{-it}}{2i}\\right)^2\\\\\n\n&=&\\frac{-1}{2^6}\\left(e^{i4t}+4e^{i2t}+6+4e^{-i2t}+e^{-i4t}\\right)\\left(e^{2it}-2+e^{-2it}\\right)\\\\\n\n&=&\\frac{-1}{2^6}\\left(e^{i6t}+2e^{i4t}-e^{i2t}-4-e^{-i2t}+2e^{-i4t}+e^{-i6t}\\right)\\\\\n\n&=&\\frac{-1}{2^5}\\left(\\cos(6t)+2\\cos(4t)-\\cos(2t)-2\\right).\n\n\\end{eqnarray*}\n\nOn en déduit :\n\n\\begin{eqnarray*}\n\n\\int_0^{\\pi/2}\\cos^4t\\sin^2tdt&=&\\frac{-1}{2^5}\\int_0^{\\pi/2}\\big(\\cos (6t)+2\\cos (4t)-\\cos (2t)-2\\big)dt\\\\\n\n&=&\\frac{\\pi}{32}.\n\n\\end{eqnarray*}\n" }, { "question": " 36 - Sommes trigonométriques [Signaler une erreur] [Ajouter à ma feuille d'exos]Enoncé Soit $n\\in\\mathbb N^*$ et $x,y\\in\\mathbb R$. Calculer les sommes suivantes :\n\n\n $\\dis \\sum_{k=0}^n \\binom{n}{k}\\cos(x+ky)$;\n\n $\\displaystyle S=\\sum_{k=0}^n \\frac{\\cos(kx)}{(\\cos x)^k}\\textrm{ et }T=\\sum_{k=0}^n \\frac{\\sin(kx)}{(\\cos x)^k},$\n\navec $x\\neq\\frac{\\pi}2+k\\pi$, $k\\in\\mathbb Z$;\n\n $\\displaystyle D_n=\\sum_{k=-n}^n e^{ikx}$ et $\\displaystyle K_n=\\sum_{k=0}^n D_k$, avec $x\\neq 0+2k\\pi$, $k\\in\\mathbb Z$.\n\n", "answer": " \n On a :\n\n\\begin{eqnarray*}\n\n\\sum_{k=0}^n \\binom{n}{k}\\cos(x+ky)&=&\\Re\\left(\\sum_{k=0}^n \\binom{n}{k} e^{ix}e^{iky}\\right)\\\\\n\n&=&\\Re e\\left(e^{ix}\\sum_{k=0}^n\\binom{n}{k} \\left(e^{iy}\\right)^k1^{n-k}\\right)\\\\\n\n&=&\\Re e\\left(e^{ix}(1+e^{iy})^n\\right)\\\\\n\n&=&\\Re e\\left(e^{ix}e^{iny/2}(e^{-iy/2}+e^{iy/2})^n\\right)\\\\\n\n&=&\\Re e\\left(e^{i(x+ny/2)}(2\\cos(y/2))^n\\right)\\\\\n\n&=&2^n\\cos(x+ny/2)\\cos^n(y/2).\n\n\\end{eqnarray*}\n\n On utilise $S+iT$ qui se calcule comme une somme géométrique :\n\n$$S+iT=\\sum_{k=0}^n \\frac{e^{ikx}}{(\\cos x)^k}=\\sum_{k=0}^n \\left(\\frac{e^{ix}}{\\cos x}\\right)^k.$$\n\nOn distingue deux cas :\n\n\n Si $x=0\\ [\\pi]$, alors $\\frac{e^{ix}}{\\cos x}=1$, et $S+iT=n+1$. On en déduit $S=n+1$ et $T=0$.\n\n Si $x\\neq 0\\ [\\pi]$, alors \n\n\\begin{eqnarray*}\n\nS+iT&=&\\frac{1-\\left(\\frac{e^{ix}}{\\cos x}\\right)^{n+1}}{1-\\frac{e^{ix}}{\\cos x}}\\\\\n\n&=&\\frac{1}{(\\cos x)^n}\\times\\frac{(\\cos x)^{n+1}-e^{ix(n+1)}}{\\cos x-e^{ix}}\\\\\n\n&=&\\frac{1}{(\\cos x)^n}\\times\\frac{\\cos^{n+1}(x)-\\cos\\big((n+1)x\\big)-i\\sin\\big((n+1)x\\big)}{-i\\sin(x)}\\\\\n\n&=&\\frac{\\sin\\big((n+1)x\\big)}{\\cos^n(x)\\sin(x)}+i\\frac{\\cos^{n+1}(x)-\\cos\\big((n+1)x\\big)}{\\cos^nx\\sin x}.\n\n\\end{eqnarray*}\n\nOn en déduit \n\n$$S=\\frac{\\sin\\big((n+1)x\\big)}{\\cos^n(x)\\sin(x)}\\textrm{ et }T=\\frac{\\cos^{n+1}(x)-\\cos\\big((n+1)x\\big)}{\\cos^nx\\sin x}.$$\n\n\n $D_n$ est une somme géométrique, de premier terme $e^{-inx}$ et de raison $e^{ix}\\neq 1$. On obtient donc\n\n$$D_n=\\frac{e^{-inx}-e^{i(n+1)x}}{1-e^{ix}}=\\frac{e^{ix/2}}{e^{ix/2}}\\times\\frac{e^{-i(n+1/2)x}-e^{i(n+1/2)x}}{e^{-ix/2}-e^{ix/2}}.$$\n\nOn en déduit que \n\n$$D_n=\\frac{\\sin\\left(\\left(n+\\frac12\\right)x\\right)}{\\sin (x/2)}.$$\n\nPour calculer $K_n$, une méthode (légèrement différente de celle de la question précédente) est d'écrire que\n\n$\\sin\\left(\\left(n+\\frac12\\right)x\\right)=\\Im m\\left(e^{i(n+1/2)x}\\right)$, puis d'utiliser une somme géométrique. On a en effet :\n\n\\begin{eqnarray*}\n\nK_n&=&\\frac{1}{\\sin(x/2)}\\Im m\\left(\\sum_{k=0}^n e^{i(k+1/2)x}\\right)\\\\\n\n&=&\\frac{1}{\\sin(x/2)}\\Im m \\left(e^{ix/2}\\sum_{k=0}^n e^{ikx}\\right)\\\\\n\n&=&\\frac{1}{\\sin(x/2)}\\Im m \\left(e^{ix/2}\\frac{1-e^{i(n+1)x}}{1-e^{ix}}\\right)\\\\\n\n&=&\\frac1{\\sin(x/2)}\\Im m\\left(e^{ix/2}\\frac{e^{i(n+1)x/2}\\sin((n+1)x/2)}{e^{ix/2}\\sin(x/2)}\\right)\\\\\n\n&=&\\frac{\\sin^2\\big((n+1)x/2\\big)}{\\sin^2(x/2)}.\n\n\\end{eqnarray*}\n\n\n" }, { "question": " 37 - Somme de modules [Signaler une erreur] [Ajouter à ma feuille d'exos]Enoncé Soit $n\\in\\mathbb N^*$; on note $\\mathbb U_n$ l'ensemble des racines $n$-ièmes de l'unité. Calculer $\\sum_{z\\in \\mathbb U_n}|z-1|$.", "answer": " Soit $k\\in\\{0,\\dots,n-1\\}$ et soit $\\omega_k=e^{2ik\\pi/n}$. Alors \n\n$$|\\omega_k-1|=|e^{i2k\\pi/n}-e^{i0}|=2|\\sin(k\\pi/n)|$$\n\nen factorisant par l'angle moitié. De plus, pour $k\\in\\{0,\\dots,n-1\\}$, $k\\pi/n\\in[0,\\pi]$ et le sinus est positif. On en déduit\n\n\\begin{eqnarray*}\n\n\\sum_{z\\in\\mathbb U_n}|z-1|&=&2\\sum_{k=0}^{n-1}\\sin(k\\pi/n)\\\\\n\n&=&2\\Im m\\left(\\sum_{k=0}^{n-1}e^{ik\\pi/n}\\right)\\\\\n\n&=&4\\Im m\\left(\\frac 1{1-e^{i\\pi/n}}\\right)\\\\\n\n&=&4\\Im m\\left(\\frac 1{-2i\\sin(\\pi/2n)e^{i\\pi/2n}}\\right)\\\\\n\n&=&2\\Im m\\left(\\frac{ie^{-i\\pi/2n}}{\\sin(\\pi/2n)}\\right)\\\\\n\n&=&2\\frac{\\cos(\\pi/2n)}{\\sin(\\pi/2n)}.\n\n\\end{eqnarray*}\n" }, { "question": " 38 - Calcul d'un cosinus [Signaler une erreur] [Ajouter à ma feuille d'exos]Enoncé A partir de la somme des racines $5-$ièmes de l'unité, calculer $\\cos(2\\pi/5)$.", "answer": " La somme des racines $5-$ièmes de l'unité est nulle. On a donc\n\n$$1+e^{2i\\pi/5}+e^{i4\\pi/5}+e^{i6\\pi/5}+e^{i8\\pi/5}=0.$$\n\nOr, \n\n$$e^{i8\\pi/5}=e^{-2i\\pi/5}\\textrm{ et }e^{6i\\pi/5}=e^{-4i\\pi/5}.$$\n\nUtilisant les formules d'Euler, on en déduit que \n\n$$1+2\\cos(2\\pi/5)+2\\cos(4\\pi/5)=0.$$\n\nOr, \n\n$$\\cos(4\\pi/5)=2\\cos^2(2\\pi/5)-1,$$\n\nce qui donne\n\n$$4\\cos^2(2\\pi/5)+2\\cos(2\\pi/5)-1=0.$$\n\n$\\cos(2\\pi/5)$ est donc une racine de l'équation $4X^2+2X-1=0$. Le discriminant de ce polynôme du second degré\n\nest $\\Delta=20$, et ses racines sont \n\n$$x_1=\\frac{-1-\\sqrt 5}4\\textrm{ et }x_2=\\frac{-1+\\sqrt 5}4.$$\n\nPuisque $\\cos(2\\pi/5)>0$, on en déduit que \n\n$$\\cos(2\\pi/5)=\\frac{-1+\\sqrt 5}4.$" } ]